Download as pdf or txt
Download as pdf or txt
You are on page 1of 294

Orthodontics

&Paediatric
Dentistry
To my parents (Declan) and to Philly, Izzy, Ollie, Mum and Dad (Peter) or their enduring love and support.

For Elsevier
Content Strategist: A lison Taylor
Content Develop m ent Sp ecialist: Sally Davies
Project Manager: Louisa Talbott
Designer/ Design Direction: Christian Bilbow
Illustration Manager: Lesley Frazier
CLINICAL P ROBLEM S OLVIN G IN DENTISTRY

SERIES
THIRDEDITION

Orthodontics
&Paediatric
Dentistry
Declan Millett
BDSc DDS FDSRCPS(Glasg) FDSRCSEng DOrthRCSEng
MOrthRCSEng FHEA
Pro essor o Orthodontics
Cork University Dental School and Hospital
University College Cork
Cork, Ireland

Peter Day
BDS PhD FDS(Paeds)RCSEng MPaedDent(RCSEng) MFDS(RCSEng)
FRCD(Canada) FIADT PGCLTHE FHEA
Associate Pro essor and Consultant in Paediatric Dentistry
School o Dentistry at the University o Leeds and Brad ord District Care NHS
Foundation Trust Salaried Dental Service
Leeds, UK
With contributions rom

Caroline Campbell
BDS MSc MPaedDent(Ed) MFDS RCPS(Glasg) FDSPaedDent(Glasg)
Consultant in Paediatric Dentistry, Honorary Senior Clinical Lecturer
Department o Paediatric Dentistry
Glasgow Dental Hospital and School
University o Glasgow
Glasgow, UK

Marie Therese Hosey


BDS MSc(MedSci) DDS FDS RCPS(Glas)
Pro essor o Paediatric Dentistry
Population and Patient Health Division
King’s College London Dental Institute
London, UK

Edinburgh London New York Ox ord Philadelphia St Louis Sydney Toronto 2017
© 2017 Elsevier Ltd . All rights reserved .
Previou s ed itions:
© 2005, 2010, reprinted 2011 (tw ice) and 2013 Elsevier Ltd . All rights reserved .

N o part of this p ublication m ay be rep rod u ced or transm itted in any form or by any m eans,
electronic or m echanical, inclu d ing p hotocopying, record ing, or any inform ation storage and
retrieval system , w ithou t p erm ission in w riting from the p u blisher. Details on how to seek
perm ission, fu rther inform ation about the Pu blisher ’s perm issions p olicies and ou r arrangem ents
w ith organizations such as the Cop yright Clearance Center and the Cop yright Licensing Agency,
can be fou nd at ou r w ebsite: w w w.elsevier.com / p erm issions.

This book and the ind ivid u al contribu tions contained in it are p rotected u nd er cop yright by the
Pu blisher (other than as m ay be noted herein).

ISBN 978-0-7020-5836-3

N otices
Know led ge and best p ractice in this eld are constantly changing. As new research and exp erience
broad en ou r und erstand ing, changes in research m ethod s, professional p ractices, or m ed ical
treatm ent m ay becom e necessary.
Practitioners and researchers m u st alw ays rely on their ow n exp erience and know led ge in
evalu ating and using any inform ation, m ethod s, com p ou nd s, or exp erim ents d escribed herein. In
u sing su ch inform ation or m ethod s they shou ld be m ind fu l of their ow n safety and the safety of
others, includ ing parties for w hom they have a p rofessional resp onsibility.
With respect to any d ru g or p harm aceutical prod u cts id enti ed , read ers are ad vised to check the
m ost current inform ation provid ed (i) on p roced ures featu red or (ii) by the m anu factu rer of each
prod u ct to be ad m inistered , to verify the recom m end ed d ose or form u la, the m ethod and d u ration
of ad m inistration, and contraind ications. It is the resp onsibility of practitioners, relying on their ow n
exp erience and know led ge of their p atients, to m ake d iagnoses, to d eterm ine d osages and the best
treatm ent for each ind ivid u al p atient, and to take all ap p ropriate safety precau tions.
To the fullest extent of the law, neither the Pu blisher nor the au thors, contribu tors, or ed itors,
assum e any liability for any injury and / or d am age to p ersons or prop erty as a m atter of p rod u cts
liability, negligence or otherw ise, or from any use or op eration of any m ethod s, p rod u cts,
instru ctions, or id eas contained in the m aterial herein.

The
publisher’s
policy is to use
Printed in China paper manufactured
from sustainable forests

Last d igit is the print num ber: 9 8 7 6 5 4 3 2 1


Contents

1 Median diastema and ectopic 24 Pain control and treatment


eruption o an upper rst planning or carious primary
permanent molar 1 teeth 140
2 Unerupted upper central 25 Facial swelling and dental
incisor 5 abscess 146
3 Absent upper lateral 26 The uncooperative child and
incisors 11 adolescent 149
by M arie Therese Hosey
4 Crowding and buccal upper
canines 16 27 Children with disabilities and
learning dif culties 157
5 Severe crowding 25 by M arie Therese Hosey
6 Palatal canines 36 28 Common medical problems
7 More canine problems 46 in children 163
8 In raoccluded primary 29 The displaced primary incisor 168
molars 51 30 The ractured immature
9 Increased overjet 55 permanent incisor crown 171
10 Incisor crossbite 62 31 The root ractured
11 Reverse overjet 66 permanent incisor 175

12 Increased overbite 73 32 The avulsed incisor 179

13 Anterior open bite 81 33 Disorders o eruption and


ex oliation 184
14 Posterior crossbite 87
34 Poor quality rst permanent
15 Bilateral crossbite 91 molars 187
16 Late lower incisor crowding 99 35 Tooth discoloration,
17 Prominent chin and TMJDS 102 hypomineralization and
hypoplasia 191
18 Dri ting incisors 109
36 Mottled teeth 195
19 Appliance -related problems 113
37 Multiple missing and
20 Tooth movement and related abnormally shaped teeth 199
problems 118
38 Amelogenesis imper ecta 203
21 Cle t lip and palate 125
39 Dentinogenesis imper ecta 206
22 Nursing and early childhood
caries 130 40 Dental erosion 209
23 High caries risk adolescents 135 41 Gingival bleeding and
by Caroline Campbell enlargement 213
• vi
CONTENTS

42 Oral ulceration 216 A4 Implications o some medical


43 Mind Maps® 219 problems or orthodontics 269

Appendices A5 Lateral cephalometric analysis 271

A1 The index o orthodontic A6 A structured dental trauma


treatment need: dental history orm 273
health component 264 Index 275
A2 Classi cation and de nitions 265
A3 Orthodontic problems:
re erral guide 268
vii •

Pre ace to the third edition

The fond ness of stu d ents for this p roblem -solving form at face of orthod ontic and p aed iatric d entistry clinical practice.
and their encou raging feed back have led u s to being asked Three new orthod ontic append ices d eal w ith classi cation
by the p u blishers to d o a third ed ition. The passage of tim e and d e nitions, im plications of com m on m ed ical problem s
has brou ght a new co-au thor, Peter Day, to the Paed iatric and referral gu id elines. In ad d ition, one new paed iatric d en-
Dentistry section and a new contribu ting au thor, Marie- tistry append ix provid es a structured d ental trau ma history
Therese H osey. It is w ith great regret that Richard Welbury, form . Tw o new chap ters ad d ress m anagem ent of the physi-
w ho co-authored the rst and second ed itions, w as u nable cally and m ed ically com prom ised child . In p romotion of
to continue in this role and w e thank him m ost sincerely for evid ence-based best p ractice and to d irect further learning,
his form er involvem ent. We are ind ebted to ou r colleagues reference lists have been u pd ated throughou t to includ e
w ho have p rovid ed valu able su ggestions regard ing this Cochrane review s, w here these have been d evelop ed .
revision. It is ou r hope that this text w ill continu e to be of use to
The stru ctu re of the chap ters rem ains u nchanged , w ith u nd ergrad u ates and to those in the early years of post-
the presentation of a clinical problem follow ed by a step-by- grad u ate training.
step lead -through assessm ent, d iagnosis and treatm ent
planning. All chap ters and related Mind Map s®, w here rel- DTM
evant, have been u p d ated . Ad d itional cases are inclu d ed in PFD
several chap ters to re ect the contem p orary and changing Cork and Leeds 2016
• viii

Pre ace to the f rst edition

Problem solving is a core skill w hich the d ental und ergrad u - Mind Map s ® are also given for each topic to provid e a
ate m u st d evelop and re ne for exam inations and everyd ay focused fram ew ork for learning and revision. Each Mind
clinical p ractice. As orthod ontics and p aed iatric d entistry Map ® links key w ord s, or key points, w hich are highlighted
interface broad ly, com bined clinical teaching and exam ina- throu ghou t the text, to create an overview of the su bject and
tions in these d isciplines are linked increasingly to encour- is d esigned to trigger inform ation recall.
age holistic p roblem solving of d ental and occlu sal p roblem s Intend ed p rim arily for the u nd ergrad u ate, w e hope this
in the child and ad olescent p atient. book w ill be of valu e also to the ju nior postgrad u ate and to
This book aim s, therefore, to ad d ress a range of com m on those p rep aring for m em bership exam inations.
clinical p roblem s encou ntered in orthod ontic and p aed iatric
d ental p ractice. The form at p rom otes a logical ap p roach to DTM
problem solving throu gh history taking, clinical exam ina- RRW
tion and d iagnosis, w hich u nd erp in the princip les of treat- Cork and Glasgow 2004
m ent p lanning for both d iscip lines. A short reference list is
provid ed w ith each chap ter to facilitate fu rther d irected
learning.
ix •

Acknowledgements

We are p articu larly gratefu l to Mrs K Shep herd and Mrs G of the illu strations. Mr K Evans also kind ly assisted w ith the
Drake for their help and su p p ort in the p rep aration of p ho- p rod uction of the app liances show n in Figs 9.4 and 13.8. Ms
tograp hic m aterial. We w ou ld also like to thank especially N Kelly is thanked esp ecially for the ed iting and com p ila-
Dr G McIntyre, Ms R Bryan, Mr J C Aird , Dr A Shaw, Miss tion of photograp hic m aterial. Thank you to stud ents and
D Fu ng, Dr T Ubaya, Dr C Cam p bell, Dr K O’Rourke, Dr P staff w ho have u nd ertaken som e or all of the care on w hich
Mu rray, Dr T McSw iney, Dr P Mu rp hy, Dr M Mead e, Dr L the cases are based on. We are also gratefu l to Buzan Centres
Darby, Dr S McMorrow, Dr S Littlew ood , Dr A Garry, Dr E Ltd for the style for the Mind Map s ®. Ou r gratitud e is
Salloum , Dr D Morris, Dr S Fayle, Dr A Mighell and Cork extend ed to the staff of Elsevier w ho have been very help fu l
University Dental School & H ospital for provision of som e throu ghout.
1
History o complaint
Brian’s p rim ary front teeth had a p leasing ap p earance w ith
a sm all m id line space in the u pp er arch; the low er p rim ary
front teeth w ere not spaced . There is no history of trau m a.
The permanent incisors erupted in their p resent positions.
Brian’s p erm anent m olars started to eru p t over a year ago
and there is no d iscomfort from any of them .

Median diastema Medical history


Brian is t and w ell.

and ectopic eruption Dental history


Brian attend s his general d ental practitioner every 6 m onths
but has not required any treatm ent.

o an upper frst Family history

permanent molar
Brian’s father had an u p p er m id line sp ace that w as closed
w ith a xed ap pliance.

Examination
Extraoral examination
Brian has a Class I skeletal p attern w ith average FMPA and
SUMMARY no facial asym m etry. Lip s are com petent w ith the low er lip
resting at the incisal third of the up p er central incisors.
Brian is almost 8 years o age. He presents with a There are no tem porom and ibu lar joint signs or sym ptom s.
gap between his upper ront teeth and crooked
lower ront teeth (Fig. 1.1); an ‘adult’ upper back Intraoral examination
tooth also does not appear to be coming through Soft tissu es are healthy and the d entition is caries-free. The
properly. What are the causes o these problems intraoral view s are show n in Figs 1.1 and 1.2.
and what treatment would you recommend? ■ What do you observe?
Low-lying maxillary labial renum.
History 6 E D C B1 1B C D
The ollowing teeth are visible:
6 E D C 21 12 C D E 6
Complaint
(note E6 present, but 6 erupting into E).
Brian’s m other noticed the gap betw een his u p p er front
teeth and the irregu larity of his low er front teeth. She is Mild lower labial segment crowding with distolabial rotations
anxiou s abou t his ap p earance and is keen for treatm ent to o 1| 1; slight spacing distal o 2 | 2.
be provid ed . Recently she has also noticed that one of his Upper median diastema with the crowns o 1 1 f ared distally.
new ‘ad u lt’ u p p er back teeth is not com ing throu gh p rop -
Class III incisor relationship.
erly com pared w ith his other u pper back tooth on the oppo-
site sid e. B| B
Crossbites .
C| C

Fig. 1.1 Anterior occlusion at presentation. Fig. 1.2 Lower occlusal view (note 6 | 6 erupted but not shown).
•2
1 MEDIANDIASTEMAANDECTOPICERUPTIONOFANUPPERFIRSTPERMANENTMOLAR

■ Wha t is the aetiology o the 1 | 1 rotations? d istal tilt and rotations of 1| 1 m ay ind icate inherent crow d -
ing. Also, there is no low er prim ate (anthropoid ) sp ace
Incisor rotations are usu ally a m anifestation of inherent
betw een the p rim ary canines and rst p rim ary m olars.
crow d ing in the arch, w hich is genetic in origin. The lack of
Spacing betw een the up per perm anent central incisors
primary low er incisor sp acing rep orted by the child ’s
( ared d istally and form erly d escribed as the ‘u gly d u ck-
m other is p red ictive of likely crow d ing of the p erm anent
ling’ stage, term s best avoid ed w ith concerned p arents) is
successors. Incisor rotations may also resu lt from ectopic
also norm al at this stage, but generalized sp acing of the
position of the tooth germ s or from the p resence of a su p er-
up p er prim ary teeth inclu d ing the u p per prim ate spaces
nu merary tooth.
(located betw een the u p p er p rim ary lateral incisors and the
■ Wha t are the possible causes o the upper up p er prim ary canines) shou ld exist.
median dia stema? Although the prim ary incisor relationship is com m only
These are listed in Table 1.1. ed ge-to-ed ge at 5–6 years w ith incisor attrition, it is not
usu al for the perm anent incisor relationship to be sim ilar.
■ Is the dental and occlusal development normal?
Rather a Class I incisor relationship should be present.
Dental d evelopm ent is norm al. Erup tion d ates of the B B
A crossbite shou ld not exist on || .
prim ary and p erm anent d entition are given in Table 1.2. C C
It is com m on for som e crow d ing to be present as the The rst perm anent m olars shou ld norm ally be in a half-
low er incisors eru pt, w hich u su ally m anifests itself as slight unit Class II relationship d u e to the ‘ ush term inal p lanes’
lingual p lacem ent and / or rotation of the teeth, bu t the slight relationship of the second p rim ary m olars.

Table 1.1 Causes o an upper median diastema


Key point
Causes Comments
On eruption:
Developmental Due to pressure o 2 2 on 1 1 roots
( ormerlyre erred to as‘uglyduckling’stage); • Some crowding o 21| 12 is usual.
tends to resolve bythe time 3 3 erupt • A median diastema between 1 1 is normal.
Dentoalveolar disproportion Small teeth in a large arch
Absent or peg shaped 2’s
■ In the developing dentition, how is space created or the
Supernumerarytooth/teeth in midline
upper permanent incisor teeth?
Proclination o 21 12 Maybe due to a digit sucking habit
Space is obtained from three sources: the spacing w hich
Prominent labial renum Implicated where there is blanching o the
should exist betw een the prim ary incisors; an increase in
incisive papilla on stretching the renumand
intercanine w id th; and by the p erm anent u p p er incisors
notching between 1 1 is seen on radiograph
eru p ting m ore labially and p roclined com p ared w ith their
Pathological Cyst/tumour
p red ecessors. On average, the u p per and low er intercanine
Juvenile periodontitis w id th increases by abou t 1–2 m m in the prim ary d entition;
a further increase of around 3 m m occu rs d u ring the m ixed
Table 1.2 Eruption dates or primary and permanent teeth d entition but is generally comp lete by abou t 9 years w hen
the u pper and low er perm anent lateral incisors are fu lly
Primary Months Permanent Years
erup ted although som e m inor increase occu rs u ntil 13 years.
Upper Upper
Central incisor 6–7 Central incisor 7–8
Lateral incisor 7–8 Lateral incisor 8–9
Key point
Canine 18–20 Canine 11–12 Intercanine width:
First molar 12–15 First premolar 10–11 • Increases ~1 2 mm during primary dentition.
Second molar 24–36 Second premolar 10–12 • Increases ~3 mm in mixed dentition.
First molar 6–7 • Is generally complete by ~9 years, with minor increase
to ~13 years.
Second molar 12–13
Third molar 17–21
Lower Lower
Central incisor 6–7 Central incisor 6–7 Investigations
Lateral incisor 7–8 Lateral incisor 7–8 ■ What investigations would you undertake? Explain why.
Canine 18–20 Canine 9–10
First molar 12–15 First premolar 10–12 Clinical
Second molar 24–36 Second premolar 11–12 • Gently pull the upper lip upwards and observe i there is
First molar 5–6 blanching o the incisive papilla rom the renal attachment.
This may implicate the renum in the possible aetiology
Second molar 12–13
o the upper median diastema. In Brian’s case, slight
Third molar 17–21
blanching o the incisive papilla was detected.
3•
MEDIANDIASTEMAANDECTOPICERUPTIONOFANUPPERFIRSTPERMANENTMOLAR 1
• Check i there is a mandibular displacement associated with the sm all existing spaces d istal to them (Fig. 1.4). Further
BB im p rovem ent in the low er incisor crow d ing is also likely
the crossbites on . I a displacement is detected,
CC u ntil about 9 years of age as the intercanine w id th increases.
early crossbite correction is indicated. Brian, however, did The up per med ian d iastem a is likely to red u ce as the
not have a mandibular displacement, which was m axillary p erm anent lateral incisors and canines erupt.
con rmed by the absence o a lower centreline shi t. Brian’s m other should be reassu red abou t this. The attach-
m ent of the m axillary labial frenu m, althou gh initially to the
Radiographic incisive papilla d u ring the p rim ary d entition, m oves to the
A left bitew ing rad iograp h cou ld be taken to assess 6 labial attached m u cosa as the p ermanent lateral incisors
p osition/ statu s of any root resorp tion of E and an u p p er eru p t and ap p roxim ate the p erm anent central incisors (Fig.
anterior occlu sal rad iograp h to assess p resence/ absence of 1.5). In a sp aced arch, this m igration of the frenum is less
an u p p er m id line su p ernu m erary. Alternatively, a d ental likely. In contrast, w here the u pp er arch is potentially
p anoram ic tom ogram , w hich is easier for the child to coop- crow d ed and the d iastem a is less than 4 m m, recession of
erate w ith, cou ld be taken initially, w hich w ill also allow the frenum and closu re of the m ed ian d iastem a m ay be
you to ascertain the p resence, p osition and form of all forthcom ing eventu ally. H ow ever, in the p resent case, as
u neru pted teeth. Brian’s father had an u pp er m ed ian d iastem a, there may be
If a supernu m erary tooth/ teeth or other pathology is a tend ency for the sp ace to persist.
observed or su sp ected on the d ental p anoram ic tom ogram ■ How common is impaction o 6?
in the anterior p rem axilla, an u p p er anterior occlusal rad io-
This anom aly of erup tion occurs in about 2–6% of child ren
grap h should be taken.
bu t has been rep orted in 20–25% of child ren w ith cleft lip
■ The dental panora mic tomogram is shown in Fig. 1.3. and / or p alate.
What do you notice?
■ Wha t are the ca uses o impaction o 6?
Normal alveolar bone levels.
Impaction o 6 is indicative o crowding.
A normally developing dentition, which is consistent with the
Both local and hereditary actors have been implicated (Table
patient’s chronological age.
1.3). A multi actorial mode o inheritance has been identi ed
Resorption o the distal root o E E. where both genetic and local actors act in combination.
Impaction o 6. ■ Describe the clinica l eatures o ectopic eruption o 6 and
cla ssif cation o this anomaly.
Diagnosis Ectopic erup tion of 6 is m anifested by eru ption m esial of its
norm al path. Com plete eru p tion of 6 is initially blocked by
■ What is the dia gnosis?
Mild Class III malocclusion in the early mixed dentition on a
Class I skeletal base with average FMPA. Mild lower labial
segment crowding; upper median diastema.
BB
Crossbites || with no mandibular displacement.
CC
Impacted 6.

■ What is the IOTN DHC gra de (see Appendix 1, p. 264)?


Expla in why.
4t – d u e to a p artially eru p ted and im pacted 6.
■ What trea tment would you advise or the labial segment
problems? Explain why.
N o treatm ent is ind icated at p resent. The m ild low er labial
segm ent crow d ing m ay red u ce slightly by d rift of 2| 2 into
Fig. 1.4 Lower occlusal view 1 year a ter presentation.

Fig. 1.3 Dental panoramic tomogram. Fig. 1.5 Anterior occlusion ollowing eruption o 2’s.
•4
1 MEDIANDIASTEMAANDECTOPICERUPTIONOFANUPPERFIRSTPERMANENTMOLAR
Table 1.3 Causes o impaction (ectopic eruption) o 6
Factor Cause
Local Signifcantlylarger 6 and more pronounced mesial angle o eruption o 6
Hereditary Familial tendency
Small maxilla

the d istal su rface of E, w hich then, in resp onse to tooth


contact, u nd ergoes resorp tion.
Ectop ic erup tion of 6 is d escribed as ‘reversible’ if d is-
im paction and fu ll eru p tion ensu e spontaneou sly. After 8
years of age, this occu rs rarely. If 6 rem ains im pacted u ntil
treated or p rem ature loss of E hap p ens spontaneou sly,
ectop ic eru ption of 6 is d escribed as ‘irreversible’. Fig. 1.6 Upper occlusal view ollowing extraction o E.

Treatment ■ How will the orthodontist manage impaction o 6 in


■ What treatment options a re there or irreversible ectopic this case?
eruption o 6? The variou s op tions regard ing d isim p action of 6 shou ld be
Without extraction o E Where im p action of 6 is m ild , a d iscu ssed w ith Brian and his p arents.
brass w ire separator m ay be tightened arou nd the contact It shou ld then be exp lained that if E becom es abscessed ,
area of E and 6 over several visits; this w ill release 6 by or attem pts to d isim p act 6 are u nsu ccessfu l, extraction of E
d isplacing it d istally. Discing the d istal surface of E and the w ill be required . Treatm ent to d eal w ith the resu ltant space
u se of a separating sp ring have also been p roposed . loss w ill be required thereafter.
If 6 exhibits marked m esial tip p ing, more active d istal Brian w as not keen for any orthod ontic treatm ent and ,
m ovem ent is requ ired . This may be achieved by a sp ring therefore, it w as d ecid ed to extract E in view of the
sold ered to a transp alatal bar u niting the D’s. The spring caries risk to 6. The consequent u p p er buccal segm ent
acts against a com posite stop bond ed to the occlu sal su rface crow d ing (Fig. 1.6) w ill be d ealt w ith in the p erm anent
of 6. d entition.
Im p ressions and a w ax registration for stu d y m od els
With extraction o E If there is m arked resorption or abscess shou ld be record ed of the d evelop ing Class III m alocclu -
formation of E, or if 6 cannot be d isim p acted w ith a sep arat- sion. This shou ld then be monitored u ntil the perm anent
ing sp ring, or if 6 is cariou s and p oor access im p ed es resto- d entition is fu lly established w hen treatm ent p lanning can
ration, extraction of E is u navoid able. As 6 erup ts w ith a be com pleted .
m esial inclination, sp ace loss occu rs rap id ly follow ing loss
of E. Consid eration shou ld be given to regaining sp ace by Primary resources and
d istalizing 6 w ith a sp ring on an u p p er rem ovable ap pliance
in cases of u nilateral loss of E. Where bilateral loss of E recommended reading
occu rs, d istal m ovem ent of 6’s m ay be achieved by sp rings Bjerklin K, Ku rol J, Valentin J 1992 Ectopic eru p tion of
sold ered to a transp alatal arch connecting both D’s or by m axillary rst p erm anent m olars and association w ith
other tooth and d evelop m ental d istu rbances. Eu r J Orthod
cervical traction to band s on 6’s. Alternatively, m anagem ent
14:369–375.
of the sp ace loss resu lting from extraction of E’s can be
Foster TD, Gru nd y MC 1986 Occlu sal changes from p rim ary to
d eferred u ntil the p erm anent d entition.
p erm anent d entitions. Br J Orthod 13:187–193.
H u ang WJ, Creath CJ 1995 The m id line d iastem a: a review of its
aetiology and treatm ent. Ped iatr Dent 17:171–179.
Key point
Ku rol J, Bjerklin K 1986 Ectop ic eru p tion of m axillary
For impacted 6 consider: rst p erm anent m olars: a review. ASDC J Dent Child
• Brass wire separator. 53:209–214.
• Discing distal sur ace o E.
• Move 6 distally.
For revision, see Mind Maps 1a and 1b,
• Extract E. pages 220 221, and Appendices 2 and
3 pages 265 268.
2
Medical history
N eil is t and w ell.

Examination
Extraoral examination
N eil has a m ild Class II skeletal p attern w ith slightly

Unerupted upper
increased FMPA. H is lip s are com p etent. N o facial asym -
m etry or abnorm al tem p orom and ibu lar joint signs or sym p -
tom s w ere d etected .

central incisor Intraoral examination


■ The appearance o the mouth is shown in Figs 2.1 and 2.2.
What do you notice?
Oral hygiene is air calculus is visible on the buccal aspect
o 6.

SUMMARY
Neil, a 9-year-old boy, presents with 1 unerupted
(Fig. 2.1). What are the possible causes and how
would you manage the problem?

History
Complaint
N eil’s m other is very concerned abou t the u neru p ted 1 as
he is 9 years old and the tooth has not yet ap peared ; 2 is
also eru p ting over B, and she d islikes the ap p earance.

History o complaint
A w as lost at abou t 6 years of age, and 1 eru p ted norm ally A
at 6.5 years of age. Unfortu nately, N eil fell over w hile
playing soccer w ith his class team 4 m onths ago and frac-
tu red 1, exp osing the p u lp, w hich w as treated by a coronal
pu lp otomy and p lacem ent of calciu m hyd roxid e.
■ Is there anything else you would wish to elicit rom the
history?
N eil’s m other shou ld be asked abou t any history of trau m a
to the prim ary incisors, p articu larly intru sion of BA .
There is no history of traum a to the prim ary d entition.
B

Fig. 2.1 Upper labial segment at presentation. Fig. 2.2 (A) Right buccal occlusion. (B) Anterior occlusion. (C) Le t
buccal occlusion.
•6
2 UNERUPTEDUPPERCENTRALINCISOR
Mild plaque deposits on most teeth associated with marginal
Box 2.1 Causes o unerupted or missing upper permanent
gingival erythema. central incisor
6 ED C 2 BA 1 2 C D E6 Missing
Early mixed dentition with present.
6 ED C 2 1 1 2 C E6 • Congenitally absent.
Restored incisal edge o 1, which also appears to be darker • Avulsed.
than the other incisors. • Extracted.
Class I malocclusion with mild lower and moderate upper Present but unerupted
labial segment crowding. • Ectopic position o the tooth germ.
Upper centreline to the right; lower centreline to the le t. • Dilaceration and/or displacement due to trauma.
Potential crowding lower le t quadrant. • Scar tissue.
Buccal segment relationship Class I bilaterally. • Supernumerary tooth.
• Crowding.
■ Why are the centrelines displa ced?
• Pathology, e.g. cyst, odontogenic tumour.
An im balance of u pper anterior tooth size (the retained A
is consid erably sm aller than an 1 ) has p rom oted the u p p er
centreline shift, bu t this has been aggravated by inherent
u pper arch crow d ing.
The low er centreline shift is d u e to early u nbalanced loss Box 2.2 Classifcation o supernumerary teeth (s/n) by
of D in a p otentially crow d ed arch. morphology and e ects on the dentition
• Conical or peg-shaped – most o ten lies between 1 1 and mayproduce no
■ Could the lower centreline shi t have been prevented?
e ect, a median diastema, incisor rotation or ailure o 1 eruption (Fig. 2.3A);
Follow ing rem oval of D the low er centreline shou ld have 75–78%o s/n.
been m onitored at review visits. D shou ld have been • Tuberculate or barrel-shaped – most usually associated with unerupted 1; ~12%
extracted to balance for loss of D w hen the centreline o s/n.
appeared to be m igrating. • Supplemental – resembles and lies adjacent to the last tooth o a series (2’s, 5’s,
■ With unilatera l loss o what other primary tooth, would you 8’s); likely to produce crowding, centreline shi t (Fig. 2.3B).
consider a balancing extraction to prevent a centerline shi t? • Odontome – may be either compound or complex; compound is more common
in the anterior maxilla; complexis more common in the premolar and molar
Where u nilateral loss of a p rim ary canine has occu rred (or areas; associated with unerupted/displaced teeth.
is planned as in the case of a p alatally ectop ic canine; see
Chap ter 6) and the contralateral tooth is not m obile, consid -
eration should be given to its rem oval to prevent loss of
the centreline. A balancing extraction of another second
Dilaceration and/or displacement due to trauma can be
primary m olar to p revent a potential centerline shift is
excluded due to the absence o a relevant history.
u nnecessary.
Scar tissue can be excluded also as this would result rom
trauma.
A supernumerary tooth (Box 2.2) is the most likely cause o
Key point unerupted 1. With an incidence o 1 3% in the premaxilla,
Always balance or unilateral loss/extraction o a primary supernumerary teeth (particularly the late- orming
canine. tuberculate type) are associated with delay or noneruption o
an upper permanent central incisor.
Crow d ing is an unlikely cau se. Althou gh the u pp er labial
segm ent is crow d ed , only very severe crow d ing w ou ld
■ Wha t are the possible causes o the unerupted 1 ? prevent 1 eru pting 2 years follow ing its expected eru ption
tim e.
These are listed in Box 2.1. Pathology is also an u nlikely cau se. There is no evid ence
■ How would you ra te the likelihood in this case o each o of alveolar exp ansion in the p rem axilla, w hich w ou ld m ost
the potential causes o unerupted 1 listed in Box 2.1? likely be d u e to cyst form ation p ossibly arising from 1, a
sup ernu m erary or od ontom e. Other rarer lesions w ou ld
Congenital absence o 1 is highly unlikely. It would be very
need to be exclud ed .
rare or 1 to be absent without other congenitally missing
teeth.
Avulsion o 1 can be excluded as there is no history o 1
having erupted or o incisor trauma.
Extraction o 1 can be excluded also. Key point
Ectopic position o the tooth germ is a possibility but is more A supernumerary tooth is the most common cause o
likely to be secondary to some pathological cause or the ailure o eruption o 1.
presence o a supernumerary tooth.
7•
UNERUPTEDUPPERCENTRALINCISOR 2
Radiographic The follow ing view s are required to d eter-
m ine the p resence/ absence of 1 and / or p ossible su p ernu -
m erary teeth:
• Dental panoramic tomogram gives a general screen o
the developing dentition allowing detection o the
presence/absence o unerupted teeth.
• Upper anterior occlusal or periapical views provide greater
detail o the anterior maxilla. In particular, the ollowing
can be assessed: the crown and root morphology o
unerupted 1, the presence o supernumerary teeth and/
or other pathology and their relation to the incisor roots,
the root and periapical status o traumatized 1. On a
panoramic radiograph these structures may be poorly
de ned due to superimposition o other anatomical
A eatures or by lying outside the ocal trough o the
tomogram. Periapical radiographs should include the
roots o adjacent teeth to determine i they were
damaged during previous trauma to 1.
Used in com bination and em ploying the principle of
vertical parallax, the d ental panoram ic tom ogram and the
u pper anterior occlu sal or periapical view s can be u sed to
localize the p osition of any u neru p ted tooth and / or su p er-
nu m erary relative to the d ental arch.

Key point
• Two radiographic views are required to localize an
B
unerupted tooth in the premaxilla using parallax.
Fig. 2.3 (A) Two conical supernumerary teeth between 1 1. • A lateral view may be required to aid localization o a
(B) Supplemental 2. dilaceration, i visible on either the dental panoramic
tomogram or on the upper anterior occlusal/periapical
■ What is the aetiology o supernumerary teeth? views.

Althou gh not entirely know n, genetic factors seem to p lay


a part w ith a m ale p red ilection. The cleft area is also fre-
qu ently affected w here the alveolu s is involved . Dichotom y ■ How would you determine the position o an unerupted
of the tooth bu d and localized ind ep end ent hyp eractivity or tooth in the anterior premaxilla using vertica l paralla x?
fragm entation of the d ental lam ina have also been su g-
If the tooth m oves in the sam e d irection as the tu be shift, it
gested ; the latter is esp ecially relevant to cleft lip and palate.
lies p alatal to the arch; if it m oves in the op p osite d irection
Mu ltip le su p ernu merary teeth rarely occu r bu t are associ-
to the tu be shift, it lies buccal to the arch. Where there is no
ated w ith cleid ocranial d ysp lasia, Gard ner synd rom e and
app arent shift in its position betw een the lm s, it lies in the
cleft lip and p alate.
line of the arch.
■ Neil’s radiographs a re shown in Fig. 2.4. Wha t do
these show?
Key point
The panoram ic tom ograp h show s all perm anent teeth to
Supernumerary teeth are more common in males.
be p resent, includ ing third m olars. Dental d evelopm ent
ap pears reasonably aligned w ith chronological age. There is
a supernu m erary tooth overlying 1 . Root resorption of the
remaining rst prim ary m olars is ad vanced , and caries is
Investigation D
evid ent in . Bitew ing rad iographs w ou ld be requ ired
ED E
■ What investigations are required? Expla in why.
for m ore accu rate assessm ent of the extent of cariou s
Clinical Palpation of the labial and p alatal m u cosae in the involvem ent of the p rim ary m olars.
1 area to d etect if the uneru p ted 1 is p resent. The u pp er anterior occlusal view show s that root resorp-
Sensibility testing of 1 to assess p u lp al statu s. 1 w as vital tion of BA is ad vanced . 1 has a norm al crow n and root form ,
on sensibility testing to all stim u li, bu t as 1 w as u neru pted , and the root canal ap p ears w id e w ith an ap ical calci c
it w as not p ossible to com p are resp onses to that tooth. Com - brid ge. A tu bercu late su pernum erary overlies the crow n of
parison w ith the 2’s ind icated that 1 had a d im inished 1 . The com posite tip rep air to 1 is visible, and root form a-
response to electric pu lp testing. tion is incom plete w ith apical narrow ing.
•8
2 UNERUPTEDUPPERCENTRALINCISOR
Upper centreline shi t to the right; lower centreline shi t to
the le t.
Buccal segment relationship Class I bilaterally.

■ What is the IOTN DHC grade (see p. 264)? Explain why.


5i – d ue to im ped ed eru ption of 1 cau sed by the presence
of a su p ernu m erary tooth.

Treatment
A ■ What a re your aims o treatment?
• Restore gingival and dental health.
• Relie o crowding.
• Correction o centrelines.
• Alignment o 1. De nitive restoration o 1.
■ What is your treatment plan?
For treatment planning, you shou ld make arrangements for
N eil to be referred for joint consu ltation by an orthod ontist,
oral surgeon and paed iatric d entistry colleague regard ing
management of unerupted 1 , associated su pernu merary, the
prognosis of 1 and the cariou s molars. The plan agreed w as:
1. Oral hygiene instruction.
2. Dietary advice with the aid o a diet diary.
B 3. Provision o a custom-made mouthguard or soccer
Fig. 2.4 (A) Dental panoramic tomogram. (B) Upper anterior practice. The t may need to be modi ed with regard
occlusal radiograph. to some aspects o the treatment given below.
4. Determine the prognosis o 1. The likelihood o apical
closure was deemed to be good, and 1 was to be
monitored radiographically at 3-month intervals.
Application of vertical p arallax to these rad iographs 5. Determine the prognosis o the second primary molars
ind icates that 1 and the sup ernum erary tooth are palatally rom bitewing radiographs. E’s were deemed to be o
p ositioned . reasonable prognosis, but E’s require ormocresol
pulpotomy and stainless steel crowns or extraction in
■ Is there any other a lternative radiographic investigation
view o the pulpal carious involvement. More than hal
you might consider? I so, why?
the root length o E| E remains, and in view o the space
Althou gh a cone beam com puted tom ographic (CBCT) loss that already exists in the lower arch, it would be wise
view m ay be consid ered in the assessm ent of u neru pted to minimize any urther extractions except in an attempt
tooth p osition, and if a su p ernu m erary tooth, or teeth, is to correct the centreline shi t.
present, a risk/ bene t analysis need s to be u nd ertaken 6. Open space or 1 and correct the upper centreline.
on a case-by-case basis. Only w hen conventional rad io- D
grap hic view s fail to provid e suf cient d iagnostic infor- 7. Taking the poor prognosis o into account, and to
D
m ation shou ld CBCT be requested . In this case it w as allow relie o upper arch crowding at this stage, to
reckoned to be u nnecessary based on the view show n in create space or centreline correction and or 1 to
Fig. 2.3. be accommodated, the ollowing extractions are
D C BA C D
indicated .
Diagnosis D
Removal o D is required to balance the extraction o
■ Wha t is your diagnosis? D. Extraction o D will balance the loss o D and tend to
Class I malocclusion on a mild Class II skeletal base with encourage correction o the lower centreline shi t.
slightly increased FMPA. 8. The supernumerary tooth will also need to be surgically
Generalized mild marginal gingivitis. removed and an attachment with a length o gold chain
should be bonded to 1, ollowed by f ap replacement
D
Caries in ; trauma to 1 involving the pulp. (closed technique). 1 should not be surgically exposed
ED E
with an open technique.
Upper and lower arch crowding. 9. In this case it will be necessary to await urther eruption
BA retained; 2 erupting labially; 1 unerupted with associated o 2 ollowing removal o BA be ore moving 2 and 12
tuberculate supernumerary. distally to create space or 1.
9•
UNERUPTEDUPPERCENTRALINCISOR 2
■ How ma y space be created or 1?
Space m ay be created u sing an u pper rem ovable appliance
or by a xed ap p liance.
■ Are there any adva ntages to use o an upper removable
appliance over a f xed a ppliance in this case and at
this stage?
A rem ovable ap p liance has the follow ing ad vantages in this
case at this stage:
There are few p erm anent teeth eru p ted , and m any of the
prim ary teeth are to be extracted ; this leaves very few teeth
on w hich the ap p liance m ay be anchored . Althou gh a xed
ap pliance cou ld be bond ed to the p erm anent incisors and
band s p laced on the rst p erm anent m olars, anchorage
w ou ld need to be reinforced by m eans of linking the rst Fig. 2.5 Upper removable appliance to open space or 1 .
m olars on either sid e of the arch by m eans of a N ance trans-
palatal arch w ith an acrylic bu tton w hich contacts the ante-
rior vau lt of the p alate. There w ill be long sp ans of the
archw ire u nsu p p orted and liable to d istortion. As the base-
plate covers the p alate, anchorage is better w ith a rem ovable
ap pliance for the tooth m ovem ents requ ired .
If d esired , a p rosthetic tooth cou ld also be ad d ed w hen
BA have been rem oved and colou r-m atched to 1; this w ou ld
enhance app liance aesthetics. It cou ld be progressively
trimm ed as 1 is brou ght into alignm ent.
The appliance m ay also be rem oved for cleaning and for
sp orts.
■ What design o upper remova ble a ppliance would you use
to achieve the desired tooth movements? Fig. 2.6 Following extraction o our rst premolars, xed
appliance therapy and urther restorative treatment to 1.
The ap p liance shou ld be d esigned w ith the aid of the
acronym ARAB (activation, retention, anchorage, baseplate)
w hile the p atient is still in the d ental chair, so that nothing
is overlooked . case at this stage. It is likely that fu rther treatm ent, p robably
• Activation: Palatal nger springs (0.5 mm stainless steel loss of a p rem olar u nit from each qu ad rant and xed ap p li-
wire) to 2 12. ance therap y, w ill be required at a later d ate, and then d etail-
• Retention: Adams clasps (0.7 mm stainless steel wire) to ing of 1 p osition can be u nd ertaken (Fig. 2.6).
6 6 . Recurved labial bow (0.7 mm stainless steel wire) ■ Wha t is the recommended root f lling ma terial or 1 during
rom mesial o each E. orthodontic tooth movement?
• Anchorage: From baseplate. When ap ical closu re is evid ent on 1, no root canal treatm ent
• Baseplate: Full palatal acrylic coverage (Fig. 2.5). is requ ired as the tooth has vital p u lp tissu e.
When sp ace for 1 has been created , a hook m ay be sol- ■ Does orthodontic tooth movement pose any risk to 1?
d ered to the labial bow to allow attachm ent of the gold
chain for 1 extru sion or the bow m ay be m od i ed to create There is an increased risk of root resorp tion. N eil and his
a bu ccal arm for this p u rp ose. m other shou ld be w arned abou t this d u ring the inform ed
consent p rocess. 1 m ay also becom e non-vital and requ ire
Key point end od ontic treatm ent.
■ Are there any precautions you would take during
Sequence in design o an upper removable appliance:
orthodontic treatment to minimize this risk?
ARAB acronym
• Activation. As w ith all orthod ontic tooth m ovem ent, excessive forces
• Retention. should be avoid ed . Sensibility testing and a periapical rad i-
ograph shou ld be taken p rior to starting treatm ent and sub-
• Anchorage.
sequ ently, for m onitoring, at 6 m onths into treatm ent. N eil
• Baseplate.
and his m other should be inform ed of this also d uring the
consent p rocess. N eil shou ld be ad vised to w ear the m ou th-
gu ard provid ed d uring contact sports to m inim ize the risk
■ Will an upper removable applia nce achieve a ll the
of repeat trau m a.
treatment objectives?
1 becam e non-vital nearing com pletion of 1 alignm ent
An up per rem ovable ap pliance w ill achieve the simp le and required end od ontic treatm ent w ith gu tta percha. This
tooth m ovem ents (tipp ing and extrusion) requ ired in this w as com pleted uneventfully.
• 10
2 UNERUPTEDUPPERCENTRALINCISOR

■ How would you ensure long-term stability o 1 ollowing Primary resources and
alignment?
recommended reading
Bond ed p alatal retention w ill be requ ired to gu arantee long- Becker A, Brin I, Ben-Bassat Y et al 2002 Closed -eru p tion su rgical
term alignm ent of 1 . techniqu e for im p acted m axillary incisors: a postorthod ontic
Labial gingivoplasty m ay be requ ired at a later stage in p eriod ontal evalu ation. Am J Orthod Dentofacial Orthop
relation to 1 to obtain coincid ence of the gingival m argins 122:9–14.
of 1 1. Flem ing PS, Xavier GM, DiBiase AT et al 2010 Revisiting the
su p ernu m erary: the ep id em iological and m olecu lar basis of
extra teeth. Br Dent J 208:25–30.
Key point Kap ila S, Conley RS, H arrell WE Jr 2011 The cu rrent status of
cone beam com p u ted tom ograp hy im aging in orthod ontics.
Sequence in management o unerupted 1: Dentom axillofac Rad iol 40:24–34.
• Obtain oral surgical/orthodontic opinion (and possibly Kind elan SA, Day PF, Kind elan JD et al 2008 Dental trau m a: an
paediatric dental opinion); i prognosis or 1 alignment overview of its in uence on the m anagem ent of orthod ontic
judged satis actory then, treatm ent. Part 1. J Orthod 35:68–78.
• Open space or unerupted 1 (may involve extractions o Mason C, Azam N , H olt RD et al 2000 A retrosp ective stud y of
primary teeth). u neru p ted m axillary incisors associated w ith su p ernu m erary
teeth. Br J Oral Maxillofac Su rg 38:62–65.
• Remove supernumerary.
Yaqoob O, O’N eill J, Gregg T et al 2010 Managem ent of
• Bond attachment to 1.
Uneru pted Maxillary Incisors. Facu lty of Dental Su rgery of
• Do not surgically expose 1. the Royal College of Su rgeons of England . Available at:
• Align 1 with appropriate appliance. http :/ / w w w.rcseng.ac.u k/ fd s/ p u blications-clinical
• Maintain 1 correction with bonded retainer. -gu id elines/ clinical_gu id elines/ d ocu m ents/
ManMaxIncisors2010p d f.
• Reassess malocclusion regarding urther treatment
needs.
For revision, see Mind Map 2, page 222.
3
Family history
Sarah’s m other also has a sm all space betw een her upp er
front teeth d u e to one m issing tooth (2 ).

Social history
Sarah is a keen clarinet player and is not m otivated to w ear
a xed ap pliance.
■ How will her instrument pla ying impa ct on

Absent upper orthodontic treatment?


Wearing an orthod ontic ap p liance w ill tem p orarily affect

lateral incisors
her m u sical p erform ance, but w ith p ractice and m otivation,
m ost w ind instru m ent players ad ju st. With a w ood w ind
instru m ent su ch as a clarinet, it is likely that Sarah w ill
ad ju st very qu ickly to w earing an orthod ontic appliance
and that p laying w ill retu rn to norm al w ithin a few
w eeks. It w ould , how ever, be ad visable to avoid tting
an ap pliance close to d ates of m u sic exam s, au d itions
or p erform ances. Any orthod ontic ap pliance, p articularly
SUMMARY xed app liances, m ay ru b the insid e of the lips and cheeks
Sa rah, aged 12 years, presents with spacing o her w hen tted , bu t w ax m ay be app lied to m inim ize this. As
upper anterior teeth (Fig. 3.1). What are the possible Sarah is not keen for a xed appliance and provid ed treat-
causes, and how may it be treated? m ent is possible w ith a rem ovable ap p liance, this could be
removed w hile p laying. In ad vance of any orthod ontic treat-
m ent, it w ould be u sefu l for the orthod ontist to ascertain
History how m any hou rs Sarah p ractises per d ay, as leaving the
app liance out for long period s is likely to imp ed e treatm ent
Complaint progress.
Sarah d oes not like the gap s betw een her u p p er front teeth.
She has ju st m oved to a new school and feels self-consciou s
abou t the ap pearance of her teeth.
Examination
History o complaint Extraoral
All p rim ary teeth w ere p resent and w ere lost norm ally. Sarah has a Class I skeletal pattern w ith average FMPA;
When her u p p er p erm anent front teeth eru p ted , there w as there is no facial asym m etry. H er lips are com petent w ith
consid erable sp acing betw een them , and this has not altered the low er lip covering the incisal third of the u p per incisors.
m u ch since then. The p erm anent teeth eru p ted at a normal The tem porom and ibular joints are sym ptom -free.
age, and none have been extracted or avu lsed .

Medical history Intraoral


Sarah is t and w ell.
■ The intraoral views are shown in Figs 3.1 and 3.2. Wha t do
these show?
Dental history
Sarah attend s her general d ental practitioner regu larly but The so t tissues appear healthy and overall oral hygiene
has had no intervention other than p lacem ent of ssu re seems good, although there are small plaque deposits labially
sealants to her rst p erm anent m olars. on the lower incisors. All teeth are o good quality, and no
caries is evident.
The ollowing teeth are present:
7 6 5 4 3 1 1 3 C4 5 6 7
.
76543211234567
There is a retained ragment o E .
There is mild imbrication o the lower incisors; the upper
arch is spaced.
The incisor relationship is Class I with a complete
overbite.
The lower centreline is shi ted slightly to the le t.
The buccal segment relationship is hal unit Class II
Fig. 3.1 Anterior occlusion at presentation. bilaterally.
• 12
3 ABSENTUPPERLATERALINCISORS

A B

C D

Fig. 3.2 (A) Lower occlusal view. (B) Upper occlusal view. (C) Right buccal occlusion. (D) Le t buccal occlusion.

■ Wha t other clinic assessment would you undertake?


Key point
The labial and palatal m u cosae in the 2 area shou ld be pal-
Congenital absence o 2’s is more common in emales.
pated for the p resence of an u neru p ted tooth or any
pathology.
■ Wha t are the possible ca uses o the upper labial
segment spa cing?
Investigations
These are listed in Table 3.1.
■ What urther investigations would you undertake?
■ Wha t is the most likely cause in this case?
Congenital absence of 2 2 is m ost likely. This is m ore Clinical
comm on in fem ales than m ales. The genetic linkage is ind i- • Mobility testing o the retained upper primary canine ( C)
cated by Sarah’s m other, w ho has absence of 2 . is required. Grade 1 mobility was detected.

Radiographic
Table 3.1 Possible causes o the upper labial segment spacing • A dental panoramic tomogram is required to determine
Cause Aetiology the presence/absence o 2’s, 8’s, supernumerary teeth or
Absence o 2 2 Hypodontia (a ects ~2%o Caucasians) – also associated with any pathology.
cle t lip and palate, Down syndrome and ectodermal dysplasia Sarah’s dental panoramic tomogram showed:
Avulsion
• Normal alveolar bone height.
Extraction
• Absence o 2 2 and third molars; short root on C.
Failure o /delayed Crowding
eruption o 2’s Ectopicposition • No pathology associated with any erupted or unerupted
Supernumerarytooth teeth.
Scar tissue Occlusal
Dilaceration • Impressions and a wax registration should be taken or
Cyst/tumour
study models to allow urther assessment o the occlusion.
13 •
ABSENTUPPERLATERALINCISORS 3
■ Wha t genes have been linked to hypodontia ?
Key point
MSX1 and PAX9 have been linked .
Management options with absent 2’s are to
■ How would you ra te the severity o the hypodontia? • Maintain or close 2 space.
Sarah has m issing 2’s and thu s has m ild hyp od ontia (one • Open space or 2 replacement.
or tw o teeth m issing). Mod erate or severe hypod ontia
ind icates three to ve or m ore than six teeth m issing,
respectively. ■ What a ctors would you consider in deciding between space
closure or space opening?
■ Are there other acial/dental/occlusal eatures associated
with hypodontia? Sarah should be seen w ith a restorative colleagu e w ho w ill
provid e inp u t regard ing the restorative im plications of each
Red u ced low er facial height, d elayed d ental d evelop m ent,
treatm ent op tion. Then, it is often w ise to u nd ertake a trial
retained p rimary teeth, sm all teeth and an increased over-
set-u p of the optim al treatm ent option using d u plicate
bite have been associated . Thinning of the hair and absence
stud y m od els and to show this to the patient, to allow a
of palm ar sw eat gland s are featu res of anhid rotic ectod er-
fu ller ap p reciation of the likely treatm ent ou tcom e.
m al d ysp lasia, w hich is associated w ith severe hyp od ontia
The follow ing factors should be consid ered :
(see Chap ter 37).
The patient’s attitude to orthodontic treatment. I the patient is
not keen to wear xed appliances, it may necessitate a
Diagnosis change in treatment plan.
■ What is your diagnosis? The anteroposterior and vertical skeletal relationships. In Class II
cases with an increased overjet, space closure is desirable as it
Class I m alocclu sion on a Class I skeletal base w ith average will eliminate the overjet, whereas in Class III cases this would
FMPA. Well cared for mou th. Mild ly im bricated low er inci- tend to worsen the incisor relationship. Space opening is
sors bu t otherw ise u ncrow d ed low er arch; sp aced u p p er optimal in Class III cases where proclination o the incisors is
arch w ith absent 2 2 and retained C. Bu ccal segm ent rela- likely to correct an anterior crossbite. Where the FMPA is
tionship is half-unit Class II bilaterally. reduced, space opening is pre erable to space closure, and
■ What is the IOTN DHC gra de (see p. 264)? Explain why. the converse is true where an increased FMPA exists.
4h – d ue to absence of 2 2 . The colour, size, shape and angulation o the canine and incisor
teeth. Where the maxillary canine is considerably darker than
the incisors and/or it has a marked canine orm, space closure
Treatment is not advisable as considerable recontouring o 3’s will be
required to enable them to resemble 2’s. Where the canine and
■ What are the trea tment options?
incisor teeth are so angulated that it is possible to reposition
These are: them into their desired locations by tipping movements, a
1. Accept the spacing not a realistic option as Sarah is removable rather than a xed appliance may be used.
concerned by it. Whether the arches are spaced or crowded, and the buccal
2. Build up the mesiodistal width o 1’s and 3’s with segment occlusion. In uncrowded or mildly crowded arches,
composite or by veneering to reduce the spacing, but where the buccal segment occlusion is Class I or at most
not to close it completely. The median diastema is too hal -unit Class II, space opening is best. Space closure is
large or restorative build up o 1 1 to look aesthetic. pre erable where more crowding exists and the buccal
Some recontouring o the cusp tips o 3’s would also be segment relationship is a ull-unit Class II.
required to improve the nal appearance.
In this case it w as d ecid ed to p roceed w ith sp ace op ening
3. Orthodontic space closure. This would require a for rep lacem ent of 2 2 , u ltim ately on resin-retained brid ges.
considerable amount o tooth movement along with C This requ ired an initial phase of d istal m ovem ent of the
extraction, the wearing o a xed appliance and reverse u pper bu ccal segm ents to achieve a Class I m olar relation-
headgear or the placement o TADs may be considered ship, extraction of C, follow ed by retraction of 3’s to a Class
to assist space closure. I relationship w ith 3’s and space op ening for 2’s replace-
4. Orthodontic space opening (this would require m ent. Im p ortantly, overbite red u ction w as also u nd ertaken
extraction o C) or replacement o 2’s on resin-retained in conju nction w ith these tooth m ovem ents to p rovid e sp ace
bridges, by xed bridgework or by implants in late for the m etal fram ew ork of the resin-retained brid ges.
teenage years. Replacement o 2’s by autotransplantation Id eally, a xed ap pliance w ould be ind icated to achieve
o lower premolars (see Chapter 2) is not a viable these objectives, bu t as Sarah w as not keen for this form of
consideration as (i) the lower arch does not warrant treatm ent, an accep table thou gh not op tim al outcom e w as
premolar extractions and (ii) root ormation on lower d eem ed achievable by u p p er rem ovable ap p liance therap y.
premolars is in advance o the ideal stage (two-thirds to
three-quarters complete). ■ Could treatment time ha ve been shortened by an earlier
interceptive measure?
As op tion 2 w ill only partly ad d ress Sarah’s concerns it
has to be ru led ou t. The choice then is betw een the tw o Rem oval of C’s is a u sefu l intercep tive m easu re in the
orthod ontic op tions. early m ixed d entition w hen sp ace closu re is p lanned as it
• 14
3 ABSENTUPPERLATERALINCISORS
encou rages 3’s to eru p t m ore m esially. In this case, how ever, ■ What design o upper removable applia nce would you
space op ening for 2’s rep lacem ent w as p lanned . H ence consider or these tooth movements?
rem oval of C at age 10 is likely to have allow ed 3 to erupt
Palatal nger springs to 31|13 (0.5 mm stainless steel wire).
in a m ore d istal p osition than it is now bu t w ou ld also have
encou raged m esial d rift of the u p p er left buccal segm ent. So Adams clasps 6 6 (0.7 mm stainless steel wire) with
on balance, this extraction at an earlier stage is u nlikely to headgear tubes soldered to the clasp bridges.
have shortened treatm ent tim e. Long labial bow with ‘U’loops (0.7 mm stainless steel wire)
rom 4 to 4 .
Key point Flat anterior biteplane to hal the crown height o 1 1 and
extended 3 mm urther palatally than the maximum overjet
With absent 2’s consider:
measurement. This is an important component o the
• Patient’s attitude to orthodontic treatment.
appliance to ensure that overbite reduction is maintained,
• Skeletal relationships. creating su cient interocclusal clearance or placement o
• Colour, size, shape and angulation o 3 and 1. the metal ramework on the resin-bonded bridges.
• Crowding/spacing.
■ When space has been created or 2 2 , what should
• Buccal segment occlusion.
be done?
The p atient shou ld be seen again w ith a restorative col-
■ How could the upper buccal segments be moved dista lly leagu e to ensure that the tooth m ovements achieved w ill
using a removable appliance to achieve a Class I molar allow restorative treatm ent to proceed as planned . Then a
relationship? removable retainer shou ld be tted for 6 m onths carrying
replacem ent 2 2 and ensu ring that space for them is main-
An u pper rem ovable ap pliance w ith bilateral screw s to
tained by p lacing w ire sp u rs in contact w ith the ad joining
m ove 6 5 4 and 4 5 6 d istally is an op tion. Anchorage need s
teeth (Fig. 3.3).
to be reinforced by allow ing provision for head gear to
be attached to the ap pliance. The app liance shou ld also
incorp orate: Key point
• Adams clasps (0.7 mm stainless steel wire) with headgear
tubes soldered to 6’s clasp bridges; Adams clasps (0.7 • Always place wire spurs on the removable retainer, to
mm stainless steel wire) to 4’s also. the teeth adjoining the 2 space a ter space opening.
• Short labial bow 3 to 3 (0.7 mm stainless steel wire). • Retain or 6 months be ore replacing 2 on resin-retained
bridge (underline 2).
• Flat anterior biteplane to hal the crown height o 1 1
and extended 3 mm urther palatally than the maximum
overjet measurement.
When there is evid ence of fu ll-tim e ap p liance w ear, head - ■ What design o resin-retained bridge is required?
gear should be tted for anchorage w ith an up w ard d irec-
tion of p ull to prevent the appliance becom ing d islod ged Maintenance of closu re of the m ed ian d iastem a requ ires
w hen the head gear is being w orn. perm anent retention. A bond ed p alatal retainer fram ew ork
linking 1 1 together is ind icated along w ith resin-retained
■ Wha t orce and dura tion o headgear wear is required or
brid ges w ith single w ing, off 3 3 . It is better that 1 1 are
anchorage?
retained as a sep arate u nit rather than risk the retention
A force of 250–350g p er sid e for 8–10 hou rs p er d ay is integrity and su ccess of the brid ges by incorporating 1 1
requ ired . retention in the brid ge d esign.
■ Wha t precautions must be adhered to when Im p lant rep lacem ent of 2 2 later is u nlikely as the roots
prescribing hea dgea r? of 31 13 are tipp ed tow ard the 2 2 sp ace, com p rom ising
access for im plant p ositioning. The nal resu lt w ith 2 2
Tw o safety m echanism s m u st be tted to the head gear
replaced on ad hesive brid gew ork is show n in Fig. 3.4.
assem bly, p referably a safety release sp ring m echanism
attached to the head cap and a facebow w ith a locking
d evice. Verbal and w ritten safety instru ctions m ust be
issued to both p atient and p arent(s)/ gu ard ian. The head -
gear should be checked at each visit.
When com p liance w ith head gear w ear is evid ent, then
Sarah shou ld be instru cted to turn each screw once per
w eek. C shou ld be extracted and acrylic relieved to allow
for p otential d istal d rift of 3 as the bu ccal segm ents are
retracted to Class I. Som e over-retraction is ad visable to
allow for any slight anchorage slip d u ring the next p hase of
treatm ent w hen 3’s w ill be retracted to a Class I relationship
w ith 3’s ; 1’s w ill be ap p roxim ated and overbite red u ction
w ill be m aintained . Fig. 3.3 Upper removable appliance retainer with replacement 2’s.
15 •
ABSENTUPPERLATERALINCISORS 3
Carter N E, Gillgrass TJ, H obson RS et al 2003 The
interd iscip linary m anagem ent of hyp od ontia: orthod ontics.
Br Dent J 194:361–366.
H arrison JE, Bow d en DE 1992 The orthod ontic/ restorative
interface. Restorative p roced ures to aid orthod ontic treatm ent.
Br J Orthod 19:143–152.
Khalaf K, Miskelly J, Voge E, et al 2014 Prevalence of hypod ontia
and associated factors: a system atic review and m eta-analysis.
J Orthod 41:299–316.
Kohich KO Jr, Kinzer GA, Janakievski J, 2011 Congenitally
m issing m axillary lateral incisors: restorative rep lacem ent. Am J
Fig. 3.4 Final restorations. Orthod Dentofac Orthop 139:435–445.
Mossey PA 1999 The heritability of m alocclu sion: p art 2. The
in u ence of genetics in m alocclu sion. Br J Orthod 26:195–203.
■ What additional actors would need to have been
considered i replacement o 2’s by implants was the Qad ri S, Parkin N A, Benson PE 2016 Sp ace closing versu s sp ace
opening for bilateral m issing up per laterals - aesthetic
pre erred pla n?
ju d gm ents of layp eop le: a w eb-based su rvey. J Orthod 43
It w ould be necessary to w ait u ntil facial grow th has red u ced (2):137–146.
to ad u lt levels. In a girl, grow th of the m axilla is com p lete Robertsson S, Mohlin B 2000 The congenitally m issing u p per
by abou t 15 years of age and m and ibu lar grow th u su ally 2 lateral incisor. A retrosp ective stu d y of orthod ontic space
years later, w hereas in boys m axillary grow th u su ally con- closu re versu s restorative treatm ent. Eu r J Orthod 22:697–710.
tinues to about 17 years of age and m and ibu lar grow th Silveira GS, De Alm eid a N V, Pereira DMT et al 2016 Prosthetic
until 19 years or later. This is esp ecially im p ortant as an rep lacem ent vs sp ace closu re for m axillary lateral incisor
agenesis: A system atic review. Am J Orthod 150, 228–237.
im plant has no eruptive potential and w ill be left behind if
erup tion of ad jacent teeth is still continu ing, com p rom ising Zachrisson BU, Rosa M, Toreskog S 2011 Congenitally m issing
m axillary lateral incisors: canine su bstitu tion. Am J Orthod
aesthetics.
Dentofac Orthop 139:434–444.
Asid e from su f cient d istance betw een the roots of the
ad jacent teeth, sp ace m u st also exist betw een the tooth
crow ns w ith su f cient interocclu sal clearance. Ad d itionally, For revision, see Mind Map 3, page 223.
am p le bu ccopalatal w id th and height of the alveolar bone
is requ ired in the 2 area.

Primary resources and


recommended reading
British Orthod ontic Society 2007 Ad vice for m u sicians.
Available at: w w w.bos.org.uk/ orthod onticsand you /
orthod onticsforschools/ ad viceform u sicians.htm .
4
Medical history
Gem m a has had asthm a since she w as 5 years of age and
uses a salbu tam ol (Ventolin) inhaler; otherw ise she is t
and w ell.
■ What a re the implications or orthodontic trea tment
with a sthma?
These are sum m arized in Ap pend ix 4.

Crowding and buccal Dental history


Gem m a has attend ed for rou tine d ental exam inations since

upper canines
she w as 3 years old bu t has not u nd ergone any active d ental
treatm ent.

Examination
Extraoral
CASE1 Gem m a has a Class I skeletal p attern w ith average FMPA.
There ap p ears to be slight facial asym m etry w ith the chin
point d eviated m ild ly to the right. The lips are com petent.
SUMMARY N o temp orom and ibu lar signs or sym p tom s w ere d etected
Gemma, an 11-year-old girl, attends or a 6-month or rep orted .
dental assessment at your practice with both upper Gem m a and her m other w ere u naw are of Gem m a’s
slight facial asym m etry and have noticed no change in her
permanent canines erupting buccally (Fig. 4.1).
facial ap p earance over recent years.
What is the cause, and how may it be treated?
■ Would you be concerned by the mild acial asymmetry?

History A mild d egree of facial asym m etry is norm al, and as facial
appearance is rep orted ly u naltered for several years, there
Complaint is no cau se for concern.
Gem m a d oes not like the ‘squint’ ap pearance of her top and
bottom teeth, in p articular the p osition of the u p p er eye Intraoral
teeth, w hich she says ‘look like fangs’. ■ Gemma’s intraoral views are shown in Figs 4.1 and 4.2.
What do you notice?
History o complaint Generalized marginal gingival erythema.
The crooked ness of Gem m a’s teeth has been getting w orse
Plaque deposits visible on several teeth, notably both 3’s.
for the p ast year. The ap p earance of her u p p er teeth has
becom e of m ore concern to her in recent m onths w hen both There are no restorations, and there is no obvious caries.
u pper eye teeth started to eru p t. She is now teased at school Gemma is in the late mixed dentition stage with the
and called ‘Fangs’, w hich annoys her. 6 5 4 C3 2 1 1 2 3 4 5 6
Gem m a’s m other rep orts that her d au ghter ’s baby teeth ollowing teeth present: (note 5
7 6 5 4 3 2 1 1 2 3 4 E6 7
w ere also slightly crooked . Both she and Gem m a are very and 7 7 are partially erupted).
keen for treatm ent.
The lower labial segment is moderately crowded with 2 | 2
bodily displaced lingually and 1| 1 slightly mesiolabially rotated.
3 is distally angulated; 3 is mesially angulated.
The lower right buccal segment is also crowded with
insu cient space or 5 ; the lower le t buccal segment is
uncrowded with E present.
The upper labial segment is moderately crowded with 1 1
slightly mesiolabially rotated and 3 3 erupting buccally; C is
present.
3 is upright and 3 is slightly distally angulated.
The upper buccal segments are aligned.
In occlusion, there is a Class I incisor relationship.
The overbite is average and complete.
Fig. 4.1 Anterior occlusion at presentation. The lower centreline is slightly to the right.
17 •
CROWDINGANDBUCCALUPPERCANINES 4

Fig. 4.3 Dental panoramic tomogram.

• Retention o the primary canine this usually leads to


slight buccal displacement o 3.

Key point
Buccal displacement o 3 is more usual in a crowded arch.

Investigations
B ■ Wha t investigations would you request and why?
A d ental panoram ic tom ogram is requ ired to provid e a
general view of the d evelop ing d entition and to con rm the
p resence and position of all u neru pted perm anent teeth.
■ Gemma’s dental panoramic tomogram is shown in
Fig. 4.3. What do you notice?
Alveolar bone level is normal.
Presence o a ull complement o developing permanent
teeth, including third molars.
C All teeth appear caries- ree.

Diagnosis
■ Wha t is your diagnosis?
Class I malocclusion on a Class I skeletal base with average
FMPA, with the chin point displaced slightly to the right.
Generalized marginal gingivitis.
Moderate upper and lower arch crowding with the lower
centreline shi ted slightly to the right.
Right molar relationship is Class III; le t molar relationship is
D
Class I.
Fig. 4.2 (A) Lower occlusal view. (B) Upper occlusal view. (C) Right
■ What is the IOTN DHC grade and why (see p. 264)?
buccal occlusion. (D) Le t buccal occlusion.
Expla in why.
4d – d ue to severe d isplacements of teeth, greater than 4 mm.
The right molar relationship is Class III, and the le t molar
relationship is Class I.
Treatment
■ What are the possible reasons or 3’s erupting buccally?
■ Wha t treatment is likely to be required in this case?
• Crowding buccal displacement o 3’s is o ten a
Expla in why.
mani estation o inherent crowding in the upper arch. A
contributory actor is 3 being the last tooth to erupt Extractions are requ ired to relieve the m od erate crow d ing.
anterior to the rst permanent molars. Fixed app liance therap y is ind icated in view of the d istal
• 18
4 CROWDINGANDBUCCALUPPERCANINES
angu lation of m ost canines, the rotations of the central inci- As Gem m a has m od erate low er labial segm ent crow d ing,
sors, the bod ily lingu al d isplacem ent of 2’s and the cen- sp ace w ill be requ ired to achieve alignm ent.
treline shift. ■ What possible means are there o creating space?
■ Wha t would you do now?
Extractions.
Explain to the patient the likely plan or correction o her Arch expansion (laterally and/or anteroposteriorly).
malocclusion.
Distal movement o the molars.
Arrange or several visits o oral hygiene instruction by the
Enamel stripping.
practice hygienist, and assuming that oral hygiene improves
satis actorily, take upper and lower impressions and a wax Any combination o the above.
registration or study models. Expansion of the low er intercanine w id th is unstable as
Arrange re erral to an orthodontist and enclose the study is forw ard m ovem ent of the low er labial segm ent (w ith a
models and dental panoramic tomogram. few excep tions w hich w ill be d ealt w ith elsew here); d istal
m ovem ent of the low er rst p erm anent m olars is d if cu lt
Write a re erral letter to the orthodontist (Fig. 4.4).
w ithout extraction of low er second perm anent m olars and
■ Wha t aims o treatment do you think will be proposed by is und ertaken rarely. Enam el stripp ing is u sually only con-
the orthodontist? sid ered in ad u lts to gain 1–2 m m of space in total. In view
of these consid erations, extractions are the only realistic
Relie o crowding.
op tion of gaining sp ace in Gem m a’s case.
Upper and lower arch alignment.
Correction o lower centreline.
Correction o right molar relationship.
Key point
Closure o any residual spacing. Always consider the lower arch f rst in treatment planning.

■ Describe how you would approach treatment planning.


1. Consider the lower arch f rst and plan the lower labial ■ What a ctors govern the choice o extraction?
segment. As the latter is in a narrow zone o so t tissue
balance between the lips and the tongue, it is best to Prognosis o teeth.
consider this sacrosanct. First the alignment o the labial Site o crowding.
segment must be assessed, and i it is crowded, as in
Degree o crowding.
Gemma’s case, the degree o crowding must be assessed
to ascertain i this is su cient to warrant extractions. Individual tooth position, e.g. grossly displaced or ectopic
teeth.
In this case there are no low er teeth of p oor p rognosis,
and in view of the site and d egree of crow d ing, the low er
rst p rem olars w ou ld be the teeth of choice for extraction.
Practice address
■ Why are f rst premolars commonly chosen or extraction?
Date
They are in the middle o the arch and, there ore, provide
space or relie o moderate labial and buccal segment
Dear crowding.
Re [patient’s name, address, date of birth] The contact point between the canine and second premolar
I would be grateful if you could see Gemma for orthodontic assessment is as good as between the canine and the rst premolar.
and treatment.
I a canine is mesially angulated, considerable scope exists or
Gemma is very concerned about the crowding of her teeth. Apart from
using a salbutamol (Ventolin) inhaler for asthma, she is in good health. spontaneous alignment o the labial segment as the canine
uprights into the extraction space. For maximum
Gemma’s oral hygiene is improving following several visits to our hygenist.
She has a caries-free dentition. She is very keen for treatment and is spontaneous improvement, it is best to extract the rst
prepared to wear fixed appliances.
premolars as the permanent canines are erupting.
She has a Class I malocclusion on a Class I skeletal base with average
FMPA and the chin point slightly to the right. The upper and lower arches Any residual space is not at the ront o the mouth and is
are moderately crowded and the lower centreline is slightly to the right.
The right molar relationship is Class III; the left molar relationship is Class I. likely to close urther with mesial dri t o the buccal segments.

I enclose current study models and a recent dental panoramic tomogram.


2. Imagine the corrected position o 3. 3 is mesially
angulated and will upright spontaneously ollowing
Yours sincerely removal o 4, thereby providing space or labial
segment alignment; 3 , however, is distally angulated
and will require bodily retraction with a xed appliance.
3. Mentally reposition 3 to be in a Class I relationship with the
corrected position o 3. Space is required in Gemma’s
case or this. Extraction o both upper rst premolars
Fig. 4.4 Example o a re erral letter. should provide adequate space or retraction o 3′s. As
19 •
CROWDINGANDBUCCALUPPERCANINES 4
3 is upright and 3 is distally angulated, xed appliance
therapy is indicated to e ect this movement.
4. Plan the upper labial segment. The incisors are mildly
crowded and slightly rotated, so xed appliance therapy
is required to produce ideal alignment.
5. Decide on the f nal molar relationship. As upper and
lower rst premolar extractions are planned, the nal
molar relationship should be Class I. Closure o residual
buccal segment spacing ollowing the extractions will
require xed appliance therapy.
6. Assess the anchorage needs. As almost all o the upper A
rst premolar extraction spaces will be required or
relie o upper arch crowding, and retraction o the
upright/distally angulated 3′s is needed, anchorage
would be best rein orced with a palatal arch attached
to bands on 6′s or temporary anchorage devices (TADs).
7. Plan retention. The prognosis is avourable, but bonded
retention to the lower labial segment would be wise in
view o the bodily lingual displacement o 2 | 2 . Upper
and lower vacuum- ormed retainers should also be
provided to be worn at night or the rst 12 months
ollowing debond; the lower is designed to t over the B
bonded retainer. Therea ter, retention should continue
on a night-only basis, reducing to alternate nights or Fig. 4.5 (A) Post-treatment: right buccal occlusion.
another year and then to twice weekly. Gemma’s (B) Post-treatment: anterior occlusion.
responsibility with regard to retention should be
explained be ore treatment starts.
■ Gemma’s f na l occlusion is shown in Fig. 4.5. What
undesirable sequelae o treatment are visible?
Key point
Several teeth are affected by w hite spot lesions or d em iner-
• Always plan anchorage at the treatment planning stage.
alization, ind icating early cariou s involvem ent.
• The amount o space and type o intended tooth
■ How common is this with f xed appliance therapy and
movement in uence anchorage demands.
which teeth are a ected mostly?
• Always consider retention in the treatment plan.
The reported prevalence is betw een 2% and 96%. Upp er
lateral incisors and low er canines are affected m ost
comm only.
■ What is the f nal orthodontic treatment plan likely to be?
■ How may the problem be prevented or minimized?
N o ap p liance therap y w ou ld be consid ered u ntil Gem m a
has d em onstrated that she is cap able of m aintaining a Care ul patient selection; ensure a high standard o oral
high stand ard of oral hygiene. Then the orthod ontic plan hygiene pre-treatment.
w ou ld be: Advise the patient that zzy drinks and sugary oods should
8. Fit palatal arch or anchorage or place TADs (see not be consumed between meals.
Chapter 5). The teeth should be brushed with a f uoridated denti rice
9. Extraction o our rst premolars (provided there is a ter each meal.
satis actory cooperation with wear o the palatal arch).
Regular surveillance o oral hygiene and oral hygiene
TADs could be placed, instead o the palatal arch, at the
instruction should be undertaken by a hygienist throughout
time o the upper arch extractions.
treatment.
10. Upper and lower xed appliance therapy.
Daily use o a f uoride mouthrinse (0.05% sodium f uoride) is
11. Lower canine to canine bonded retainer with upper
recommended during treatment.
and lower vacuum- ormed retainers.
Moderate quality evidence exists that application o a f uoride
■ What risks should the patient be wa rned o regarding f xed
(0.1% f uoride) varnish around the brackets on a 6-weekly
appliance orthodontic treatment?
basis is e ective.
The p atient shou ld be w arned of the risk of:
■ How may these ‘white spots’ be managed?
Enamel demineralization.
Usually, follow ing rem oval of the ap pliances, the w hite
Root resorption.
spots regress slightly as m aintenance of an im p roved stand -
Loss o tooth vitality. ard of oral hygiene is facilitated . App lication of high-
Relapse. concentration u orid e varnish is inad visable at this stage as
• 20
4 CROWDINGANDBUCCALUPPERCANINES
it lead s to hyp erm ineralization of the lesions, w hich m akes
them m ore obviou s.
Where the w hite sp ot lesions are extensive and p ose an
obviou s aesthetic insu lt, acid -p u m ice abrasion w ith 0.2%
hyd ro u oric acid m ay be carried ou t. In rare severe cases,
veneers or com p osite restorations are likely to be required .

Key point
Demineralization with f xed appliances:
• Is common (2 96% prevalence).
• Mostly a ects 2′s and 3 ’s.
• Is best prevented by care ul patient selection and
dietary advice.

CASE2 B

SUMMARY Fig. 4.6 (A) Dental panoramic tomogram. (B) Upper anterior
Aoi e, a 12-year-old girl, presented with crowded occlusal radiograph.
upper teeth. Her mother reported a possible
nickel allergy.
■ How would you deal with the history o a possible The u p p er anterior occlu sal rad iograp h reveals:
nickel a llergy?
Tapered roots o the upper incisors.
Aoife shou ld be referred to a d ermatologist for assessm ent 3 to be in contact with 2 root which is distally angulated.
and patch testing.
Fortu nately, no allergy to nickel w as rep orted . ■ Where is 3 located? Expla in how this assessment is made
and your reasoning.
■ How common is nickel allergy?
3 is buccally p laced . This assessm ent is m ad e by com paring
N ickel allergy is m ore comm on in fem ales (10–30%) than
the p osition of 3 on the panoram ic lm and the occlusal lm
m ales (~2%), p ossibly d u e to increased contact from nickel-
using vertical parallax. The x-ray tube has m oved u p
containing jew ellery.
betw een the panoram ic (taken at −10° to the occlu sal plane)
■ What implica tions does a nickel allergy ha ve or and the occlu sal (taken at 65–70° to the occlu sal p lane). This
orthodontic mana gement? w ou ld be a 75–80° tu be shift. The position of 3 relative to
These are su m m arized in Append ix 4. the ad jacent 2 is then assessed . An object closer to the x-ray
beam (bu ccally p laced ) w ill seem to m ove in the op posite
d irection as the tube shift. This is evid ent for 3.
■ What do you notice in Fig. 4.7?
Key point
Right bu ccal occlusion show ing:
A suspected nickel allergy:
• Should be assessed by a dermatologist. Mild marginal gingival erythema with apparent bulge over 2 .
• Is more common in emales. No caries.
• May require use o nickel- ree appliance components. Crowded lower arch with mesially angulated 3 (lower arch
was overall moderately crowded).
Spacing between 4 and 2 but insu cient or 3 ; 2 distally
angulated and mesiopalatally rotated.
■ Wha t do you notice on the radiographs shown in Fig. 4.6?
Class 1 incisor and molar relationship; average overjet; overbite
The d ental p anoram ic tom ogram show s: average (1 covers hal lower incisor crown) and complete.
Normal alveolar bone height.
7 6 5 4 C2 1 1 2 3 4 5 6 7
■ What a re the aims o treatment?
erupted.
76543211234567 The aim s of treatm ent are to:
Four third molars developing. Improve oral hygiene.

All permanent teeth o good quality; some root shortening o Relieve upper and lower arch crowding with alignment o 3 .
C but o reasonable length. Closure o residual spaces.
21 •
CROWDINGANDBUCCALUPPERCANINES 4

Fig. 4.7 Right buccal occlusion at presentation. A

Fig. 4.9 (A) Post-treatment: upper occlusal view.


(B) Post-treatment: anterior occlusion.

Fig. 4.8 Upper occlusal view ollowing extraction 4 4 and closed


surgical exposure o 3 .
CASE3
SUMMARY
Triona, a 13-year-old girl, presents with 3 and 3
Fou r rst prem olars w ere extracted follow ed by xed erupting buccally. How will you manage the
ap p liance m echanics. problem?
3 remained unerupted 9 months a ter extraction o 4 . How ■ Wha t do you notice in Fig. 4.10?
would you manage 3 ?
Mild generalized marginal gingiva erythema; caries- ree
It w ou ld be ad visable to assess 3 w ith an oral su rgeon
dentition with ssure sealants in all rst permanent molars.
and d iscu ss surgical exp osu re.
■ What types o surgical exposure are there or a buccally Mild lower arch crowding; severe upper arch crowding
positioned ca nine? with 3 and 3 erupting buccally. (Cusp tip o 3 just visible.)
Lower incisors appear slightly retroclined and upper incisors
Open or closed exp osu re are the p ossible su rgical options.
appear upright to retroclined.
With op en exp osu re, an ap ically rep ositioned ap w ou ld be
requ ired . With closed exp osu re, a buccal ap is raised and Class III incisor relationship; with 2 1 1 in crossbite (reverse
an attachm ent w ith gold chain bond ed to 3 to facilitate trac- overjet was 1 mm on 1 ); minimal overbite (~1 mm measured
tion, follow ed by replacem ent of the ap . Du e to operator clinically on 1 ).
preference, a closed exp osu re w as agreed u p on (Fig. 4.8). Centreline shi t (upper was 3 mm to the le t clinically).
■ How may 3 be aligned? Right molar relationship slightly Class II; le t molar relationship
Initially, 3 m ay be aligned partially by attaching the gold slightly Class III with 5 erupting potentially in crossbite.
chain to the base archw ire; a bracket w ill then need to be
■ Is it possible to ma ke a reliable assessment o the incisor
bond ed to 3 and a light (typ ically 0.012-in or 0.014-in nickel
inclination to the underlying denta l bases rom intraoral
titanium ) archw ire ‘piggy-backed ’ to the base archw ire
images o the dentition in occlusion?
to im prove alignm ent until 3 can be fu lly engaged in a
0.019 × 0.025-in stainless steel w ire (assu m ing an 0.022 × It is not possible to tell the u pper and low er incisor inclina-
0.028-in slot). tion reliably from intraoral clinical im ages as there are
The nal occlusion is show n in Fig. 4.9. no reference p lanes visible (Frankfort or m axillary and
• 22
4 CROWDINGANDBUCCALUPPERCANINES

B
A

Fig. 4.10 (A) Right buccal occlusion. (B) Anterior occlusion. (C) Le t buccal occlusion. (D) Upper occlusal. (E) Lower occlusal.

m and ibu lar p lanes resp ectively). Only in the broad est sense rad iograp hs of 2 to ensu re that the root statu s rem ains
m ay com m ent be m ad e regard ing how the inclination of the unchanged .
incisors ‘ap p ears’. This w as found to be the case.
■ Wha t clinical assessment should you underta ke o 2? A lateral cep halom etric rad iograph is required to
provid e m ore inform ation abou t the severity of the Class
2 should be checked for m obility in case it is being resorbed III m alocclu sion and the extent of any d entoalveolar
by 3. If this is su spected , sensibility tests shou ld also be com p ensation.
u nd ertaken.
■ What is your interpretation o the ollowing
cephalometric f ndings?
Investigations SN A = 81°; SN B = 79.5°; SN -m axillary p lane = 8°; 1 to m axil-
lary p lane = 105°; 1 to m and ibu lar p lane = 90°; m axillary
■ The patient presented with a denta l panoramic radiograph
m and ibu lar p lanes angle (MMPA) = 28°; facial prop ortion
taken 6 months previously by her general dental
= 57%.
pra ctitioner. What radiographic investiga tions would you
The skeletal p attern is m ild ly Class III (AN B = 1.5°; SN A-
request and why?
SN B) d u e to very slight m and ibu lar p rognathism. Com -
It is not necessary to rep eat the p anoram ic lm . This revealed pared w ith m ean valu es, the u p per and low er incisors are
all perm anent teeth to be d evelop ing norm ally and to be of somew hat retroclined , bu t w ithin the norm al range, and the
good qu ality. The root statu s of 2 also app eared to be sou nd 1 angle is not com p ensating for the MMPA (shou ld be
w ith no evid ence of root resorp tion from 3. 120−28° = 92°). Both the MMPA and facial proportion are
As 6 m onths has elap sed since the last rad iograp h w as slightly increased from norm al values, but both are w ithin
taken of 2, it w ould be sensible to take tw o p eriap ical the norm al range.
23 •
CROWDINGANDBUCCALUPPERCANINES 4
• Arch expansion and extractions expansion will
Diagnosis reduce the need to extract 4 4 , so 5 5 may be
considered.
■ What is your orthodontic diagnosis?
• Arch expansion and distal movement o the le t buccal
Class III malocclusion on a mild Class III dental base with segment the ormer is as previously outlined; the latter
slightly increased acial proportions. may be achieved by placing a TAD between and pushing
Mild marginal gingivitis. Mild lower arch and severe upper against it with coil spring on a xed appliance to move
arch crowding with 3 excluded buccally. 4 5 6 distally.
Upper centreline shi t o 2 mm to the le t. Triona’s m other w as keen to avoid extractions, if p ossi-
ble. Follow ing consid eration of all op tions, it w as agreed to
Right molar relationship is slightly Class II, and le t molar
p roceed w ith an initial non-extraction ‘therap eutic d iagno-
relationship is slightly Class III.
sis’ ap proach (Fig. 4.11).
■ What is the IOTN DHC grade? Expla in your reasoning. ■ What a ppliance is shown in Fig. 4.11A? How is it constructed
4d – d u e to severe d isp lacem ent of teeth greater than 4 m m and how is it activated?
(betw een 2’s and 3’s bilaterally). This is a qu ad helix ap p liance and has been cu stom -m ad e in
the laboratory; p reform ed types, how ever, also exist. The
qu ad helix is attached to band s cem ented to a m olar on each
Treatment sid e of the arch. For a cu stom -m ad e app liance, an im pres-
■ What are your aims o treatment? sion m u st be taken w ith the band s in p lace, then the band s
are rem oved and located carefu lly in the im pression before
Improve oral hygiene. being d isinfected and sent to the laboratory. For the p re-
Relie o upper and lower arch crowding. formed typ es, attachm ents are w eld ed to the palatal sur-
Establish a positive overjet and overbite. faces of the band s into w hich slot the arm s of the qu ad helix;
this facilitates rem oval for ad ju stm ents. Both ap pliance
Correction o the upper centreline.
types are m ad e of 1 m m stainless steel w ire. Typ ically, acti-
Correction o the molar relationships to Class I. vation is achieved by ‘expand ing’ the app liance around half
Retain the correction. a tooth w id th on each sid e. Although activation m ay be
achieved intraorally w ith triple-beak p liers, it is d if cu lt to
Monitor third molars.
gauge the am ou nt of activation by that m ean; it is u su ally
■ What possible options are there to relieve the upper arch p referable to rem ove the appliance every second visit and
crowding and correct the upper centreline? activate it extraorally.

Sp ace is required in ord er to ad d ress these treatm ent aims. ■ Wha t is the most likely initial a ligning archwire shown
Possible m eans to provid e this are: in Fig. 4.11C? What a vourable properties ha s it got or
• Extractions removal o a premolar on either side o alignment?
the upper arch. Removal o 5 5 is usually considered This is most likely a nickle–titaniu m archw ire of sm all d iam -
in Class III camouf age, but anchorage would need to eter, probably 0.012-in. Its p roperties inclu d e exibility
be rein orced (with either a palatal arch or TADs) i w ithout und ergoing d eform ation, and it exerts a light force
their removal was considered. Removal o 4 4 , situated w hen tied into d isp laced teeth.
right beside where space is needed, would acilitate
alignment o 3 3, and although not all the 4 space ■ How would you re-evaluate treatment progress ollowing
is required or relie o crowding in the upper right upper arch expa nsion a nd a lignment?
quadrant, the residual space will be utilized or Interim record s should be taken for further treatm ent
upper centreline correction. With both extraction planning. These are clinical photographs (extraoral and
options, the upper arch would be aligned inside intraoral view s), stu d y m od els, and a lateral cep halom etric
the lower. rad iograp h.
• Arch expansion both anteroposterior and lateral The lateral cephalom etric m easurem ents shou ld be com -
expansion o the upper arch would correct the anterior pared w ith those taken at the start of treatm ent. Any change
crossbites as well as the tendency to buccal crossbite. It in SN B, AN B and MMPA shou ld be noted . The incisor incli-
will also provide urther space or 3, which may erupt. nations shou ld also be assessed relative to the m axillary and
Care is required as arch expansion may cause the palatal m and ibular planes resp ectively. 1 inclination w ill ind icate
cusps o the maxillary molars to tip buccally, which will how m u ch their inclination has changed d u ring u p p er arch
compromise the already tenuous overbite. A potential alignm ent, and 1 inclination w hen consid ered w ith the AN B
advantage o this approach is that extractions are and MMPA angles w ill ind icate if any d entoalveolar com -
delayed while urther mandibular growth is observed p ensation has occu rred since treatm ent started and w hether
when it will become more apparent whether fu rther orthod ontic com p ensation is feasible or d esirable. If
orthodontic correction is possible or not. Such a there has been m inim al/ no change in SN B, AN B and MMPA
‘therapeutic diagnosis’approach to treatment is o ten then the p rospect of orthod ontic correction is good . Other-
advisable in a Class III malocclusion where the extent o w ise any attem p t at orthod ontic cam ou age should be con-
urther mandibular growth is unknown. sid ered w ith cau tion d u e to the likely guard ed prognosis;
• 24
4 CROWDINGANDBUCCALUPPERCANINES

A A

Fig. 4.11 (A) Mid-treatment: upper occlusal view; note eruption o C


3 . (B) Mid-treatment: initial aligning archwire. (C) Mid-treatment:
Fig. 4.12 (A) Post-treatment: right buccal occlusion. (B) Post-
upper arch alignment.
treatment: anterior occlusion. (C) Post-treatment: upper occlusal
view.

Primary resources and


in su ch circu m stances, it w ou ld be w ise to d elay any fu rther recommended reading
intervention and review at a later stage w hen grow th is Benson PE, Parkin N , Dyer F et al 2013 Flu orid es for the p revention
com p lete. of early tooth d ecay (d em ineralised w hite lesions) d u ring xed
brace treatm ent. Cochrane Database of Syst Rev Issu e 12. Art N o:
■ When is mandibular growth completed? CD003809. DOI: 10.1002/ 14651858.CD003809.p u b3.
Mand ibu lar grow th is u su ally com p lete by abou t 17 years H afez H S, Shaaraw y SM, Al-Sakiti AA et al 2012 Dental crow d ing
of age in girls and abou t 2 years later in boys. as a caries risk factor: a system atic review. Am J Orthod .
As reassessm ent revealed that grow th had not been Dentofacial Orthop 142:443–450.
ad verse, it w as d ecid ed to p roceed w ith low er xed appli- Little RM, Wallen TR, Reid el RA 1981 Stability and relapse of
m and ibu lar anterior alignm ent- rst p rem olar extraction cases
ance therap y and orthod ontic cam ou age on a non-
treated by trad itional ed gew ise orthod ontics. Am J Orthod
extraction basis. The nal occlusion is show n in Fig. 4.12.
80:349–365.
■ What will determine whether the correction remains sta ble? Mitchell L 1992 Decalci cation d u ring orthod ontic treatm ent w ith
xed ap p liances – an overview. Br J Orthod 19:199–205.
The am ou nt of overbite and the buccal segm ent interd igita-
Rahilly G, Price N 2003 N ickel allergy and orthod ontics. J Orthod
tion w ill d eterm ine the stability of the anterior and p osterior
30:171–174.
crossbites resp ectively, bu t both w ill also be greatly in u -
Stephens CD 1989 The use of natural spontaneous tooth movement
enced by the extent and d irection of fu rther m and ibu lar
in the treatment of malocclusion. Dent Update 16:337–338, 340–342.
grow th. The latter w ill have the nal say w ith regard to
long-term stability. For revision, see Mind Map 4, page 224.
5
She fractured her right w rist in a fall from her m ountain
bike 4 m onths ago and has been attend ing for physiotherap y
at the local hosp ital since the cast w as rem oved . Mobility
is alm ost back to norm al now, bu t she has d if culty w ith
som e proced ures, su ch as toothbru shing. Otherw ise she is
t and w ell.
■ What implications does the medical history ha ve or any
proposed orthodontic treatment?

Severe crowding Amy’s card iologist should be consulted regard ing the
card iac statu s and the p ossible need for antibiotic p rop hy-
laxis for p roced u res likely to p rod u ce bacteraem ia becau se
of the potential risk of infective end ocard itis. The N ational
Institute for H ealth and Clinical Excellence (N ICE, w hich
governs clinical practice in England and Wales) guid elines
(2016) d o not recom m end rou tine antibiotic p rophylaxis for
those und ergoing d ental p roced ures; the American H eart
Association (AH A) recom m end s antibiotic cover only for
those at high risk.
CASE1 Am y’s card iologist con rm ed that her card iac m u rm u r
had fu lly resolved and that antibiotic p rop hylaxis w as not
requ ired prior to any d ental or orthod ontic (separator p lace-
SUMMARY
m ent, tting/ removal of band s) proced u res.
Amy, an almost 11-year-old girl, presents with As excellent oral hygiene is essential w ith any orthod on-
marked space shortage or both unerupted 3′s tic treatm ent, the im p act of lack of optim al w rist m obility
(Fig. 5.1). What has caused this problem and how on oral hygiene shou ld be assessed . Assistance w ith tooth-
may it be treated? brushing by a parent m ay be requ ired until w rist m obility
is fully restored ; com p ared w ith m anual bru shes, pow ered
brushes w ith a rotation oscillation action provid e better
History plaque rem oval in the short term and better protection
against gingivitis in the short and long term .
Complaint Am y’s m other is alread y assisting w ith her d au ghter ’s
Am y d oes not like the appearance of the upp er ‘sid e teeth’ toothbru shing u sing a pow ered toothbrush.
being besid e her upp er front teeth; the ‘sid e teeth’ she thinks
‘look like tw o row s of teeth’. She also d oes not like the Dental history
crooked ness of her low er front teeth.
Am y is a regular attend er at her general d ental p ractitioner.
She had several of the baby back teeth extracted a few years
History o complaint ago and has som e ssu re sealants placed in the rst perm a-
Am y has become aw are of the w orsening app earance of her nent molars. She brushes her teeth tw ice p er d ay, but her
teeth over the p ast year. In that tim e, she has lost som e baby m other says that she need s remind ing abou t toothbru shing,
teeth and the new teeth have com e throu gh crooked . w ith w hich her m other cu rrently assists her.
Am y’s m other reports that her d aughter ’s baby teeth
looked good w ith only mild irregularity of the low er front
teeth. Both she and Am y are keen for treatm ent. Examination
Medical history Extraoral
Am y has a Class I skeletal pattern w ith average FMPA,
Am y’s m other reports that her d au ghter had a heart m u rm ur
average low er facial height and no facial asym m etry. H er
as a baby and attend ed a card iologist at the local hospital.
lip s are com p etent w ith the low er lip resting in the m id labial
third of the u pper central incisors.
N o abnorm al tem p oromand ibu lar joint signs or sym p -
tom s w ere d etected .

Intraoral
■ The intraoral views are shown in Figs 5.1 and 5.2. Describe
what you see.
Poor oral hygiene with plaque deposits visible on several
teeth and associated generalized marginal gingival erythema.
Stained occlusal ssures in 6 , brown staining on the mesial
Fig. 5.1 Anterior occlusion at presentation. o 6 and decalci cation at the gingival margin level on the
• 26
5 SEVERECROWDING
Table 5.1 Factors a ecting the rate o space loss ollowing early loss
o a primary molar
Factor Ef ect*
Age at loss The younger the age at loss, the greater the potential or space loss
Degree o crowding The more crowded the arch, the more space that will be lost
Tooth extracted Earlyloss o an E, rather than o a D,is likelyto lead to more space
loss (see belowregarding arch). 5’s mayerupt and be excluded
palatally/linguallyor be impacted; 5 maybe in crossbite; centreline
shi t i asymmetrical extraction and in case o E, i earlyloss be ore
age 7
Arch romwhich Greater loss is likelyin the upper, rather than in the lower arch, as
A tooth is lost mesial dri t tendencyis greater in the ormer
Type o occlusion Less space loss will occur where good buccal interdigitation exists

*The e ects listed above are those that are, in general, likelyto result romeach o the actors
given. Individual variation with regard to outcome is, however, possible.

Mild lower labial segment crowding with 1 distolabially


rotated; 3 slightly mesially inclined and 3 upright.
Lower right and le t buccal segments exhibit mild crowding.
B
Overall, summing the labial and buccal segments, the lower
arch has moderate to severe crowding.
Severe upper labial segment crowding; 2′s very slightly
mesiolabially rotated; both 3′s are about to erupt buccal to
the line o the arch.
Upper buccal segments are not crowded; there is
mesiopalatal rotation o 5 and distopalatal rotation o 4 ; small
amount o space on either side between the premolar teeth
and between the rst premolars and lateral incisors.
Class I incisor relationship; overbite is average and complete;
C
lower centreline shi ted slightly to the right.
Class I molar relationship on right and le t.

■ What is the likely cause o the enamel hypoplasia on 5 ?


This is m ost likely d u e to p u lpal p athology in the overlying
E , affecting am elogenesis, often referred to as a Tu rner ’s
tooth or Tu rner ’s hyp op lasia.
■ What a re the likely causes o the severe upper
arch crowding?
• Inherent dentoalveolar disproportion this is genetically
D determined and represents a mismatch in tooth size and
the size o the alveolus. Added to this, the maxillary
Fig. 5.2 (A) Lower occlusal view. (B) Upper occlusal view. (C) Right canines are the last permanent teeth to erupt anterior to
buccal occlusion. (D) Le t buccal occlusion. the rst permanent molars and are o ten squeezed
buccally in a crowded arch.
• Early loss o primary teeth this leads to mesial dri t o
the buccal segments and aggravates crowding.
buccal aspects o 6 | 6; ssure sealants are visible in the • Supernumerary teeth and megadont teeth are other
occlusal sur ace o the rst permanent molars. 5 has a mildly causes o crowding but are not relevant in this case.
hypoplastic palatal cusp (this was non-carious).
• Any combination o the previously mentioned causes.
All permanent teeth present rom the second permanent
■ What a ctors in uence the ra te o space loss ollowing early
molar to the second permanent molar in the lower arch (5’s
loss o a primary mola r? What are the e ects o ea rly loss o
are partially erupted); in the upper arch, all permanent teeth
a primary mola r?
(except 3’s) erupted rom second permanent molar to second
permanent molar. These are given in Table 5.1.
27 •
SEVERECROWDING 5
To correct the lower centreline.
Key point
To close residual buccal segment spacing, maintaining Class I
On average, space loss is greater ollowing extraction o a incisor and molar relationships.
primary molar:
■ Wha t is your treatment plan?
• The younger the age at extraction.
• In the upper rather than the lower arch. 1. Provide oral hygiene instruction and dietary advice.
2. Topical f uoride (Duraphat) application to early enamel
• In a crowded arch.
lesions on lower rst permanent molars.
• Where the second rather than the f rst primary molar is
extracted. 3. Composite restoration o the palatal aspect o 5 .
• Where there is poor occlusal interdigitation. 4. Re erral, with the recently taken radiographs, to an
orthodontist or urther assessment and management o
the severe crowding.
■ What are the likely causes o the upper premolar rotations? ■ Explain the treatment options or Amy’s severe upper arch
crowding and modera te to severe lower arch crowding.
Developmental where the tooth germ is rotated in its crypt,
What are the implica tions o ea ch option?
which could be a mani estation o inherent crowding.
Acquired due to early loss o the primary predecessor, most Althou gh Am y’s principal concern relates to her u pper arch
likely due to caries, which removes its main guidance into crow d ing, it is essential that treatm ent planning begins in
occlusion and allows the premolar to initially rotate in the tooth the low er arch. (See p. 18 for an exp lanation of this and
crypt, and become characteristically mesiopalatally rotated later options for relief of crow d ing.) Based on the assessm ent of
due to mesial dri t pressure o the rst permanent molars. the overall severity of the low er arch crow d ing, extraction
of rst prem olars w ill be requ ired . This w ill relieve im pac-
A retained prim ary m olar or its root fragm ent m ay also
tion of the second prem olars and provid e space for the
ind u ce rotation of the su ccessor if it fails to erup t in the
canines to m ove d istally to facilitate labial segm ent align-
correct p osition.
m ent. The latter is m ore likely to occur sp ontaneously on
the right sid e d ue to the uprighting of the slightly m esially
Investigations angu lated 3 . Provid ed oral hygiene has im proved suf -
ciently, xed app liances w ill then be requ ired because of the
■ What investigations would you request and why? angu lation of 3, rotation of 1, the need for centreline cor-
A d ental p anoram ic tom ogram is requ ired to d eterm ine the rection and space closure.
presence and p osition of all u neru p ted teeth. Bitew ing rad i- Follow ing on the sam e schem e given on p ages 18–19 for
ograp hs w ou ld be ad visable in view of the ssu re and treatm ent planning, after you :
m esial su rface staining observed related to 6 and 6, resp ec- Imagine the corrected position o 3 (in this case taking the need
tively. These w ill also allow the statu s of 5 to be assessed . to move the lower centreline to the le t), the next step is to
The presence of all perm anent teeth inclu d ing third Mentally reposition 3 to be in a Class I relationship with the
m olars w as con rm ed ; enam el caries w as d etected in 6 and corrected position o 3. For Amy, space is required to
6 (this had not p rogressed throu gh to d entin). achieve this.
■ How would you assess the spa ce required in the upper arch?
Diagnosis As sp ace is at a p remium , the d istance from the d istal of the
■ What is your diagnosis? lateral incisor to the m esial of the rst p erm anent m olar
should be m easured w ith ne pointed stainless steel
Class I malocclusion on a Class I skeletal base with d ivid ers.
average FMPA. This m easured 16 m m on the right and left sid es.
Generalized marginal gingivitis; enamel caries in 6 | 6. ■ Is this su cient to achieve the trea tment objectives?
Moderate to severe lower arch crowding; severe upper arch Rem oval of both u p p er rst p rem olars w ill p rovid e sp ace
crowding with 3′s unerupted but positioned buccally; lower for alignm ent of 3′s, bu t it w ill be insu f cient to achieve this
centreline displaced slightly to the right. comp letely and obtain a Class I canine relationship w ith the
Class I molar relationship on right and le t. 3| 3 p ositions corrected u nless all the space from the u p per
■ What is the IOTN DHC gra de (see p. 264)? Explain why. arch extractions is m aintained . N o space loss is perm issible.
Anchorage reinforcem ent is requ ired . It is im p ortant to
5i – d u e to the im p acted m axillary canines. realize that rem oval of an u pper rst p rem olar creates
u sually 7 m m of sp ace, bu t the average m esiod istal w id th
Treatment of the perm anent m axillary canine is 8 m m – so generally,
on average, 15 m m of space is required to accom m od ate 3
■ What are the aims o treatment? and 5. Fortu nately, in Am y’s case, there is a sm all am ount
of sp ace present in the u pp er prem olar areas w hich w ill
To improve oral hygiene and restore dental health.
assist correct p ositioning of 3’s w ith the aligned 3’s .
To relieve crowding. Rem oval of both u p p er canines is another p ossibility; if
To align upper and lower arches. they eru pt bu ccal to the line of the arch in a few m onths’
• 28
5 SEVERECROWDING
tim e, they cou ld be extracted , or if not, su rgical rem oval ■ I a n upper removable a ppliance spa ce mainta iner
cou ld be u nd ertaken. Up p er xed app liance therap y w ou ld were to be considered, what would be your design?
then be required to rotate the rst prem olars slightly m esio- What instructions would you issue regarding appliance
palatally to hid e the p alatal cu sp and occu p y a greater wear?
m esiod istal w id th akin to that of the extracted canine; the
Activation: there are no active components.
palatal cusp of 4′s shou ld also be grou nd to avoid interfer-
ences in lateral excu rsions. Bond ed retention w ould also be Retention: Adams clasps 6|6 (0.7 mm stainless steel wire) with
need ed to m aintain the nal p osition of 4′s. headgear tubes soldered to the 6′s clasp bridges; f at stops
Rem oval of u pp er 4’s ad d resses Amy’s concern and is distal o 2|2.
likely to give the better nal ap p earance. Anchorage: ull palatal acrylic coverage; also headgear to be
added to t into the molar clasp tubes.
Baseplate: ull palatal acrylic coverage.

Key point The u p p er rem ovable ap p liance shou ld be w orn fu ll-tim e


except for contact sports and after m eals w hen it shou ld be
An upper f rst premolar extraction space will not removed for cleaning. Sticky and hard food s as w ell as zzy
accommodate an upper permanent canine. d rinks shou ld be avoid ed w hen the ap pliance is w orn.
Details regard ing head gear force and w ear for anchorage
reinforcem ent as w ell as safety p recau tions necessary are
given on page 14.

■ Finalize your treatment planning. ■ Describe the alternatives to this appliance.


The next step s are: Sp ace cou ld be m aintained w ith head gear alone tted to
Plan the upper labial segment. The mild rotation o 2’s is best band s on the up per rst p erm anent m olars, w ith a force of
dealt with by xed appliance therapy. 200–250 g per sid e, w orn 10–12 hou rs per d ay. Shou ld the
head gear not be w orn as requ ired in this case, then m esial
Decide on the f nal molar relationship. With our rst d rift of the second p rem olar and rst p erm anent m olar w ill
premolar extractions, this should be Class I; residual space comp rom ise sp ace requ ired for 3′s.
closure in the lower arch will necessitate xed appliance A N ance bu tton p alatal arch sold ered to band s on the
therapy. up per rst p erm anent m olars is an alternative (Fig. 5.3),
Assess the anchorage needs. The high anchorage demands in bu t as space is critical in this case, sold ering a transpalatal
the upper arch have already been identi ed. Options are a arch also to the m olar band s m ay su pp ort anchorage further
space maintainer with anchorage support (headgear), (Fig. 5.11). The N ance bu tton palatal arch provid es anchor-
headgear alone, Nance palatal arch with/without transpalatal age throu gh m ucosal contact of the acrylic bu tton w ith
arch or temporary anchorage devices (TADs). Anchorage the anterior vau lt of the p alate w hile the p alatal arch m ain-
demands in the lower arch are modest; there is no need or tains the interm olar d istance, p reventing m esial d rift and
anchorage rein orcement. m olar tipp ing. This is assisted fu rther by the ad d ition of a
Plan retention. Upper and lower vacuum- ormed retainers transp alatal arch, straight across the p alate, linking 6 to 6.
(Essix retainers) should be adequate to be worn night-time Another op tion is to place a TAD d istal to the 6’s on either
only or 1 year ollowed by a second year o every second
night wear.

■ Wha t is the f nal orthodontic trea tment plan?


Assu ming that Am y’s oral hygiene im proves and is m ain-
tained at a high stand ard follow ing instru ction, then the
plan w ou ld be as follow s:
1. Anchorage rein orcement in the upper arch by one o
several means (see above).
2. Extraction o our rst premolars.
3. Upper and lower xed appliances.
4. Upper and lower Essix retainers.

Key point
Where anchorage is at a premium, always rein orce
anchorage be ore any extractions or relie o crowding.
Fig. 5.3 Nance button palatal arch.
29 •
SEVERECROWDING 5
effectiveness betw een the ap p liances in term s of anchorage
su pp ort, bu t there w ere m ore problem s reported w ith head -
gear and N ance bu ttons than w ith TADs.
■ Are there any risks with TADs?
Risks inclu d e:
• Screw breakage during insertion (5%).
• Root contact during placement (should heal
unevent ully).
• Failure necessitating TAD replacement/removal (~14%).
• In ection (unlikely i the area around the screw is brushed
gently with a small headed toothbrush and f uoride
toothpaste and chlorhexidine mouthrinse (0.2%) is used
or rst 5 days a ter insertion).
• Screw loosening (may be replaced in original or di erent
Fig. 5.4 Lingual arch. position).

sid e and u se it to keep the 6’s and second prem olars from ■ I Amy were issued an upper removable space mainta iner
m oving m esially. with headgear support, how would you know at her
2-week review whether the a pplia nces were being worn
■ I all the spa ce rom lower premolar extra ctions ha d been
as instructed?
required or lower labial segment a lignment, how could
anchorage have been rein orced there? Amy shou ld be speaking norm ally w ith the u pper rem ov-
able ap pliance in p lace; she shou ld also be able to rem ove
A low er rem ovable ap p liance is not w ell tolerated d u e to
and insert the app liance u naid ed by a m irror. The baseplate
encroachm ent on tongu e sp ace, interference w ith sp eech
should have lost its shine; there shou ld be evid ence of m ild
and d if cu lty w ith achieving good ap p liance retention d u e
gingival erythem a along the palatal m argins of the app li-
to the lingu al inclination of the m olars. A low er lingual arch
ance and at the posterior extension of the basep late (if the
sold ered to band s on the rst p erm anent m olars is a better
app liance had a bite p latform , there w ou ld be m arks from
op tion (Fig. 5.4). Alternatively, the second perm anent
the occlu sion also). She should also be able to insert, assem -
m olars cou ld be bond ed or band ed and ligated to the rst
ble and rem ove the head gear easily. If it is being w orn as
perm anent m olars. Another op tion is to p lace a TAD into
instru cted , the head cap w ill have signs of w ear and the
the retrom olar area on each sid e and use it to keep the rst
head gear tu bes in the rem ovable ap p liance shou ld be free
perm anent m olars and second p rem olars from m oving
of any food d ebris from insertion of the facebow.
m esially w hile p rovid ing anchorage for canine retraction.
The occlusion follow ing appliance w ear w ith head gear
support and extraction of four rst premolars is show n in
Key point Fig. 5.5. The lower centreline shift corrected without appli-
ance treatment. At this stage, Amy indicated that she was
Options or rein orcing anchorage in upper arch:
suf ciently pleased w ith the improvement in her dental
• Upper removable appliance space maintainer with appearance that she did not wish to proceed to further xed
headgear support. appliance therapy. This was fortunate as she was not keen on
• Headgear to molar bands. assisted tooth brushing by her mother and had struggled, due
• Nance button palatal arch (with/without transpalatal to continuing problems w ith right w rist mobility, to maintain
arch). a high standard of oral hygiene during removable appliance
• TAD. treatment, despite the use of a pow ered toothbrush.
Options or rein orcing anchorage in lower arch: An Essix retainer w as tted in the u p p er arch only; Am y
• Lingual arch. w as instru cted to w ear this at night-tim e only for 12 m onths
• Bond/band 7 ’s and ligate to 6 ’s. initially. Arrangem ents w ere m ad e to review her occlu sion
at that stage.
• TAD.
■ Wha t is an Essix reta iner, and what are its potential
adva ntages over a Ha wley retainer in the upper a rch? Aside
■ How e ective are TADs a t rein orcing anchorage? How do rom the usual advice regarding reta iners, what specif c
they compare to other methods o anchora ge advice should the pa tient be given regarding this retainer?
rein orcement? An Essix retainer is a clear vacu u m-form ed therm oplastic
retainer. Som e of its potential ad vantages over a H aw ley
Evid ence ind icates that TADs are an effective, non-com p liant
retainer are:
m eans of reinforcing anchorage or of m oving u p p er rst
perm anent m olars d istally. A recent rand om ized clinical • Better aesthetics.
trial com pared anchorage sup plem entation w ith head gear • Less di culty with speaking.
or N ance button p alatal arch or TADs, in cases that requ ired • Cheaper.
m axim u m anchorage su p p ort; there w as no d ifference in • Easier to make.
• 30
5 SEVERECROWDING

■ What do you notice in Fig. 5.6?


Poor oral hygiene with plaque deposits on most teeth.
Generalized marginal gingival erythema.
76543211234567
visible (note 3 cusp tip).
76543211234567
Caries- ree dentition.
Moderate lower arch crowding with 3 impacted but erupting.
A Severe upper arch crowding; 5 excluded palatally with 6 and
4 in contact; 6 rotated mesiopalatally; median diastema
~2 mm; 3 buccal and mesially inclined.
Class I incisor relationship; overjet 2 mm (measured clinically);
overbite average and complete; centreline shi t (lower was
~4 mm to the right).
Right molar relationship is Class II; le t molar relationship is
Class I.
Crossbite 6 (5 also in crossbite).
B
■ What is the IOTN (DHC) grade (see p. 264)? Expla in why.
4d – d u e to contact point d isplacem ent betw een 5 and 4 , or
betw een 3 and 4. Althou gh 3 is imp acted , it is partially
erupted and so w ould score 4t and not 5i (if 4 m m or less
betw een 4 and 2.5i).

■ What a re the possible ca uses o the crowding and tooth


displacements in the upper a nd lower arches?
The palatal exclusion of 5 is m ost likely d u e to early loss of
E in an inherently crow d ed arch. Sp ace closure has been
C comp lete w ith 6 and 4 in contact. Factors in u encing the
rate of sp ace closu re have been given in Table 5.1. N ote the
Fig. 5.5 (A) Post-treatment: right buccal occlusion. characteristic m esiopalatal rotation of 6 follow ing early loss
(B) Post-treatment: anterior occlusion. (C) Post-treatment: le t of E .
buccal occlusion. Roger ’s m other con rmed that E had been extracted at
age 5 d u e to caries.
3 is the last tooth to eru p t in the u p per left quad rant and
is buccally d isplaced d u e to the inherent crow d ing.
Vacu um -formed retainers appear m ore effective than The lack of sp ace for 3 m ay be linked to the low er cen-
H aw ley retainers at m aintaining correction of the u p p er and treline shift (see possible cau ses below ). N ote the d istal tilt
low er labial segm ents, w ith greater effectiveness in the of 2 . Early loss of low er p rim ary m olars app ears unlikely
u pper than the low er arch. d u e to the reasonably aligned bu ccal segm ents.
The retainer m u st not be w orn w hile eating and m u st
■ Why has 6 rotated mesiopalatally?
never be w orn w hile consu m ing beverages, esp ecially car-
bonated d rinks, as w ith these there is a high risk of enam el As the p alatal root is the largest, the tooth rotates arou nd
d em ineralization. this rather than arou nd either of the bu ccal roots.

■ What is unusual a bout the eruption pa ttern in

CASE2 the lower arch?


Usually the canines eru p t before the p rem olars (the op p o-
site is tru e in the u pper arch) so it is u nusual that 3 is
SUMMARY im pacted . See Chapter 1 for the average eru ption d ates of
Roger has recently relocated to your area rom teeth.
another country. He presents with a tooth erupting
■ What is the likely cause o the lower centreline shi t?
in his palate and an impacted 3 . What are the
causes o these problems, and how may they be Prem atu re loss of C d ue to inherent crow d ing is a likely
cause. Early u nbalanced loss of C or D d u e to caries is
managed?
another p ossibility as E w as lost early d u e to caries. N o
Roger has a Class I skeletal pattern with average low er prim ary teeth had been extracted .
FMPA and lower acial height; there is no acial Other cau ses of a centreline shift are given in Chap ter 14,
asymmetry. Box 14.1, p age 88.
31 •
SEVERECROWDING 5

Fig. 5.6 (A) Right buccal occlusion. (B) Anterior occlusion. (C) Le t buccal occlusion. (D) Upper occlusal view. (E) Lower occlusal view.

Key point
6 rotates mesiopalatally with mesial dri t ollowing early
loss o E.

Investigations Fig. 5.7 Dental panoramic tomogram.

■ A dental pa noramic tomogram (DPT) was taken by Roger’s


previous dental pra ctitioner a ew months ago. You request ■ Do you require any urther ra diogra phs?
this by email. What do you notice in Fig. 5.7?
It is not possible to see the roots of 5 and 4 clearly; tw o
The DPT show s: periap ical rad iograp hs w ith a tu be shift cou ld be taken to
Normal alveolar bone height. assess this m ore com pletely and to exclu d e any resorption.
Should the intraoral view s not provid e su f cient inform a-
All erupted permanent teeth sound.
tion, a CBCT cou ld be taken.
5 overlapping 4 ; impacted 3 . Periap ical view s ind icated no root resorption of either
Four third molars developing. p rem olar.
• 32
5 SEVERECROWDING

Diagnosis
■ Wha t is your diagnosis?
Class I malocclusion on a Class I skeletal base with
average FMPA.
Generalized mild marginal gingivitis.
Moderate lower and severe upper arch crowding with the
lower centreline shi ted to the le t.
Right buccal segment relationship is Class II; le t buccal
Fig. 5.8 Digital simulation o potential outcome ollowing
segment relationship is Class I.
extractions and 3 alignment.
Buccal crossbite 6 and 5 .

align and 3 to erup t. The m esial angulation of 3 is favou r-


Treatment able to allow it to m ove d istally follow ing 4 extraction, and
it shou ld align consid erably u nd er cheek pressu re. Althou gh
■ What are your aims o treatment?
this m ay be incom plete, it w ill im prove 3 position greatly.
Establish good oral hygiene. This p lan accep ts the low er centreline shift and the very
Relie o crowding. m ild crow d ing of the low er left bu ccal segm ent. These
shortcom ings m u st be exp lained to Roger and his p arents
De-rotation and crossbite correction o 6 .
shou ld he w ish to p roceed . A d igitally sim u lated im age of
Correction o lower centreline. the p otential ou tcom e of this p lan is show n in Fig. 5.8.
Establish Class I right and le t molar relationships.
Retain.

■ Wha t is your treatment plan?


CASE3
1. Oral hygiene instruction. SUMMARY
2. Correct the crossbite on 6 and derotate 6 . Conal, a 12-year-old boy, presents complaining
54 about the crowding o his upper and lower ront
3. Extract .
44 teeth. He is keen or treatment. What has caused
4. Upper and lower xed appliances. this problem and how may it be treated?
5. Upper and lower vacuum- ormed retainers. Conal has a mild Class III skeletal pattern with
■ How could the crossbite be corrected on 6 and 6 be slightly increased FMPA and lower acial height;
derotated? there is no acial asymmetry. His lips are competent
The crossbite m ay be corrected by cem enting band s to 6 and and no temporomandibular joint signs or
6 and ru nning cross-elastics from an attachm ent on the symptoms were recorded.
palatal of 6 band to the hook on the bu ccal of 6 band . In ■ What do you notice in Fig. 5.9?
ord er to facilitate band p lacem ent and to ensu re that m ove-
m ent of 6 is not obstru cted by 5 , it w ou ld be sensible to Mild marginal gingival erythema.
have 5 extracted p rior to band p lacem ent. The cross-elastics 6 has an amalgam restoration. 6 6 are ssure sealed.
should be w orn fu ll-tim e, inclu d ing at m eals. Crossbite
Severe lower and upper arch crowding with all canines
correction is likely to take a short tim e, follow ing w hich
erupting buccally.
the elastics shou ld be d iscontinued for a m onth to assess
stability. Derotation of 6 w ill likely occu r w ith increasing Class III incisor relationship; reduced and complete overbite;
d iam eter of the archw ires w hen the rem aind er of the xed centreline shi t (lower was 3 mm to the le t and upper was
appliances has been p laced . 1 mm to the right).
An alternative is to u se a qu ad helix ap p liance and ad ju st 6 and 2 in crossbite.
it to both d erotate 6 and to m ove it buccally. There is, Buccal segment relationship is Class I bilaterally.
how ever, likely to be som e recip rocal u nw anted bu ccal
m ovem ent of 6 by this m eans. ■ What should you check or with the crossbite on the 2’s?
Roger w as a keen ru gby p layer, and althou gh he w anted You shou ld check if there is a m and ibu lar d isp lacem ent
to im p rove the app earance of his teeth, he w as not p rep ared present on closing.
to w ear xed appliances. N o m and ibular d isplacem ent w as d etected .
■ Is there another option you could consider to address ■ What is the likely cause o the upper and lower a rch
his wishes? crowding?
An extraction-only p lan cou ld ad d ress several issues. Inherent d entoalveolar d isproportion is the m ost likely
Removal of 5 4 and 4 w ill relieve crow d ing and allow 3 to cau se.
33 •
SEVERECROWDING 5

A B

C
D

Fig. 5.9 (A) At presentation: right buccal occlusion. (B) Anterior occlusion. (C) Le t buccal occlusion. (D) Lower occlusal view. (E) Upper
occlusal view.

■ What is the IOTN (DHC) gra de (see p. 264)? Explain why. A lateral cep halom etric lm is also requ ired to assess
m ore fu lly the Class III skeletal p attern and the angu lations
4d – d u e to contact p oint d isp lacem ent >4 m m (present on
of the u pp er and low er incisors to their resp ective d ental
several teeth, bu t greatest w ith 3 and ad jacent teeth).
bases. This revealed the follow ing: SN A = 81°; SN B = 81°;
SN -m axillary plane = 5°; MMPA = 29°; 1 to m axillary plane
Investigations = 112°; 1 to m and ibu lar plane = 91°; interincisal angle = 130°;
facial p rop ortion = 58%.
■ What investigations would you request and why?
■ How would you assess the long term prognosis o 6 ?
A d ental p anoram ic view w ou ld be ad visable to check the
cond ition of all eru p ted teeth and to ascertain if third m olars As the patient’s general d ental p ractitioner, you shou ld
are d evelop ing. check the clinical notes regard ing the last tim e 6 w as
This revealed all teeth includ ing third m olars to be d evel- restored . This shou ld have record ed w hether an ind irect
op ing norm ally and in the correct p ositions w ith no ap p ar- p ulp cap w as u nd ertaken or w hether the pu lp w as exposed
ent resorp tion of the 2’s by 3’s. The restoration in 6 w as d u ring the restorative p roced u re. You should also ask Conal
d eep . A su bsequent periap ical rad iograp h of 6 ind icated if he is having any sym p tom s associated w ith 6 , then check
second ary caries. the bu ccal su lcu s for any sw elling or sinu s related to 6 , as
• 34
5 SEVERECROWDING
w ell as the restoration for integrity of the m argins, and 3. Upper and lower xed appliances.
assess w hether 6 is tend er to p ercu ssion. 4. Upper and lower vacuum ormed retainers.
The clinical notes record ed a p u lp exp osu re and a d irect 5. Monitor third molars.
pu lp cap , so althou gh the tooth has been sym p tom -free, the
■ How may anchorage be rein orced in the upper and
prognosis w ou ld be som ew hat gu ard ed .
lower arches?
■ What is your interpretation o the cephalometric f ndings?
Means to reinforce anchorage in the up per and low er arches
Relative to Caucasian norm s, SN A is average and SN B is have been ou tlined on p age 29.
slightly increased , ind icating m ild m and ibu lar prognath- A lingual arch and a N ance palatal arch w ith transpalatal
ism. The skeletal p attern is m ild ly Class III (SN A − SN B = bar from 7 to 6 w ere placed (Figs 5.10 and 5.11).
AN B = 0°). SN to m axillary p lane is slightly less than
■ Are there any means by which anchorage demands may be
average; MMPA is slightly increased ; 1 to m axillary p lane is
reduced in the upper arch?
slightly increased , and 1 to m and ibu lar p lane is slightly
red u ced . The interincisal angle is red u ced and facial p ropor- Extraction of 4 in ad d ition to 6 w ou ld red u ce anchorage
tion is increased . The 1 angu lation is com p ensating for the d em and s in the u p p er right bu ccal segm ent.
m ild ly increased MMPA (120° − 91° = 29°). ■ Does this option ha ve a ny other potential benef ts? Are
there any risks?

Diagnosis It w ould m ake alignm ent of 3 m u ch easier, as the 4 extrac-


tion sp ace is ad jacent. 7 shou ld also end u p in a Class I
■ Wha t is your diagnosis? relationship w ith 6 .
Class III malocclusion on a mild Class III skeletal base with As the incisor relationship is m ild ly Class III, closure of
slightly increased FMPA. three extraction sp aces (6 4 4) in the u pper arch m ay ru n the
risk of bringing the incisors to an ed ge-to-ed ge relationship
Generalized mild marginal gingivitis.
or even into a slight reverse overjet.
Restored 6 with guarded long-term prognosis.
Severe upper and lower arch crowding with upper and lower
centreline shi ts (lower 3 mm to the le t, upper 1 mm to the
right).
Crossbite o 2 and 2 with no associated mandibular
displacement.
Molar relationship is Class I bilaterally.

Treatment
■ What are the aims o treatment?
Improve oral hygiene.
Relieve crowding.
Align upper and lower arches.
Correct centrelines.
A
Maintain Class I buccal segment relationship.

■ I 6 is removed in view o its guarded prognosis, wha t


implica tions will that have or trea tment and the f nal
outcome?
Anchorage d em and s are alread y high in the upp er arch and
extraction of 6 rather than 4 w ill increase that fu rther on the
u pper right sid e as both p rem olars w ill requ ire retraction
before 3 can be aligned . So althou gh 6 is a larger tooth
m esiod istally than 4 , the sp ace created is further from the
site w here space is required and its rem oval w ill m ake treat-
m ent m ore com p lex. If 6 is extracted , the cu sp tip of 5
should occlu d e w ith the bu ccal groove of 6 at the end of
treatm ent.
■ Wha t is your treatment plan? B
1. Rein orce upper and lower arch anchorage.
Fig. 5.10 (A) Lingual arch. (B) A ter the removal o 4 4 (note small
6 4
2. Extract . amount o space remaining).
4 4
35 •
SEVERECROWDING 5
Key point
Extraction o a f rst permanent molar in an upper arch with
severe labial segment crowding will complicate anchorage
requirements.

The occlusion follow ing canine retraction and alignment


is show n in Fig. 5.11. The nal occlu sion is show n in
Fig. 5.12.

Primary resources and


recommended reading
British Orthod ontic Society 2009 Patient Inform ation Lea et:
Fig. 5.11 Nance palatal arch with transpalatal arch ( ollowing Orthod ontic Mini-Screw s. Lond on: British Orthod ontic Society.
removal o 6 4 and retraction o 5 4 3 3; note minimal residual Jam bi S, Walsh T, Sand ler J et al 2014 Reinforcem ent of anchorage
space). d uring xed brace treatm ent w ith im plants or other su rgical
m ethod s. Cochrane Database of Syst Rev Issu e 8. Art N o:
CD005098. DOI: 10.1002/ 14651858.CD005098.p u b3.
Row land H , H itchens L, William s A et al 2007 The effectiveness
of H aw ley and vacuum -form ed retainers: a single-center
rand om ized controlled trial. Am J Orthod Dentofacial Orthop
132:730–737.
Sand ler J, Mu rray A, Thiru venkatachari B et al 2014 Effectiveness
of 3 m ethod s of anchorage reinforcem ent for m axim um
anchorage in ad olescents: a 3-arm m u lticenter rand om ized
clinical trial. Am J Orthod Dentofacial Orthop 146:10–20.
Thornhill MH , Dayer M, Lockhart PB et al 2016 A change in
the N ICE gu id elines on antibiotic p rop hylaxis. Br Dent J
A 221:112–114.
Yaacob M, Worthington H V, Deacon SA et al 2014 Pow ered versus
m anu al toothbru shing for oral health. Cochrane Database of
Syst Rev Issu e 6. Art N o: CD002281. DOI:10.1002/ 14651858.
CD002281.p u b3.

For revision, see Mind Map 5, page 225.

Fig. 5.12 (A) Post-treatment: anterior occlusion. (B) Right buccal


occlusion (note position o 5 ). (C) Le t buccal occlusion.
6
Medical history
Diane is t and w ell.

Dental history
Diane is a regu lar attend er at her general d ental p ractitioner
bu t has never had any d ental treatm ent.

Palatal canines Examination


Extraoral
Diane has a Class I skeletal pattern w ith average FMPA and
low er facial height and no facial asym m etry. H er lip s are
com p etent w ith the low er lip at the level of the incisal third
of the u p p er incisors.
There is a slight lateral m and ibu lar d isplacement to the
4
left on closure on .

CASE1
4
Intraoral
SUMMARY ■ The intra oral views are shown in Figs 6.1 a nd 6.2. Describe
what you see.
Diane, a 15-year-old girl, presents with both upper
primary ca nines retained (Fig. 6.1). What is the Oral hygiene is air with mild marginal gingival erythema
related to 2 2 and the upper le t buccal segment teeth.
cause and what treatment possibilities are there?
No obvious buccal swellings in the C areas, but there seem to
be mucosal swellings palatal to C2 2C, perhaps indicating
History the position o unerupted 3’s.
Diane is concerned about the size of the baby u pper ‘eye’ Slight enamel demineralization buccally on 6 6.
teeth that are present and by the spaces on either sid e of her
7 6 5 4 C2 1 1 2 C4 5 6 7
u pper tw o front teeth. She is not bothered by the sm all sp ace erupted.
7654321 1234567
betw een the u pp er front teeth. C is also slightly loose, and
she is w orried in case it is lost, p rod u cing a big sp ace. Mild lower labial segment crowding; 1| 1 very slightly
mesiolingually rotated; lower buccal segments spaced.
History o complaint Upper arch uncrowded; spacing in the upper labial
Diane has been aw are that the baby ‘eye’ teeth shou ld have segment.
been lost a few years ago. H er p reviou s general d ental prac- Class I incisor relationship with a centreline shi t (clinically the
titioner, w ho retired last year, ad vised her that these teeth lower centreline was 1.5 mm to the le t).
w ou ld eventu ally fall ou t by them selves and that w hen
Buccal segment relationship is Class I bilaterally; lingual
the new ‘eye’ teeth cam e throu gh, she w ou ld then need a
crossbite o 4 with 4 ; buccal crossbite o 6 with 6.
brace to close the sp aces betw een her top teeth. There is no
history of trau m a to C C areas, and all other prim ary teeth ■ What a re the potential causes o C’s being retained?
w ere lost natu rally. All p erm anent teeth have eru p ted on
Absence o 3’s this is highly unlikely (0.3% o Caucasians).
sched u le.
She has noticed that C has been loose interm ittently for Ectopic position o 3’s this is the most likely cause (1 2% in
the p ast 18 m onths. It d oes not app ear to have got looser in Caucasians with 8% o these being bilateral).
recent m onths. Diane is very keen to im p rove the appear- ■ What a ctors are implicated in pa la tal ca nine ectopia?
ance of her u p p er teeth.
The aetiology of p alatal canine ectop ia is obscu re bu t m ost
probably mu ltifactorial. Possible cau sative factors are:
1. Genetic palatally displaced 3 appears to result rom a
polygenic mode o inheritance, with associated
anomalies including incisor-premolar hypodontia,
peg-shaped 2 (see below), in raoccluded primary molars,
impacted 6, other ectopic teeth and transposition (see
Chapters 1, 7 and 8). Class II division 2 malocclusion is
also associated with an increased incidence o palatal 3.
Lending urther support to a genetic aetiology is that
palatal 3 has a predilection or those o European origin
Fig. 6.1 Anterior occlusion at presentation. as well as those with a amilial tendency, with emales
37 •
PALATALCANINES 6
3. 3 has the longest path o eruption o any permanent
tooth.
4. Arch length discrepancy palatal displacement o 3’s has
been mostly associated with an uncrowded or spaced
arch. Note the spacing present in Diane’s upper arch.
5. Trauma to the maxillary anterior area at an early stage o
development this has been suggested, but there is no
history o trauma in this case.
6. Peg-shaped, short-rooted 2’s or absent 2’s guidance or
3 is reduced where these eatures are evident, doubling
the incidence o palatal impaction o 3.

A
Key point
Palatal displacement o 3:
• A ects Europeans, emales and both sides o the arch
more commonly.
• Is more common in an uncrowded arch.
• Is associated with small, absent or abnormal ormation
o 2’s, hypodontia, impacted 6 in raoccluded primary
molars, other ectopic teeth and Class II division 2
malocclusion.

B N ote in Diane’s case, the m esiod istal w id th of 2’s w ere


the sam e as those of 2’s, ind icating that 2’s are sm aller than
average and that a tooth-size d iscrepancy (TSD) or Bolton
d iscrep ancy exists betw een the u p p er and low er labial
segm ent teeth.
■ Wha t is the prevalence o TSD, and which teeth are most
commonly a ected?
Betw een 5% and 14% of the p op u lation have a signi cant
overall TSD, w hereas 20–30% have a signi cant anterior
TSD (see below ). Althou gh a TSD is m ost com m only d ue to
a size anom aly of the u p per lateral incisor, prem olars or
C
other teeth m ay also be responsible.
■ How would you assess or a TSD?
Quick-check method
• For anterior TSD: compare the width o the upper and
lower lateral incisors; i the upper lateral incisor is not
wider than the lower, a TSD exists.
• For posterior TSD: compare the width o the upper
and lower second premolars; these should be about
equal size.

D Computational method A tooth-size analysis, often referred


to as a Bolton analysis after its d evelop er, m ay also be per-
Fig. 6.2 (A) Lower occlusal view. (B) Upper occlusal view. (C) Right
formed . The m esiod istal w id th of each perm anent tooth,
buccal occlusion. (D) Le t buccal occlusion.
exclu d ing second and third m olars, is m easu red and then
the su m m ed w id ths of the m axillary to m and ibu lar teeth
a ected more than males and occurrence being more are com p ared w ith a stand ard table. This allow s calcu lation
common on both sides o the maxillary arch than one of Bolton anterior (canine to canine) and overall ( rst m olar
would envisage. to rst m olar) ratios as follow s:
2. Crypt displacement where the position o 3 is grossly (Sum mandibular anteriors)/(sum maxillary anteriors) × 100 =
displaced, this may be an aetiological actor. anterior ratio (%)
• 38
6 PALATALCANINES
(Sum mandibular 6 6)/(sum maxillary 6 6) × 100 = overall assessm ent of 3 position, althou gh its p otential for align-
ratio (%) m ent is presented m ore favourably as 3 ap p ears at a m ore
Bolton obtained an anterior ratio o 77.2 ± 1.65% and an obtu se angle to the occlu sal p lane and less close to the
overall ratio o 91.3 ± 1.91%. m id line. The root length of C, the vertical and m esiod istal
position of 3 relative to the incisor roots and the axial angu -
Discrep ancies greater than 2SDs (2 stand ard d eviations)
lation and apex location shou ld be assessed . An up per ante-
beyond these m ean valu es have been regard ed as clinically
rior occlu sal rad iograph or a periapical lm of each 3 is
relevant to treatm ent p lanning. Tooth-size analysis m ay also
useful for d etecting incisor resorption and d eterm ining the
be und ertaken using d igital m od els; the m easurem ents are
prognosis of the C’s. Either of these view s, used in com bina-
as accurate and reliable as those obtained from p laster
tion w ith the p anoram ic view and ap p lication of p arallax (a
m od els.
palatal 3 m oves w ith the tu be shift), can be u sed to locate
■ Wha t are the implications o a TSD? the 3’s. Localization of a palatally im p acted canine, how ever,
The teeth m ust be proportional in size to ensure good occlu- has been show n to be best u nd ertaken u sing the com bina-
sion. Rarely is a TSD of less than 1.5 m m of signi cance w ith tion of an occlu sal and a p eriap ical rad iograp h allow ing
regard to treatm ent p lanning, but w here larger d iscrep an- horizontal p arallax. A lateral cep halom etric rad iograph is
cies exist, ad justm ent of the m esiod istal tooth w id th through not ind icated in Diane’s case, bu t w here it is justi ed on
either ad d ition to the enam el (e.g. com posite bu ild -u ps or clinical grou nd s, it provid es valu able inform ation about the
porcelain veneers) or enam el rem oval (e.g. interd ental position of 3’s, w hen u sed in com bination w ith the p ano-
enam el strip p ing/ rep roxim ation) m ay be requ ired to close ram ic view.
or open sp aces in the opp osing arch. Three-d im ensional evalu ation of the canine p osition and
any su sp ected resorption to the roots of other teeth is p ro-
vid ed by CBCT, w hich cou ld in tim e, rep lace the rad io-
Key point graphic view s given above for canine localization (Fig. 6.3).
The recent d evelop m ent of a low d ose protocol (~50% less)
TSD: for CBCT of the anterior m axilla w ith an im p acted canine
• May be assessed comprehensively by a Bolton analysis: in paed iatric d entistry m ay facilitate this. Using CBCT, the
mean anterior ratio is 77.2 ± 1.65%; mean overall ratio is incid ence of incisor root resorption from an im pacted m axil-
91.3 ± 1.91%. lary canine has been estim ated to be as high as 68%, m ore
• Is rarely signif cant i <1.5 mm. than ve tim es higher than that rep orted from conventional
• Clinically signif cant or treatment planning i >2SDs rad iograp hs. A m ore robu st research base for orthod ontic
beyond Bolton mean values. ap plication of this techniqu e, how ever, is requ ired before it
can be ad vocated m ore w id ely.

Investigations Key point


■ What investigations would you underta ke rega rding the Using CBCT:
reta ined C’s? Explain why.
• Incidence o incisor root resorption associated with an
It w ou ld be essential to d eterm ine if 3’s are p resent and to ectopic maxillary canine may be f ve times higher than
localize their p osition. Initial assessm ent shou ld be clinical, with conventional radiographs.
and w here su spicion of 3 d isp lacem ent exists, rad iographic
exam ination should follow.

Clinical ■ Does CBCT have a ny other uses in orthodontics a side rom


Palp ation of the buccal sulci and palatal m ucosae in the assisting with the localization o unerupted teeth a nd a ny
u pper canine regions, as w ell as observation of the 2 inclina- associated pathology?
tion, u su ally p rovid es a reasonable gu id e to the p robable
CBCT also assists app raisal of alveolar bone height, w id th
position of an u neru p ted 3. Labial d isp lacem ent of 2 crow n
and volu m e, w hich m ay be bene cial for cases requ ir-
ind icates 3 to be lying high and bu ccal over 2 root, or low
ing com bined su rgical–orthod ontic m anagem ent, cleft lip
and palatal.
and palate for alveolar bone grafting, and orthod ontic–
restorative care for im plant planning. In certain other cases,
Radiographic it may also be u sefu l for assessment of the airw ay or the
Tw o lm s taken w ith either a vertical or a horizontal tu be temp orom and ibular joint.
shift are requ ired to assess accurately the location of
■ How does the ra diation dose rom CBCT compare with that
u neru p ted 3’s. Alternatively, cone beam com p u ted tomog-
o a DPT?
rap hy (CBCT) m ay be u sed , bu t its use is generally con ned
to cases w here there is specu lation of root resorption of Althou gh the effective rad iation d ose is less (generally
ad jacent teeth or w here u ncertainty rem ains regard ing 3 50–500 µSv) than that of a conventional CT, one exp osu re
position, having screened initial conventional rad iograp hs. equates to abou t 2–8 conventional panoramic rad iograp hs
A d ental p anoram ic tom ogram (DPT) gives a general good (3–24 µSv).
39 •
PALATALCANINES 6

A
A

Fig. 6.4 (A) Dental panoramic tomogram. (B) Upper anterior


occlusal radiograph.

in view of the association of palatal 3’s w ith these anom a-


lies. From 9 years of age, p alpation for u neru pted 3’s should
be carried ou t rou tinely. Im p ortantly, the p osition of 3 m ust
C be localized before consid ering any intercep tive extractions.
Fig. 6.3 (A) Another case: dental panoramic tomogram showing Rad iograp hic investigation is requ ired w hen a d ifference is
3 overlying 2. (B) Upper anterior occlusal radiograph indicating d etected on clinical p alp ation of the u p p er bu ccal su lcu s
3 to be slightly palatal with possible resorption o 2. (C) CBCT betw een op posite sid es of the arch.
image showing 3 to be palatal to 2 and indicating the site and Where 3 is d isp laced palatally in an u ncrow d ed arch, in
extent o resorption o 2. a child aged 10–13 years old , rem oval of C m ay lead to 3
reverting to a normal path of eru ption. The am ou nt of
im p rovem ent d ep end s on the d egree of overlap of 3 over 2
root, w ith a better p rognosis w hen 3 overlies the d istal
■ Are there any other disadva ntages to CBCT in orthodontics?
rather than the m esial half of 2 root. Although im provem ent
Cu rrently, CBCT u nits are costly and consid erable tim e in 3 position m ay occu r even w here 3 is m arked ly d isp laced ,
m ust be allocated to view, and rep ort on, all the d ata sp ecialist ad vice mu st be obtained before rem oval of C.
obtained , in line w ith m ed icolegal requ irements. Consid eration m u st be given to balancing the extraction of
■ Dia ne’s DPT and upper anterior occlusal radiograph are C w ith rem oval of the op p osite C to p revent a centreline
shown in Fig. 6.4. What are the eatures o note? shift. N ormally, follow ing extraction of C, clinical and rad io-
grap hic re-evalu ation should be u nd ertaken at 6-m onthly
• Four developing third molars. intervals. If no im p rovem ent in 3 position is observed on a
• Presence o 3 3, which are palatal. DPT w ithin 12 m onths, alternative treatm ent is requ ired .
• Resorption o the roots o C C. ■ How strong is the evidence to support extraction o C’s as
■ How may pa la tal ectopia o 3 be intercepted? an interceptive mea sure or palatally displaced canines?
Early d etection of an abnorm al erup tion path of 3 is essen- Clinical exp erience ind icates that there is evid ence to
tial in ord er to p rovid e, if ap p rop riate, an op p ortunity for supp ort this intervention. H ow ever, a Cochrane review con-
intercep tive m easu res to be u nd ertaken. If 2 is peg-shaped , clud ed that cu rrently there is no rm research base from
sm all or absent, then extra vigilance is requ ired from age 8 controlled clinical trials on w hich to base this p ractice.
• 40
6 PALATALCANINES
unerupted and displaced palatally); lower centreline shi t to
Key point the le t.
Removal o C’s between 10 and 13 years o age may Class I molar relationship bilaterally; lingual crossbite o 4
encourage improvement in the position o a palatally with 4; buccal crossbite o 6 with 6.
ectopic canine, although there is no f rm research base
or this. ■ What is the IOTN DHC grade (see p. 264)? Explain why.
5i – d u e to im p ed ed eruption (ow ing to palatal ectopia)
of 3’s.
When 3 d isp lacem ent is associated w ith crow d ing, elimi-
nation of crow d ing and sp ace m aintenance, if requ ired , m ay
stim u late 3 p osition to im p rove. Treatment
■ What mana gement options are there or Diane’s unerupted
Key point 3’s? What a re the indica tions or each option?

In planning treatment or a palatally ectopic canine, assess These are sum m arized in Table 6.1.
the ollowing on radiograph:
• The root length o C and root status o incisors. Key point
• The vertical and mesiodistal position relative to the Surgical exposure and orthodontic alignment o a palatal 3
incisor roots. requires a well-disposed patient with good oral hygiene
• The axial angulation. and dentition.
• The apex location.

■ Which option would you avour?


As Diane is a highly m otivated patient w ith a high stand ard
Diagnosis of general d ental care and the roots of C’s are resorbing,
■ Wha t is your diagnosis? w ith the 3’s in reasonably favou rable p ositions for ortho-
d ontic alignm ent, surgical exposu re of 3’s and orthod ontic
Class I malocclusion on a Class I skeletal base with average alignm ent w ou ld be op tim al.
FMPA.
4 ■ What a re the ideal aims o trea tment?
Lateral mandibular displacement on closure on .
4 Alignment o 3’s.
Marginal gingivitis related to 2’s and upper le t buccal
segment teeth; enamel demineralisation buccally on 6’s. Build up 2’s to increase mesiodistal width.

Mild lower labial segment crowding but spaced buccal Correction o crossbites on 4 6 .
segments; uncrowded upper arch with C’s retained (3’s Correction o lower centreline shi t.

Table 6.1 Management options, with indications, or palatally displaced unerupted 3’s*
Option Indications Comments
Earlyremoval o C’s See comments in the text in relation to interceptive treatment Not a viable option in this case as Diane is 15 years old
Retain 3 and observe Patient not keen or treatment Need to monitor radiographicallythe unerupted 3 or cystic degeneration and/or
Pathologyor resorption o adjacent teeth not evident root resorption o incisors

Good aesthetics/prognosis o C’s or 2 and 4 in good contact


3 severelydisplaced with no associated pathologyevident
Surgical exposure o 3’s and Highlymotivated patient with excellent general dental health Prognosis is good: the nearer 3 is to the occlusal plane, 3 overlaps at most the distal
orthodonticalignment Spaced arch or possible to create space; vertical, anteroposterior and hal o 1 root, when 3 long axis is ≥30° to the midsagittal plane, when root o 3 is
transverse position o 3 crown and root avourable not dilacerated or ankylosed or 3 apexis not more distal than 5. Bond gold chain,
bracket or magnet to 3 at surgery; alignment o 3 maycommence with removable
appliance but fxed appliance required to align 3 apex
Remove 3 Patient not keen or alignment o 3 and radiographic evidence o associated Prosthetic replacement o Crequired when lost
cystic degeneration
Hopeless prognosis or alignment o 3, 2 and 4 in good contact, or good
root length on Cwith good aesthetics, or patient willing to undergo fxed
appliance therapyto substitute 4 or 3. Earlyresorption o adjacent teeth
Transplant 3 Adequate space in arch or 3 Prognosis best i root o 3 is 50–75% ormed, minimal handling o 3 root at surgery,
Intact removal o 3 possible and rigid splinting is avoided

Adequate buccal/palatal bone

*Mayneed to address anyassociatedTSD.


41 •
PALATALCANINES 6
For treatm ent p lanning, Diane shou ld be seen by an rectangu lar stainless steel stabilizing archw ire (0.019 ×
orthod ontist, oral su rgeon and restorative colleagu e to 0.025-in stainless steel in an 0.022 × 0.028-in slot) can be
d iscu ss m anagem ent of 3’s and 2’s. Orthod ontic alignm ent p laced .
of 3’s, follow ing their su rgical exp osu re, w as agreed . Then arrange for surgical exp osu re of 3’s. If tem porary
Bu ild -u p of 2’s m esially w as to p reced e this. Mid -treatm ent, anchorage d evices (TADs), rather than a palatal arch, are
after 3’s w ere across the occlu sion, bu ild -u p of 2’s d istally u sed for anchorage, these could be p laced at the sam e time
w as p lanned . Diane, how ever, d ecid ed not to have 2 m esial as su rgical exp osu re of 3’s.
build -u p s, and the likely effect of this on the nal resu lt w as ■ Wha t methods o surgical exposure are there?
explained to her.
The need for low er centreline correction shou ld be Three m ethod s exist:
4 1. Open surgical exposure ollowed by spontaneous
reassessed follow ing crossbite correction on . eruption. 3 needs to be o correct angulation or this to
4
succeed.
■ How would you proceed with treatment?
2. Open surgical exposure o 3 with packing.
Create space for 3’s alignment. This w ill be obtained by m oving 3. Closed surgical exposure o 3 with attachment bonded
2 2 slightly m esially. As they are d istally angu lated , m esial
during surgery.
tip ping only is requ ired . These m ovem ents, as w ell as
With op en exp osure, the p alatal m u cosa overlying 3 is
palatal m ovem ent of 4 and bu ccal m ovem ent of 6, cou ld be
excised and a su rgical p ack is sutu red in p lace for 7–10 d ays.
accom p lished easily by u p p er rem ovable ap p liance therapy.
Follow ing rem oval of the pack, 3 can be allow ed to erup t
Alternatively, a xed ap pliance m ay be u sed for these
for usually u p to 3 m onths before bond ing an attachm ent to
m ovem ents.
comm ence traction. 3 is then aligned orthod ontically above
■ Detail the design o a suita ble removable applia nce. the m u cosa.
With closed exp osure, after u ncovering 3, an eyelet
Activation attachm ent w ith a gold chain is bond ed to either the bu ccal
Palatal nger springs (0.5 mm stainless steel wire to move 2’s or palatal asp ect of 3, d ep end ing on ease of access. 3 is
mesially). then m oved orthod ontically beneath the m u cosa into
Buccally approaching spring (0.7 mm stainless steel wire) with alignm ent.
‘u’loop to 4 . ■ Wha t is the evidence rega rding open versus closed
exposure o palata lly displaced 3’s?
Screw section to move 6 buccally.
A recent m u lticentre rand om ized clinical trial fou nd no d if-
Retention ference in p eriod ontal health of p alatally d isp laced canines
treated w ith an open or closed surgical techniqu e. Su rgical
Adams clasps 6 6 (0.7 mm stainless steel wire).
exposu re prod u ced a sm all aesthetic im pact, bu t this d id not
Southend clasp 1 1 (0.7 mm stainless steel wire). d iffer betw een op en or closed techniqu es.

Anchorage
From baseplate. Key point
Surgical exposure o 3, whether by open or closed
Baseplate technique, has no signif cant e ect on:
Full palatal acrylic coverage. • Periodontal health.
• Dental aesthetics.
Posterior bite plat orms ~2 mm in thickness to acilitate
crossbite correction on 4 6. The acrylic needs to be relieved
palatal and occlusal to 4 .

■ What instructions would you give the patient regarding ■ How may the 3’s be aligned?
turning o the screw?
Elastic traction m ay be ap p lied from the attachm ent bond ed
It should be turned one qu arter tu rn once per w eek (this is to 3’s to the archw ire (Fig. 6.5) or to a TAD. Light forces
~0.25 m m ). (20–60 g) shou ld be used . When m ovem ent of 3’s is evid ent,
■ When the crossbites on 4 6 ha ve been corrected, what C’s should be extracted . Once 3’s are close to the line of the
would you do? arch, a bracket shou ld be bond ed to the m id -bu ccal aspect
of each tooth. It is essential that the roots of 3’s are ad e-
Red u ce the posterior capping to half its height at one visit qu ately torqu ed to nalize their p ositioning.
and then rem ove it com p letely at the follow ing visit to allow
■ Wha t actors may you consider or reta ining 3’s in their
the p osterior occlusion to settle. It w ould then be ad visable
corrected positions?
to place an upp er xed appliance u nless this w as placed at
the outset for the initial m ovem ents outlined above. A trans- Asid e from fu ll correction of torqu e, early correction of rota-
palatal arch, attached to band s on 6’s, shou ld be cem ented tions shou ld be u nd ertaken, follow ed by circum ferential
for anchorage. Brackets should be bond ed to all other up per breotom y to 3’s and then the p rovision of a bond ed
teeth except 7C C7 and alignm ent continu ed u ntil a retainer.
• 42
6 PALATALCANINES

beyond the m esial aspect of 2 root, and the root ap ex


d oes not go beyond 5 root. 3 crow n lies in the m id to apical
third of 2, and the angulation of 3 to the m id sagittal p lane
is >30°.
■ What other investigations would you require or
treatment pla nning?
A lateral cephalogram is required to provid e fu rther infor-
m ation on the skeletal p attern and to assess m ore fu lly the
incisor angu lations.
Fig. 6.5 Mid-treatment view. ■ What is your interpretation o the ollowing
cephalometric f ndings?
SN A = 78°; SN B = 75°; AN B = 3°; SN -Max plane = 7°; MMPA
= 22°; 1 to m axillary plane = 99°; 1 to m and ibu lar plane =
CASE2 88°; interincisal angle = 151°.
Relative to Cau casian m ean values, SN A and SN B are
red uced bu t w ithin the norm al range. AN B ind icates a
SUMMARY Class 1 skeletal pattern, but ap plication of the Eastm an cor-
Paula, an 11-year-old girl, presented with 3 rection, as the SN to Maxillary plane angle, is w ithin the
unerupted. How will you manage the problem? range of 8° ± 3°, ind icating a m ild Class II skeletal pattern
■ The intraoral views at presentation are shown in Fig. 6.6 a nd (3° + 1.5° = 4.5°; 1.5° arises from half the d ifference of
the radiogra phs in Fig. 6.7. Describe what you see. SN A from the average SN A value of 81°). Com p ared w ith
Caucasian norm s, the MMPA angle is red u ced ; u p p er and
Generalized slight gingival erythema. low er incisor inclinations are d ecreased ind icating retrocli-
Caries- ree dentition. nation. The 1 incisor angle is m ore retroclined than it shou ld
be for the MMPA; the 1 incisor angle should be 98° (120° −
654321 12456
visible. 22°). The interincisal angle is increased ind icating a d eep
654321 123456 overbite.
Moderate lower arch crowding/moderate upper arch ■ What is your dia gnosis?
crowding with 3 erupting buccally; 2 2 mesiolabially
rotated. An 11-year-old girl w ith a Class II d ivision 2 m alocclu sion
on a m ild Class II skeletal base w ith d ecreased MMPA.
Class II division 2 malocclusion; deep and compete overbite;
Generalized m ild m arginal gingivitis. Mod erate u pper and
upper centreline shi t slightly to the le t.
low er arch crow d ing w ith 3 u neru p ted and p alatally d is-
Buccal segment relationship appears to be almost Class I placed . The up p er centreline is slightly to the left. Bu ccal
bilaterally; (both, however, were hal Class II clinically). segm ent relationship is m ild ly Class II bilaterally.
The dental panoramic tomogram and maxillary anterior ■ What is the IOTN (DHC) grade (see p. 264)? Expla in why.
occlusal radiograph show:
5i – d u e to the im p acted 3.
Normal alveolar bone height.
All teeth including third molars to be present.
Caries- ree dentition. Treatment
3 palatal; roots o 1 1 appear narrow. ■ What a re your aims o treatment?
■ Wha t risk actor is evident or 3 being pa la tal? Improve oral hygiene.
Paula has a Class II d ivision 2 m alocclu sion. Relieve upper and lower arch crowding.
■ Wha t a spects o Class II division 2 malocclusion ha ve been Reduce the overbite.
proposed as predisposing to this risk? Align the arches, including 3.
Class II d ivision 2 m alocclu sion has a strong genetic linkage Correct the upper centreline.
and is associated w ith m u ltip le d ental anom alies inclu d ing
Correct the molar and incisor relationships to Class I.
palatal 3. The anterior transverse w id th of the u pp er arch,
especially in the canine/ rst p rem olar area, is w id er in Retain and monitor third molars.
Class II d ivision 2 m alocclu sion than in other m alocclu sions Pau la and her m other w ere keen on avoid ing extraction
and afford s 3 greater scop e to w and er in the anterior m axilla. of any p erm anent teeth, if possible.
In ad d ition, d u e to their retroclination, the gu id ance to 3 ■ How would you relieve lower and upper arch crowding?
provid ed by the root of 2 is lacking.
In Class II d ivision 2 m alocclusion, a non-extraction
■ Is the position o 3 a vourable or orthodontic alignment?
approach is favou red (see Chapter 12). Fitting an u p per
The position of 3 is reasonably favou rable for alignm ent removable ap pliance w ith a at anterior bite plane w ou ld
(see Table 6.1). The crow n of 3 only extend s m arginally free the anterior occlu sion and allow the low er incisors to
43 •
PALATALCANINES 6

A
B

Fig. 6.6 (A) Anterior occlusion at presentation. (B) Right buccal occlusion. (C) Le t buccal occlusion. (D) Upper occlusal view. (E) Lower
occlusal view.

A B

Fig. 6.7 (A) Dental panoramic tomogram at presentation. (B) Upper anterior occlusal radiograph.
• 44
6 PALATALCANINES
p rocline unrestrained by the u p p er incisors. This w ill ■ What procedure would you pre er?
p rovid e som e space for relief of low er arch crow d ing, bu t
Cu rrent evid ence ind icates that it is a m atter of personal
the alignm ent of the teeth w ill require a xed appliance.
preference w hether to u se an op en or closed su rgical tech-
As a non-extraction approach is being ad opted in the
niqu e. In this case, after d iscu ssion betw een the orthod ontist
low er arch, a sim ilar strategy shou ld be consid ered for the
and oral su rgeon, open exp osure w as favoured .
u pper arch.
■ How may 3 be brought across the occlusion?
■ How could you create space or 3 a lignment?
When su f cient sp ace has been created for 3 and a m inim u m
This w ou ld involve d istalizing the u p p er rst p erm anent
of an 0.018-in stainless steel archw ire has been in place
m olars initially follow ed by retraction of the p rem olars to
(assu m ing an 0.022 × 0.028-in slot), orthod ontic traction m ay
op en sp ace for 3. This cou ld be accom p lished by head gear,
be ap p lied to 3 either by an au xillary w ire attached over the
TADs or head gear in com bination w ith a rem ovable (Ten
base archw ire or by elastic chain (Fig. 6.9A). The form er
H ove ap p liance; Fig. 6.8A) or xed ap p liance (Fig. 6.8B).
involves ‘p iggy-backing’ an 0.012-in or 0.014-in nickel–tita-
■ When should 3 be surgically exposed? nium archw ire on an 0.018-in stainless steel w ire, or one of
The typical treatm ent sequ ence w ou ld be to rstly create greater d im ensions.
space for 3 and then to organize for su rgical exposu re of It w ill also be necessary to d isengage the occlu sion to
3, follow ed by a 2–3-m onth p eriod to allow 3 eru p tion p rior allow 3 to be free of occlu sal interference d u ring m ovem ent
to the app lication of traction to align 3. H ow ever, in this across the low er arch. This can be achieved by one of the
case, as no extractions are p lanned for the u p per arch, it follow ing short-term m easu res: p rovision of a low er rem ov-
w ou ld be bene cial to have 3 su rgically exposed at the start able app liance w ith buccal cap ping of 2–3 m m to ‘clip over ’
of treatm ent so 3 can then eru p t w hile other asp ects of the the low er xed ap p liance; bond ing of glass ionom er cem ent
treatm ent are p rogressing. Op en or closed exp osure shou ld to the occlu sal su rfaces of the rst p erm anent m olars or
be d iscussed w ith the surgeon. bond ing of stainless steel ‘bite turbos’ to the palatal aspect
of the u p p er central incisors (Fig. 6.9B). The last option
given is the m ost favourable in this case as it d oes not com -
prom ise overbite red u ction.

Fig. 6.8 (A) Upper occlusal view showing space created by


distalization o 6’s by Ten Hove appliance and headgear. (Note: 3
erupting ollowing open surgical exposure). (B) Le t buccal B
view showing space or 3 then being created by retraction
o 4 with elastomeric chain as urther space is opened with Fig. 6.9 Upper occlusal views: (A) showing elastomeric traction to
nickel titanium coil spring. (Note anchorage was supported by 3 with bite turbos bonded palatal to 1 1; (B) with 3 moved
maintenance o headgear wear at night-time). across the occlusion.
45 •
PALATALCANINES 6
■ What are the adva ntages o nickel–titanium a rchwires ■ What type o retainer would you consider?
in 3 alignment?
A low er bond ed retainer from 3 to 3 and an u p p er bond ed
N ickel–titaniu m archw ires offer greater exibility and retainer from 2 to 2 should be placed in view of the planned
greater resistance to d eform ation than stainless steel arch- p roclination of the low er labial segm ent and the initial rota-
w ires. Fu rtherm ore, even if the archw ire is d e ected several tions on 2 and 2. This shou ld be supp orted by provision of
m illimetres, as in this case to engage the attachm ent on 3, an u pp er H aw ley retainer w ith a at anterior bite plane to
a light force is app lied w ithou t d eforming the w ire. be w orn at night only and a low er vacu um form ed retainer
also to be w orn at night only; both of these retainers should
t over the xed retainers.
The occlu sion follow ing alignm ent of 3 is show n in
Fig. 6.10.

Primary resources and


recommended reading
Flem ing PS, Sharm a PK, DiBiase AT 2010 H ow to … m echanically
eru p t a p alatal canine. J Orthod 37:262–271.
H id algo Rivas JA, H orner K, Thiru venkatachari B et al 2015
Develop m ent of a low -d ose p rotocol for cone beam CT
exam inations of the anterior m axilla in child ren. Br J Rad iol
1054:20150559.
H u sain J, Bu rd en D, McSherry P 2010 Managem ent of the
A p alatally ectop ic m axillary canine. Faculty of Dental Su rgery of
the Royal College of Su rgeons of England . Available at:
http :/ / w w w.rcseng.ac.u k/ fd s/ p u blications-clinicalgu id elines/
clinical_gu id elines/ ind ex.htm l.
Isaacson KG, Thom AR, Atack N E et al 2015 Orthod ontic
Rad iograp hs: Gu id elines, 4th ed . British Orthod ontic Society,
Lond on.
Kokich VG, Sp ear FM 1997 Guid elines for m anaging the
orthod ontic-restorative p atient. Sem in Orthod 3:3–20.
N oar J, Pabari S 2013 Cone beam com pu ter tom ograp hy – current
u nd erstand ing and evid ence for its orthod ontic app lications?
J Orthod 40:5–13.
Othm an SA, H arrad ine N W 2006 Tooth-size d iscrep ancy and
B Bolton’s ratios: a literatu re review. J Orthod 33:45–51.
Parkin N , Benson P 2011 Cu rrent id eas on the m anagem ent of
p alatally d isplaced canines. Fac Dent J 2:24–29.
Parkin N , Furness S, Shah A et al 2012 Extraction of p rim ary
(baby) teeth for u neru p ted p alatally d isp laced p erm anent canine
teeth in child ren. Cochrane Database Syst Rev Issue 12. Art.
N o.: CD004621, DOI:10.1002/ 14651858.CD004621.pu b3.
Parkin N A, Milner RS, Deery C et al 2013 Period ontal health of
p alatally d isp laced canines treated by op en or closed su rgical
techniqu e: a m u lticenter, rand om ized controlled trial. Am J
Orthod Dentofacial Orthop 144:175–184.
Parkin N A, Freem an JV, Deery C et al 2015 Esthetic ju d gem ents
of p alatally d isp laced canines 3 m onths postd ebond after
surgical exp osu re w ith either a closed or an op en technique.
C
Am J Orthod Dentofacial Orthop 147:173–181.
Fig. 6.10 Post treatment. (A) Right buccal occlusion. (B) Anterior
occlusion. (C) Le t buccal occlusion. For revision, see Mind Map 6, page 226.
7
654321 1
7654321 12 visible with 3 erupting mesial o 2 .
Uncrowded upper and lower arches: space mesial to 3 ; 2
bodily displaced lingually and tilted distally; 3 rotated
distobuccally; space between 11; Class I incisor relationship.
Molar relationship is slightly Class III; 3 in crossbite with 2 .

■ What do you notice in Fig. 7.2?

More canine problems Mild marginal gingival erythema related to most teeth, more
marked interproximally.
654C21 12
visible with 4 erupted orward o 3 and
654321 123
displaced palatally.
Moderate lower and upper arch crowding; mesiopalatal
rotation o 3 ; distopalatal rotation o 2 ; mesiolabial rotation o
1 . Class I incisor relationship.
Molar relationship is Class I (4 was in buccal crossbite
CASES1 AND2 with 4 ).

■ What is the term used to describe the anomaly in position


SUMMARY o the ca nine teeth? How common is this?
Two similar ca nine-related problems are shown. The term u sed is transposition (positional interchange of
What is the cause o each, and how may they be tw o ad jacent teeth or tooth d evelopm ent/ erup tion in a posi-
managed? tion normally occu pied by a non-ad jacent tooth). In the
■ Wha t do you notice in Fig. 7.1? general population, prevalence remains u nd er 1% bu t varies
accord ing to the sam ple investigated .
Fair oral hygiene with marginal gingival erythema related to
several teeth.

A A

B B

Fig. 7.1 (A) Case 1 at presentation: right buccal occlusion. (B) Case Fig. 7.2 (A) Case 2 at presentation: right buccal occlusion. (B) Case
1 at presentation: lower sectional occlusal view. 2 at presentation: upper sectional occlusal view.
47 •
MORECANINEPROBLEMS 7
■ Which arch and which teeth are a ected mostly? Is there a Table 7.1 Factors to consider in treatment planning or transposition
gender di erence in incidence? Factor(s) Reason(s)
Transp osition is m ore com m on in the u pp er arch, w here it Underlying malocclusion, acial These will in uence need or extraction(s)
m ost com m only affects the canine and rst p rem olar, fol- aesthetics, degree o crowding
low ed by the canine and lateral incisor. In the low er arch, it Stage o dental development When root development is complete, interception (by
seem s to affect exclu sively the canine and lateral incisor. The and position o root apices extraction o primaryteeth) is unlikelyto lead to
left sid e seem s to be favoured in the u pper arch and the spontaneous improvement in tooth position
right sid e in the low er arch. With complete transposition and root apices closed,
A fem ale p red ilection has been highlighted in som e acceptance o transposition maybe best due to the root
stud ies, w hereas others have ind icated either no d ifference resorption and periodontal risks (e.g. gingival recession,
in gend er p revalence or a m ale pred ilection. alveolar dehiscence) involved in correction
Dental morphology I transposition is to be maintained, reshaping is
■ What is the aetiology o this anomaly?
necessaryto disguise or incorrect tooth position
Althou gh several theories have been p rop osed – inter- Occlusal considerations Judicious grinding o the palatal cusp o a maxillary
change of d evelop ing tooth bu d s, altered eru p tion paths, frst premolar will be required where it is aligned in the
presence of retained p rim ary teeth, trau ma – the aetiology canine position
is now su ggested to be m u ltifactorial w ith involvement
of comp lex relationship s betw een genetic and environm en-
tal factors. There is evid ence of associated gend er pred ilec-
tion, hypod ontia, p eg-shap ed m axillary lateral incisors and
retained prim ary teeth.
Key point
Management options or transposition:
• Intercept.
• Accept/tooth reshaping.
Key point
• Extract most displaced tooth.
Transposition: • Align orthodontically: consider relative position o root
• In maxilla: more commonly a ects 3 and 4 than 3 and 2. apices.
• In mandible: a ects 3 and 2 almost exclusively.
• Prevalence: less than 1%.
• Aetiology: genetic and environmental.
■ How would you ma nage Ca se 1 and Case 2?
Requ est a p eriap ical rad iograp h of the transp osed teeth to
■ Could you classi y this anomaly? d eterm ine p osition of the root ap ices.
• Case 1: this indicated that the apex o 2 was slightly
The transposition may be partial or comp lete, the ap ices of ahead o the long axis o 3 ; root ormation was nearing
the affected teeth being transposed in the latter. completion. No root resorption was observed.
■ What actors would you consider in treatment? • Case 2: this indicated that hal the root length o C was
remaining; root apices o 4 were marginally ahead o 3
These are given in Table 7.1.
root; no root resorption was visible.
■ What are the trea tment options?
Treatm ent options are to align the transposed teeth in
These are as follow s: their transp osed position or to correct the transp ositions.
Interceptive treatment: i detected early (on average, between • Case 1: in view o the relation o the 2 apex to the 3
6 and 8 years o age), extraction o primary teeth may be root, it was decided to proceed with orthodontic
undertaken in an attempt to guide the transposed teeth to alignment, correcting the transposition.
their normal positions while ensuring that space is • Case 2: a periodontal specialist’s opinion deemed that as
maintained by either an upper removable appliance/palatal 4 was quite markedly palatal (almost in line with the
bar or lingual arch. This approach is only possible where the palatal cusp o 5 ), there was adequate alveolar bone to
teeth a ected are tilted so their roots are near the desired align 3 in its correct position without risk o gingival
positions (sometimes called pseudotransposition). recession/alveolar dehiscence. To encourage 3 to move
Acceptance: especially i transposition and root ormation are mesially, C was removed initially and an upper
complete, ollowed by reshaping o incisal/occlusal sur aces removable space maintainer was tted. It was decided to
and/or composite additions to camouf age or tooth position. commence treatment on a non-extraction basis and to
review need or extractions based on urther
Extraction o the most displaced tooth: this strategy has been
cephalometric evaluation o pro le and incisor
recommended where the arch is crowded or or caries;
inclinations when the arches were aligned.
appliance therapy may be required therea ter.
■ What appliance type will be required? Explain why.
Orthodontic alignment: whether the a ected teeth are aligned
in their transposed positions or whether these are corrected, Fixed ap p liance therap y is ind icated in both cases in view
depends on the relative position o the root apices. of the need for bod ily m ovem ent. These are: Case 1: the
• 48
7 MORECANINEPROBLEMS

Fig. 7.4 (A) Pre-treatment ollowing surgical exposure o 3. (B)


Following xed appliance alignment accepting the transposed
positions o 3 and 2.
B

Fig. 7.3 (A) Case 1 ollowing xed appliance alignment. (B) Case 2
ollowing xed appliance alignment.

CASE3
SUMMARY
positions of 3 and 2 to be corrected ; rotational correction of Adrienne, an 11.5-year-old girl, presents with
3 ; sp ace closure. Case 2: p ositions of 43 to be corrected ; mobile 2’s and C’s with 3’s unerupted and not
rotational correction of 321 ; correction of incisor relation- palpable buccally (Fig. 7.5). You order a dental
ship; space closu re. panoramic tomogram and upper anterior occlusal
In Case 1, the u pp er arch also required xed ap pliance radiograph.
therap y to close the m ed ian d iastem a. In Case 2, the low er
arch requ ired xed ap pliance alignm ent.
■ What do you notice in Fig. 7.5?

■ How would you check that the positions o the corrected Mild marginal gingival erythema.
transposed teeth are optimal? 654C21 12C456
Palpate the labial/ bu ccal su lci for root p osition of the cor- 654321 123456 .
rected teeth (Case 1: 3 and 2 ; ad d itional labial crow n torqu e Attrition o C’s.
m ay be requ ired in a rectangu lar steel or TMA archw ire to
Uncrowded upper and lower arches.
m axim ize root p ositions; Case 2: 4 and 3 ; ad d itional bu ccal
root torque m ay be required to 4 and palatal root torqu e to Class I (tending toward Class III) incisor relationship.
3 in rectangu lar steel or TMA w ires for m axim u m Molar relationship Class I bilaterally.
correction).
■ Why are radiographs requested?
Take a periapical radiograph to check root alignment and any
root resorption o the corrected teeth. To localize the position of the unerup ted 3’s and to d eter-
m ine the cau se of m obility of 2’s.
Check unctional occlusion, lateral and protrusive movements,
to ensure no occlusal inter erences. ■ What do you notice on the radiographs (Fig. 7.6)?
The occlusion ollowing xed appliance alignment or Case 1 Normal alveolar bone height.
and Case 2 is shown in Fig. 7.3A and B, respectively. All permanent teeth present except or third molars; all
An example o a case where the transposition o 3 and 2 was erupted permanent teeth appear to be caries- ree with
accepted is shown in Fig. 7.4. possible exception o 6.
49 •
MORECANINEPROBLEMS 7
■ Wha t is the most likely ca use o root resorption o the
incisors?
It is probably cau sed by a com bination of inherent pressure
d u e to m igration of the d isp laced , eru p ting canines and
their p hysical contact w ith the incisor roots.
■ Wha t is the incidence o root resorption o 2’s by ectopic
3’s? What sites a re most commonly a ected? Is there a
gender predilection?

A Depend ing on the m od ality of d iagnosis and pop ulation


samp led , an incid ence of betw een 12% and 68% has been
reported , w ith the apical and m id d le third s of the incisor
roots m ost com m only affected . It is m ore com m on in
fem ales.
■ How accura te is the in ormation rega rding resorption o 2’s
rom the ra diogra phs?
Due to su p erimp osition of the m alp ositioned canine, esp e-
cially w hen it is bu ccal or palatal to the incisor root, the true
extent of the inju ry may be obscu red . If, how ever, the angu -
lation of 3 to the m id line is greater than 25°, the risk of
B incisor resorp tion increases by 50%.
Even w ithout overlap ping teeth, intraoral rad iographs
Fig. 7.5 (A) Case 3 at presentation: right buccal occlusion. (B) Case m ake it d if cu lt to d etect resorp tion on the p alatal sid e.
3 at presentation: le t buccal occlusion.

Key point
Incisor resorption by an ectopic maxillary canine:
• Has an incidence o 12 68%.
• Is more common in emales.
• Risk increases by 50%, i angulation o 3 to the midline
is greater than 25°.

A ■ How may more detailed in ormation regarding 2’s


resorption be obtained?
Cone beam com p u ted tom ograp hy (CBCT; see p. 38) has
proved u seful.

Key point
Detection o incisor root resorption may be:
• Di cult on conventional radiographs.
• Facilitated by CBCT.

■ Wha t other investigations would you do in relation to 2’s?


B Sensibility tests and period ontal assessm ent (pocket d epths/
Fig. 7.6 (A) Case 3 at presentation: dental panoramic tomogram. bleed ing on probing/ attachm ent loss) should be u nd er-
(B) Case 3 at presentation: upper anterior occlusal radiograph. taken. It w ould also be w ise to enqu ire regard ing bru xing
habits.
Both 2’s w ere vital on electric p ulp testing; asid e from
m ild bleed ing on p robing consistent w ith slight m arginal
gingivitis, no period ontal p ocketing of >2 m m or loss of
Root resorption o C21 2C with less than hal the root length
attachm ent w as noted ; no bruxing habits w ere rep orted .
o 2 remaining; slight pipette root morphology o 1 and
apical curvatures developing on 5 45. ■ Wha t are the treatment options in rela tion to 2’s?
3’s palatally positioned (3 much more so than 3 by parallax; A ccept and monitor: this is not ad visable as the resorption of
see p. 7 and p. 38). 2’s is likely to w orsen d u e to p resence of 3’s im pinging on
• 50
7 MORECANINEPROBLEMS
their roots. Sw ift intervention is requ ired as the p rogression
of incisor resorp tion can be rap id .
Extraction o C2 2C: both 3’s may erupt spontaneously, or 3
may erupt and 3 may require surgical exposure. In view o
the lack o upper arch crowding, it would be di cult to then
close the upper labial segment spacing by xed appliance
therapy without creating a reverse overjet; opening space or
the lateral incisors, to be replaced with resin-retained bridges
or by implants at a later stage, would likely be a better option.
Extraction o C’s and surgical exposure o 3’s: orthodontic
alignment by xed appliance therapy would then be required;
lower arch xed appliance therapy may also be required to
detail the occlusion.
There is a risk of fu rther root resorp tion to 2’s by aligning
3’s; how ever, if this occu rs and p rognosis of 2’s is d eem ed
hopeless, retaining 2’s for as long as p ossible w ill p reserve Fig. 7.7 Case 3: upper anterior occlusal radiograph mid-treatment.
alveolar bone for possible im p lant p lacem ent later. Other-
w ise 2’s may be rep laced on resin-retained brid ges.
After d iscu ssion w ith Ad rienne and her m other, they
d ecid ed to p roceed w ith the last op tion. They w ere both
w arned in relation to the p ossible resorp tion risk to several
other teeth d u e to their root m orp hology.

Key point
Swi t intervention is required or incisor resorption by an
ectopic 3.

■ How would you minimize a nd monitor resorption o the


upper incisors during orthodontic treatment?
This is d ealt w ith on p age 120.
Fig. 7.8 Case 3: occlusal view ollowing surgical exposure o 3’s
■ What is the short- to medium-term prognosis o 2 with the
and during xed appliance alignment.
markedly resorbed root?
From the lim ited evid ence available in the literatu re, this
exposu re and xed ap pliance alignm ent of 3’s (Fig. 7.8).
shou ld be reasonable. In a Sw ed ish stu d y, even in cases of
There w as m inor fu rther resorp tion of the other u p p er
severe resorption, the incisor roots show ed good healing
incisors.
w hen assessed at a m ean tim e of 3.5 years (range 2–10 years)
after treatm ent w ith xed ap p liances. Su ch healing w as
observed in m ost cases after m anagem ent of the ectop ic Primary resources and
canine, either by surgical exp osu re and orthod ontic align- recommended reading
m ent or by su rgical rem oval. The resorbed incisors w ere Alqerban A, Jacobs R, Lam brechts P et al 2009 Root resorp tion of
incorporated in the orthod ontic ap pliance system , and the m axillary lateral incisor cau sed by im p acted canine: a
end od ontic treatm ent w as not ind icated to arrest further literatu re review. Clin Oral Investig 13:247–255.
root resorption. Bjerklin J, Bond em ark L 2008 Ectop ic m axillary canines and root
resorption of ad jacent incisors. Does com p u ted tom ograp hy
(CT) in u ence d ecision-m aking by orthod ontists? Sw ed Dent J
Key point 32:179–185.
Ciarlantini R, Melsen B 2007 Maxillary tooth transp osition: correct
Severely resorbed maxillary incisors:
or accep t? Am J Orthod Dentofacial Orthop 132:385–394.
• May heal a ter management o the associated ectopic
Ely N J, Sherriff M, Cobourne MT 2006 Dental transp osition as a
canine. d isord er of genetic origin. Eu r J Orthod 28:145–151.
• May be incorporated in an orthodontic appliance. Falahat B, Ericson S, Mak D’Am ico R et al 2008 Incisor root
• Do not require endodontic treatment to arrest urther resorp tion d u e to ectop ic m axillary canines: a long-term
root resorption. rad iograp hic follow -u p. Angle Orthod 78:778–785.
Peck S, Peck L 1995 Classi cation of m axillary tooth
transp ositions. Am J Orthod Dentofacial Orthop 107:505–517.
Rad iograp hic follow -u p d u ring treatm ent (Fig. 7.7)
show ed m inim al change in 2 root resorption after surgical For revision, see Mind Map 7, page 227.
8
inform ed . To red uce exposure, m ore frequ ent cleaning of
the su rgery w ith a p rotein w ash and changing of the air
lters is recom m end ed . Latex-free prod u cts shou ld be kept
separately in a screened area aw ay from latex prod u cts.
Em ergency d ru gs and resu scitation equ ip m ent shou ld also
be latex-free. Sp eci cs regard ing orthod ontic m anagem ent
are su m m arized in Ap pend ix 4.
Fortu nately, Aileen w as d eem ed not to requ ire any spe-
ci c p recau tions.

In raoccluded primary Dental history

molars
She is a regu lar attend er at the fam ily’s general d ental prac-
titioner. N o d ental treatm ent has been requ ired to d ate.

Family history
Aileen’s m other has several p erm anent teeth m issing, and
these have been replaced by brid gew ork.

SUMMARY Examination
Aileen is 11 years old. She is re erred by her general Extraoral examination
dental practitioner or in raoccluded lower primary
Aileen has a m ild Class II skeletal pattern w ith average
molars (Fig. 8.1). What is the cause, and how would FMPA and no facial asym m etry. The lip s are incom p etent
you treat it? w ith the low er lip lying at the incisal ed ges of the u pper
incisors. There are no tem p orom and ibular joint signs or
History sym ptom s.

Complaint Intraoral examination


Aileen is unconcerned by the p osition of her back teeth. Soft tissu es of the tongu e, oor of m outh, palate/ oropharynx
and the oral m u cosa are healthy. The intraoral view s are
History o complaint show n in Figs 8.1 and 8.2.
Aileen and her m other w ere u naw are of any problem w ith ■ What do you see?
her m olars u ntil this w as brou ght to their attention recently Plaque deposits on many teeth with associated marginal
by their general d ental p ractitioner. There is no d iscom fort gingival erythema.
associated w ith these teeth and they are not loose.
Dentition appears caries- ree; ssure sealants are present
occlusally in the rst permanent molars.
Medical history
6E4321 1234E6
Ap art from a possible latex allergy, Aileen is t and w ell.
■ What implications does this have or her mana gement? 6E4321 1234E6 erupted.
Uncrowded lower labial segment; E E in raoccluded;
Aileen shou ld be referred to a clinical im m u nologist, aller-
uncrowded upper arch.
gist or d erm atologist for testing. If a latex allergy is con-
rmed , the clinical team and rad iograp hers shou ld be Mild Class II division 1 incisor relationship (overjet is 4.5 mm
measured clinically); overbite slightly increased and
complete.
Lower centreline to the right.
First molar relationship: right hal unit Class II with 6E in
crossbite; le t Class I.

■ What is the prevalence o in ra occlusion o primary molars?


Betw een 1% and 9% of child ren seem to be affected , bu t
p revalence estimates vary.
■ What is the a etiology o in raocclusion o prima ry molars?
Is it linked to any other anomalies?
Cu rrent ep id em iological evid ence su ggests a genetic link
and an association w ith p alatally d isplaced canines (see
Chap ter 6), ectopic erup tion of rst perm anent m olars (see
Fig. 8.1 Lower occlusal view at presentation. Chap ter 1) and absent p rem olars.
• 52
8 INFRAOCCLUDEDPRIMARYMOLARS

Investigations
■ What investigations would you undertake? Explain why.

Clinical
Assess:
1. Mobility o –E’s i these are mobile, this tends to indicate
that they are close to ex oliation and that the permanent
successors are present.
2. Extent o in raocclusion o –E’s i these teeth are in
danger o submerging below gingival level, their removal
A is indicated.
3. I –E’s are ankylosed typically a ‘tin-can’sound is audible
when the occlusal sur ace is percussed with the stainless
steel handle end o a dental mirror and the sound
compared with that obtained rom percussion o
adjacent ully erupted teeth.
4. Overeruption o opposing teeth this could lead to
inter erences in unctional occlusion and present
di culties i prosthetic replacement o –E’s spaces is
required in the absence o 5’s.

B
Key point

With in raoccluded E’s, assess:

• Mobility o E’s.
• Extent o in raocclusion.

• I E’s are ankylosed.
• Overeruption o opposing teeth.
• I 5’s are present.

C
Radiographic
1. A dental panoramic tomogram to determine i
unerupted teeth are present, in normal developmental
position and o normal orm and size.
2. A lateral cephalometric radiograph may be required later
i xed appliance therapy is planned and the patient is
keen to proceed. It would allow more accurate
determination o the skeletal pattern in the
D
anteroposterior and vertical dimensions and or the
Fig. 8.2 (A) Upper occlusal view. (B) Right buccal occlusion. incisor inclinations to be assessed.

(C) Anterior occlusion. (D) Le t buccal occlusion. Both E’s w ere fou nd to be non-m obile and w ere not
infraocclu d ed below gingival level, bu t clinically both w ere
ankylosed .
■ Why does in raocclusion o prima ry molars occur?
■ The dental panoramic tomogram is shown in Fig. 8.3. What
Separate phases of resorption and repair occu r in the exfolia- are the f ndings o note?
tion of p rim ary teeth. Althou gh resorp tion p red ominates in
m ost cases, som etim es rep air p revails tem p orarily lead ing to • Dental development corresponds with chronological age.
ankylosis of a p rim ary m olar. As alveolar grow th and erup- • Extensive resorption o the roots o E’s; short roots on –E’s.
tion of the ad jacent teeth continu e, the tooth infraocclu d es. • Absent 5’s and all third molars.
• Absence o periodontal ligament space related to E E.
Key point ■ What is the prevalence o hypodontia in the permanent
In raocclusion o a primary molar is due to ankylosis o the dentition? Which teeth and gender does hypodontia a ect
tooth while alveolar growth and eruption o the adjacent most commonly?
teeth continues. A recent system atic review found the prevalence of hypo-
d ontia in the perm anent d entition to be around 6% w ith a
53 •
INFRAOCCLUDEDPRIMARYMOLARS 8

Fig. 8.4 Fixed appliances.

Fig. 8.3 Dental panoramic tomogram. Treatment


■ Wha t treatment options are there or the lower arch?
signi cant d ifference betw een continents; it is highest in Expla in why.
Africa (~13%) and low est in Latin Am erica (~4%), w ith a In view of the lack of crow d ing:
prevalence in Eu rop e of 7%. In Cau casians, third m olars are 1. Accept the position and status o E E, realizing their poor
m ost com m only affected (20–30%) follow ed by 5 (3%), then long-term prognosis due to the short root length, but
2 (2%) and 5 (less than 2%). Fem ales are affected m ore than build up E E with occlusal onlays in composite to bring
m ales, and tooth size in the rem aind er of the d entition tend s them into occlusion. This procedure has been shown to
to be red uced . improve longevity o in raoccluded molars. Maintaining
E E rather than extracting them also preserves alveolar
Key point bone. When eventually they are lost, resin-bonded or
conventional bridgework or implants may be used to
Hypodontia: replace the missing units. Aileen and her mother would
• Prevalence: ~6% in permanent dentition and di ers need to be aware o the implications o this treatment
signif cantly by continent. proposal over the li etime o the dentition, including the
• Frequency: 8’s, then 5, 2, 5. need or replacement o any prosthesis as required.
• Females more than males. 2. Extract E E in view o their poor long-term prognosis and
as in raocclusion is likely to progress with the absence o
5’s. Then, close the extraction spaces with a lower xed
Follow ing p erusal of the p anoram ic tom ogram and pre- appliance. This has the advantage o removing the need
liminary d iscussion of treatm ent options w ith Aileen and or a prosthesis, but a retainer would need to be worn
her m other, a lateral cep halom etric rad iograp h w as taken. post-treatment or several years at night to minimize the
Analysis revealed the follow ing: likelihood o space opening. Alternatively bonded
SN A = 82°; SN B = 76.5°; AN B = 5.5°; 1 to m axillary p lane retainers could be placed on the buccal aspects o
= 112°; 1 to m and ibular plane = 92°; MMPA = 26°; facial 6 4 4 6 to maintain space closure.
prop ortion = 55%. ■ Wha t implications do these options have or the upper arch?
■ What do these values tell you (see p. 270)?
I –E’s are retained, the slight overjet increase could be
They con rm the clinical im p ression of a m ild Class II skel- accepted as the teeth are aligned, provided the patient is in
etal p attern w ith average FMPA. Incisor inclinations to their agreement.
u nd erlying d ental bases are also w ithin the norm al range. I –E’s are to be extracted and a lower xed appliance planned,
it would be sensible to resort to an upper premolar extraction
Diagnosis on either side in the upper arch (probably 5’s in view o the
small overjet and absence o crowding, although it will be
■ What is your diagnosis? necessary to await their eruption) and proceed to xed,
appliance therapy to achieve Class I molar and incisor
Mild Class II division 1 malocclusion on a mild Class II skeletal
relationships.
base with average FMPA.
Following several visits to the hygienist, Aileen’s oral hygiene
Generalized marginal gingivitis, uncrowded lower arch with
–’s. improved, and having considered all options, she decided to
in raoccluded E
proceed with xed appliance therapy (Fig. 8.4).
Uncrowded upper arch. First molar relationship right hal unit
Class II with 6E in crossbite; le t Class I. ■ What type o f xed a ppliance is shown in Fig. 8.4? Wha t
means a re there to close premolar extraction spaces with
Hypodontia o 5’s and third molars.
this a ppliance type? What method is most e ective?
■ What is the IOTN (DHC) gra de (see p. 264)? Explain why.
This is a p re-ad ju sted ed gew ise ap pliance. Space closing
4h – d ue to absent 5’s . slid ing mechanics w ith this app liance m ay be u nd ertaken
• 54
8 INFRAOCCLUDEDPRIMARYMOLARS

Fig. 8.5 Another case with Ni-Ti coils used or the closure o
premolar extraction spaces.

in each qu ad rant by one of the follow ing m ethod s, each of


w hich is attached from the rst m olar band hook to the hook
on the canine bracket or to a sold ered hook on the archw ire
(so-called p osted archw ire): B
• Polyurethane powerchain narrow spaced polyurethane
powerchain is stretched to about double its resting
length.
• Active ligatures a grey elastic module is stretched by a
ligature to double its resting length (otherwise known as
a Berman ligature).
• Nickel-titanium (NiTi) springs attached as or the active
ligatures (Fig. 8.5).
A force of 100–200 g has been recomm end ed . Class II
elastics m ay also be u sed to assist w ith closure of upper and C
low er p rem olar extraction sp aces.
Fig. 8.6 (A) Post-treatment: right buccal occlusion.
A rand om ized clinical trial com p ared the three op tions
(B) Post-treatment: anterior occlusion. (C) Post-treatment: le t
listed above. The m ost rap id rate of sp ace closure w as
buccal occlusion.
achieved w ith N iTi sp rings and w as consid ered the treat-
m ent of choice. Elastic chain, how ever, w as as effective and
is cheaper. The ad d ition of Class II elastics d id not seem to
affect the rate of sp ace closu re.

Primary resources and
The occlu sion follow ing rem oval of E E, then 5 5 and E’s
recommended reading
and xed app liance therap y is show n in Fig. 8.6.
Bjerklin K, Al-N ajjar M, Karested t H et al 2008 Agenesis of
■ I 5’s had been present radiographically, what would have m and ibu lar second p rem olars w ith retained p rim ary m olars: a
been your treatment plan? longitud inal rad iograp hic stu d y of 99 su bjects from 12 years of
– age to ad ulthood . Eu r J Orthod 30:254–261.
Ankylosis of E’s is likely to be tem porary w hen perm anent
– Dixon V, Read MJF, O’Brien KD et al 2002 A rand om ized clinical
successors exist, and E’s should exfoliate w ithin a norm al
– trial to com p are three m ethod s of orthod ontic sp ace closu re.
time fram e. The p osition of E’s should be m onitored until J Orthod 29:31–36.
then, and if the infraocclu sion p rogresses, extraction is rec-
– H u d son AP, H arris AM, Morkel JA et al 2007 Infraocclu sion of
omm end ed , particu larly if the crow n of E m oves to lie p rim ary m olars: a review of the literatu re. SADJ 62:114,
below gingival level (reinclu sion) and / or apical closu re is 116,118–122.
alm ost com plete on 5. Khalaf K, Miskelly J, Voge E et al 2014 Prevalence of hyp od ontia
and associated factors: a system atic review and m eta-analysis.
J Orthod 41:299–316.
Key point Ku rol J, Koch G 1985 The effect of extraction of infraocclu d ed
Management options or in raoccluded Ē: d ecid uou s m olars: a longitu d inal stu d y. Am J Orthod 87:46–55.

• 5 present, no reinclusion: allow Ē to ex oliate. Patel A, Burd en DJ, Sand ler J 2009 Med ical d isord ers and
orthod ontics. J Orthod 36:1–21.
• 5 present, and reinclusion: extract or surgically
remove Ē.
• 5 absent: retain and place onlay: For revision, see Mind Map 8, page 228.
• extract and space close.
• extract and prosthetic replacement.
9
■ How common is bullying among school children? What
in uence do prominent teeth have, and does bullying have
any long-term consequences?
Using a valid ated m easu re, the prevalence of bullying
among 8–18-year-old s in 11 European cou ntries w as found
to be 20.6%. In a recent UK stu d y, nearly 13% of child ren
aged 10–14 w ho w ere assessed for orthod ontic treatment
had been bullied d u e to their m alocclu sion, and bu llying

Increased overjet w as signi cantly associated w ith an increased overjet. The


negative im pact on self-esteem and oral health–related
qu ality of life w as also rep orted by those w ho had been
bu llied . Long-term effects of bu llying into ad u lthood have
also been review ed and have ind icated im p acts on physical
and m ental w ell-being.
■ Wha t is the risk o trauma with an increa sed overjet?
For an overjet of 3–4 m m , the risk is d oubled and increases
for an overjet of 5–7 m m .
■ Wha t is the signif cance o the history o teasing and
SUMMARY incisor trauma?
Emma, aged 11, is teased at school about her
It w ou ld be im portant to ascertain the intensity of the teasing
prominent upper ront teeth (Fig. 9.1). What are the and w hether Em m a is bu llied at school abou t her teeth. If
possible causes, and how may it be treated? so, it w ould be p ru d ent not to d elay treatm ent especially
as 1 1 have also suffered trau m a. There is a risk of teeth
History w ith a history of trau m a becom ing non-vital in the fu ture.
This shou ld be assessed before, and m onitored d u ring,
Complaint any orthod ontic treatm ent. Traum atized teeth also have a
Em m a’s u pp er front teeth stick out. H er mother is very greater risk of root resorption d u ring orthod ontic treatm ent.
concerned abou t her d au ghter ’s ap p earance and is anxiou s Althou gh su ggested as a risk factor for orthod ontically-
for her to be treated . ind uced tooth reabsorp tion, p reviou s trau m a seem s an
u nlikely cau se (see p . 120). Em m a and her m other should
be ad vised accord ingly as part of inform ed consent prior
History o complaint
to treatm ent.
The u p per front teeth have alw ays been p rom inent, even
w hen the p rim ary incisors w ere p resent. Em m a is teased
Medical history
abou t her teeth at school, and the teasing is u p setting her.
She recently fell in the school yard and hit her tw o up per Em m a has had asthm a since she w as 4 years old . This is
front teeth on the grou nd . Fortu nately there w as only m anaged w ith a salbu tam ol inhaler (Ventolin).
m inimal incisal enam el d amage to 1 1.
■ Is teasing the same as bullying?
Teasing is som etim es confu sed w ith bu llying, bu t they are Key point
not the sam e. Teasing has been d escribed as am bigu ou s An increased overjet increases the risk o incisor trauma
social exchange that m ay be friend ly, neu tral or negative, and may predispose a child to teasing.
w hereas bu llying is d em arcated by rep eated aggressive
behaviou r or intentional harm over tim e and is character-
ized by an im balance of p ow er.

Examination
Extraoral
Em m a’s fu ll facial and p ro le view s are show n in Fig. 9.2.
■ How would you assess Emma’s skeletal pattern?
The skeletal pattern is the relationship of the m and ibular to
the m axillary d ental base in all three p lanes of sp ace – anter-
oposterior, vertical and lateral. Assessm ent shou ld be mad e
w ith Em ma seated u p right in the natu ral head position (the
p osition in w hich the head is supp orted naturally w hen
Fig. 9.1 Right buccal occlusion at presentation. looking straight ahead at a d istant object); the lip s shou ld
• 56
9 INCREASEDOVERJET
ront o chin) acial thirds; this may be increased
(Class II; convex pro le), average (12° + 4°; Class I) or
reduced (Class III; concave).
Emma has a Class II skeletal pattern.
2. Vertical.
Lower acial height. The distance rom the mid-eyebrow
level to the base o the nose (upper acial height) should
equal that rom the base o the nose to the in erior aspect
o the chin (lower acial height). The lower acial height is
reduced when the latter measurement is reduced, and
the converse is true when this distance is increased.
Frank ort-mandibular planes angle (FMPA). With a nger
along the lower border o the mandible and a ruler placed
along the Frank ort plane (lower border o the orbit to the
superior aspect o the external auditory meatus), project
both o these lines backwards in the imagination to
A estimate the FMPA. The FMPA is then classi ed as average
(both lines intersect at the back o the skull, occiput),
reduced (both lines meet beyond occiput) or increased
(both lines meet anterior to occiput).
Emma has a slightly reduced lower acial height and
FMPA.
3. Transverse. Stand directly behind the patient and look
down across the ace, checking the coincidence o the
midlines o the nose, upper and lower lips and midpoint
o the chin. Alternatively assess the ace rom the ront. It
is important to note that slight acial asymmetry is
common. The location (upper, middle or lower acial third)
and extent o any asymmetry should be recorded. Emma’s
chin point is marginally to the right. As this is very mild
and has not been noticed by her or her mother be ore
now, and as a slight degree o acial asymmetry is
regarded as normal, there is no cause or concern.
No mandibular deviation on closure or
B temporomandibular signs/symptoms were detected.
The lips are habitually competent with the lower
Fig. 9.2 (A) Full ace view. (B) Pro le.
lip tending to lie under the upper incisors at rest
(Fig. 9.2B).
be at rest and the teeth in m axim u m interd igitation. Assess-
m ent shou ld be as follow s: Intraoral
1. Anteroposterior. Viewing the so t tissue acial pro le in
most cases allows the ollowing classi cation to be made: ■ The intra oral views are shown in Figs 9.1 a nd 9.3. Wha t do
these show?
Class I: the mandible lies 2 3 mm behind the maxilla.
Class II: the mandible lies more than 2 3 mm behind the There are plaque deposits on several teeth and overall mild
maxilla. marginal gingival erythema.

Class III: the mandible lies less than 2 3 mm behind the All teeth appear to be o good quality.
maxilla. Emma is in the permanent dentition with
Due to variation in lip thickness, this method is not 654321123456
present. (Note 7 7 are erupting.)
always reliable and palpation o the alveolar bases over 654321123456
the apices o the upper and lower incisors in the midline The upper and lower arches are uncrowded.
has been claimed to give a better estimate o skeletal There is a Class II division 1 incisor relationship with increased
pattern. overjet (measured 7 mm clinically); the overbite is increased
Two other assessments may also be made: and complete. The buccal segment relationship is a hal -unit
1. The relationship o the lips to zero meridian (a true Class II bilaterally. There is a lingual crossbite (scissors bite)
vertical rom so t tissue nasion): the upper lip should a ecting 4.
be on, or slightly ahead, and the chin just behind.
■ What a re the causes o an increa sed overjet?
2. The angle o acial convexity o the middle (mid-
eyebrow to base o nose) to lower (base o nose to These are given in Table 9.1.
57 •
INCREASEDOVERJET 9
Investigations
■ What radiographs are indica ted?
A d ental p anoram ic tom ogram is requ ired to check the pres-
ence, p osition, d evelop m ental stage and any abnorm alities
of crow n and root of uneru p ted teeth. Untreated caries
should also be noted and bitew ing rad iographs requ ested ,
if necessary. In view of the history of trau m a to the u pper
incisor area, a p eriap ical view or an u pp er anterior occlu sal
rad iograp h shou ld be taken and exam ined for p ossible
ap ical p athology.
A A lateral cephalom etric rad iograph is ind icated as there
is an anteroposterior and a vertical skeletal d iscrep ancy. In
ad d ition, anterop osterior m ovem ent of the incisors is
p lanned .
The nd ings of the cep halom etric analysis are:
SN A = 82°; SN B = 76°; SN to m axillary p lane = 9°; MMPA
= 22°; 1 nomenclatu re to m axillary p lane = 114°; 1 to m an-
d ibu lar p lane = 92°; facial p roportion = 52%.
■ Wha t do these indicate (see p. 270)?
ANB value o 6° (SNA minus SNB) indicates a Class II skeletal
pattern.
Reduced MMPA and acial proportion.

B
Relative to m ean Cau casian valu es, the upp er incisors are
proclined (bu t w ithin the norm al range) and the low er inci-
Fig. 9.3 (A) Anterior occlusion. (B) Le t buccal occlusion. sors are slightly retroclined . Although w ithin the norm al
range the 1 to m and ibu lar p lane m ust be consid ered w ith
the MMPA as there is an inverse relationship betw een the
tw o valu es. 1 to m and ibu lar p lane (93°) and MMPA (27°)
should total 120° or, alternatively, 1 to m and ibu lar p lane
angle shou ld be 120° – MMPA. H ence in this case, the 1 to
Table 9.1 Causes o an increased overjet m and ibu lar p lane angle shou ld be 120° – 22° = 98°. At 92°,
Cause Aetiology it is retroclined .
Skeletal pattern Maybe Class I, IIor III ■ Wha t other important in ormation rega rding growth
potential may be obtained rom the lateral cephalometric
I Class II, mandibular defciencyis almost entirely
the primarycause but maybe excessive horizontal f lm? How is this assessed?
maxillarygrowth or a combination o the two actors It may also be u sed to d eterm ine skeletal m atu rity. This is
So t tissues* Lower lip lying under the upper incisors to create an assessed by the cervical vertebral m atu ration (CVM) ind ex
anterior oral seal will procline the upper incisors and w hich has ve stages, each w ith m orphological changes in
retrocline the lower incisors (likelyi there is a Class II the second , third and fourth cervical vertebrae. The peak in
skeletal pattern, reduced lower acial height and lip m and ibu lar grow th occu rs betw een CVM stage II and CVM
incompetence)
stage III; CVM stage V occurs 2 years after the p eak.
Hyperactive lower lip will retrocline the lower incisors
■ How valid and reliable is the CVM index?
Primaryatypical swallowing pattern (endogenous
tongue thrust) will tend to procline upper (but also CVM has been show n to have high valid ity and
lower) incisors reprod ucibility.
Digit sucking habit I present or more than 6 hours out o 24, it will ■ Would you consider a ny other investigations?
procline upper incisors, retrocline lower incisors,
It w ou ld be w ise to d o sensibility tests of 1 1. These proved
create an anterior open bite and a tendencyto buccal
segment crossbite
p ositive for all tests, w ith no m arked d ifference in record -
ings betw een teeth.
Overjet increase is o ten asymmetrical due to digit
positioning
Crowding Labial displacement o upper incisors and/or lingual
displacement o lower incisors Diagnosis
Anycombinationo the above
■ Wha t is the diagnosis?
*E ects determined principallybythe skeletal pattern, and therea ter bythe manner in
Emma has a Class II division 1 malocclusion on a mild Class II
which an anterior oral seal is produced.
skeletal base with reduced FMPA.
• 58
9 INCREASEDOVERJET
There is generalized marginal gingivitis. gests that earlier treatm ent red u ces the incid ence of incisor
1 1 have su ered recent trauma. trau m a bu t does not offer any other ad vantages com p ared
w ith treatm ent in early ad olescence. Em m a has recently
There is no crowding o the upper and lower arches.
su ffered u p per incisor traum a, and starting treatm ent earlier
The buccal segment relationship is a hal -unit Class II m ay have averted this; how ever, she is still w ithin the scope
bilaterally with a lingual crossbite o 4. of the ages d eem ed ‘early treatment’. Although this review
rep orted no p sychosocial bene ts of early or later treatm ent,
■ Wha t is the IOTN (DHC) grade (see p. 264)? Explain why.
recent longitu d inal d ata from a UK stu d y on bu llied chil-
4a – d u e to overjet >6 m m bu t ≤9 m m. d ren fou nd that intercep tive treatm ent for an increased
overjet red u ced the p revalence of bu llying and im p roved
oral health–related qu ality of life.
Treatment
■ What types o unctional appliances are there? Which is the
■ Wha t actors other than increased overjet predispose to most popular?
upper incisor trauma?
Fu nctional app liances m ay be classi ed as tooth-borne (e.g.
The risk is doubled where the overjet exceeds 9 mm. Tw in-Block appliance, m ed iu m op ening activator, H erbst)
Lip incompetence due to the absence o lip protection. or soft tissu e borne (e.g. Frankel). The H erbst is a xed
Gender o the patient boys experience more upper incisor fu nctional ap pliance; all others are rem ovable, although the
trauma than girls. Tw in-Block appliance m ay also be cem ented in p lace. In
the UK, the Tw in-Block app liance is m ost p op u lar, bu t the
■ What are the aims o treatment? H erbst ap p liance is favou red in N orth Am erica.
To reduce the overbite and overjet to establish a Class I ■ Describe the records you would take to allow abrication o
incisor relationship. a Twin-Block a ppliance?
To correct the buccal segment relationship to Class I. The record s required are w ell-extend ed u pper and low er
4 alginate im p ressions as w ell as a w ax registration taken
To correct the crossbite on .
4 w ith the m and ible postu red forw ard , usu ally to an ed ge-to-
ed ge incisor relationship , the bite op en abou t 8 m m in the
■ Wha t treatment would you advise? Explain why. p rem olar areas w ith no appreciable shift in the u pper and
Em m a’s m alocclu sion shou ld be am enable to correction by low er d ental m id lines. This ‘w orking bite’ may be record ed
grow th m od i cation w ith fu nctional ap pliance therap y. by softening several layers of w ax in w arm w ater, form ing
Favou rable features are that Em m a is likely to be grow ing this to a horseshoe shape ind exed rm ly over the u pper
and is ap p roaching the p u bertal grow th sp u rt. The skeletal teeth and nally gu id ing the m and ible to the correct antero-
pattern is m ild ly Class II d u e to m and ibu lar retrusion rather p osterior, vertical and lateral position by checking the
than m axillary protru sion. The arches are u ncrow d ed and relationship of the centrelines and the incisal op ening. Alter-
aligned ; the low er incisors are slightly retroclined ; the natively, layers of w ax m ay be ad apted to a proprietary
bu ccal segm ent relationship is a half-u nit Class II, so a bite registration fork, w hich has grad u ated m arkings to
m od est shift of the arch relationship is requ ired for it to be facilitate assessm ent of the p ostu red m and ibu lar position.
corrected to Class I. The w ax registration shou ld then be chilled , exam ined for
Fu nctional ap pliances are u su ally contraind icated w here ad equ ate d ental registration and re-checked for accuracy in
the low er incisors are p roclined , as they ind u ce fu rther the m outh before forw ard ing w ith the im pressions to the
proclination throu gh generation of Class II interm axillary laboratory. Where the overjet is large, the m and ible m ay be
traction. Follow ing fu nctional ap p liance therap y, xed ad vanced to 70–80% of m axim um protrusion to facilitate
appliances m ay be requ ired to d etail the occlu sion. It w ou ld p atient com fort.
be ad visable then to retain the resu lt by night-only w ear of ■ On issuing the Twin-Block appliance, what instructions
a functional ap pliance u ntil grow th is com p lete. would you give Emma ?
The instructions w ould be as follow s:
Key point • The appliance should be worn ull-time, including at
mealtimes, rom insertion. The only times it is removed
A unctional appliance:
are a ter meals or cleaning and or contact sports, during
• Aims to ‘modi y’ growth.
which times it should be stored in the hard plastic tub
• Is only e ective in growing children, pre erably just provided.
prepubertal.
• Speaking and eating will be di cult or the rst ew days
but will improve i you persevere.
• Avoid eating hard or sticky oods or consuming zzy
■ Should Emma have been treated earlier? What evidence is drinks while wearing the appliance as these are likely to
available regarding this? damage the appliance and/or your teeth. The appliance
A recent Cochrane review com p ared the effects of orthod on- and the teeth should be cleaned thoroughly a ter every
tic treatm ent for child ren w ith prom inent upp er front teeth meal.
w hen treatm ent is started betw een 7 and 11 years com p ared • Mild jaw discom ort and muscle tenderness are common
w ith treatm ent started in early ad olescence. Evid ence su g- or the rst ew days but reduce a ter that. It may be
59 •
INCREASEDOVERJET 9
necessary to take a mild analgesic, as required, during Dentoalveolar (~70–80%)
this ‘settling-in’period. Retroclination o upper incisors/proclination o lower incisors.
• Should a sore spot develop or there be any breakage o
Promotion o mesial and upward eruption o lower posterior
the appliance, you should contact us immediately by
teeth (see below).
telephone to arrange an appointment to have any
adjustments carried out. Distal movement o the upper molars.

■ How does a Twin-Block work a nd wha t e ects does it Upper arch expansion.
produce?
The Tw in-Block ap p liance consists of u p p er and low er
Key point
ap pliances incorporating bu ccal blocks w ith interfacing
inclined p lanes (at abou t 70°), w hich p ostu re the m and ible A unctional appliance or Class II correction:
forw ard on closu re (Fig. 9.4). This ap p liance w orks by u sing • Postures the mandible downward and orward.
the forces generated by the orofacial m u scu lature, tooth • Generates intermaxillary traction.
erup tion and d entofacial grow th. The u p p er m id line expan- • Uses, removes or modif es orces o the oro acial
sion screw is u su ally ad ju sted once p er w eek by the p atient musculature, tooth eruption and dento acial growth.
until the arch w id ths are coord inated w ith the m and ible
postu red forw ard in a Class I incisor relationship . In this
case, no exp ansion w as requ ired in view of the scissors bite
4 ■ How do the e ects produced by a Twin-Block a ppliance
on . The effects are u su ally as follow s:
4 di er rom those o other unctional applia nces?

Skeletal (~20–30%) The recent Cochrane review also assessed the effect of
Forward growth o the mandible. orthod ontic treatm ent for p rom inent u p per front teeth
w hen u nd ertaken by d ifferent orthod ontic app liances.
Increase in lower anterior acial height.
Fu nctional app liance treatm ent, irresp ective of type, in early
ad olescence app ears to prod u ce som e m inor bene cial skel-
etal changes. When the Tw in-Block w as com pared w ith
other ap pliances, there w as no d ifference in overjet, bu t the
Tw in-Block p rod u ced a statistically signi cant greater
red u ction in AN B, althou gh this w as sm all. There w ere no
ad vantageou s effects of other fu nctional app liances com -
pared w ith the Tw in-Block.
■ Following overjet correction by Twin-Block therapy, wha t
occlusa l anomaly is usually mani est posteriorly in the
dental arches?
A posterior op en bite is u su ally p resent bilaterally d u e to
the buccal blocks.
■ How may this be corrected?
There are three possible m eans available to allow for correc-
A tion of the posterior open bite by eru ption of the bu ccal
segm ent teeth:
• The patient may be instructed to proceed to part-time
wear o the appliance.
• The Adams’clasps on the lower molars may be removed
initially and then acrylic trimmed progressively over a
period o a ew months, rom the undersur ace o the
lower block and the lower sur ace o the upper block,
until a posterior occlusion is established.
• Wear o the Twin-Block appliance may be ceased and the
patient tted with an upper Hawley retainer (Adams
clasps 0.7 mm on 6 6, labial bow 0.7 mm 3 to 3) with a
‘steep and deep’anterior inclined biteplane, which aims
to maintain overjet correction by posturing the mandible
orward while encouraging eruption o the lower buccal
segment teeth. Full-time wear o the appliance is
B required, except or contact sports and teeth cleaning,
until a well-interdigitating posterior occlusion is
Fig. 9.4 Twin-block appliance. (A) Upper occlusal view. (B) Lower established. Then night-only wear o the appliance, until
occlusal view. The design is modi ed rom the original growth has ceased or until a second phase o treatment
developed by Clark. commences, is permissible.
• 60
9 INCREASEDOVERJET

■ I there is no progress at 6 months, what action would interincisal angle is created. A use ul rule o thumb with
you take? tipping movement is that each millimetre o upper incisor
retraction approximates to a 2.5° change in angulation. With
Lack of overjet correction cou ld be d u e to p oor p atient
an original overjet o 7 mm and a target overjet o 3 mm
response to the ap pliance, im p rop er d esign or p oor
(representing a 4 mm reduction), this would equate to a 10°
comp liance. Treatm ent shou ld be d iscontinu ed and a re-
change producing a nal incisor angulation o 104°. This value
evalu ation m ad e. The p atient’s stand ing height shou ld be
is just within the normal range (109° ± 6°), and the incisors
record ed and com pared w ith the pre-treatm ent m easure-
would be quite upright. An upper incisor angulation o 95° to
m ent. This w ill give an ind ication of grow th over the inter-
the maxillary plane is regarded as the limit or acceptable
vening p eriod . Provid ed Em m a rem ains keen for orthod ontic
retraction by tipping movements.
treatm ent, new record s, includ ing a p rogress cep halom etric
rad iograp h, shou ld be taken and analysed to allow for a ■ What a ctors govern stability o the corrected overjet?
new treatm ent p lan to be d evised . For the best p rospects of stability the interincisal angle
■ What other treatment options are there? shou ld be w ithin norm al lim its (135° ± 10°) and the overjet
comp letely red uced w ith the incisors in soft tissue balance,
I a design problem with the appliance is identi ed as the i.e. no tongu e thru st and the low er lip covering at least
cause o lack o treatment progress, then remaking the one-third of the labial su rface of the u pp er incisors. A
appliance to incorporate appropriate design modi cations period of retention w ill nonetheless be requ ired , and this
could be undertaken and treatment recommenced. shou ld extend u ntil grow th is com p lete follow ing fu nctional
I poor compliance is to blame or no progress, then the appliance therapy. Most p atients, how ever, usu ally p roceed
reason(s) should be ascertained rom discussion with the d irectly to a second p hase of xed ap p liance therap y to
child. I lack o motivation or interest in treatment is the d etail the occlu sion follow ed by retention.
cause, it would be prudent to avoid any urther appliance
therapy until such time as the child has a change o heart
Key point
regarding orthodontic treatment.
A ter unctional appliance therapy with or without
Orthodontic camou age by retraction o the upper incisors
subsequent f xed appliance therapy:
into rst premolar extraction spaces, accepting the Class II
skeletal pattern. Importantly, this treatment should not be • Ensure the upper incisors are in so t tissue balance and
detrimental to acial aesthetics. Although some amount o controlled by the lower lip.
tipping movement o the upper incisors is permissible, xed • Retain until growth is complete.
appliances would be required to ensure an optimal

Fig. 9.5 (A) A ter unctional appliance therapy: pro le. (B) A ter unctional appliance therapy: le t buccal occlusion. (C) A ter unctional
appliance therapy: anterior occlusion.
61 •
INCREASEDOVERJET 9
The p ro le and occlu sion follow ing fu nctional ap p liance Petti S 2015 Over tw o hund red m illion inju ries to anterior teeth
therapy are show n in Fig. 9.5. attribu table to large overjet: a m eta-analysis. Dent Trau m atol
31:1–8.

Primary resources and Seehra J, Flem ing PS, N ew ton T et al 2011 Bu llying in orthod ontic
p atients and its relationship to m alocclu sion, self-esteem and
recommended reading oral health-related qu ality of life. J Orthod 38:247–256.
Baccetti T, Franchi L, McN am ara JA Jr 2005 The cervical vertebral Shah AA, Sand ler J 2009 H ow to … take a w ax bite for a Tw in
m aturation (CVM) m ethod for the assessm ent of optim al Block ap p liance. J Orthod 36:10–12.
treatm ent tim ing in d entofacial orthopaed ics. Sem in Orthod
Thiru venkatachari B, H arrison JE, Worthington H V et al 2013
11:119–129.
Orthod ontic treatm ent for p rom inent u p p er front teeth (Class II
Clark W 2010 Design and m anagem ent of Tw in Blocks: re ections m alocclusion) in child ren. Cochrane Database of Syst Rev Issu e
after 30 years of clinical u se. J Orthod 37:209–216. 11. Art N o: CD003452. DOI: 10.1002/ 14651858.CD003452.p ub3.
DiBiase AT, Sand ler PJ 2001 Malocclu sion, orthod ontics and Wolke D, Lereya ST 2015 Long-term effects of bu llying. Arch Dis
bullying. Dent Up d ate 28:464–466. Child 100:879–885.
Flem ing PS, Scott P, DiBiase AT 2007 H ow to … m anage the
transition from fu nctional to xed app liances. J Orthod
34:252–259. For revision, see Mind Map 9, page 229.
10
Examination
Extraoral
The skeletal pattern is Class I w ith an average FMPA. There
is no facial asym m etry. The lip s are com petent.
There are no abnorm al tem p orom and ibular joint signs or
sym p tom s.

Incisor crossbite Intraoral


■ What eatures are visible on the intraoral views
(Figs. 10.1 and 10.2)?
Oral hygiene is fair. Marginal gingival erythem a is evid ent
related to the incisor teeth. There is m arked attrition of
C C, w ith cariou s involvem ent of C m esially and D D d is-
tally. With the excep tion of the low er right quad rant w here
D has been lost, 6EDC21 are p resent in each qu ad rant.
Gingival recession ap p ears to be evid ent on the labial
SUMMARY aspect of 1 . 1 is d isp laced labially; the low er arch ap p ears
u ncrow d ed . 2 2 are rotated m esiolabially; 1 is d isp laced
Matthew is 8 years old. He presents with an upper
incisor in crossbite (Fig. 10.1). What is the cause, and
how would you manage it?

History
Complaint
Matthew ’s m other is concerned that her son’s u p p er front
teeth are not straight and is anxious for treatm ent to be
u nd ertaken soon.

History o complaint
A
1 eru p ted insid e the low er teeth. There is no history of a fall
or other trau m a to the prim ary p red ecessor or to 1 . A w as
lost over a year ago, a little later than A.

Medical history
Matthew is in good health.

Dental history
D w as extracted uneventfully u nd er local anaesthesia 8
m onths ago.

Fig. 10.1 Anterior occlusion at presentation. Fig. 10.2 (A) Upper occlusal view. (B) Right buccal occlusion.
(C) Le t buccal occlusion.
63 •
INCISORCROSSBITE 10
slightly palatally, and there is a sm all m ed ian d iastem a.
Otherw ise the u p p er arch app ears uncrow d ed . The incisor
Key point
relationship is Class I and 1 is in crossbite; 2 is p artially Where an incisor crossbite is present, check:
erup ted w ith the d istoincisal aspect in crossbite w ith C . • Periodontal status o lower incisors.
■ What is the prevalence o anterior crossbite reported • I an edge-to-edge incisor occlusion is achievable.
in the literature? • I mandibular displacement is present.
Dep end ing on the racial grou p , age at assessm ent and if an • Amount o overbite.
ed ge-to-ed ge incisor relationship w as inclu d ed or not, the • Incisor inclination.
reported p revalence varies from 2.2–11.9%. • Amount o space required or correction.
■ What specif c ea tures would you check? Explain why.
1. The periodontal status o 1 degree o mobility and
pocket probing depth associated with 1 should be
assessed to determine its prognosis as it is being
displaced labially by def ecting occlusal contact Investigations
(see below) and gingival recession is present. ■ What special investigations would you request? Why?
1 exhibited grade 2 mobility, but probing pocket
depth was less than 2 mm, indicating good periodontal As a d ental p anoram ic tom ogram taken 6 m onths prior to
prognosis in the event the crossbite relationship is this visit by a previou s general d ental p ractitioner (see
corrected. below ) w as available for inspection, a rep eat lm of this
1 nature w as not ind icated on clinical grou nd s. From this
2. Is it possible to achieve an edge-to-edge relationship on ?
1 rad iograp h it w as possible to check the presence/ absence
I so, this indicates that only a small amount o labial of p erm anent teeth and w hether there w as a su pernu m erary
movement o 1 is required to correct the crossbite tooth p resent in the up p er m id line. Shou ld there have been
1 any susp icion of the latter, a m axillary anterior occlu sal
relationship. An edge-to-edge relationship o was view w ould be ind icated to note the relation of the su per-
1
easily achievable. nu merary tooth to the roots of the u p per incisors. N o su per-
nu merary tooth w as evid ent in the u pp er m id line. A
3. Is there a mandibular displacement on closure? I the
p eriap ical rad iograph of the low er central incisors is not
mandible is shi ted anteriorly or laterally on closure
requ ired as the clinical exam ination d oes not lend signi -
1 2
rom initial tooth contact, on or into maximum cant cau se for concern to the p rognosis of these teeth.
1 C
Bitew ing rad iographs should be taken to d iagnose accu -
interdigitation, early treatment to eliminate the
rately the extent of carious involvem ent of the prim ary
displacement is indicated on dental health grounds. The
m olars.
rationale or this approach is that in susceptible
individuals, mandibular displacement on closure due to ■ The dental panoramic tomogram taken 6 months prior to
premature tooth contact(s) may lead eventually to this visit is shown in Fig. 10.3. What does it show?
temporomandibular joint dys unction syndrome. A 3 mm Normal alveolar bone height, except or apparent angular
anterior mandibular displacement on closure was bone de ects related to the mesial aspects o 6 6. (Both teeth,
1 however, were not mobile, and pocket depths were <2 mm
detected rom initial contact on ; there was no lateral
1 on the mesial and distal aspects o 6 6.)
mandibular displacement associated with the crossbite
D CD
2 D absent; caries in .
a ecting . E DE
C
4. The amount o overbite on 1 as the amount o All permanent teeth (except 8’s) present, o normal size and in
overbite post-treatment is a major actor governing normal developmental positions.
stability o incisor crossbite correction and as overbite
reduces when the incisor edge is moved upwards
and orward during incisor proclination, a deep
overbite pre-treatment is a avourable eature. In this
case, the overbite was 3.5 mm on 1 and there is a good
prospect o adequate overbite ollowing crossbite
correction.
5. The inclination o 1 an upper incisor that is upright or
retroclined (1) is better or proclination than an incisor
that is already labially inclined. Further proclination o the
latter may not be possible or could result in un avourable
occlusal loading.
6. The amount o space required to procline 1 space already
exists in the upper incisor area, and there is no need or
any extractions. Fig. 10.3 Dental panoramic tomogram.
• 64
10 INCISORCROSSBITE

Diagnosis
■ Wha t is your diagnosis?
Matthew has a Class I malocclusion on a Class I skeletal base
with average FMPA.
Mild marginal gingivitis related to the incisor teeth.
Gingival recession related to 1 labially.
D CD
Caries in .
E DE
Crossbite on 1 with associated mandibular displacement.
Misalignment o upper and lower labial segments. Fig. 10.4 Upper removable appliance to procline 1.
■ Wha t is the IOTN DHC grade (see p. 264)? Explain why.
4c – d u e to m and ibu lar d isp lacem ent >2 m m betw een the
retrud ed contact position (RCP) and the intercu spal posi-
tion (ICP).
■ Wha t would you deem to be the prognosis or the la bial
recession related to 1 ?
For accu rate assessm ent of the extent of the labial recession,
the soft tissu es shou ld be healthy, and at p resent gingival
in am m ation is evid ent. There ap p ears, how ever, to be
some attached gingiva labially, and the recession d oes not
Fig. 10.5 Post-treatment.
extend to the sulcu s re ection. It is also not associated w ith
a frenal p u ll. At this stage, p rovid ed oral hygiene im proves
and the crossbite is corrected , the gingival recession shou ld Retention: Adams clasps 6D D6 (clasps on 6 6 in 0.7 mm
not w orsen, although the w id th of attached gingiva w ill not stainless steel wire; clasps on D D in 0.6 mm wire).
increase.
Anchorage: rom baseplate.
■ Why is 1 in crossbite?
Baseplate: acrylic baseplate with ull palatal coverage
This is m ost likely d u e to a slightly p alatal ectop ic p osition incorporating posterior capping (~2 mm in height).
of 1 tooth bud . The appliance is show n in Fig. 10.4.
■ What will determine stability o crossbite correction on 1 ?
Treatment
Provid ed there is 2–3 m m of overbite on 1 follow ing p rocli-
■ What treatment would you provide and why? nation, the p rospect of stability is good . Su bsequ ent m an-
d ibu lar grow th m u st also be favou rable.
1. Oral hygiene instruction this is required to improve
The occlu sion follow ing crossbite correction on 1 is
gingival health and to remove the plaque insult to the
show n in Fig. 10.5.
gingival recession related to 1.
2. Caries management a diet diary should be completed
over 3 consecutive days (one o which should be a Key point
weekend day) and then appropriate dietary advice
Early treatment o an incisor crossbite is advisable i there
should be given based on the ndings (see Chapter 22).
is associated mandibular displacement and/or periodontal
Although several primary molars are carious, none have
trauma.
associated symptoms. Restorative management o
carious primary teeth is dealt with in Chapters 22
and 24.
3. Upper removable appliance therapy to procline 1 due to ■ What other treatment possibilities are there? What
the mandibular displacement that is producing evidence is there in relation to their e ectiveness?
periodontal trauma to 1, correction o the crossbite on 1 Other treatm ent op tions includ e xed appliances incorpo-
is required urgently. rating the incisors and rst p erm anent m olars (‘2 × 4’;
4. Monitor the lower centerline consider removal o D i a although 2 is only partly erup ted p resently and w ill need
centreline shi t develops. to have the bond ed attachm ent rep ositioned as treatm ent
■ Describe the a pplia nce design you would use to align 1 . progresses), u se of an inclined comp osite slop e bond ed to
the opp osing low er incisor (at about a 45° incline and
The ap p liance w ou ld have the follow ing d esign: 3–4 m m in height), a com bination of these techniques, elas-
Activation: Z spring (0.5 mm stainless steel wire) to tics attached to bond ed brackets/ bu ttons or m od i ed fu nc-
procline 1. tional app liances. A system atic review p u blished in 2011
65 •
INCISORCROSSBITE 10
ind icated that the evid ence level w as low w ith regard to Gravely JF 1984 A stu d y of the m and ibu lar closu re p ath in Angle
anterior crossbite correction, bu t evid ence favou red the u se Class III relationship . Br J Orthod 11:85–91.
of xed ap pliances. A recent rand om ized controlled trial Joss-Vassalli I, Grebenstein C, Top ou zelis N et al 2010
cond u cted in Sw ed en fou nd m inim al d ifferences betw een Orthod ontic therap y and gingival recession: a system atic
xed and rem ovable appliances for anterior crossbite cor- review. Orthod Craniofac Res 13:127–141.
rection; treatm ent d u ration w as slightly shorter (1.4 months) McCom b JL 1994 Orthod ontic treatm ent and isolated gingival
recession: a review. Br J Orthod 21:151–159.
w ith a xed ap p liance (brackets bond ed to the m axillary
incisors, p rim ary canines, prim ary rst m olar or rst prem o- Wied el A, Bond em ark L 2014 Fixed versu s rem ovable orthod ontic
ap p liances to correct anterior crossbite in the m ixed d entition –
lar, if erup ted ) than w ith a rem ovable app liance, bu t the
a rand om ized controlled trial. Eu r J Orthod 37:123–127.
latter treatm ent cost m ore. At 2-year follow -u p , stability w as
sim ilar in both grou p s.
For revision, see Mind Map 10, page 230.
Primary resources and
recommended reading
Borrie F, Bearn D 2011 Early correction of anterior crossbites: a
system atic review. J Orthod 38:175–184.
11
Medical history
Alistair is t and w ell.

Family history
Alistair ’s father reports that his ow n teeth m eet in a m anner
sim ilar to his son’s, and he also has a slightly prom inent
chin bu t is u nconcerned by it. H e had orthod ontic treatm ent
w ith extraction of tw o low er teeth and xed ap p liances

Reverse overjet w hen he w as a teenager to correct the bite of his front


teeth. H is bite changed a lot after he stop p ed w earing the
retainers.

Examination
Extraoral
Alistair has a m ild Class III skeletal pattern w ith average
FMPA (Fig. 11.2) and no facial asym m etry.

CASE1 ■ What other eatures would you check or?


• Presence/absence o a mandibular displacement on
SUMMARY closure.
Alistair, 8.5 years old, presents with a reverse • Temporomandibular joint signs/symptoms.
overjet on all o the upper incisors (Fig. 11.1). What Alistair cou ld ju st achieve an ed ge-to-ed ge incisor
relationship.
is the cause, and how may it be treated?
A 3 m m anterior m and ibu lar d isplacem ent on 1 1 w as
d etected from RCP to ICP. N o tem porom and ibu lar joint
History signs w ere noted , and Alistair reported no tem p orom an-
d ibu lar joint sym ptom s. There w as no m asticatory m uscle
Complaint tend erness.
Alistair is not bothered abou t the w ay his teeth bite together
and is not concerned abou t any aspect of his facial appear- Intraoral
ance. H is father, how ever, feels that Alistair ’s chin is som e-
■ What a re your observations rom the intra oral views
w hat p rom inent and gives the boy an aggressive-looking
(Figs 11.1 and 11.3)?
app earance. Som etim es Alistair is teased abou t his chin at
school. The soft tissues appear healthy w ith the exception of m ild
m arginal gingival erythem a related to the incisor teeth. Oral
History o complaint hygiene is fair. The d entition ap pears caries free. 6EDC21
Alistair ’s perm anent up per front teeth erupted behind his are present in each quad rant.
low er teeth. H is m other ’s recollection is that the bite of his
‘m ilk’ teeth w as sim ilar. Alistair is not bothered by the occa-
sional teasing he gets about his chin.
Alistair ’s p arents are keen for treatm ent, if p ossible, at
this stage to correct his bite and red uce the prom inence of
his chin, w hich w ou ld rem ove the sou rce of teasing at
school.

Fig. 11.1 Right buccal occlusion at presentation. Fig. 11.2 Pro le.
67 •
REVERSEOVERJET 11
to the mean (109°) bu t are w ithin the norm al range. Taking
account of the MMPA, the 1 angle shou ld be 120° − 25° =
95° but is 2° proclined at 97°. The MMPA and facial propor-
tions are slightly red u ced from average values bu t are
w ithin the norm al range.

Diagnosis
■ What is your orthodontic diagnosis?
Alistair has a Class III m alocclu sion on a Class III skeletal
Fig. 11.3 Le t buccal occlusion. p attern w ith slightly red u ced facial p roportions. There is an
anterior m and ibu lar d isp lacem ent on closu re on 1 1. Mar-
Table 11.1 Causes o reverse overjet ginal gingivitis related to the u p per and low er incisors.
Upper and low er arches exhibit m ild incisor crow d ing; the
Cause Aetiology
u pper incisors are in crossbite. Up p er and low er centrelines
Skeletal UsuallyClass IIIdue to anyo the ollowing: long mandible; are slightly d isp laced . The bu ccal segm ent relationship is
orward placement o glenoid ossa positioning the mandible Class III bilaterally.
more anteriorly; short and/or retrognathicmaxilla; short
anterior cranial base ■ What is the IOTN (DHC) grade (see p. 264)? Explain why.
Anterior mandibular Apremature contact maydisplace the mandible orward on 4c – d u e to >2 m m m and ibular d isp lacem ent betw een the
displacement on closure closure into maximuminterdigitation RCP and ICP.
Retained primaryupper These mayde ect the eruption path o their successors ■ Wha t dental health reasons a re there or
incisors palatallyinto crossbite orthodontic treatment?
Pattern/excessive Forward pattern o mandibular growth will exacerbate a
Mand ibu lar d isplacem ent on closu re m ay increase the like-
mandibular growth Class IIIskeletal pattern
lihood of tem porom and ibu lar joint d ysfunction in su scepti-
Excessive mandibular growth maybe due to excess growth
ble ind ivid u als. In ad d ition, d isplacing occlusal contacts
hormone resulting roma pituitaryadenoma
m ay lead to low er incisor m obility and contribu te to gingi-
Restraint o maxillary Found in repaired cle t lip and palate and attributed to the val recession.
growth e ect o post surgical scar tissue
■ What a ctors would you assess in orthodontic
treatment planning?

Upper and low er incisors are very m ild ly crow d ed and These are given in Table 11.2.
in crossbite. The overbite is average to slightly increased
and com plete. Up p er and low er centrelines are d isp laced .
The bu ccal segm ent relationship is Class III bilaterally.
Treatment
■ What are the possible causes o the reverse overjet? ■ Wha t orthodontic treatment would you undertake
and why?
These are listed in Table 11.1.
■ What radiographic investigations would you request In view of the alread y app arent Class III skeletal pattern,
and why? the ability of the patient to ju st achieve an ed ge-to-ed ge
incisor relationship , the inherent tend ency for d ow nw ard
A d ental p anoram ic tom ogram w ou ld be requ ired to accou nt and forw ard m and ibular grow th and the fam ily history, a
for the p resence and p osition of all the rem aining p erm a- sensible option w ould be to accept the m alocclusion for the
nent teeth. present and reassess in the light of fu rther m and ibular
A lateral cep halom etric rad iograph is ind icated to assess grow th. Alistair has not yet entered the p u bertal grow th
m ore accu rately the m agnitu d e of the Class III skeletal spurt, w hich is likely to exacerbate the Class III m alocclu -
pattern and the incisor inclinations, w hich w ill facilitate sion and chin prom inence as m and ibu lar grow th p roceed s.
treatm ent p lanning. It w ill also form a baseline from w hich On average, m and ibu lar grow th continu es until 19 years of
treatm ent progress/ grow th changes can be evaluated by age in boys, bu t it m ay progress for longer.
comp arison w ith fu tu re cep halom etric lm s.
The panoram ic rad iograph show ed all perm anent teeth
to be d evelop ing. Key point
■ What is your interpretation o the ollowing cephalometric Class III malocclusion in the mixed dentition is likely to
f ndings? worsen with mandibular growth, especially in boys.
SN A = 80°; SN B = 82°; 1 to m axillary plane = 106°; 1 to
m and ibu lar p lane = 97°; maxillary m and ibu lar planes
angle = 25°; facial p rop ortion = 53%. As the p arents are keen for treatm ent if possible, to try
The skeletal pattern is Class III (SN A − SN B = AN B = −2°) to red u ce any fu rther teasing at school, another op tion to
d u e to m ild m axillary retrognathism and m and ibu lar p rog- consid er w ou ld be grow th m od i cation by fu nctional ap p li-
nathism . The u p per incisors are slightly retroclined relative ance therapy (Fränkel III or FR III, w here FR stand s for
• 68
11 REVERSEOVERJET
Table 11.2 Factors to assess in treatment planning continu ing grow th and of the need for reassessm ent in the
Factor light of ensuing grow th. This form of treatm ent shou ld only
Degree o anteroposterior and Most important actor be u nd ertaken by a specialist and only w hen Alistair ’s oral
vertical skeletal discrepancy hygiene has im proved .
Re ected directlyin acial and dental appearance;
patient’s perception o these will in uence complexity ■ How would you take a wax registration or this appliance?
o treatment undertaken The m and ible is rotated d ow nw ard and backw ard u ntil the
Potential direction and extent o Assess relevant amilyhistory, age and gender o the incisors are brought to an end -to-end relationship or better,
uture acial growth patient and vertical acial proportions w ith the bite open about 2 m m . Alistair m ay be instructed
Reverse overjet is likelyto worsen with a orward to place the tip of his tongu e at the back of the hard p alate
growth rotation and horizontal pattern o mandibular and to m aintain it there w hile closing slow ly into a horse-
growth, usuallyobserved when the anterior acial shoe of softened w ax p laced over the u p p er teeth u ntil the
height is reduced or average d esired p osition is reached . This w ax registration should
The converse is likelywhere there is an increased then be chilled in cold w ater and its accu racy re-checked
vertical acial height in the m ou th before forw ard ing to the laboratory w ith
Incisor inclinations I dentoalveolar compensation is alreadymarked, im pressions of the d ental arches to allow for appliance
urther orthodonticcompensation is unlikelyto be constru ction.
stable or to produce an aesthetic result ■ How much should Alista ir wear this appliance?
Amount o overbite The deeper the overbite, the better the likelihood o
stable correction o the reverse overjet H e shou ld bu ild up w ear over the rst w eek so that it is
w orn for at least 14 hou rs ou t of 24. Encou ragem ent shou ld
Abilityto achieve edge to edge I this is not possible, correction o the incisor
be given to increase w ear to fu ll-tim e w ith the exception of
incisor contact relationship bysimple means is unlikely
m ealtim es and d u ring sports, althou gh this may prove d if-
Degree o upper and lower arch Delayupper arch extractions until the reverse overjet cult for som e child ren. As its nam e im p lies, the Fu nction
crowding is corrected, as this mayprovide space or relie o
Regu lator w as d esigned w ith the intention of altering fu nc-
mild/moderate crowding
tion of the circu m oral and m asticatory m u scu lature. For this
I extractions are undertaken in the upper arch only, reason, the p atient shou ld be instru cted to ‘exercise’ these
the reverse overjet mayworsen bythe upper labial
m uscles by gently op ening and closing into the ap pliance.
segment moving palatally
Alistair shou ld be given a tim e sheet to allow him to record
I mid upper arch extractions are necessary, extraction the nu m ber of hou rs of w ear per d ay. This shou ld be
o 4 4 is usuallyadvisable to allow or correction o inspected at each visit and u sed to encou rage p rogress.
the incisor relationship
■ What e ects will this appliance have?
The resp onse is a d ow nw ard and backw ard rotation of the
m and ible accom panied by an increase in facial height. Evi-
d ence su ggests that m and ibu lar grow th m ay be constrained
by the FR3 bu t it d oes not prom ote forw ard m axillary
grow th. The low er incisors are up righted , and the upp er
incisors m ay be proclined slightly. The upp er molars shou ld
erupt m ore than the low er ones.

Key point
The FR3 appliance or correction o Class III malocclusion:
• May constrain mandibular growth.
• Does not promote orward maxillary growth.
• Produces mostly dentoalveolar changes.
Fig. 11.4 Fränkel Ill appliance.

■ What other treatment options are there?


Fu nction Regu lator; Fig. 11.4) to correct the incisor relation-
ship becau se: An alternative ap proach to try to m od ify grow th w here
• Skeletal pattern is mildly Class III. m and ibu lar excess exists is by chincu p therap y. This requ ires
specialist m anagem ent. The line of force app lication shou ld
• An anterior mandibular displacement exists on closure,
be oriented below the cond yle to prod u ce d ow nw ard and
i.e. Alistair can achieve edge-to-edge incisor contact.
backw ard rotation of the chin. As a resu lt, low er anterior
• MMPA is slightly reduced.
facial height is increased bu t chin p rom inence is red u ced
• Upper incisors are not proclined. simu ltaneou sly. In essence, the ap pliance w orks in exactly
• Lower incisors are very mildly proclined. the sam e w ay as fu nctional ap p liances for m and ibu lar p rog-
• Overbite is average to slightly increased. nathism . As a signi cant am ount of force from the chincu p
It is essential, how ever, that Alistair and his parents is transferred to the base of the low er alveolar p rocess, the
are aw are of the need for p rolonged retention d uring low er incisors are also u prighted .
69 •
REVERSEOVERJET 11
Prognosis
■ Wha t actors will in uence sta bility o the corrected incisor
rela tionship?
The amount of overbite is im portant in the short term , bu t
the pattern of facial grow th, in particu lar the magnitu d e and
d irection of m and ibu lar grow th, w ill in u ence longer-term
stability.
The pro le and occlu sion follow ing crossbite correction
are show n in Fig. 11.5.

CASE2
SUMMARY
A Daniel, a 9.1-year-old boy, presents with all o the
upper incisors in crossbite. Daniel is unconcerned
by this. He is in good health. His mother is keen or
treatment to correct Daniel’s bite.

History
History o complaint
All of the perm anent upper front teeth eru pted insid e the
low er teeth; the ‘baby’ teeth had the sam e bite.
B

Fig. 11.5 (A) Post-treatment: pro le view. (B) Post-treatment: Medical history
occlusion. Daniel is in good health.

Dental history
H e has not had any p reviou s d ental treatm ent.
If the p atient is not keen to try grow th m od i cation or
the parents express concern abou t the need for p rolonged Examination
retention of the corrected incisor relationship , the m alocclu -
sion should be accep ted for the present. Arrangem ents Extraoral
shou ld be m ad e to review occlu sal d evelop m ent and to Daniel has a Class III skeletal pattern with average FMPA and
m onitor facial grow th. In Alistair ’s case it w ou ld be w ise to no facial asymmetry. The Class III pattern appears to be
review his occlu sion in 18 m onths (at age 10) to check par- mostly maxillary rather than mandibular in origin, with
ticularly on the position of the u nerupted perm anent m axil- mid face attening rather than a prominent chin. Lips are com-
lary canines and to m easure the reverse overjet. petent. There is no mandibular displacement on closure and
Once the p erm anent d entition is established , provid ed no abnormal temporomandibular joint signs or symptoms.
the reverse overjet has not w orsened m arked ly and the chin
■ Wha t do you notice in Fig. 11.6?
prom inence has not increased greatly, consid eration cou ld
be given to rem oval of 4 4 only, in conju nction w ith up p er Oral hygiene is poor with plaque deposits on most teeth and
and low er xed app liance therapy to correct the incisor associated generalized mild marginal gingival erythema. The
relationship . If the u pp er arch is crow d ed , 5 5 m ay be dentition appears caries- ree. EDC21 are visible in each
rem oved also, bu t it is w ise to d elay the d ecision regard ing quadrant. (6’s were erupted but not shown).
the need for any u pper arch extractions until the reverse Upper and lower incisors are very mildly crowded and in
overjet has been corrected . It is im p ortant to assess the crossbite as is the mesial aspect o C. The overjet is reversed
pattern of m and ibu lar grow th from an u p d ated cep halom - (measured 2 mm); the overbite is average to slightly reduced
etric rad iograp h p rior to this treatm ent ap p roach, and if and appears complete on 2 . The buccal segment relationship
there is any concern abou t it, treatm ent shou ld be d elayed is slightly Class III.
u ntil grow th is alm ost com plete.
■ Wha t radiogra phic investigations would you request
If the reverse overjet increases consid erably w ith fu rther
and why?
grow th, a com bined orthod ontic and su rgical app roach m ay
be requ ired for correction, d epend ing on the p atient’s con- A d ental p anoram ic tom ogram shou ld be requ ested to
cerns. This treatment w ou ld not be u nd ertaken u ntil m an- ascertain the p resence and p osition of all the uneru pted
d ibu lar grow th is com p lete in the late teens. perm anent teeth.
• 70
11 REVERSEOVERJET

Fig. 11.6 Pre-treatment: right buccal occlusion.

This show ed all p erm anent teeth to be p resent.


A lateral cep halom etric lm w ou ld also be requ ired to
inform the d iagnosis more fu lly w ith regard to the skeletal
com p onents of the Class III skeletal p attern and the extent
of any com p ensation exhibited by the incisor inclinations; Fig. 11.7 Delaire-type acemask.
both these factors w ill help treatm ent planning.
■ As Daniel is o mixed-race origin, how valua ble would
cephalometric data be?
Mild m arginal gingivitis. Up per and low er arches are m ild ly
The cep halom etric nd ings shou ld only be com p ared w ith crow d ed , and the bu ccal segm ent relationship is slightly
approp riate racial norm s. Daniel’s m other w as Cau casian, Class III.
bu t his father w as African. As there are no norm s available
for this racial m ix to w hich com p arison can be m ad e, com -
parison to Cau casian norm s is not app rop riate. As clinical
Treatment
assessm ent takes p reced ence and is m ore valuable than any ■ What treatment would you consider? Explain why?
cephalom etric analysis, treatm ent p lanning shou ld be based
on the clinical nd ings. Protraction (reverse-pu ll) head gear, otherw ise know n as
A cephalom etric rad iograph w as taken to act as baseline facem ask, treatm ent w ould be suitable. This treatm ent is
from w hich to m onitor m and ibu lar grow th. m ost ap p rop riate w ith m ild to m od erate Class III skeletal
problem s d ue to m axillary retrusion, in the early m ixed d en-
■ What is your interpretation o the ollowing
tition (and preferably no later than 10 years) as it attem p ts
cephalometric f ndings?
to stim u late grow th at the m axillary su tu res to bring the
SNA = 78°; SN B = 79°; 1 to m axillary p lane = 105°; 1 to m axilla forw ard . Daniel satis es all these criteria.
m and ibu lar p lane = 89°; m axillary m and ibu lar planes ■ What is the design o the protra ction headgea r appliance?
angle = 26°; facial p rop ortion = 54%.
Interp retation can only be in the broad est sense d u e to Several d esigns are com m ercially available and are ad ju st-
Daniel’s m ixed racial backgrou nd . With that in m ind , the able at the chair-sid e. All incorporate forehead and chin
follow ing should be interpreted w ith great cau tion. Based pad s. The Delaire-typ e has tw o vertical rod s located to
on the valu es given and consid ering Cau casian norm s, the the lateral asp ects of the head w hich connect to the
skeletal pattern is Class III (SN A − SN B = AN B = −1°) d u e pad s and allow s ad ju stm ent vertically for op tim al t. (Fig.
to the slightly m ore m axillary retrognathism than m and ibu- 11.7) If the plastic p ad s prod u ce skin irritation, pad d ing
lar p rognathism . Relative to the m ean value (109°), the m ay need to be ad d ed or ventilation holes d rilled in the
u pper incisors are slightly retroclined but are w ithin the chin p ad .
norm al range. Taking into accou nt the MMPA, the 1 angle The ad ju stable m id line crossbow is connected via elastics
should be 120° − 26° = 94° bu t is 5° retroclined at 89°. The to vestibu lar hooks (located in the region of D D) on a m ax-
MMPA and facial p rop ortions are only slightly red u ced illary splint w hich is either cem ented or bond ed , u su ally to
from average valu es bu t are w ithin the norm al range. 6ED DE6; the app liance m ay or may not incorp orate an
expansion screw (Fig. 11.8). Althou gh it w as form erly
ad vised that rapid m axillary expansion be carried ou t
Diagnosis simu ltaneou sly w ith facem ask treatm ent, this has now been
show n not to be necessary.
■ Wha t is your diagnosis?
An alternative, rail-style d esign of facem ask is m ore
Daniel has a Class III m alocclu sion on a Class III skeletal stream lined , easier to ad ju st and ap pears to be m ore com -
base w ith average FMPA and no m and ibu lar d isplacem ent. fortable for sleep ing (Fig. 11.9).
71 •
REVERSEOVERJET 11

Fig. 11.8 RME appliance.

B
Fig. 11.9 Rail-style acemask.
Fig. 11.10 Seven months later on removal o RME (A) pro le and
(B) right buccal occlusion.
Key point
Facemask (reverse pull headgear) treatment may be SN B change 0.7°) w ith an overall m ean skeletal change
considered or: (AN B) of abou t 2°. The occlusal plane rotates u p w ard and
forw ard w ith a d ow nw ard and backw ard rotation of the
• Mild to moderate Class III malocclusion due to maxillary
m axilla. Dentally, the overjet is increased and the low er
retrusion.
incisors are retroclined by about 5°.
• ≤10 years.
The facial and occlu sal ou tcom es show n 7 m onths later
and at 1-year follow -u p are show n in Figs 11.10 and 11.11,
respectively.

■ How does this work? ■ Does treatment pose any risk to the ja w joints? Are there
any psychologica l benef ts?
Forces of abou t 400 g p er sid e are app lied by elastics for 14
hou rs p er d ay from the vestibu lar hooks on the bond ed In the one rand om ized controlled trial that record ed tem po-
m axillary splint/ exp and er to the crossbow of the facem ask. romand ibu lar joint signs and sym ptom s w ith facem ask
These p u ll at 30° in a d ow nw ard and forw ard d irection to treatm ent com pared w ith an u ntreated control grou p, no
ad vance the m axilla. ad verse effects w ere reported at 15-m onth and 3-year
follow -up . The sam e trial found no im provem ent in the
■ What e ects does it have?
child ’s self-esteem at either point in tim e. At 15-m onth
Cephalom etric nd ings ind icate that, com p ared w ith an review, a signi cant red u ction w as record ed in the im pact
u ntreated control grou p, facem ask treatm ent p rod u ces a of malocclu sion scores relative to controls, w hich allud es to
sm all am ou nt of m axillary p rotraction (m ean SN A change less u nease abou t d ental app earance, bu t this effect w as not
1.4°), slight backw ard m ovem ent of the m and ible (m ean present at 3-year review.
• 72
11 REVERSEOVERJET

■ Are there a ny alternatives to a cemask treatment that may


produce the same, or greater, outcome?
Greater m axillary protraction has been reported by the fol-
low ing means:
Application o the acemask to miniplates placed either
posteriorly in the maxilla at the base o the zygomatic arch or
anteriorly in the maxilla above the incisors.
Application o Class III intermaxillary elastic traction rom
miniplates located at the base o the zygomatic arch to
screws positioned in the anterior mandible, usually mesial to
the canines. This is known as bone anchored maxillary
protraction (BAMP).

■ What a re the adva ntages and disadva ntages o BAMP?


The m ajor ad vantages are that fu ll-tim e elastic traction is
possible w ith no need for an extraoral app liance. In ad d i-
tion, although m ore effective than treatm ent by conven-
tional facem ask therap y w ith m ore skeletal m ovem ent than
from a facem ask attached to m inip lates in the anterior
A m axilla, both p lacem ent and rem oval of the m iniplates
requires a su rgical p roced ure.

Primary resources and


recommended reading
Battagel JM 1993 The aetiological factors in Class III m alocclu sion.
Eu r J Orthod 15:347–370.
Cevid anes L, Baccetti T, Franchi L et al 2010 Com p arison of tw o
p rotocols for rap id m axillary p rotraction: bone anchors versu s
face m ask w ith rap id m axillary exp ansion. Angle Orthod
80:799–806.
Jam ilian A, Cannavale R, Piancino MG et al 2016 Method ological
B qu ality and ou tcom e of system atic review s rep orting on
orthopaed ic treatm ent for Class III m alocclusion: Overview of
Fig. 11.11 At 1-year review: (A) pro le and (B) right buccal occlusion. system atic review s. J Orthod 43:102–120.
Levin AS, McN am ara JA Jr, Franchi L et al 2008 Short-term and
long-term treatm ent ou tcom es w ith the FR-3 app liance of
■ How success ul is a cemask treatment or Class III
Frankel. Am J Orthod Dentofacial Orthop 134:513–524.
malocclusion in the short term?
Mand all N , Cou sley R, DiBiase A et al 2016 Early class III
Facem ask treatm ent ap p ears to be 70% su ccessful at p rotraction facem ask treatm ent red u ces the need for
15-m onth and 3-year follow -u p . orthognathic su rgery: A m u lti-centre, tw o-arm p arallel
rand om ised , controlled trial. J Orthod 43:164–175.
■ What is the long-term success rate o this trea tment?
Mand all N A, Cou sley R, DiBiase A, et al 2012 Is early class III
The longest follow -u p to d ate has been for 6 years for p rotraction facem ask treatm ent effective? A m u lticentre,
patients treated in the trial m entioned above. Alm ost 70% rand om ised , controlled trial: 3-year follow -u p . J Orthod
of those treated by p rotraction facem ask m aintained a p osi- 39:176–185.
tive overjet. Panel consensu s ind icated that 36% of those Watkinson S, H arrison JE, Fu rness S et al 2013 Orthod ontic
treated by protraction facem ask need ed orthognathic treatm ent for p rom inent low er front teeth (Class III
m alocclu sion) in child ren. Cochrane Database of Syst Rev Issu e
surgery versu s 66% of the control grou p . Patients in the
9. Art N o: CD003451. DOI: 10.1002/ 14651858.CD003451.pu b2.
control grou p w ere 3.5 tim es m ore likely to need surgery.
Yang X, Li C, Bai D et al 2014 Treatm ent effectiveness of Fränkel
Self-esteem , how ever, d id not d iffer signi cantly betw een
fu nction regu lator on the Class III m alocclu sion: a system atic
the grou ps.
review and m eta-analysis. Am J Orthod Dentofacial Orthop
146:143–154.
Key point
Facemask treatment or Class III malocclusion in the early For Revision See Mind Map 11, page 231.
mixed dentition:
• Is 70% success ul at 3 year ollow-up.
• Has no adverse e ect on the TMJs.
• Reduces the need or orthognathic surgery.
12
Family history
H arry’s father has the sam e arrangem ent of the u pp er ante-
rior teeth as his son. H e had four teeth rem oved and treat-
m ent w ith xed app liances as a teenager. H ow ever, the
treatm ent resu lt relap sed and the u pper front teeth have
largely returned to their original positions. H arry’s m other
is keen that this d oes not hap p en to her son.

Increased overbite Examination


Extraoral
■ Ha rry’s prof le view is shown in Fig. 12.2. What do you notice
about the anteroposterior skeleta l pattern and
the lips?
H arry has a Class II skeletal p attern w ith slightly red u ced
FMPA.
The lips are com petent.
CASE1 Intraoral
SUMMARY ■ The appearance o the mouth is shown in Figs 12.1 a nd 12.3.
Harry, aged 10 years and 6 months, presents with What do you see?
crowded upper teeth and a deep traumatic So t tissues appear healthy apart rom mild gingival erythema
overbite (Fig. 12.1). What has caused these related to the upper incisors.
problems, and how may they be treated? The oral hygiene is air with no visible carious lesions.
There are no restorations visible.
History Harry is in the mixed dentition with the ollowing teeth

Complaint visible: 6E4321 124E6 (21 1 2 are erupted but are covered
6E43 3456
H arry d oes not like the ap p earance of his u p p er teeth and by the upper incisors).
has recently com p lained abou t the gu m behind his u p p er
Mild upper labial segment crowding.
front teeth being sore. H is m other is keen for treatm ent.
Class II division 2 malocclusion with deep complete overbite.
History o complaint Buccal segment relationship is a hal -unit Class II bilaterally.
H is p rim ary teeth w ere m ild ly irregu lar. H is perm anent
u pper front teeth erup ted in a ‘crooked ’ p osition and have
not changed since.

Medical history
H arry is t and w ell.

Dental history
H arry is a regu lar attend er at his general d ental p ractitioner
and has not requ ired any d ental treatm ent so far.

Fig. 12.1 Anterior occlusion at presentation. Fig. 12.2 Pro le at presentation.


• 74
12 INCREASEDOVERBITE
is aware o him having any nocturnal bruxing habit. I
wear o the incisors is observed, site and extent should
be noted or uture monitoring; as this is better
undertaken rom dental casts, impressions should be
recorded o the dental arches to allow these to be
constructed. There was no noticeable incisor wear, and
no bruxing habit was reported.
3. Assessment o the amount o upper and lower arch
crowding. Space may be obtained to relieve mild/
A moderate lower arch crowding and level an increased
curve o Spee by some proclination o the lower labial
segment and by a small amount o intercanine width
expansion, which appear to be stable in this
malocclusion. There ore, in Class II division 2
malocclusion, lower arch extractions should generally
only be undertaken i crowding is severe. In this case,
lower arch extractions should be considered with great
reservation, as this will allow the lower labial segment to
drop lingually and aggravate an already traumatic
overbite. There was 2 mm o lower labial segment
B crowding, and space analysis indicated su cient space
or the canines and premolars (21 mm present in each
Fig. 12.3 (A) Right buccal occlusion. (B) Le t buccal occlusion.
quadrant; 21 mm is required, on average, to
accommodate 3, 4, 5).
Table 12.1 Causes o increased and traumatic overbite in Class II
division 2 malocclusion
Key point
Cause Aetiology
In Class II division 2 malocclusion:
Skeletal: anteroposterior AClass IIskeletal pattern in combination with a reduced lower
and vertical acial height • Beware o lower arch extractions only, as a deep
overbite may become traumatic.
Growth pattern An anterior mandibular growth rotation tends to increase
overbite • Some proclination o 2 11 2 and mild lower intercanine
expansion are o ten possible and stable.
So t tissues E ects are via the skeletal pattern – reduced lower acial
height leads to a high lower lip line that will retrocline the
upper incisors, leading to overbite increase
4. Radiographic investigations. The ollowing views are
Ahyperactive high lower lip, in association with a reduced required:
lower acial height, leads to bimaxillaryretroclination
• Dental panoramic tomogram, to account or the
Dental actors Absence o a well defned cingulumstop on the upper incisors presence/absence, position and orm o all
leads to continued eruption o the lower incisors, increasing
unerupted teeth. This showed: normal alveolar bone
overbite
height; a normal developing dentition with a ull
complement o teeth. No tooth appeared to be in
an ectopic position or to be o abnormal size or
■ What are the possible causes o the tra umatic overbite? shape.
In a Class II d ivision 2 m alocclu sion, several factors contrib- • A lateral cephalometric radiograph to assess the
u te to an increased overbite. These are listed in Table 12.1. anteroposterior and vertical skeletal relationships
■ What urther investigations would you underta ke? and the inclination o the incisor teeth to their
underlying dental bases.
1. Assessment o the extent o so t tissue trauma palatal to
the upper incisors and labial to the lower incisors rom ■ What is your interpretation o the ollowing cephalometric
the increased overbite. Periodontal pocket depth should f ndings (see p. 270)?
be recorded in these areas and any gingival recession SN A = 81°; SN B = 74°; AN B = 7°; MMPA = 22°; 1 to m axillary
should be noted. Incisor mobility should also be p lane = 99°; 1 to m and ibu lar p lane = 88°; interincisal
assessed. Although there was evidence o tooth angle = 162°; facial p roportion = 51%.
impingement on the gingivae palatal to 11 and labial to Relative to Cau casian norms, this ind icates SN A is
1, periodontal probing depths did not exceed 2 mm and average; SN B is red u ced ; AN B is increased , ind icating a
there was no gingival recession or incisor mobility. Class II skeletal pattern; MMPA is red u ced , w hich in con-
2. Assessment o tooth sur ace wear on the upper and ju nction w ith the Class II skeletal p attern is contribu ting to
lower incisors this may be on the labial aspect o the the increased overbite; 1 to m axillary p lane is retroclined ; 1
lower and/or the palatal aspect o the upper incisors as to m and ibu lar p lane is retroclined and not com p ensating
well as the incisal edges. Harry should be asked about comp letely for the red u ced MMPA; the interincisal angle is
any bruxing habit, and his mother should be asked i she increased ; facial proportion is red uced .
75 •
INCREASEDOVERBITE 12
u pper incisor teeth should be overproclined slightly to
Diagnosis allow for som e retroclination u nd er the in u ence of the
fu nctional ap p liance.
■ What is your diagnosis?
The patient shou ld be instructed to tu rn the m id line
Class II division 2 malocclusion in the late mixed dentition on expansion screw one quarter tu rn p er w eek. Arch coord ina-
a Class II skeletal base with reduced FMPA. tion should be m onitored at each recall by getting the patient
Mild marginal gingivitis related to the upper incisors. to posture the m and ible forw ard u ntil the m olars are in a
Class I relationship and checking to ensu re that the bu ccal
Traumatic overbite onto gingivae palatal to 1 1 and labial
segm ent teeth are not in crossbite. As the u pp er arch in Class
o 1.
II Division 2 m alocclusion is usually square-shap ed and the
Mild upper and lower arch crowding. low er arch is ‘u ’-shap ed , only a sm all am ou nt of upper arch
Buccal segment relationship is hal -unit Class II bilaterally. exp ansion is generally requ ired to achieve coord ination of
the arch w id ths w ith the m and ible postu red forw ard .
■ What is the IOTN (DHC) gra de (see p. 264)? Explain why. An activator-type fu nctional ap pliance is particu larly
4f – d u e to the trau m atic overbite. u sefu l in this type of m alocclusion. A H erbst ap p liance is
less recom m end ed as it m ay tend to d ep ress u p p er m olars
and inhibit correction of the d eep bite problem . For this
Treatment case, the construction bite for the activator ap pliance shou ld
be taken w ith the m and ible postured forw ard ed ge-to-ed ge
■ What are your aims o treatment? w ith the incisors 3–4 m m ap art and the centrelines correct.
Aims of treatm ent are to: If the centrelines had been d iscrepant by several m illim etres
initially, the bite should not comp ensate for this.
Improve oral hygiene.
The d esign of a m ed ium opening activator to be used
Relieve upper and lower arch crowding. here w ou ld be as follow s:
Reduce the overbite. • Adams clasps and occlusal rests to 6 6 (0.8 mm stainless
Correct the incisor relationship to Class I. steel wire).
• Labial bow 3 to 3 (0.8 mm stainless steel wire); palatal
Correct the molar relationship to Class I.
bow 2 to 2 (0.8 mm stainless steel wire).
Retain the corrected occlusion. • Acrylic palatal baseplate, deep acrylic capping o the
■ How do you propose to achieve these aims? lower incisors and canines with acrylic struts joining the
As H arry is grow ing and has Class II and d eep bite skeletal upper to the lower part o the appliance. The acrylic
problem s, grow th m od i cation by a fu nctional app liance should be heat-cured.
w ou ld be the treatm ent of choice. A p relim inary phase of An alternative to u sing an upp er rem ovable appliance
u pper rem ovable appliance therapy w ill be required to align (w ith or w ithou t a sectional xed ap p liance to 21 12) to
1 1 by p roclination and to exp and the u p p er arch slightly, p rocline and align the u p per incisors, follow ed by an
allow ing the m and ible to be p ostu red forw ard w ith the arch activator-type app liance, is to u se a m od i ed Tw in-Block
w id ths coord inated for the constru ction bite for the fu nc- ap pliance to achieve the d esired tooth m ovem ents. The
tional ap p liance. Alternatively, a sectional xed ap p liance d esign of this ap p liance incorp orates Z sp rings to p rocline
m ay be u sed in conju nction w ith the u p p er rem ovable 1 1 and has no labial bow ; it m ay also be u sed w ith or
ap pliance to align and procline 21 12 p rior to fu nctional w ithout a sectional xed appliance to the u pper incisors.
ap pliance treatm ent. Follow ing this, nal d etailing of the ■ Wha t are the goals o the unctional appliance trea tment?
occlu sion w ith fu ll u p p er and low er xed ap p liances is
To correct the skeletal Class II problem by di erential growth
likely to be requ ired prior to proceed ing to retention.
o the jaws, in particular addressing the mandibular retrusion.
■ Describe the design o applia nces you would use.
To increase the lower acial height and to correct the deep
The u p p er rem ovable ap p liance to p rocline 1 1 w ould have bite by preventing incisor eruption, controlling eruption o
the follow ing d esign: the upper posterior teeth while allowing eruption o the
• Activation: Z springs to 11 (0.5 mm stainless steel wire). lower posterior teeth. This di erential control o incisor/molar
Midline expansion screw. eruption aims to rotate the occlusal plane in a manner that
• Retention: Adams clasps 64 6 (0.7 mm stainless steel allows Class II correction.
wire). 4 is insu ciently erupted at present to clasp. To convert the incisor and molar relationships to Class I.
• Anchorage: rom baseplate.
• Baseplate: ull palatal acrylic coverage with f at anterior
biteplane, initially to hal the height o 11. A measurement
o the overjet plus 3 mm should be orwarded to the
Key point
laboratory at the time o appliance construction to ensure
adequate posterior extension o the biteplane. Class II division 2 malocclusion in the mixed dentition may
As treatm ent p rogresses, ad d ition of cold -cu re acrylic to be amenable to unctional appliance correction, taking
the at anterior bitep lane m ay be m ad e at the chairsid e, advantage o acial growth to aid in overbite reduction.
until suf cient overbite red u ction has been achieved . The
• 76
12 INCREASEDOVERBITE

Fig. 12.4 (A) Post-treatment. Pro le. (B) Post-treatment: le t buccal occlusion. (C) Post-treatment: anterior occlusion.

■ Why may a later phase o f xed appliance therapy


be required? CASE2
Rotational correction, esp ecially of 2 2, and d etailing of the
SUMMARY
buccal segm ent occlusion requ ire the u se of xed ap pliance
therapy. Gillian is 14 years old and does not like the
■ Wha t aspects o the corrected occlusion a re prone to appearance o her teeth. She had 6 extracted
relapse? How may you try to prevent/minimize relapse? 4 years ago due to ca ries. Both she and her
Rotations o 2 2 pericision (severing o the ree gingival
mother would pre er to avoid any urther
bres) should be undertaken a ew months prior to xed extractions with orthodontic treatment i at all
appliance removal as it reduces the relapse tendency. A possible. What are the causes, and how may it
bonded retainer, however, will be required to maintain upper be managed?
labial segment alignment in the long term.
Overbite the tendency or anterior mandibular growth Examination
rotation to continue into late teens and beyond will
encourage the overbite to increase. To combat this, a f at Extraoral
anterior biteplane should be incorporated on an upper Gllian has a m ild Class II skeletal p attern w ith slightly
Hawley retainer (designed to t around the upper bonded red u ced FMPA and low er facial height. The low er lip lay at
retainer) to be worn at night until growth has reduced to the m id d le to gingival third of the u pper incisors. There w as
adult levels. no facial asym m etry. There w ere no temp ororm and ibular
joint signs or sym p tom s.

Key point ■ What do you notice in Fig. 12.5?


The corrected aspects o Class II division 2 malocclusion Fair oral hygiene with generalised mild marginal gingival
most prone to relapse are: erythema.
• Rotational correction o 2 2. Apart rom upper le t quadrant where 6 has been extracted,
• Overbite reduction. 1234567 erupted in all quadrants.
6 , 6 6 and 5 restored; caries 4.
Severe lower arch crowding with 2 lingually displaced; 3
The facial p ro le and the occlu sion after fu nctional upright and 3 distally angulated; 2 mesiolingually rotated;
app liance, follow ed by xed ap p liance therap y, are show n moderate upper arch crowding with 3 3 buccally displaced;
in Fig. 12.4. 3 upright and 3 distally angulated.
77 •
INCREASEDOVERBITE 12

B
A

Fig. 12.5 Case 2 at presentation. (A) Anterior occlusion. (B) Right buccal occlusion. (C) Le t buccal occlusion. (D) Upper occlusal view.
(E) Lower occlusal view.

Class II division 2 incisor relationship; deep complete overbite;


2 in crossbite with 3 (there was a 2.5 mm mandibular Diagnosis
displacement between RCP and ICP).
■ Wha t is your diagnosis?
Right molar and canine relationships, Class I; le t canine
Class II division 2 malocclusion on a mild Class II skeletal base
relationship is Class III.
with reduced FMPA and lower acial height.
A lateral cephalometric radiograph was taken to assist with
Generalized mild marginal gingivitis.
diagnosis and treatment planning.
Caries 4.
■ What is your interpretation o the ollowing
Severe lower and moderate upper arch crowding with a deep
cephalometric f ndings?
complete overbite.
SN A = 82°; SN B = 77°; MMPA = 23°; 1 to m axillary plane = Crossbite o 2 with anterior mandibular displacement on
98°; 1 to m and ibu lar p lane = 88°; interincisal angle = 153°; closing.
facial p rop ortion = 52%.
Buccal segment relationship is Class I on the right and Class III
Relative to m ean Cau casian valu es, SN A is slightly
on the le t.
increased and SN B m ild ly red u ced . The skeletal p attern
(SN A-SN B) is m ild ly Class II (AN B 5°). The u p per and ■ What is the IOTN DHC gra de (see Appendix A1)? Explain why.
low er incisors are retroclined ; the low er incisor angle is not 4c – d u e to the m and ibu lar d isp lacem ent.
comp ensating for the red u ced MMPA; w ith an MMPA of
■ Wha t is the aetiology o the deep overbite?
23°, the low er incisor angle shou ld be 97° (120° – 23°). The
interincisal angle is increased , and facial p rop ortion is Several factors have contribu ted : m ild Class II skeletal
slightly red u ced . p attern, red u ced MMPA and facial proportions, high low er
• 78
12 INCREASEDOVERBITE
lip line, retroclination of the u p p er and low er incisors p ro- effect of also extru d ing the molars. Use of Class II or Class
d u cing a steep interincisal angle (Table 12.1). III interm axillary elastics w ill extru d e low er m olars or
■ Wha t implications does the lower incisor inclination have up p er molars, respectively, and assist w ith overbite red uc-
on treatment pla nning? tion. Care m ust be taken, how ever, to p revent extrusion of
the u p p er incisors w ith the elastics w hich w ill offset efforts
In view of the m arked retroclination of the low er incisors, to red u ce the overbite.
there is scop e for these to be proclined . This w ill p rovid e
sp ace for relief of the low er labial segm ent crow d ing and Lower incisor proclination Moving the low er incisors labi-
alignm ent of the low er arch; any d ecision regard ing possible ally w ill red uce a d eep overbite bu t need s to be und ertaken
low er arch extractions shou ld be d elayed u ntil this has been w ith cau tion as u ncontrolled movem ent is likely to lead to
achieved . relapse. In Class II d ivision 2 m alocclu sion, w here the low er
incisors may be retroclined by the u pp er incisors, som e
■ I a non-extraction, approach is adopted or the lower arch,
low er incisor proclination may occu r naturally by freeing
what impact has this or the upper arch?
the anterior occlu sion. This m ay be achieved by m oving the
Either no fu rther extractions are u nd ertaken or extraction of up p er incisors forw ard , by d isengaging the anterior teeth
u pper rst p rem olars is consid ered ; in the latter case, the w ith a at anterior bite plane on an up per rem ovable appli-
nal m olar occlu sion w ou ld be a fu ll u nit Class II bilaterally. ance, or by p lacing bite tu rbos (see Fig. 6.9A).
If only 4 is rem oved , consid ering the p reviou s loss of 6, the
m olar relationship w ill be Class II on the right and Class I
Key point
on the left.
Fu rther up per arch extractions w ou ld be u nw ise as the Overbite reduction by incisor intrusion is di cult to
u pper incisors are alread y very retroclined . achieve.
■ What options are there to reduce the deep overbite?
These are sum m arised in Box 12.1.
Incisor intrusion This requ ires xed ap p liances. In reality, Treatment
true incisor intru sion is d if cu lt to achieve and ‘relative’ ■ What a re the aims o treatment?
intrusion occurs. The incisors are held vertically as vertical
facial grow th occu rs w ith som e m olar extru sion, as this These are to:
occu rs m ore read ily than incisor intru sion. Placing a reverse Improve oral hygiene.
cu rve of Sp ee in a low er rectangu lar archw ire and / or an Restore 4.
increased cu rve of Sp ee in an u p p er rectangu lar archw ire
Relieve upper and lower arch crowding.
w ill achieve ‘relative’ incisor intru sion (Fig. 12.6A).
Use of tem p orary anchorage d evices (TADs) or au gm ent-
ing m olar anchorage, by typ ical ad d ition of second m olars
(Fig. 12.6B) to the anchor u nit, w ill increase the likelihood
of incisor intru sion and lim it m olar extru sion. Attem pts to
achieve tru e incisor intru sion by the u se of u tility arches,
w hich p u sh the incisors against the m olars, avoid ing the
bu ccal segm ent teeth, have achieved p artial su ccess as
limited m olar extru sion also takes p lace.
Molar eruption In a grow ing p atient, freeing the p osterior
occlusion w ith a at anterior bite p lane on an u p p er rem ov-
able ap p liance w ill allow the m olar teeth to eru p t, thereby A
red u cing the overbite; this is accom plished by a sim u ltane-
ous increase in low er facial height. The interincisal angle
should also be red u ced w ith rm incisor contact to m aintain
the ou tcom e.
Molar extrusion Use of cervical head gear w ith a d ow nw ard
and backw ard pu ll w ill extru d e the u pp er m olars and
increase low er facial height, an effect that is com p ensated
for in a grow ing child . Sim ilarly, u se of m echanics listed
above to attem p t intru sion of incisors has in m ost cases the

Box 12.1 Options o overbite reduction


Incisor intrusion.
Molar eruption. B
Molar extrusion.
Fig. 12.6 (A) Mid-treatment: anterior occlusion. (B) Mid-treatment:
Incisor proclination.
lower occlusal view.
79 •
INCREASEDOVERBITE 12
Reduce the overbite. headgear wear, otherwise known as a Ten Hove
Correct the incisor relationship to Class I. appliance (Fig. 12.8).
• TADs.
Maintain right molar relationship Class I; close 6 space and
Another Class II d ivision 2 case w ith increased overbite
establish Class I buccal segment relationship.
(Figs 12.9–12.12) is show n and w as treated by u sing a Ten
Retain the corrected occlusion. H ove ap p liance for d istal m ovem ent of the u p p er bu ccal
■ How will these be achieved? segm ents (Fig. 12.10), follow ed by xed ap pliance therap y
(Figs 12.11 and 12.12). See also Chap ter 6 (Case 2).
Gillian’s oral hygiene and caries statu s w ill be review ed
■ Wha t guidance does the best current evidence provide
follow ing tw o visits to the d ental hygienist and restoration
regarding management o Cla ss II division 2 malocclusion?
of 4. If oral hygiene is satisfactory, arrangem ents can be
m ad e for xed ap p liance therap y to be com m enced on a A recent system atic review ind icated that the evid ence w ith
non-extraction basis. The u p p er xed ap pliance can be regard to treatm ent and stability of this m alocclusion is
placed initially to ad vance and p rocline the u p p er incisors
to allow placem ent of the low er xed ap pliance.
■ Why may a non-extraction approach be avoured in Cla ss II
division 2 malocclusion?
Althou gh evid ence is slim and of low qu ality, prop osed
bene ts of a non-extraction ap p roach inclu d e less or no risk
of the overbite increasing, overbite red u ction is favou red ,
there is unlikely to be any u nfavou rable retraction of the lips
and , as there are no extraction sp aces to close, treatm ent
d u ration is u nlikely to be p rotracted .
Gillian’s nal occlu sion is show n in Fig. 12.7A and B.
■ What other non-extraction options are there or the upper
arch in Class II division 2 malocclusion?
Asid e from the op tions show n in Cases 1 and 2, other pos-
sibilities are d istalization of the u p p er bu ccal segm ents by
either: Fig. 12.8 Ten Hove appliance.
• An upper removable appliance with 0.7 mm springs to
banded 6’s supported by night-time (8 10 hours)

A A

B B

Fig. 12.7 (A) Post-treatment: anterior occlusion. (B) Post-treatment: Fig. 12.9 (A) Pre-treatment: right buccal occlusion. (B) Pre-
le t buccal occlusion. treatment: le t buccal occlusion
• 80
12 INCREASEDOVERBITE

A A

B
B
Fig. 12.12 (A) Post-treatment: right buccal occlusion.
Fig. 12.10 Following retraction o 6’s: (A) right buccal occlusion; (B) Post-treatment: le t buccal occlusion.
(B) le t buccal occlusion.

• Retain long-term with a bonded retainer with or without


an upper removable appliance with f at anterior
biteplane.

Key point
A non-extraction approach is avoured in management o
Class II division 2 malocclusion.

A Primary resources and


recommended reading
Dyer FM, McKeow n H F, Sand ler PJ 2001 The m od i ed tw in block
app liance in the treatm ent of Class II d ivision 2 m alocclu sions.
J Orthod 28:271–280.
Gianelly AA 1998 A strategy for non-extraction Class II treatm ent.
Sem in Orthod 4:26–32.
Kim TW, Little RM 1999 Postretention assessm ent of d eep
overbite correction in Class II d ivision 2 m alocclu sion. Angle
Orthod 69:175–186.
Lap atki BG, Mager AS, Schu lte-Moenting J et al 2002 The
im portance of the level of the lip line and resting lip p ressu re in
Class II d ivision 2 m alocclu sion. J Dent Res 81:323–328.
B
Millett DT, Cu nningham SJ, O’Brien KD et al 2012 Treatm ent and
Fig. 12.11 Mid-treatment with xed appliances. (A) Right buccal stability of Class II d ivision 2 m alocclu sion in child ren and
occlusion; note active “Berman”ligatures in place or space ad olescents: a system atic review. Am J Orthod Dentofacial
Orthop 142:159–169.e9.
closure. (B) Le t buccal occlusion with Class II elastic.
N g L, Major PW, H eo G et al 2005 Tru e incisor intru sion achieved
lim ited and highly biased . The best gu id elines that w ere d uring orthod ontic treatm ent: a system atic review and
forthcom ing from this review w ere to: m eta-analysis. Am J Orthod Dentofacial Orthop 128:212–219.
• Treat in a timely manner to maximize growth potential. Selw yn-Barnett BJ 1996 Class II/ Division 2 m alocclu sion: a
m ethod of p lanning and treatm ent. Br J Orthod 23:29–36.
• Treat pre erably non-extraction.
• Reduce the overbite and correct the interincisal angle. For revision, see Mind Map 12, page 232.
13
Dental history
Gerald is a regu lar attend er at his general d ental practi-
tioner and has cooperated w ell w ith previous d ental
treatm ent.

Examination
Extraoral
Anterior open bite ■ Gera ld’s acia l prof le is shown in Fig. 13.2. What do
you notice?
Gerald has a m ild Class II skeletal pattern w ith increased
FMPA and increased low er anterior facial height. The lip s
are com p etent.
There w as no facial asym m etry. Mou th op ening w as
w ithin norm al d im ensions, and there w as no tem porom an-
d ibu lar joint tend erness or crepitu s. N o m asticatory m u scle
tend erness w as noted .
SUMMARY ■ Wha t other eatures should you assess? Explain why.
Gerald is 11 years old. He presents with no contact 1. The swallowing pattern. Where there is a space between
o his incisor teeth (Fig. 13.1). Identi y the cause(s) the upper and lower anterior teeth, swallowing is likely
and discuss the treatment options. to be achieved by orward positioning o the tongue
between the anterior teeth to achieve an oral seal. This is
particularly so where the vertical acial proportions are
History increased as the likelihood o lip incompetence is greater.
Although such behaviour o the tongue is in most cases
Complaint adaptive, in rare instances an endogenous (primary)
Gerald com p lains that his front teeth d o not m eet. This tongue thrust exists. It has been suggested that this is
em barrasses him w hen eating as he cannot bite into food . associated with lisping and some proclination o the
H is p arents are also concerned by this and by lisp ing d u ring upper and lower incisors. Any attempt in these cases to
sp eech, w hich they attribu te to the p osition of his front close the open bite is doomed to ail as the tongue will
teeth. They are anxiou s for treatm ent. return the incisors to their original positions.
2. Speech. By asking Gerald to count rom 60 to 70 aloud or
History o complaint to say ‘Mississippi’, the degree o sibilance (lisping) can be
Gerald ’s p arents rep ort that his p rim ary incisors d id not detected. The tongue position during speech should also
m eet either, bu t the sp ace betw een the u pp er and low er be observed.
p erm anent incisors app ears to have increased in the past
year. H is lisp has also becom e m ore noticeable. H e has no
history of thu m b or d igit su cking.

Medical history
Gerald is t and w ell.

Fig. 13.1 Anterior occlusion at presentation. Fig. 13.2 Pro le.


• 82
13 ANTERIOROPENBITE
Gerald had a tongu e to low er lip sw allow ing p attern, and ■ What a re the possible ca uses o an anterior open bite?
the lisp w as d eem ed to be m ild .
These are given in Table 13.1.
■ Wha t occlusal anomalies a re associated with speech
problems? Are the latter likely to resolve i any underlying
malocclusion is treated?
Key point
A persistent digit-sucking habit o ten produces an
Althou gh speech problem s are associated w ith incisor
asymmetrical anterior open bite.
sp acing, Class II d ivision 1 m alocclu sion, Class III m alocclu -
sion and anterior op en bite, they d o not occu r in all
ind ivid u als w ith these occlu sal anom alies. Fu rtherm ore,
correction of these occlu sal p roblem s is no gu arantee that Table 13.1 Causes o anterior open bite
the associated sp eech p roblem w ill resolve satisfactorily. Cause Aetiology
Where sibilance is ju d ged to be marked , referral to a sp eech Skeletal Increase in lower anterior acial height such that the compensatory
therap ist w ould be p ru d ent, althou gh treatm ent m ay d o abilityo the incisors to erupt into contact is exceeded. This maybe
little to im p rove m atters. worsened bya downward and backward pattern o acial growth
So t tissues Rarelyendogenous tongue thrust (Fig. 13.4)
Intraoral Habits Non nutritive sucking habit (NNSH) – pacifer (dummy; Fig. 13.5),
■ Wha t other eatures do you see (Figs 13.1 and 13.3)? blankets, digit (Fig. 13.6) or thumb
Persistent digit sucking habit, which o ten leads to an
Mild marginal gingival erythema related especially to the asymmetrical anterior open bite (Fig. 13.7)
incisors and 4 . Localized ailure o Occurs in cle t lip and palate, although in other cases there maybe
No caries is visible. alveolar development no known cause

Mild spacing o the upper and lower labial segments;


mesiolabial rotations o 11.
Class I incisor relationship.
Anterior open bite (measured clinically = 6 mm rom the
1
mesioincisal aspects o .
1
Class III molar relationship bilaterally.

B Fig. 13.4 Five-year-old boy with no history o a non-nutritive


sucking habit: (A) anterior open bite in the primary dentition.
Fig. 13.3 (A) Right buccal occlusion. (B) Le t buccal occlusion. (B) Suspected to be due to an endogeneous tongue thrust.
83 •
ANTERIOROPENBITE 13
■ How common are non-nutritive sucking habits (NNSHs)
in children?
The reported incid ence varies d epend ing on the age and
region assessed , bu t N N SH s are very com m on in early
child hood . An incid ence of abou t 80% has been reported in
the 5 m onths after birth in a Sw ed ish stud y and of arou nd
70% in 2–5-year-old N orth American child ren. The inci-
d ence, how ever, red u ces w ith age. Alm ost 50% of 4-year-
old s still su ck a d igit or paci er, red u cing to 12% past the
age of 7 and to alm ost 2% by 12 years.
■ Do NNSHs always produce a malocclusion?
Fig. 13.5 Anterior open bite in a 3-year-old due to dummy
Where there is a history of a N N SH , there is a greater likeli-
sucking (note the buccal crossbite o C and le t buccal segment).
hood of d eveloping a m alocclu sion than w ithou t. The longer
the habit continues, the greater its im pact on the d evelop ing
m alocclu sion, bu t it is im p ortant to realize that the effect is
ad d itive to any und erlying p rimary skeletal cau se and d oes
not lead p red ictably to a m alocclu sion.
■ Wha t are the e ects o protra cted pacif er use or a
persistent digit-sucking ha bit on the occlusion other than
creating a n anterior open bite?
Application of pressure from an object su ch as a paci er or
d igit d isru pts norm al erup tion. Developm ent of a posterior
crossbite is associated w ith protracted paci er u se. Persist-
ent d igit su cking m ay resu lt in retroclination of the low er
incisors, p roclination of the u pp er incisors, increased overjet
and a u nilateral bu ccal segm ent crossbite w ith associated
m and ibu lar d isp lacem ent (see Chapter 14).
Fig. 13.6 Anterior open bite due to sucking o two digits (second
and third ngers); note abnormal shape o 2 .
Key point
Non-nutritive sucking habits:
• Are common in early childhood.
• Reduce with age.
• Do no predictably lead to malocclusion.

Investigations
■ Wha t special investigations would you require? Explain why.
Adental panoramic tomogram is required to indicate what
other teeth have yet to erupt and to check their
A
developmental position and orm.
Alateral cephalometric radiograph is required to assess more
ully the extent o the anteroposterior and vertical skeletal
discrepancies, as well as the relationship o the incisors to the
underlying dental bases.
The d ental p anoram ic tom ogram show ed :
Normal alveolar base height.
A normal dentition. The developmental age matches Gerald’s
chronological age.
B
Third molars are present.
Fig. 13.7 (A) Anterior open bite due to thumb sucking. The cephalom etric analysis revealed :
(B) Following cessation o the habit and xed appliance therapy. SN A = 82°; SN B = 76°; AN B = 6°; MMPA = 34°; 1 to m axil-
Note that acid pumice microabrasion was used to improve the lary p lane = 111°; 1 to m and ibu lar plane = 86°; interincisal
appearance o 11 (see Chapter 36). angle = 126°; facial p rop ortion = 60%.
• 84
13 ANTERIOROPENBITE

■ What is your interpretation o these f ndings? crib associated w ith high-p u ll chincu p are able to correct
anterior op en bite.
Relative to m ean valu es for Cau casians, SN A is slightly
increased and SN B is slightly red u ced , but both are w ithin
the norm al range; AN B is increased , ind icating that the skel- Key point
etal p attern is m ild ly Class II; 1 to m axillary p lane show s
Unless an anterior open bite is due to a habit, treatment is
that it is proclined relative to the m ean but w ithin the
complex.
norm al range; 1 to m and ibu lar p lane show s that it is retro-
clined relative to the m ean, bu t it is at the correct angle to
comp ensate for the increased MMPA (120° − 34° = 86°); rela-
tive to m ean valu es, the interincisal angle is red u ced ■ What mea ns have been tried in a n a ttempt to stop a
but w ithin the norm al range and the facial proportion is digit-sucking ha bit?
increased .
These range from sim ple strategies, su ch as covering the
d igit w ith a p laster or p ainting the d igit w ith an u np leasant-
Diagnosis tasting su bstance, to attem pts to m od ify behaviou r w ith
cognitive behavioural therapy, rew ard -based strategies and
■ Wha t is your diagnosis? the use of positive reinforcem ent. The u se of intraoral app li-
Class I malocclusion on a mild Class II skeletal base with ances has also been tried either to stop the d igit being pu t
increased FMPA. in the habit p osition or to lessen the satisfaction felt in
u nd ertaking the habit.
Marginal gingivitis related to the incisors and 4 .
■ I the anterior open bite had been due to digit sucking,
Mild mesiolabial rotations o 1 1 with spacing o the upper what treatment would you recommend?
and lower labial segments. Anterior open bite.
A recent Cochrane systematic review conclud ed that p rovi-
Buccal segment relationship is Class III bilaterally.
sion of a xed habit breaker (palatal crib or arch; Fig. 13.8A
■ Wha t is the IOTN (DHC) grade (see p. 264)? Explain why. and B), p sychological intervention (p ositive or negative
reinforcem ent) or both seem to be effective in assisting chil-
4e – d u e to the anterior op en bite.
d ren w ith stop p ing a N N SH ; this, how ever, w as based on
low -qu ality, highly biased evid ence. If the habit ceases, the
Treatment
■ What treatment would you consider?
As the anterior op en bite is not d u e to d igit su cking, treat-
m ent is likely to be com p lex. Gerald has anterop osterior and
vertical skeletal p roblem s, the latter being m ore m arked ,
w ith a d ow nw ard and backw ard p attern of facial grow th
that has p rod u ced the anterior op en bite. An attempt, there-
fore, to achieve incisor contact m ay be by grow th m od i ca-
tion aim ing for effective control of m axillary vertical skeletal
and d ental grow th.
This w ou ld requ ire sp ecialist care. A fu nctional ap p liance
w ith p osterior bite blocks, e.g. a Tw in-Block app liance,
w ould be op tim al. As the incisor relationship is Class I and
the m olar relationship s are Class III, no forw ard p osturing
A
of the m and ible for the registration bite is ad visable. The
bite, how ever, m u st be op ened beyond the norm al resting
vertical d im ension so that m olar eru p tion is prevented . As
the app liance hold s the m and ible in this position, a vertical
intru sive force is exerted on the p osterior teeth m ed iated by
the stretch of the m u scles and other soft tissu es. Gerald
should be instructed to w ear the app liance fu ll-tim e, inclu d -
ing for m eals. The anterior teeth are allow ed to eru p t w hile
eruption of the p osterior teeth is inhibited , thereby red u cing
the anterior op en bite. This is su p p lem ented by a tend ency
for m and ibu lar grow th to be p rojected anteriorly w hile
vertical control of m axillary skeletal and d ental grow th
is effected . In this case, high-p u ll head gear should not be
ad d ed to the ap p liance. As the m olar relationship s are Class
III, any m olar d istal m ovem ent is not ind icated . B
Cu rrently, there is w eak evid ence that both the Fu nction
Regu lator IV (FR IV) w ith lip -seal training and the p alatal Fig. 13.8 (A) Palatal crib. (B) Modi ed palatal arch.
85 •
ANTERIOROPENBITE 13
anterior open bite w ill u su ally red uce sp ontaneou sly, guard ed . Gerald and his parents should be m ad e aw are of
althou gh this is likely to take several years. this before treatm ent com m ences. Provid ed there is excel-
lent coop eration w ith ap p liance w ear and vertical facial
grow th is favou rable, functional ap pliance treatm ent has
Key point a reasonable chance of success at this stage. H ow ever, a
Currently, low-quality evidence indicates that e ective second phase of treatm ent w ith xed app liances is likely to
strategies to assist a child in stopping a digit-sucking habit be requ ired to d etail the occlu sion. As these app liances d o
include: not control eru p tion so favou rably, posterior bite blocks or
• Fixed habit breaker. sim ilar com ponents w ill also be required d uring that phase
• Positive or negative rein orcement. of treatm ent to m aintain the correction achieved in the
earlier p hase. Thereafter, bite blocks w ill also need to be
• Both.
incorporated in any retainer. Long-term retention w ill be
requ ired to avert the possible unfavourable effects of su b-
sequent vertical facial grow th.
■ How would you manage a parent who is concerned a bout
The lisp m ay imp rove w ith closu re of the anterior open
an anterior open bite produced by either a dummy or digit
bite, but Gerald and his p arents should not have high expec-
sucking habit in the prima ry dentition (Figs 3.4 and 3.5)?
tations regard ing this.
It w ould be pru d ent to ad vise the parent of the child that ■ Are there any other treatment options?
this is a very com mon issue at this stage of d evelopm ent
and that, as the incid ence of N N SH s d ecreases throu ghou t If Gerald d oes not cooperate w ith functional appliance
the m ixed d entition, a ‘w ait and see’ ap p roach w ou ld be w ear, treatment for the anterior open bite by a speci c typ e
bene cial. Gentle p ersu asion to d iscontinu e the habit may of xed appliance m echanics w ith m u ltiloop archw ires
also be consid ered , bu t as m ost child ren are likely to ou tgrow (Kim m echanics) m ay be consid ered , m ost likely in conjunc-
the habit, it w ou ld be im portant not to m agnify the issu e for tion w ith the rem oval of the second or third m olars. This
the child . If either habit stop s, the anterior op en bite w ill ap proach to ap p liance treatm ent requ ires specialist training.
red u ce. Many changes w ill occu r w ith grow th of the face The objective is to correct the cant of ind ivid ual occlusal
and the eru ption of the p erm anent teeth, so the m alocclu - p lanes, u p righting the teeth in relation to the bisecting
sion can be reassessed in light of these changes at age 8 occlu sal plane. As this hap p ens, there is a red u ction in pos-
(w hen the incisors and rst p erm anent m olars eru p t); terior facial and d entoalveolar heights, w hich red uces the
shou ld there be a d isp lacem ent of the low er jaw d u e to nar- anterior open bite; anterior vertical elastics then bring the
row ing of the u pp er arch by the habit, it can be assessed and incisors into contact. Im p ressive and stable correction of
corrected at that tim e. Any concerns regard ing sw allow ing m arked anterior op en bite, in ad olescents and ad u lts, has
pattern and sp eech are also likely to change as the p erm a- been reported u sing this techniqu e. Sim ilar results can be
nent teeth eru p t and cou ld be assessed by a sp eech thera- achieved w ith ‘rocking horse’ archw ires u sed in com bina-
pist, also at that tim e, if d esired . tion w ith anterior vertical elastics.
Molar intru sion m ay also be achieved u sing tem p orary
■ What is the likely prognosis o treatment in Gerald’s case?
anchorage d evices (TADs; Fig. 13.9) in the form of either
As the anterior open bite is qu ite m arked and the vertical screw s or plates. To m axim ize m olar intrusion and p revent
skeletal p attern is m od erately increased , the prognosis is u nw anted bu ccal tipping of the m olars, use of bu ccal and

A B C

Fig. 13.9 TADs used to close anterior open bite: lateral cephalometric radiograph (A) pre-treatment; (B) ollowing molar intrusion with
TADs (arrowed) and palatal arch; (C) nearing the completion o xed appliance treatment with extraction o our premolars.
• 86
13 ANTERIOROPENBITE
palatal im p lants has been ad vised ; the skeletal anchorage, m axillary teeth. Am J Orthod Dentofacial Orthop
how ever, for m olar intru sion need s to be maintained d u ring 138:396–398.
the early retention phase in an attem p t to red u ce relap se. Borrie FBP, Bearn DR, Innes N PT et al 2015 Interventions for the
Shou ld the anterior op en bite w orsen consid erably, a cessation of non-nu tritive su cking habits in child ren. Cochrane
com bined orthod ontic su rgical ap p roach m ay be sou ght Database of Syst Rev Issu e 3. Art N o: CD008694. DOI: 10.1002/
14651858.CD008694.p u b2.
w hen grow th is com p lete.
British Orthod ontic Society 2012 Du m m y and thu m b
su cking habits. Patient inform ation lea et. Available at:
w w w.bos.org.u k.
Johnson N C, Sand y JR 1999 Tooth p osition and sp eech – is there a
Key point relationship ? Angle Orthod 69:306–310.
Management options or anterior open bite may be: Kim YH 1987 Anterior openbite and its treatm ent w ith m u lti-loop
• Accept. ed gew ise archw ire. Angle Orthod 57:290–321.
• Habit breaker. Lentini-Oliveira D, Carvalho FR, Qingsong Y et al 2007
Orthod ontic and orthop aed ic treatm ent for anterior op en bite in
• Growth modif cation.
child ren. Cochrane Database of Syst Rev Issu e 9. Art N o:
• Orthodontic camou age, with or without TADs. CD005515. DOI: 10.1002/ 14651858.CD005515.p u b3.
• Surgery. Lop ez-Gavito G, Wallen TR, Little RM, et al 1985 Anterior
op en-bite m alocclu sion: a longitu d inal 10-year p ostretention
evalu ation of orthod ontically treated p atients. Am J Orthod
87:175–186.
Mizrahi E 1978 A review of anterior op en bite. Br J Orthod 5:21–27.
Primary resources and N gan P, Field s H W 1997 Op en bite: a review of etiology and
recommended reading m anagem ent. Ped iatr Dent 19:91–98.

Baek MS, Choi Y-J, Yu H -S et al 2010 Long-term stability


of anterior open-bite treatm ent by intrusion of posterior For revision, see Mind Map 13, page 233.
14
Examination
Extraoral
Kirsten has a m ild Class III skeletal pattern w ith slightly
increased FMPA; the chin p oint is d isp laced slightly to the
right. The lips are incom p etent but habitu ally held together.
There is a tongu e to low er lip sw allow ing pattern. There
is no m asticatory m u scle tend erness, temp orom and ibular

Posterior crossbite joint tend erness or crep itu s, and m outh opening is not
restricted .
■ What other eature would you check or, bearing in mind
the history? Explain why.
It is im p ortant to check if there is m and ibular d isplacem ent
on closu re as this w ou ld ind icate that the facial asym m etry
is more an app arent than a true skeletal asym m etry. For the
former, orthod ontic correction of an associated crossbite
should be straightforw ard , bu t for the latter, fu rther inves-
tigations w ou ld be required to d etermine if the asym m etry
SUMMARY is p rogressive and more com p lex treatm ent w ou ld be
Kirsten is 7 years old. She presents with a crossbite requ ired to ad d ress the facial and occlu sal problem s.
o the right buccal segments (Fig. 14.1). What will be Early correction of a crossbite w ith a m and ibu lar d is-
your assessment and management options or this placem ent is ind icated to allow the occlu sion to d evelop
in an u nd isplaced p osition. It is likely also to red u ce the
problem?
possibility of d evelop m ent of tem porom and ibu lar joint d ys-
fu nction synd rom e, w hich m ay occur in su scep tible ind i-
History vid u als in w hom this occlu sal d iscrep ancy exists.
An anterolateral m and ibu lar d isp lacem ent on closu re
Complaint C
w as d etected on w ith an associated 3 m m shift betw een
Kirsten’s m other is concerned abou t the w ay her d aughter ’s C
teeth bite together. She has noticed that Kirsten’s jaw m oves retrud ed contact position (RCP) and intercusp al position
to one sid e as she closes her m ou th. This m akes her face (ICP).
ap pear crooked , a fu rther cause of anxiety for her m other.

History o complaint
Kirsten u sed to su ck her thu m b u ntil 5 months ago w hen Key point
1 1 started to eru p t. H er m other has becom e m ore aw are of Early correction o a posterior crossbite with associated
her d au ghter ’s ‘d eviated bite’ in the p ast year and w on- mandibular displacement is advisable.
d ered if the thu m b-su cking habit cou ld have contribu ted to
the problem .

Medical history
Kirsten is t and w ell. Intraoral
■ Wha t eatures are evident on the intraoral views (Figs 14.1
Family history and 14.2)?
There is no fam ily history of facial asym m etry.
Mild marginal gingival erythema related to the erupting
permanent incisors, but otherwise the so t tissues appear
healthy. There is no caries visible.
Upper arch seems V-shaped anteriorly; lower arch is more
U-shaped anteriorly.
6EDCB1 are present in both upper quadrants and in the lower
right quadrant; 12CDE6 are present in the lower le t quadrant.
Spacing o the upper and lower incisors with distopalatal
rotation o 1.
Class III incisor relationship.
Anterior open bite.
Fig. 14.1 Anterior occlusion at presentation. Lower centreline shi t to the right.
• 88
14 POSTERIORCROSSBITE
Table 14.1 Causes o buccal segment crossbite
Cause Aetiology
Skeletal Mismatch in the widths o the dental arches and/or an anteroposterior
skeletal discrepancy– buccal and anterior crossbites are most
commonly ound in Class IIImalocclusion. Rarely, mandibular growth
restriction ollowing condylar trauma or hemimandibular hypertrophy
maybe implicated, both producing asymmetry
So t tissues/habit With a digit sucking habit, the tongue position is lowered with the
teeth apart, and cheekcontraction is unopposed during sucking,
narrowing the upper arch slightly
A

■ What actors may be implicated in the aetiology o the


crossbite?
These are sum m arized in Table 14.1.
■ What is the most likely ca use o the posterior crossbite in
this case?
The thu m b-sucking habit (see also Chap ter 13).

B
Key point
Fig. 14.2 (A) Right buccal occlusion. (B) Le t buccal occlusion. A digit-sucking habit may lead to posterior crossbite with
associated mandibular displacement.

Box 14.1 Causes o a lower centreline shi t


• Unbalanced loss o C, Dand possiblyE; the age at extraction, the degree o ■ What is a possible explanation or the ma ndibular
crowding and the tooth extracted (the more anterior, the greater the e ect) displacement being on a primary ca nine rather than on
in uence the extent o centreline shi t. the molars?
• Unilateral retained primary incisor, canine or molar. As the corners of the m ou th exp erience the greatest cheek
• Hypodontia o an incisor or premolar. p ressu re d uring thu m b su cking, it seem s plau sible for
• Supplemental incisor or premolar. greater narrow ing to occu r across the canines rather than
• Lateral mandibular displacement on closure producing unilateral buccal segment the m olars, resulting in a V-shaped u p per arch (see Fig.
crossbite (o ten secondary to digit- or thumb-sucking habit). 14.1); hence p rem atu re occlu sal contact on one of the
• Early unilateral condylar racture leading to defcient growth on the a ected side. p rimary canines is the likely trigger for a lateral m and ibular
d isp lacem ent on closu re.
• Hemi acial microsomia.
A narrow, V-shaped up per arch is m ore likely to resu lt
• Hemimandibular hypertrophy(known ormerly as condylar hyperplasia). Cause is
w hen the thu m b is sucked intensely than w hen it ju st rests
entirely unknown. Most likely in emales between the ages o 15 and 20, but
may occur in either sex as late as the early thirties.
in the m ou th.
Kirsten’s m other rep orted that Kirsten u sed to su ck her
thum b for at least 12 hou rs per d ay for the past several years
bu t gave u p the habit in recent m onths.
Hal -unit Class II right molar relationship with buccal crossbite
o the right buccal segments (note palatal inclination o the
teeth).
Investigations
Class III le t molar relationship. ■ What special investigations would you undertake and why?
■ How would you assess the centrelines? A d ental p anoram ic tom ogram (Fig. 14.3A) is requ ired to
su rvey the d evelop ing d entition for any abnorm alities of
The up per and low er centrelines shou ld be coincid ent w ith
tooth nu m ber, size and p osition.
each other and w ith the m id line of the face. Both of these
Im p ressions of the d ental arches and a w ax registration
aspects shou ld be assessed , the latter by rst looking at the
in maximu m intercu spation shou ld be taken to allow stu d y
patient anteriorly and then d ow n on the face from above.
m od els to be constructed . These w ill allow a thorou gh
With stu d y m od els alone, it is not p ossible to d eterm ine the
occlu sal assessm ent to be p erform ed and w ill act as a base-
relation of the d ental m id lines to the facial m id line.
line record of the m alocclu sion.
In this case the low er centreline is d isp laced to the right
by about half the w id th of a low er incisor. ■ What does the denta l panoramic tomogra m show?
■ What are the possible causes o a lower centreline shi t? Normal alveolar bone height.

These are listed in Box 14.1. In this case it is d u e to the lateral All permanent teeth developing apart rom third molars.
aspect of the m and ibu lar d isp lacem ent on closu re. No apparent caries or other pathology.
89 •
POSTERIORCROSSBITE 14
Treatment
■ Wha t treatment plan would you propose?
Oral hygiene instruction to improve gingival health.
Correction o the right buccal segment and anterior
crossbites.
Regular review o the developing occlusion.
■ How may the crossbites be corrected? Describe the design
A o any a ppliance you would use.
Possible ap proaches to treatm ent are as follow s:
1. As the digit-sucking habit has been abandoned and the
mandibular displacement arises rom premature contact
C
on , judicious grinding o their cusp tips may remove
C
the occlusal inter erence and correct the buccal segment
crossbite, although the success rate with this treatment
has varied considerably, rom 27 64%. Placing composite
onlays to allay mandibular shi t where occlusal
inter erences exist has also been tried but with outcomes
in erior to active appliance therapy.
B The permanent incisors are erupted insu ciently, at
present, to consider their proclination and which means
Fig. 14.3 (A) Dental panoramic tomogram. (B) Repeat o right hal will be best.
dental panoramic tomogram. 2. As the upper buccal segment teeth are not tilted
buccally, upper arch expansion by a midline screw in an
■ What is the most likely reason or the blurred image on the upper removable appliance may be considered. The
right hal o the dental pa noramic tomogram? appliance may be clasped on 6D D6 (Adams clasps: 6 6
0.7 mm stainless steel wire; D D 0.6 mm stainless steel
Movem ent of the p atient d u ring im age cap tu re, m ost p rob-
wire). Buccal capping will acilitate tooth movement by
ably d ue to sw allow ing, is the likely cau se.
disengaging the posterior occlusion. Kirsten should be
■ Why wa s a right hal , rather than a ull, dental panora mic encouraged to turn the screw one quarter turn twice
tomogram retaken (Fig. 14.3B)? weekly (0.25 mm o activation with each turn) until the
In line w ith cu rrent rad iology gu id elines, the rad iation crossbite is corrected. A small amount o overexpansion
d osage to the p atient shou ld be as low as reasonably achiev- is advisable as some relapse is to be expected. Then the
able for the d iagnostic p u rp oses requ ired . Retaking the capping should be reduced to hal its height at one visit
right-half im age only satis es d iagnostic need s here. and removed completely at the ollowing visit to allow
the buccal segment teeth to erupt into occlusion.
Provided there is a well-interdigitating buccal segment
Diagnosis occlusion, the appliance should then be worn as a
retainer at night or several months; 6 months o
■ What is your diagnosis?
retention will likely su ce.
Class III malocclusion on a mild Class III skeletal base The nal occlusion ollowing crossbite correction by
with slightly increased FMPA; anterolateral mandibular an upper removable appliance is shown in Fig. 14.4.
C Correction o the incisor relationship and improvement
displacement on closure on .
C in 1 alignment occurred spontaneously.
Mild marginal gingivitis related to the erupting incisors. 3. Alternatively, upper arch expansion may be undertaken
Spacing o the upper and lower incisors with 1 distopalatally using a quadhelix (see Fig. 4.11A). This consists o bands
rotated. cemented to the rst permanent molars with soldered
Anterior open bite; lower centreline shi t to the right. arms, which in this case should extend orward to the
palatal aspects o the C’s. Alternatively, a pre ormed
Buccal segment relationship hal -unit Class II on the right and quadhelix may be employed that ts into sheaths
Class III on the le t. soldered to the palatal aspects o the molar bands,
Buccal crossbite o the right buccal segments associated with allowing easy removal or adjustment. Activation is
the mandibular displacement. usually hal a tooth width on each side. To acilitate
crossbite correction, it may be necessary to disengage
■ What is the IOTN (DHC) gra de (see p. 264)? Explain why.
the buccal occlusion temporarily by placing glass
4c – d u e to m and ibu lar d isp lacem ent >2 m m betw een the ionomer cement on the occlusal sur aces o the molars.
RCP and the ICP. Once the crossbite has been corrected, the cement may
• 90
14 POSTERIORCROSSBITE

A A

B B

Fig. 14.4 Post-treatment (A) Right buccal occlusion. (B) Anterior Fig. 14.5 (A) Pre-treatment: anterior occlusion (note C retained
occlusion. and mobile; 3 unerupted). (B) Post-treatment: anterior occlusion.

be removed and the quadhelix should be rendered over another w ith regard to the other ou tcom es assessed
passive prior to cementation or 3 6 months as a (intercanine exp ansion, stability of crossbite correction,
retainer. signs and sym p tom s of tem p orom and ibu lar joint d ysfu nc-
Another case treated by a quad helix is show n in tion, signs and sym ptom s of resp iratory d isease and quality
Figs 14.5A and B. of life).
■ What will determine i the corrected buccal segment
crossbite is likely to be stable?
Key point
Good buccal segm ent interd igitation and absence of any
Management options or a unilateral posterior crossbite d isp lacing occlu sal contacts.
with associated mandibular displacement in the mixed
dentition:
• Grind C’s. Key point
• URA with midline screw.
Correction o a unilateral buccal segment crossbite in the
• Quadhelix.
mixed dentition may be more success ul with a quadhelix
than with a removable expansion plate.

■ Based on current evidence, what treatment moda lity is


most e ective or correction o a bucca l segment crossbite
in the mixed dentition? Primary resources and
A recent system atic review su ggests that crossbite correc- recommended reading
tion and exp ansion of the interm olar w id th for child ren in Agostino P, Ugolini A, Signori A et al 2014 Orthod ontic treatm ent
the early m ixed d entition (aged 8–10) m ay be m ore su ccess- for p osterior crossbites. Cochrane Database of Syst Rev Issu e 8.
fu l w ith a qu ad helix than w ith an u p p er rem ovable exp an- Art N o: CD000979. DOI: 10.1002/ 14651858.CD000979.p u b2.
sion ap p liance, althou gh the qu ality of evid ence w as low to Prim ozic J, Ovsenik M, Richm ond S et al 2009 Early crossbite
m od erate. Com p ared w ith rem ovable exp ansion p lates, a correction: a three-d im ensional evalu ation. Eu r J Orthod
quad helix app liance m ay achieve 1.15 m m m ore m olar 31:352–356.
expansion and m ay be 20% m ore likely to correct crossbites.
There w as insu f cient evid ence to favou r one intervention For revision, see Mind Map 14, page 234.
15
she snores and is prone to d ay-tim e d row siness, w hich her
teachers have noticed . Otherw ise she is w ell. H er m other
w ond ers if the narrow appearance of Jean’s u pper teeth is
related to her m ou th breathing and snoring. She is keen
to know if Jean’s m outh breathing and snoring cou ld be
im p roved by any brace treatm ent.
■ Wha t is the relevance o Jean’s mode o breathing to
snoring and to her complaint?

Bilateral crossbite Comp u lsive mouth breathing, d u e to inability to breathe


through the nose, m ay contribu te to an altered head p osture
and low tongu e position; this m ay lead to unop posed action
of bu ccinator mu scu latu re and bilateral narrow ing of the
u pper arch creating bilateral bu ccal crossbites.
Inability to breathe throu gh the nose is also linked w ith
snoring, w hich is associated w ith sleep apnoea, a m ajor
cau se of d ay-tim e d row siness. Prolonged in am m ation of
the nasal m ucosa associated w ith allergies (in Jean’s case
there is a history of asthma) or chronic infection (she is
CASE1 prone to u pp er resp iratory tract infections) could prod uce
some d egree of nasal obstru ction and lead to m ou th breath-
ing. The norm ally large p haryngeal tonsils or ad enoid s in
SUMMARY
child ren m ay also contribu te.
Jean has just turned 12. She presents with bilateral
buccal crossbites (Fig. 15.1). What are the possible
Key point
causes, and how could it be managed?
• Mouth breathing may contribute to altered head
posture, low tongue position and bilateral buccal
History crossbite, but it is not the sole or even the major cause
o such crossbites.
Complaint
Jean d oes not like the ‘narrow ’ ap p earance of her u p p er
teeth, esp ecially w hen she sm iles.
Dental history
History o complaint Jean is an irregu lar attend er at her general d ental p racti-
H er mother says that Jean’s ‘baby’ teeth had the sam e tioner. She has ssure sealants to several m olar teeth and
ap pearance, and she also has a sim ilar ap p earance of her one lling. There is no history of d igit-su cking.
u p per teeth to her d au ghter. Jean’s teeth eru p ted in the p osi- ■ How may a digit-sucking habit cause a bucca l segment
tion they are in at p resent. There is no history of traum a to crossbite?
her u pp er jaw, and birth w as norm al.
This is given in Table 14.1 (p . 88).

Medical history Social history


Jean is asthm atic and u ses a salbu tam ol (Ventolin) inhaler.
Jean has tw o you nger sisters. N either of them have had
She is prone to u pper respiratory tract infections and is
orthod ontic treatm ent, and neither of them have teeth like
u nable to breathe throu gh her nose. H er m other reports that
hers.

Examination
Extraoral
Jean has a Class I skeletal p attern w ith slightly increased
FMPA and no facial asym m etry. Lip s are incom p etent. There
are no signs or sym ptom s associated w ith the tem porom an-
d ibu lar joints.

Intraoral
■ The appearance o the teeth on presentation is shown in
Figs 15.1 and 15.2. Describe wha t you see.
Fair oral hygiene with generalized marginal gingival
Fig. 15.1 Case 1: anterior occlusion at presentation. erythema.
• 92
15 BILATERALCROSSBITE
Table 15.1 Possible causes o bilateral buccal crossbite
Bilateral buccal
crossbite Possible causes
Skeletal Mismatch in relative widths o arches or anteroposterior discrepancy
(commonlyassociated with Class IIImalocclusion)
So t tissues Possible role o adenoids/tonsils (see text)
Lowtongue position possiblydue to altered head posture
associated with mouth breathing
Scar tissue o cle t repair restraining growth in upper arch width

A
Radiographic
A d ental p anoram ic tom ogram w ou ld be u sefu l to d eter-
m ine the presence, p osition and form of u neru pted third
m olars. A lateral cephalom etric rad iograp h w ill also be
requ ired to ascertain the inclination of the u p p er and low er
incisors to their respective d ental bases.
The d ental panoram ic tom ogram ind icated fou r third
m olars of good form and p osition to be p resent.
The cephalom etric nd ings w ere as follow s:
SN A = 81°; SN B = 79°; MMPA = 28°; 1 to m axillary plane
= 113°; 1 to m and ibu lar plane = 93°; interincisal angle = 138°;
facial prop ortion = 56%.
B ■ What is your interpretation o these f ndings?
Fig. 15.2 (A) Case 1: right buccal occlusion. (B) Case 1: le t buccal Relative to m ean valu es for Cau casians, SN A is norm al; SN B
occlusion. is slightly increased ; AN B (SN A − SN B) = 2°, ind icating a
Class I skeletal p attern; MMPA is slightly increased ; 1 to
m axillary p lane is increased , so the up per incisors are
slightly proclined ; 1 to m and ibu lar p lane is norm al; interin-
654321 123456 cisal angle is slightly increased ; facial proportion is slightly
visible (please note 7 7 are also erupted).
7654321 1234567 increased .
All teeth visible appear caries- ree; amalgam restoration in 6 . All valu es are w ithin the norm al range for Cau casians.

Moderate lower arch crowding; 11 appear slightly small


(contact point displacements were 3 mm between 1 and 2; Diagnosis
also between 2 and 3); mild upper arch crowding.
■ What is your dia gnosis?
Class I incisor relationship; average and complete overbite;
upper and lower centreline shi ts (upper appears to the le t, Class I malocclusion on a Class I skeletal pattern with slightly
lower to the right); 2 2 in crossbite. increased FMPA and no acial asymmetry.

Molar relationship is Class I on right and Class III on le t with Generalized marginal gingivitis.
6543 456 Moderate lower labial segment crowding: mild upper labial
bilateral buccal crossbite a ecting .
6543 56 segment crowding with 2’s in crossbite; upper and lower
centreline shi ts.
■ What are the possible causes o bilateral buccal crossbite?
Molar relationship is Class I on the right and Class III on le t
Factors im plicated in the aetiology of a bilateral bu ccal
6543 456
crossbite are given in Table 15.1. with bilateral buccal crossbite a ecting .
6543 56
■ What is the IOTN (DHC) grade (see p. 264)? Expla in why.
Investigations
3d – d ue to contact point d isp lacem ent betw een 1 and 2;
■ What investigations would you underta ke in relation to the also betw een 2 and 3.
bilateral buccal crossbite? Explain why.

Clinical Treatment
It w ou ld be imp ortant to ascertain if there is an associated ■ What a re the aims o treatment?
m and ibu lar d isp lacem ent, althou gh this is rare w ith bilat-
To improve oral hygiene.
eral crossbites. It is m ore u su al to have a m and ibu lar d is-
placem ent associated w ith a u nilateral bu ccal crossbite. To correct the bilateral buccal crossbite.
N o m and ibu lar d isp lacem ent w as noted . To relieve upper and lower arch crowding.
93 •
BILATERALCROSSBITE 15
Table 15.2 Management options or bilateral buccal crossbite
Management option Indications Comments
Accept and monitor Patient not keen or correction. Not an option here as Jean is keen or correction
Part o underlying skeletal IIIproblemwhich is likelyto
worsen with mandibular growth, especiallyin males
Removable appliance with midline Primary/earlymixed dentition Rate o expansion must be quite slowand orce employed low: otherwise retention o
screwor heavymidline spring appliance compromised byhigher expansion orces
Compliance with wear and activation maybe problematic
Not cost e ective as o ten lengthytime required to produce desired expansion
Quadhelix Pre erred approach in earlymixed dentition Made o 1 mmstainless steel wire attached to bands cemented to molar tooth on each side
3–5 mmmaxillaryexpansion required (mainlydental but Delivers ewhundred grams o orce
some skeletal expansion)
Teeth pre erablytipped palatallybut molar inclination maybe Produces e cient slowexpansion
adjusted with fxed appliances later Mayderotate molars
Maybe adjusted to give more expansion posteriorlyor anteriorly
Maybe custom made or pre ormed types available
Activation hal a tooth width each side
Rapid maxillaryexpansion (RME) Child/adolescent Produces ~ equal amounts o dental/skeletal expansion
Minimal/no palatal tipping o the buccal segment teeth, i.e. Maybe undertaken using a banded or bonded appliance (the latter limits the amount o
skeletal crossbite downward backward rotation o the mandible)
>5 mmtransverse maxillaryexpansion required
Mild anterior maxillarycrowding The older the age at expansion, the less likelythe increase in vertical acial height will be
recovered bysubsequent growth
Surgicallyassisted rapid maxillary Skeletallymature patient with severe skeletal crossbite in Not an attractive option i urther maxillaryrepositioning in the anteroposterior or vertical
expansion (SARPE) whomsegmental expansion in Le Fort Iosteotomymight plane is required later
compromise blood supplyto segments
>5 mmtransverse maxillaryexpansion required

To align upper and lower arches with centreline


correction.
Key point
To correct the le t buccal segment relationship to Class I. • Mid-palatal suture opening by RME is almost 100%
guaranteed be ore age 15.
■ What treatment would you provide?
Oral hygiene instru ction by a hygienist. Provid ed the m ar-
ginal gingivitis is corrected then proceed to correction of the
■ Describe the design o the a ppliance you would use. What
bilateral bu ccal crossbite (the need for extractions for relief
instructions would you give Jean regarding this appliance?
of crow d ing and centreline correction shou ld be review ed
follow ing crossbite correction). The mechanism of m id -palatal su tu re sep aration is expan-
■ What options are there or management o the bilateral sion by a screw bu ilt into a xed ap pliance that is attached
buccal crossbite? Which would you choose? rigid ly to as m any teeth as p ossible.
The appliance m ay only com prise m etal or acrylic fram e-
The op tions are given in Table 15.2. In view of the severity w ork against the teeth, w hich d oes not contact the palatal
of the crossbite and the d esire to sim u ltaneou sly, if p ossible, m u cosa or m ay be m ad e w ith acrylic p alate-covering
im prove nasal breathing, rap id m axillary exp ansion (RME) shelves. The latter type m ay, in theory, p rod u ce m ore bod ily
w ou ld be the p referred choice of treatm ent. positioning of the alveolar p rocesses bu t m ay im pinge on
■ What actors should be checked be ore using this the palatal tissu es. For that reason, app liances that are tooth-
trea tment? What are the chances o opening the mid- borne are preferred .
palatal suture in this pa tient? The appliance used in Jean’s case w as as follow s
(Fig. 15.3):
It is p articularly ad visable to check that there is ad equate
bu ccal su pp orting bone and w id th of attached gingiva on • Activation: mid-palatal expansion screw (Hyrax).
all of the u p p er bu ccal segm ent teeth. Before the age of 15, • Retention: bands on 4’s and 6’s.
the chances of successfu l opening of the m id -palatal suture • Anchorage: 4’s and 6’s and the joining metal struts
are alm ost 100% bu t red u ce after that d u e to greater inter- between the bands on these teeth.
d igitation of the su tu res. • Baseplate: no palatal acrylic or buccal capping.
• 94
15 BILATERALCROSSBITE
m icrofractures of the interd igitating bone spicules. Because
closu re begins p osteriorly in the m id -p alatal su tu re and the
other m axillary stru ctu res also exert a bu ttressing effect in
this region, the sutu re opens w id er and faster anteriorly,
often p rod u cing a med ian d iastem a. Som e overcorrection is
ad visable (m axillary p alatal cu sps in line w ith m and ibular
bu ccal cu sps) as there is a strong relap se tend ency d u e to
palatal soft tissu e elasticity.

Key point
Rapid mid-palatal suture expansion:
• Exerts a orce o 10 20 lb (approximately 4.5 9 kg) over
2 3 weeks.
Fig. 15.3 Case 1: rapid maxillary expansion appliance with Hyrax • Produces ~0.5 mm movement per day.
screw. • Creates a median diastema.
• Has a strong relapse tendency, requiring overcorrection.

■ How will you retain the crossbite correction?


The exp ansion ap p liance shou ld be rend ered p assive and
remain in place for 3 m onths as a retainer. On its rem oval,
a rem ovable retainer w ith p alatal acrylic coverage shou ld
be tted unless fu rther treatm ent is being carried ou t im m e-
d iately, in w hich case a heavy expand ed m axillary archw ire
m ay be p laced for retention. A m od i ed transpalatal arch
w ith arms extend ing to the mesial of the 4’s or a 1 m m stain-
Fig. 15.4 Case 1: anterior occlusion ollowing RME and prior to
less steel archw ire throu gh the head gear tu bes are alterna-
proceeding to upper/lower xed appliance therapy.
tives w hile light w ires align the rem aining teeth. On
comp letion of treatm ent, an u p per H aw ley retainer (see
p. 123) or a tted p alatal arch m ay be p laced .
■ Wha t should Jean be advised o rega rding the e ects The occlu sion at rem oval of the RME ap p liance and
o expansion? im med iately p rior to proceed ing to further xed appliance
therap y is show n in Fig. 15.4.
As the su tu re expand s m ore anteriorly than posteriorly, an
u pper m ed ian d iastem a w ill u su ally d evelop w ithin d ays of
appliance activation (Fig. 15.3). Follow ing correction of the
bilateral crossbite and several m onths retention, d u ring
CASE2
w hich tim e the app liance is left in p lace, the pu ll of the
gingival bres and som e skeletal relapse w ill close the
SUMMARY
d iastem a (Fig. 15.4). Aidan is 19 years old. He presents concerned
With transverse m axillary expansion, som e m olar extru - about the appearance o his top teeth and his bite
sion is likely and cu sp al interferences are created that cau se (Fig. 15.5). How may it be treated?
the m and ible to rotate d ow nw ard and backw ard . This w ill
■ What are the main ea tures o note in Fig. 15.5?
red u ce the overbite.
■ Describe how the a ppliance works. Narrow maxillary arch.
Bilateral posterior buccal crossbite, extending to include 3 .
It separates the m id -palatal su tu re as if on a hinge at the
nasal base. Rap id exp ansion is achieved u sing forces of Upper labial segment crowding with incisor rotations; very
10–20 lb (ap proxim ately 4.5–9 kg) over 2–3 w eeks w ith the mild lower labial segment crowding.
patient tu rning the screw tw ice d aily (~0.5 m m m ovem ent Class III incisor relationship with 2 2 in crossbite (distal aspects
per d ay). This contrasts w ith slow expansion (force of 2–4 lb o 11 also in crossbite).
(0.9–1.8 kg) over abou t 2.5 months w ith the screw being
Upper and lower centreline discrepancy (appears that lower
tu rned once every other d ay, 1 m m per w eek). Minim al
centreline is shi ted to the right).
suture d isrup tion takes place w ith slow expansion.
With a screw d evice for rap id exp ansion, force is trans- Minimal overbite.
m itted rst to the teeth and then to the su tu re, prod u cing Lateral open bites.
95 •
BILATERALCROSSBITE 15

Fig. 15.5 Case 2: anterior occlusion at presentation.

History
Complaint
Aid an has been concerned for som e tim e regard ing his B
d ental ap pearance. H e now has a new job w ith a retail rm
and feels embarrassed w hen he sm iles. A p revious ortho- Fig. 15.6 (A) Case 2: expansion appliance in situ a ter surgery.
d ontist told him that he w ou ld need to w ait u ntil he w as in (B) Case 2: completion o alignment by xed appliances.
his late teens before anything cou ld be d one regard ing the
bite of his front teeth.
■ What makes a smile attractive?
Treatment
On sm iling, the full height of the u p per incisors shou ld be
seen (usu ally at rest, there is 1 m m incisor show in m ales ■ Why is RME not easible?
and 3 m m incisor show in fem ales). Although som e gingival
In ad u lts the m id -p alatal and lateral m axillary su tu res are
d isp lay is accep table, the interp roximal gingivae only
w ell-interd igitated , w hich afford increased resistance to the
shou ld be visible for op tim al aesthetics. The contou r of the
orthop aed ic-typ e p alatal exp ansion (RME) that m ay be used
u pper incisor ed ges should m atch that of the low er lip
in ad olescents.
(smile arc), w ith no u p p er incisor/ low er lip contact. In the
transverse d im ension, the sm ile shou ld inclu d e at m inim um ■ How is SARPE undertaken?
the u p per rst p rem olars. The d istance betw een the insid e Originally, surgically assisted rapid p alatal exp ansion
of the cheek and the m axillary p osterior teeth (bu ccal cor- (SARPE) w as u nd ertaken u sing bone cu ts only in the lateral
rid or), p articu larly the p rem olars, shou ld be narrow. m axillary bu ttress to d ecrease resistance su ch that in ad u lts
the m id -palatal suture cou ld be m icrofractu red by forced
opening. Althou gh this is com m only successfu l in p atients
u nd er 30 years old , in old er patients there is a risk of
Key point u nw anted fractures in other areas. Currently, the p roced u re
is often u nd ertaken u sing cu ts as for a Le Fort I osteotom y
An attractive smile comprises:
bu t w ithou t m axillary d ow n-fractu re. The m axilla is thu s
• Full length o the upper incisors. able to w id en in a p rocess akin to d istraction osteogenesis
• Related interproximal gingivae. (see p. 97) as resistance is provid ed by the soft tissues only.
• Upper incisor edges contour matches that o the The RME appliance is cem ented p rior to su rgery, and activa-
lower lip. tion of the screw u su ally com m ences w ithin 2 d ays at the
• Extends laterally to include no less than upper f rst same rate as for non-su rgically assisted RME. Fixed appli-
premolars. ances are required to com plete alignm ent (Fig. 15.6).
• Narrow buccal corridors. The im p roved occlu sion on com p letion of treatm ent is
show n in Fig. 15.7.
■ How stable is SARPE?
SARPE app ears to be m ore stable than su rgical w id ening of
the m axilla. The latter has a high relap se tend ency d u e to
Medical and dental history elastic rebou nd of the stretched p alatal m u cosa. Even after
Aid an is in good health. H e is u nable to breathe throu gh SARPE, w ear of a palate-covering retainer for at least the
his nose. rst p ost-su rgical year is recom m end ed to control relap se.
• 96
15 BILATERALCROSSBITE

A A

Fig. 15.7 (A) Case 2 post-treatment: anterior occlusion. (B) Case 2


post-treatment: le t buccal occlusion.

Key point
SARPE:
C
• Is indicated or bilateral posterior crossbite correction in
adults.
• Allows widening o the maxilla against only so t tissue
resistance.
• Appears to be more stable than surgical widening o
the maxilla.

CASE3
SUMMARY
Simon is 13 years old. He presents complaining
D
about the prominence o his top teeth and the
crooked lower back teeth (Fig. 15.8). What are the Fig. 15.8 (A) Case 3 at presentation: right buccal occlusion.
causes, and what options are there or (B) Case 3 at presentation: anterior occlusion. (C) Case 3 at
management? presentation: le t buccal occlusion. (D) Case 3 at presentation:
lower occlusal view.
■ What are the main eatures o note in Fig. 15.8?
Enamel racture in incisal third o 1 with composite
restoration ( 1 had su ered trauma 2 years previously but was Bilateral lingual crossbite (also re erred to as scissors bite or
symptom- ree). Brodie bite) a ecting 54 5.
Caries 5 distally.
■ Given the position o 5 45, what would you enquire about?
Mild upper arch crowding; moderate lower arch crowding.
It w ou ld be u sefu l to know if there had been early loss of
Class II division 1 malocclusion; increased overjet; slightly –
both low er E’s w hich w ou ld encourage m esial d rift of 6s
increased overbite. and lingu al d isp lacem ent of 5s.
Canine relationship hal -unit is Class II bilaterally. Sim on had E DE rem oved w hen he w as 7 years old .
97 •
BILATERALCROSSBITE 15
clip (Speed ) or retaining springs (Sm art-Clip ). These replace
Key point the stainless steel ligatures or elastom eric m od ules that are
Lingual crossbites are associated with Class II malocclusions. u sed (the latter m ainly) to retain the archw ire in the bracket
slot w ith conventional non-self-ligating system s. Claim ed
ad vantages inclu d e faster ligation, low er friction, faster
treatm ent, less pain and few er app ointm ents. Com p ared
w ith conventional pre-ad ju sted ed gew ise brackets, cu rrent
Extraoral p rospective evid ence for self-ligating brackets ind icates
Sim on has a Class II skeletal pattern w ith slightly red u ced no clinically signi cant d ifference in treatm ent d u ration
FMPA; there is no facial asym m etry. betw een the tw o system s; sim ilar effects on arch form for
N o m and ibular d isplacem ent or tem poromand ibu lar
joint signs or sym p tom s w ere d etected .
■ What are the causes o bilateral lingual crossbite? Table 15.4 Management options or bilateral lingual crossbite
These are given in Table 15.3. In Sim on’s case, a com bina- Option Possible indications
tion of local and skeletal causes are im plicated in the bilat- Extraction Single tooth on either side a ected and completelyexcluded
eral lingu al crossbite. lingually
■ What treatment options are there or correction o bilateral Reciprocal movement o Depends on inclination o teeth, number o teeth a ected and
lingua l crossbite? a ected upper and lower presence/absence o underlying Class IIskeletal discrepancy
teeth Fixed appliances required
It is im portant to ascertain the nu m ber of teeth affected and
to check the inclination of the affected teeth. These w ill help Growth modifcation An appliance that in uences so t tissue orce balance maybe
to ind icate the severity of any u nd erlying skeletal com po- use ul, eg, Frankel II
nent and w hether recip rocal m ovem ent of the affected teeth Orthodontic camou age Extraction/non extraction therapydepending on local/skeletal
w ith the op posing teeth only w ill su f ce for correction. components
Managem ent op tions are given in Table 15.4. An appliance that acilitates lower interpremolar expansion
In view of the u nd erlying Class II skeletal problem , maybe help ul, eg, Damon appliance
Simon w as treated rst by grow th m od i cation w ith a Midline distraction Severe mandibular crowding with well aligned upper arch,
Frankel II appliance follow ed by non-extraction u pper and osteogenesis v shaped mandible and narrowmandibular arch with bilateral
low er xed ap p liance therap y (Fig. 15.9). scissors bite a ecting several teeth
Orthognathic surgery Adult skeletal Class IIwith mandibular defciency

Key point
Correction o a bilateral lingual crossbite:
• Single tooth a ected on each side: consider extraction
o displaced teeth.
• Several teeth a ected on each side: consider
combination o buccal movement o a ected lower
teeth/palatal movement o a ected upper teeth.

■ Simon was treated with the f xed appliances shown in


Fig. 15.9. What type o a ppliance is this? What are its
claimed adva ntages compared with conventional f xed
appliance systems?
A
This is a self-ligating xed ap p liance (Dam on). The archw ire
is not pressed rm ly against the bracket base but is held in
place by either a rigid (as w ith the Dam on system ) or spring

Table 15.3 Causes o bilateral lingual crossbite


Local Bilateral earlyloss o E’s mayallow5s to be
displaced completelylingually
Skeletal Mismatch in relative width o arches or a Class II
skeletal discrepancy
Combination o local and skeletal B
Rarer causes Pierre Robin anomaly(mandibular retrognathia, cle t Fig. 15.9 (A) Case 3: mid-treatment with Frankel II.
palate, glossoptosis)
(B) Mid-treatment with xed appliances.
• 98
15 BILATERALCROSSBITE
each system ; m od est tim e-saving for tying and untying self- form ation of new bone (osteogenesis) and soft tissu e (his-
ligating brackets, bu t tim e saving varies w ith bracket d esign. togenesis) to create new p eriosteu m over the d istracted
More w ell-d esigned p rosp ective clinical trials u sing id enti- area. Althou gh in concept m and ibu lar m id line d istraction
cal w ire sequ ences and m echanics are requ ired to p rovid e is basically the sam e as SARPE, in contrast to the m axilla no
m ore robu st d ata regard ing self-ligating brackets. lateral surgical d isjunction is perform ed as the m and ible is
■ How does distra ction osteogenesis work, and what a re the not connected rigid ly to the skull. Distraction of the m and i-
complication risks o mandibular midline distraction? ble, therefore, w ill not be parallel bu t w ill alw ays rotate
arou nd the cond yles. Vertical cu ts are m ad e throu gh the
Based on the m anip u lation of a healing bone, d istraction m and ibu lar facial and lingual cortical p lates, usu ally extend -
osteogenesis stretches an osteotom ized site before calci ca- ing all the w ay through the sym physis. Follow ing a 5–7-d ay
tion has taken place to generate form ation of ad d itional latency period , d istraction begins by tu rning the screw
bone and investing soft tissu e. As there is insu f cient soft tw ice per d ay (0.5 m m ) u ntil the d esired m ovem ent is
tissue in the m and ibu lar sym physeal area to cover a bone achieved . The d istractor m ay be tooth-borne or screw ed to
graft necessary for w id ening the m and ibu lar sym physis by the bone; in the latter case, it is rem oved about 4 m onths
orthognathic surgery, d istraction afford s the op p ortu nity for post-operatively.
Com plications of m and ibu lar m id line d istraction, arising
w ithin 2 w eeks post-operatively, are relatively rare and
m ainly of a m ild or transient natu re. Only 3% of patients
experience m ore seriou s d amage d u e to fractured incisor
roots or gingival recession. Mand ibu lar m id line d istraction,
thu s, appears a relatively safe m ethod for expansion of the
m and ible.
Sim on’s nal occlu sion is show n in Fig. 15.10. A w ell-
interd igitating buccal occlusion w ill facilitate stability of
bilateral lingual crossbite correction.

Primary resources and


recommended reading
A Battagel JM 1996 Obstru ctive sleep apnoea: fact not ction.
Br J Orthod 23:315–324.
Flem ing PS, Johal A 2010 Self-ligating brackets in orthod ontics: A
system atic review. Angle Orthod 80:575–584.
Freem an DC, McN am ara JA Jr, Baccetti T et al 2009 Long-term
treatm ent effects of the FR-2 ap p liance of Frankel. Am J Orthod
Dentofacial Orthop 135:570.e1–e6.
H erold JS 1989 Maxillary exp ansion: a retrosp ective stud y of
three m ethod s of exp ansion and their long-term sequ elae. Br J
Orthod 16:195–200.
Lagravere MO, Major PW, Flores-Mir C 2005 Long-term d ental
B
arch changes after rap id m axillary exp ansion treatm ent: a
system atic review. Angle Orthod 75:155–161.
Kou d staal MJ, Poort LJ, van d er Wal KG et al 2005 Su rgically
assisted rap id m axillary exp ansion (SARME): a review of the
literatu re. Int J Oral Maxillofac Su rg 34:709–714.
Magnu sson A, Bjerklin K, N ilsson P et al 2009 Su rgically assisted
rapid m axillary exp ansion: long-term stability. Eu r J Orthod
31:142–149.
Parekh SM, Field s H W, Beck M et al 2006 Attractiveness of
variations in the sm ile arc and bu ccal corrid or sp ace as ju d ged
by orthod ontists and laym en. Angle Orthod 76:557–563.
Vig KWL 1998 N asal obstru ction and facial grow th: the strength
of evid ence for clinical assu m p tions. Am J Orthod Dentofacial
Orthop 113:603–611.
C
von Brem en J, Schafer D, Kater W et al 2008 Com p lications of
Fig. 15.10 (A) Case 3 post-treatment: right buccal occlusion. m and ibu lar m id line d istraction. Angle Orthod 78:20–24.
(B) Case 3 post-treatment: anterior occlusion. (C) Case 3 post-
treatment: le t buccal occlusion. For revision, see Mind Map 15, page 235.
16
p roblem s, but he w ond ers if they are m aking his low er front
teeth crooked .

Medical history
Graham is t and w ell.

Dental history
Tw o years ago Graham had a course of xed appliance

Late lower therapy to close a large space betw een 1 1, follow ed by


com p osite bu ild -u p of 2 2.

incisor crowding Examination


Extraoral examination
Graham has a Class I skeletal pattern w ith average FMPA
and no facial asym m etry. Lips are com petent. There are no
signs or sym p toms associated w ith the tem porom and ibu lar
joints.
SUMMARY
Graham is almost 20 years old. He presents with Intraoral examination
crowding o his lower incisors (Fig. 16.1). What is the The ap pearance of the teeth on p resentation is show n in
Figs 16.1 and 16.2.
cause, and how would you treat it?
■ What do you notice?

History So t tissues appear healthy with the possible exception o


mild gingival erythema related to 34 associated with slight
Complaint plaque deposits; otherwise oral hygiene appears good; slight
Graham is concerned abou t the crow d ing of his low er front gingival recession on 4.
teeth and w ond ers if it w ill get w orse. All lower permanent teeth present; 21 123456 visible in upper
arch (note 7 was erupted).
History o complaint Lower labial segment crowding with 8 8 erupting; worst
H is low er front teeth w ere straight until 18 m onths ago, incisor contact point displacement was 2.5 mm.
w hen he noticed crow d ing d evelop ing. H e now nd s it
Upper le t quadrant appears aligned.
m ore d if cu lt to keep his low er front teeth clean. Calcu lu s
build -up also seem s to occu r m ore easily on the insid e of Class I incisor relationship, although it appears to be tending
the low er teeth, w hich he nd s annoying. H e is also aw are towards Class III.
of the tw o w isd om teeth eru p ting at the back of his low er Le t buccal segment relationship is Class III.
jaw for the p ast 18 m onths. These d o not cau se him any
■ Is development o lower incisor crowding common in the
late teens?
In m od ern p opu lations, there is a strong tend ency for
crow d ing of the low er incisor teeth to d evelop in the late
teens. This occurs even if the teeth w ere w ell aligned or
spaced initially, lead ing usu ally to m ild crow d ing, w hereas

Fig. 16.1 Lower occlusal view at presentation. Fig. 16.2 Le t buccal occlusion.
• 100
16 LATELOWERINCISORCROWDING
initial m ild crow d ing tend s to becom e w orse. Low er labial
segm ent crow d ing also occu rs in patients in w hom extrac- Investigations
tions for relief of crow d ing and xed appliance therapy
■ What investigations would you undertake? Explain why.
have been u nd ertaken (u nless a bond ed lingu al retainer has
been placed ). Radiographic
■ Wha t are the possible causes o late lower incisor A d ental p anoram ic tom ogram w ou ld be u sefu l to assess the
crowding? position and inclination of the p artially eru pted low er third
The aetiology is m u ltifactorial and the m ajor theories p ro- m olars. As a rad iograp h w as taken prior to com m encing
posed are as follow s: orthod ontic treatm ent, this shou ld be consu lted rst, and
the need for a fu rther rad iograp h shou ld be assessed based
Late mandibular growth. Forward mandibular growth,
on the clinical assessm ent and the rad iograp hic nd ings.
especially i a growth rotation is also present, and where
maxillary growth has stopped, aided by lip pressure, tends to
Study models
reposition the lower incisors lingually, reducing arch length
and causing anterior crowding. Currently it is thought that as Upper and low er im pressions and a w ax registration in
the lower incisors, and perhaps the whole lower dentition, centric occlu sion shou ld be taken for stu d y m od els to record
move posteriorly relative to the mandibular body late in accu rately the current m alocclu sion and to aid fu rther treat-
mandibular growth, lower incisor crowding almost always m ent p lanning.
develops. The greater the magnitude o late mandibular ■ What do you notice on the denta l panoramic tomogram
growth when other growth has essentially stopped, the taken pre-orthodontic treatment (Fig. 16.3)?
greater the likelihood o developing late lower incisor Normal alveolar bone height.
crowding. On average, in a male patient, as is the case here,
mandibular growth is complete by 19 years o age, but it may All teeth present and o good quality.
continue or longer. 8 is slightly mesioangularly impacted.
Gingival and/or occlusal orces. Pressure rom the transseptal 8 is upright.
bres and/or the anteriorly directed component o occlusal Bone overlies the distal hal o the crown o both lower third
orces lead to mesial migration o the dentition. molars.
Lack o approximal attrition in modern diet. This was thought to
explain absence o lower labial segment crowding in Diagnosis
Australian aborigines but has not been borne out by research
ndings. ■ What is your dia gnosis?
Reduction in intercanine width. The mandibular intercanine Class I malocclusion on a Class I skeletal base with average FMPA.
width reduces gradually during the teenage years, and this
Lower incisor crowding.
decrease progresses at a reduced rate into adult li e. The
result is an increase in lower labial segment crowding, most Impacted third molars.
marked during the late teens. Le t buccal segment relationship is Class III.
The presence o third molars. Their role is controversial. Two ■ What is the IOTN DHC grade (see p. 264)? Explain why.
theories abound:
3d – based on the 2.5 m m w orst contact p oint d isplacem ent
• They may exert mesial pressure on the lower dentition betw een the low er incisor teeth.
during eruption, tending to produce lower labial ■ What a re the management options or la te lower incisor
segment crowding. crowding?
• They may prevent the lower teeth rom moving distally
in response to orces generated by mandibular growth or Accept and monitor. As this is a normal maturational change in
so t tissues. the lower arch, it would be sensible in a late teenager, and
However, late lower incisor crowding develops even when
third molars are absent so their presence is not the critical
actor in the aetiology o the problem. The amount o late
mandibular growth is paramount.

Key point
Possible actors in the aetiology o late lower incisor
crowding include:
• Late mandibular growth.
• Gingival and/or occlusal orces.
• Reduction in intercanine width.
• Third molars?
Fig. 16.3 Dental panoramic tomogram.
101 •
LATELOWERINCISORCROWDING 16
where the crowding is mild, to keep it under observation. low er labial segm ent alignm ent w hile p reserving the integ-
Where more marked crowding is present, intervention may rity of the bu ccal segm ents.
be considered. ■ Would you advise removal o the lower third molars?
Interproximal stripping. This is only acceptable in an adult with
Low er third m olars shou ld not be rem oved in an attem pt to
mild lower incisor crowding. By removing 0.25 mm at most
p revent further increase in low er labial segm ent crow d ing,
rom the mesial and distal aspects o each incisor, up to 2 mm
as their relation to this aspect of m alocclusion is u nproven.
o space can be obtained or relie o crowding. Incisor
Current gu id elines ad vise rem oval of low er third m olars
alignment may then be achieved by either a sectional xed
only if they are associated w ith recurrent ep isod es of peri-
appliance or a removable appliance. For ultimate aesthetics, a
coronitis or other p athology. In this case, as neither of these
lingual xed appliance may be used. A removable appliance
ap ply at p resent, the low er third m olars should be retained
clasped on the rst permanent molars, with an acrylated
and their p osition review ed if sym ptom s d evelop.
labial bow or Invisalign®(a clear vacuum- ormed
thermoplastic aligner) made to t a duplicate study model
with the incisors aligned with their mesial and distal sur aces
reduced, are alternative approaches. With Invisalign®, however, Key point
several appliances will be required to achieve incremental Management options or late lower incisor crowding:
change (~0.25 0.3 mm per aligner worn or about 20 hours • Accept and monitor.
per day and changed about every 2 weeks) until the nal • Interproximal stripping with appliance therapy.
alignment has been realized. A bonded retainer will be
• Lower incisor extraction with appliance therapy.
required to maintain the result long term.

■ Aside rom enhanced a esthetics, are there other


adva ntages o lingual appliances?
■ How would you manage the lower incisor crowding?
Tooth position is easier to d iscern, as the labial crow n is
clearly visible. There is also no risk of labial enam el d em - As the low er labial segm ent crow d ing is m ild , Graham
ineralization. There are, how ever, som e shortcom ings of should be ad vised to accept it for the p resent. It shou ld be
lingual orthod ontics and clear aligner therap y; these are kept und er review, and if the crow d ing increases, consid era-
given in Chapter 18, p age 112. tion could be given to treatm ent.
Extraction of a lower incisor. Where the low er labial crow d - The eru p tion of the low er third m olars shou ld be
ing is m arked , rem oval of a low er incisor m ay be ind icated m onitored .
to give su f cient space for alignm ent of the rem aining u nits.
It is ad visable to carry out a d iagnostic w ax-u p on a d up li- Primary resources and
cate set of stu d y m od els to ascertain w hat the nal resu lt is recommended reading
likely to be and to ensu re the p atient is hap p y w ith this
Dacre JT 1985 The long-term effects of one low er incisor
before p roceed ing w ith the extraction. Unless the incisor to extraction. Eu r J Orthod 7:136–144.
be extracted is com p letely exclud ed from the arch and the Ghaem inia H , Perry J, N ienhu ijs MEL et al 2016 Su rgical rem oval
remaining incisors are w ell aligned , xed ap p liance therapy versu s retention for the m anagem ent of asym p tom atic d isease-
is inevitably ind icated to d etail the p osition of the rem aining free im p acted w isd om teeth. Cochrane Database of Syst Rev
labial segm ent teeth. Bond ed lingu al retention w ill then be Issu e 8. Art N o: CD003879. DOI:10.1002/ 14651858.CD003879.
requ ired . p u b4.

■ Are there any possible undesirable e ects rom extraction H arrad ine N W, Pearson MH , Toth B 1998 The effect of extraction
of third m olars on late low er incisor crow d ing: a rand om ized
o one permanent lower incisor?
controlled trial. Br J Orthod 25:117–122.
The p atient shou ld be w arned of the p ossibility of tw o Little RM, Ried el RA, Artu n J 1988 An evalu ation of changes in
und esirable sequ elae: the u p p er labial segm ent m oving m and ibu lar anterior alignm ent from 10 to 20 years
palatally, w ith resu ltant m isalignm ent, in resp onse to the p ostretention. Am J Orthod Dentofacial Orthop 93:423–428.
low er labial segm ent being aligned slightly lingu ally and / N H S Centre for Review s and Dissem ination, York 1999
or a slight increase in overjet. Prop hylactic rem oval of im p acted third m olars: is it ju sti ed ?
Extraction of lower premolars. Where the bu ccal segm ent Br J Orthod 26:149–151.
occlu sion is w ell interd igitated and crow d ing is con ned to Richard son ME 2002 Late low er arch crow d ing: the aetiology
the low er labial segm ent, it is p referable to avoid low er review ed . Dent Up d ate 29:234–238.
m id -arch extractions becau se these w ill d isru p t the bu ccal
occlu sion. Instead rem oval of a low er incisor w ill exped ite For revision, see Mind Map 16, page 236.
17
History o complaint
Jocelyn has been m ore aw are of her p rom inent chin and her
bite since she w as in her last year at school. After consu lta-
tion w ith an orthod ontist at age 12, she had tw o u pper teeth
rem oved to p rovid e sp ace for the u p p er eye teeth (she had
both low er rst perm anent m olars extracted at age 8 becau se
of d ecay.) She d id not w ear any braces on her teeth. She w as
ad vised to w ait u ntil she w as in her late teens to have the

Prominent chin bite of her front teeth reassessed . In the past 6 m onths she
has becom e qu ite self-consciou s abou t her facial ap pear-
ance, althou gh she feels that her chin d oes not app ear to

andTMJDS
have becom e m ore prom inent in the past 4 years.
■ What questions would you ask a bout the
temporomandibula r joint pa in?
When and how the pain started.
Type and duration o pain.
Frequency.
SUMMARY Is it localized? Site o radiation.
Jocelyn, aged 23, is re erred by her general dental Associated symptoms, e.g. muscle pain, click, jaw locking,
practitioner because o her prominent chin (Fig. trismus.
17.1) and pain in her le t temporomandibular joint Aggravating actors, e.g. stress.
(TMJ). What are the causes and how would you Any habits, e.g. nail-biting, bruxism, pen-chewing.
manage these problems? Relieving actors, e.g. heat, analgesics (type and amount).
The p ain started su d d enly w hen Jocelyn w as p rep aring
History for exam inations in her rst year at u niversity. She has had
interm ittent d iscomfort in her left jaw joint since then, but it
Complaint has been of a m ild natu re. The d iscom fort is an ache that is
Jocelyn’s m ain concern is that she d oes not like the p rom i- p rincipally in the left jaw joint area but rad iates to the jaw
nent ap p earance of her chin and her u p p er teeth biting m uscles on that sid e. It d oes not keep her aw ake at night, bu t
insid e her low er teeth. She has pain in her left jaw joint and she feels it is w orse in the m orning. Typ ically it lasts for a few
has som e d if cu lty chew ing. She is also aw are that she has hours and then d isap p ears. It tend s to retu rn w hen Jocelyn
a lisp , w hich she d islikes. is stressed by w ork. She is aw are of grind ing her teeth w hen
stressed . She d oes not engage in chew ing pencils or p ens or
nail-biting. She feels that in the past year the p ain has
recu rred m ore frequ ently and has becom e w orse. Chew ing
hard food or op ening her m ou th too w id e m akes the p ain
w orse; one or tw o p aracetam ol tablets usually relieves the
ache. Jocelyn is also aw are that she has a jaw click.

Medical history
Jocelyn is t and w ell.

Dental history
Jocelyn is a regu lar attend er at her general d ental p racti-
tioner and bru shes tw ice d aily.

Family history
Jocelyn’s sister also has a prom inent chin, bu t not as m arked
as hers, and her sister ’s bite w as corrected w ith xed braces.

Examination
Extraoral examination
■ What do you observe rom Jocelyn’s prof le view (Fig. 17.1)?
Class III skeletal pattern w ith average FMPA. Com -
Fig. 17.1 Pro le at presentation. p etent lips.
103 •
PROMINENTCHINANDTMJDS 17
■ Based on the history, what other aspects would you
assess extraora lly?
Temporomandibular joints. Opening and lateral mandibular
movements should be assessed by rst observing the patient
rom the ront and second by palpation o the condylar heads
while listening or the presence o crepitus or a joint click. As
symptoms are present, the masticatory muscles should also
be palpated. A le t TMJ click and le t masseteric tenderness
were detected.
Mandibular path o closure. The path o closure rom rest
position to maximum interdigitation should be assessed,
noting any anterior or lateral mandibular displacement A
produced by a premature contact.
In this case there is an anterior m and ibu lar d isp lacem ent
7
on closu re on contact of (centric relation to occlusion
8
shift of ~3 m m ).

Intraoral examination
The intraoral view s are show n in Fig. 17.2.
■ Wha t do you see?
Gingival tissues appear healthy; gingival recession related
63 356 B
to .
7
Oral hygiene appears overall good, apart rom slight plaque
765321 12356
deposit on 3 are visible.
8754321 123457
No obvious caries.
Very mild lower incisor crowding with spacing between the
4’s and 5’s.
Very mild upper arch crowding with slight mesiolabial
rotations o 2 2.
Class III incisor relationship with reverse overjet (measured
4 mm clinically), reduced overbite and coincident dental
centrelines. C

Class I canine relationship bilaterally. Fig. 17.2 (A) Right buccal occlusion. (B) Anterior occlusion. (C) Le t
7 buccal occlusion.
Buccal crossbite a ecting .
8
■ What is the most likely cause o the considerable spacing
in the lower premola r area s with 5’s dri ted into contact
with 7 ’s?
w hich m ay transm it its effect by a p arafu nctional habit,
Early rem oval of 6E E6 in an u ncrow d ed arch is the m ost stem m ing from a d isplacing occlusal contact in su scep tible
likely exp lanation for the sp acing. An uncrow d ed low er 7
ind ivid u als. In this case a d isp lacem ent exists on on
arch is com m on in Class III m alocclu sion. 8
■ What occlusal eatures ma y predispose to closu re. There are also anterior and p osterior crossbites
temporomandibula r joint dys unction syndrome? p resent.
■ Why was Jocelyn advised to wa it until her late teens
Crossbites, Class III m alocclu sion and anterior open bite
or reassessment?
have been show n to have a signi cant association w ith tem -
porom and ibu lar joint d ysfu nction synd rom e (TMJDS) in In a Class III m alocclu sion, the am ou nt of reverse overjet
some stu d ies w hile others have fou nd no link betw een tend s to increase w ith forw ard m and ibu lar grow th d u ring
signs and sym p tom s of TMJDS and m and ibu lar d isp lace- teenage years. Waiting u ntil m and ibu lar grow th is essen-
m ent. The aetiology of TMJDS is m ultifactorial w ith the tially com pleted , w hich is generally abou t 17 years of age
im plicated involvem ent of psychological, trau m atic and in girls and 19 years of age in boys, has three ad vantages.
occlu sal elem ents. The m ost salient factor is p robably stress, First, it allow s treatment planning to be und ertaken w ith
• 104
17 PROMINENTCHINANDTMJDS
reasonably stable facial and occlu sal characteristics. Second , Table 17.1 Investigations required in combined orthodontic-surgical
if treatm ent is und ertaken, it safegu ard s against relap se d u e planning*
to further grow th. Third , the m agnitu d e of occlu sal change Investigation required Reason
d u e to m and ibu lar grow th in u ences w hether treatm ent Thorough clinical assessment o acial To locate anycranial, maxillary, nasal, mandibular
can be u nd ertaken by orthod ontic m eans alone or w hether ormin ull ace and profle or chin de ormities
a com bined orthod ontic-su rgical ap p roach is necessary.
To assess the height and width proportions o the
Orthod ontic treatm ent involves ind ucing d entoalveolar ace, interalar distance, nasolabial angle, upper
com p ensation for the und erlying skeletal p attern, bu t if incisor exposure, relation o upper dental midline
u nsuccessfu l d u e to continu ed ad verse m and ibu lar grow th, to other acial midlines, the ormand tone o the
the com pensation w ou ld need to be u nd one as p art of pre- so t tissues
surgical orthod ontics. This m ay involve op ening up low er Assessment o TMJ To record signs and/or symptoms o TMJ
prem olar extraction sp aces. H ence the d ecision to treat a dys unction. Treat these conservativelyi possible
Class III m alocclu sion in early teenage years by orthod ontic prior to treatment; i marked occlusal problems
cam ou age that inclu d es low er arch extractions m u st be contributing to dys unction, aimto correct these
m ad e w ith great cau tion. with treatment
In this case the severity of the anteroposterior skeletal Dental panoramictomogram(DPT) To assess the general dental status and prognosis
pattern, the likely pattern of mand ibu lar grow th, the am ou nt o the dentition as well as the position o
of d entoalveolar com p ensation, the am ou nt of overbite and unerupted third molars (bitewing or periapical
the relative absence of crow d ing (follow ing the rem oval of radiographs maybe required depending on
u pper rst prem olars and low er rst perm anent m olars) clinical and/or DPTfndings)
w ou ld all have been consid erations in d elaying any fu rther Lateral cephalometric radiograph To ascertain the aetiologyo the malocclusion
orthod ontic intervention. Whether the patient cou ld achieve and to acilitate surgical planning
an ed ge-to-ed ge incisor relationship w ou ld also need to Facial and dental photographs To record the acial and dental characteristics o
have been assessed . the malocclusion
To allowsurgical planning bymatching the
digital profle acial image with the lateral
cephalometrictracing
Studymodels and duplicates mounted To allowthorough orthodontic/occlusal
Key point on an articulator assessment and model surgery

Waiting until at least the late teens be ore considering an *Three dimensional (3D) acial images, where possible, and when combined with cone beam
orthodontic-surgical approach is desirable, as: computed tomography, allow3Dsurgical planning, although urther development is required to
• Facial and occlusal characteristics stabilize. optimize this.

• It sa eguards against relapse due to urther growth.


• Camou age or surgery is determined by the extent o
mandibular growth.

The cephalom etric values are as follow s:


SN A = 79°; SN B = 85°; AN B = −6°; 1 to m axillary p lane =
113°; 1 to m and ibu lar p lane = 82°; interincisal angle = 139°;
Investigations MMPA = 26.5°; SN to m axillary plane = 11°; facial p roportion
= 56%.
■ What investigations are required a nd why? ■ What is your interpretation o these f ndings (see p. 270)?
The patient’s prim ary concern is about her facial ap pear- Moderately severe Class III skeletal pattern due to a
ance, and she has a m alocclu sion w hich is likely to be combination o maxillary retrognathia and mandibular
u ntreatable by orthod ontic m eans alone. A com bined prognathism. As SNA is 2° less than the mean o 81°,
orthod ontic-surgical app roach is requ ired . The investiga- application o the Eastman correction adds 1° to the ANB to
tions requ ired , in su ch cases, together w ith reasons for give a revised ANB value o −5°.
their selection, are listed in Table 17.1. If possible, three-
Slightly proclined upper incisors (although inclination is
d imensional (3D) facial im ages, at rest and sm iling, shou ld
within the normal range) and markedly retroclined lower
also be record ed at this stage to act as a baseline from w hich
incisors indicating dentoalveolar compensation or the Class
to m onitor robu stly the 3D facial changes p rod u ced by
III skeletal pattern. The lower incisor angle should be 120°
surgery and d u ring su bsequ ent follow -u p . Sim ilarly, if fea-
− 26.5° = 93.5° but is 82°.
sible, cone beam comp uted tom ograp hy (CBCT) m ay be
u nd ertaken. Interincisal angle is increased but within the normal range or
• The TMJ assessment has already been undertaken or Caucasians.
Jocelyn (p. 103). MMPA, SN to maxillary plane and acial proportion are all
• The dental panoramic tomogram showed no condylar slightly out o the mean values but lie within the normal
pathology. range or Caucasians.
105 •
PROMINENTCHINANDTMJDS 17
and she w as given app ropriate oral hygiene instru ction
Diagnosis aim ed to p revent any d eterioration in the areas affected
by gingival recession. A pre-treatment gingival graft to the
■ What is your diagnosis?
low er incisors w as not d eem ed necessary; m aintenance of
Class III malocclusion on a Class III skeletal base with average good oral hygiene p ractices d u ring orthod ontic ap p liance
FMPA. therap y w as em phasized . Period ontal review w as arranged
TMJDS with le t TMJ click. 6 m onths into orthod ontic treatm ent.

Gingival recession on 63 356 and 7 . ■ How will this case be managed?


Previous loss o 6’s and 4’s. Short term A hard fu ll coverage u p p er acrylic sp lint shou ld
Very mild upper and lower labial segment crowding. be mad e, and Jocelyn shou ld be instructed to w ear this fu ll-
time u ntil the TMJ sym p tom s subsid e. She shou ld also be
All o the upper arch in crossbite with the exception
ad vised to take a soft d iet and to avoid straining her jaw
o 65 56.
joints by, for exam ple, yaw ning w id ely. Mild analgesics
7
Buccal crossbite on with associated mandibular should also be taken as requ ired .
8
displacement. Longer term A com bined orthod ontic-su rgical-restorative
ap proach is requ ired to correct the facial and occlu sal prob-
■ What is the IOTN DHC gra de (see p. 264)? Explain why.
lems. The TMJDS sym ptom s m ay ease w ith orthod ontic
5 m – d u e to reverse overjet greater than 3.5 m m w ith treatm ent. This is d u e to tooth m ovem ent rend ering the
reported m asticatory and speech d if culties. teeth tend er on biting, so parafu nctional activity stops
becau se tooth clenching/ grind ing d oes not prod u ce the
same su bconsciou s p leasu re as previously. The imp rove-
Treatment m ent in sym p tom s m ay be transient, even if d isplacing
7
occlu sal contacts (present here on ) are elim inated .
■ What are the aims o treatment? 8
The patient shou ld be w arned about the u np red ictable
• Relie o the TMJDS.
im pact of orthognathic su rgery on TMJDS to avoid u nrea-
• Control o the gingival recession.
sonable expectations of treatm ent.
• Correction o the underlying skeletal Class III
problem.
• Establish Class I incisor and molar relationships. Key point
• Correct the buccal segment crossbites. Orthodontic treatment and/or orthognathic surgery
• Restore the lower buccal segment spaces. cannot be guaranteed to eliminate TMJDS.

■ What trea tment is required? Explain why.


A com bined orthod ontic-su rgical-restorative ap p roach is ■ Explain how you would proceed with surgical planning or
need ed d u e to: this case.
The patient’s concern relating to acial and dental
1. A team approach is required involving the orthodontist
appearance.
and the oral and maxillo acial surgeon. The input o a
The severity o the underlying Class III skeletal pattern. restorative specialist is also required in relation to
Despite the degree o dentoalveolar compensation, a reverse management o the lower premolar spacing. The
overjet o 4 mm exists. involvement o a clinical psychologist with an interest in
An ANB angle o greater than −4° and lower incisor orthognathic surgical cases would also be help ul.
angulation to the mandibular plane o less than 83° Jocelyn must understand that to obtain the best acial
have been ound indicative o an orthodontic-surgical and occlusal result possible, xed appliance therapy is
approach. essential to the overall plan.
2. Surgical planning may be undertaken by various means.
Pre-surgical lower arch decompensation will open up more
To provide in ormation about the inter-relationships o
space between the premolars and then require restorative
the dento acial complex components, namely the
management to optimize the nal occlusion.
cranium and cranial base, nasomaxillary complex and
In addition, lower incisor decompensation may run the risk o maxillary dentition, mandible and related dentition,
creating gingival recession, particularly as Jocelyn already has specialized cephalometric analyses exist. These allow
several teeth a ected by minor gingival recession. A specialist comparison o individual cephalometric data with
periodontal opinion should be sought be ore embarking on ‘normal’data that should be matched or age, gender
pre-surgical orthodontics. and racial background. Computer programs then allow
It w as d ecid ed that the sp acing in the low er bu ccal seg- surgical, skeletal and dental movements to be planned
m ents w ou ld be m anaged by resin-retained brid gew ork and displayed visually on screen be ore printing.
follow ing u prighting of the prem olars. The p eriod ontist 3. By linking the patient’s digital pro le image with the
ad vised Jocelyn to ad ju st her toothbru shing techniqu e, cephalometric tracing, specialized planning so tware can
• 106
17 PROMINENTCHINANDTMJDS

A B C

Fig. 17.3 Another case: (A) Pre-treatment: pro le. (B) Super-imposition o lateral cephalometric radiograph tracing on a digital pro le
view and morphing using Dolphin®so tware to simulate the post-treatment pro le. (C) Post-treatment: pro le.

automatically morph the image in response to planned ■ Describe the phase o pre-surgica l orthodontics.
surgical and orthodontic movement undertaken virtually
This p hase of xed ap p liance treatm ent aim s to allow the
on the computer. The likelihood o di erent treatment
jaw s to be m oved to their d esired location w ithou t interfer-
options can thus be explored. In addition, the patient
ence from tooth positions. The u p per and low er arches are
can view the nal computerized prediction to appreciate
aligned and coord inated as w ell as establishing the vertical
more clearly the possible likely outcome on pro le
and anteroposterior p osition of the incisors. This involves
aesthetics, although it should be understood that this is
d ecom p ensating for any existing d entoalveolar com p ensa-
not guaranteed (Figs 17.3A–C). Using re erence lines to
tion. For Jocelyn, this w ill involve p rim arily labial m ove-
measure distances, planned surgical movements should
m ent of the low er incisors, w hich w ill elim inate the
then be trans erred to a duplicate set o study models,
m ild crow d ing and slight up righting of the upp er incisors.
mounted on a semi-adjustable articulator in this case as
The fu ll extent of the skeletal d iscrep ancy is thu s revealed ,
maxillary surgery is planned. The ability to plan
m axim izing the extent of possible su rgical correction.
orthognathic surgery in 3D (superimposing 3D acial
Class II interm axillary traction m ay be required to aid
images and CBCT scans) is being developed and o ers
d ecom p ensation. N o extractions are ind icated in this case to
exciting possibilities.
achieve the d esired tooth m ovem ents. The gingival statu s,
4. The nal plan should be explained to Jocelyn, ensuring
labial to the low er incisors, shou ld be m onitored d u ring
that she is aware that her pro le will be worsened by
d ecom p ensation to ensu re that gingival recession d oes not
pre-surgical orthodontics and that she realizes what her
ensu e.
nal acial appearance is likely to be. Provision o an
When the requisite tooth m ovem ents have been achieved ,
in ormation leaf et and DVD on orthognathic surgery will
rigid rectangular stainless steel archw ires shou ld be u sed to
also assist in recognizing the ull consequences o what
passively stabilize tooth position. Final p re-su rgical record s
treatment involves and are an important part o
– stu d y m od els, photograp hs and a cep halom etric lm –
obtaining in ormed consent.
w ill then be taken to assess the changes that have occu rred
and to d ecid e if the original surgical plan w ill be follow ed
or requ ire som e am end m ent. H ooks shou ld be attached
to the archw ires ju st p rior to su rgery u nless brackets w ith
Key point an integrated hook on each tooth have been u sed . The
Planning orthodontic-surgical treatment: hooks facilitate interm axillary xation and / or elastic trac-
• Requires a team approach. tion p ost-op eratively.
• May be assisted by matching the digital acial prof le
■ What surgical procedures are likely to be required?
image with the lateral cephalometric tracing and using
computer prediction so tware. Le Fort I advancement.
Mandibular setback.
107 •
PROMINENTCHINANDTMJDS 17
Key point
Pre-surgical orthodontics:
• May involve extractions.
• Usually involves decompensation or any dentoalveolar
compensation.
• Aligns and coordinates arches or arch segments.
• Establishes the vertical and anteroposterior position o
the incisors.
• Place rigid rectangular archwires with ball-hooks (unless
brackets with integrated hooks are used) immediately
pre-surgically.

■ What orm o splint and f xation is likely to be required? A


An interocclu sal acrylic w afer, fabricated to t articu lator
m ou nted casts positioned to the d esired occlu sal resu lt, is
recom m end ed rou tinely to ensu re accu racy of the p ost-
su rgical resu lt. Follow ing Le Fort I ad vancem ent and m an-
d ibu lar setback osteotom ies, m ini-p lates and m ini-plates/
screw s are likely to be requ ired resp ectively for xation.
Intermaxillary elastics then d irect the teeth into the requ ired
position and assist w ith jaw fu nction.

■ Describe the post-surgical orthodontic pha se.


With som e m ild jaw exercises, m ou th op ening is usu ally
satisfactory w ithin a few w eeks. Lighter, round stainless
steel archw ires shou ld then be p laced to allow for occlu sal B
settling, often aid ed by the u se of p osterior box elastics w ith
a slight anterior force vector, w hich help s m aintain the sagit- Fig. 17.4 (A) Post-treatment: pro le. (B) Post-treatment: occlusion.
tal correction. When good interd igitation has been achieved ,
elastic w ear shou ld be d iscontinu ed . Rarely w ill this p hase
of treatm ent take longer than 6 months to com plete. A
period of retention is then requ ired , no d ifferent from that
for other ad u lts w ho have com p leted rou tine orthod ontic Key point
treatm ent. Su rgical follow -u p shou ld be for a m inim um of Stability is enhanced when surgical movement is modest
2 years. and does not induce so t tissue tension.

■ What actors in uence post-surgical stability?


Stability w ill be in u enced by:
• Orthodontic and surgical plans being correct,
realistic, well-integrated and undertaken
competently.
Primary resources and
• Modest surgical movement no greater than recommended reading
6 mm in any direction in the maxilla or 8 mm in British Orthod ontic Society 2012 Tem p orom and ibu lar Disord ers
the mandible. This does not place the so t tissues (TMDS) and the Orthod ontic Patient. Ad vice sheet. British
Orthod ontic Society, Lond on.
under tension, and the condyles are not distracted
at surgery. Cevid anes LH C, Tu cker S, Styner M et al 2010 Three-d im ensional
surgical sim u lation. Am J Orthod Dentofacial Orthop
• Absence o tongue thrust, previous surgical
138:36–371.
scarring.
H ajeer MY, Millett DT, Ayou b AF et al 2004 Ap p lications of 3D
• Patient compliance with all aspects o treatment, im aging in orthod ontics: Part II. J Orthod 31:154–162.
particularly post-surgical wear o elastic traction. H u nt N P, Ru d ge SJ 1984 Facial p ro le and orthognathic su rgery.
• Adequate xation. Br J Orthod 11:126–136.
• The post-surgical pro le and occlusion are shown in Lu ther F 2007 TMD and occlu sion p art I. Dam ned if w e d o?
Fig. 17.4. Occlu sion: the interface of d entistry and orthod ontics; TMD and
• 108
17 PROMINENTCHINANDTMJDS
occlusion p art II. Dam ned if w e d on’t? Functional occlu sal Ryan F, Shu te J, Ced ro M et al 2011 A new style of orthognathic
p roblem s: TMD epid em iology in a w id er context. Br Dent J clinic. J Orthod 38:124–133.
202:E2, Br Dent J 202:E3.
Prof t WR, White R, Sarver D 2003 Contem p orary Treatm ent of
Dentofacial Deform ity. Mosby, St Lou is. For revision, see Mind Map 17, page 237.
18
Dental history
Iain has been a regu lar attend er at another d ental practice
for m any years before m oving to you r area. H e is highly
m otivated and d oes not w ish any tooth to be lost. 1 w as
trau m atized in his early tw enties and has becom e p rogres-
sively d arker, bu t he is u nconcerned by this. Iain had ortho-
d ontic treatm ent w ith xed ap p liances as a teenager becau se
his u pp er and low er teeth w ere crooked . N o teeth w ere

Dri ting incisors extracted , and he w ore rem ovable retainers for 1 year (6
months fu ll-tim e and 6 m onths part-tim e) follow ing treat-
ment. After he stopped w earing the retainers, his bite
changed and his orthod ontist said it w as d u e w as d u e to his
low er jaw grow ing m ore.

Medical history
Iain has d iabetes w hich is w ell controlled by insulin. H e is
otherw ise t and w ell.

SUMMARY Social history


Iain, a 51-year-old man, presents with spacing and Iain sm okes 10 cigarettes a d ay and has d one so for 30 years.

mobility o his upper incisors (Fig. 18.1). What is the


cause, and what can be done? Examination
Extraoral
History There are no p alpable subm and ibu lar or cervical lym ph
nod es.
Complaint
Iain com plains of the sp acing of his u pper front teeth and Intraoral
looseness of all his front teeth, p articu larly of 2 and 2. H e
is self-consciou s of the spacing and is w orried in case his ■ Wha t do you notice in Figs 18.1 a nd 18.2?
front teeth fall ou t. Oral hygiene appears air with generalized interproximal
staining. Interdental gingivae related to the lower central and
History o complaint lateral incisors appears slightly oedematous.
H e has noticed increasing m obility of his u p p er front teeth Generalized gingival recession.
over the p ast few m onths. The sp acing betw een the teeth
Heavily restored dentition with 1 discoloured.
ap peared at the sam e tim e and is becom ing p rogressively
w orse. There is no pain associated w ith the m obility of the 7654321 present in all quadrants; 8 also in lower right
teeth, bu t eating has becom e u ncom fortable. H e is also quadrant.
aw are of m obility of his low er front teeth and of several Mild lower incisor crowding.
u p per and low er back teeth. Recently he has exp erienced an
Uncrowded upper arch with incisor spacing.
u np leasant taste in his m ou th, w hich app ears to be d erived
from the u p p er front teeth. Class III incisor relationship with minimal overbite and overjet
except or 2 1; upper and lower centrelines are not coincident.
Class I molar relationship bilaterally.

■ Ba sed on wha t you know so a r, what are the possible


actors implicated with respect to mobility and dri ting
o 21 12?
Chronic periodontal disease is a de nite possibility when the
overall periodontal condition is observed.
Periapical periodontitis is another possibility, but this would
also tend to lead to extrusion o these teeth, which is not
markedly evident.
Root resorption. This would have to be extensive to produce
such spacing and mobility.
Root ractures. This is not a possibility in view o the history.
Other pathology such as radicular cyst or bony lesions are
Fig. 18.1 Occlusion at presentation. rarer possibilities.
• 110
18 DRIFTINGINCISORS

A B

C D

Fig. 18.2 (A) Lower occlusal view. (B) Upper occlusal view. (C) Right buccal occlusion. (D) Le t buccal occlusion.

■ Wha t would you check or specif ca lly in relation to


the history?
Degree o any tooth mobility.
Presence, extent and location o plaque and/or supragingival/
subgingival calculus deposits.
Fig. 18.3 Full-mouth periapical radiographs.
Presence/site o bleeding on probing.
Presence/site o periodontal purulent exudate.
Iain has right and le t group unction in lateral excursions.
Presence/site o a sinus and/or associated exudate. There are no occlusal inter erences in protrusion. There is no
Presence/site o any deep carious lesion. bruxism, pen-chewing or nail-biting habit.
Occlusal actors that may contribute to tooth mobility, e.g.
displacing occlusal contacts and/or a bruxing habit.
Investigations
Other habits, e.g. pen-chewing, nail-biting.
Calculus is present subgingivally on all teeth with generalized ■ What other investigations would you ca rry out? Why?
4 6-mm pocketing and delayed bleeding on probing. There is Full-m ou th periap ical rad iograp hs are requ ired to assess
a purulent discharge rom the periodontal pocket on the accu rately the period ontal status, particu larly the alveolar
mesial aspect o 2 . No sinuses are present. There are no bone height as w ell as the presence of any p eriapical
carious teeth. pathology.
Upper and lower molars, premolars and canines exhibit grade ■ Full-mouth periapica l radiographs are shown in Fig. 18.3.
I mobility buccolingually but not vertically. Upper and lower What do you see?
incisors have grade I mobility buccolingually but not vertically
Generalized horizontal bone loss o at least 50% with angular
except or 2, which exhibits grade II mobility buccolingually 2 1
and vertically. bone de ects a ecting .
63 26
111 •
DRIFTINGINCISORS 18
70 80% alveolar bone loss a ecting the lower incisor teeth. Referral for joint periodontal/orthodontic consultation. Iain’s
Heavily restored dentition, but no caries visible. cond ition requ ires sp ecialist care in view of the ad vanced
natu re of the p eriod ontal d isease.
■ Wha t periodonta l treatment do you envisage will
Diagnosis be required?
■ What is your diagnosis? Oral hygiene instruction, particularly in the use o
Chronic moderate periodontitis with localized areas o interproximal cleaning aids.
advanced disease. Full-mouth scaling and root planing.
Class III malocclusion with mild lower incisor crowding and Reassessment.
spacing/dri ting o the upper incisors. Consider localized surgery to those areas where response to
Upper and lower centreline shi ts. primary phase therapy is inadequate, i.e. bleeding/purulent
Buccal segment relationship is Class I bilaterally. exudate on probing persists.

■ With loss o periodontal attachment, how ma y labial ■ How would you describe the prognosis o Iain’s dentition?
dri ting o the incisors occur?
Prognosis d epend s on the response to initial therap y and
Alveolar bone loss compromises the ability o the teeth to p atient factors such as m otivation tow ard s m aintenance of
deal with so t tissue and occlusal orces, thereby leading to a very high stand ard of oral hygiene and cessation of the
tooth movement. sm oking habit.
A traumatic occlusion may result rom extrusion as a Based on the am ou nt of alveolar bone loss, the prognosis
consequence o periodontal disease. Dri ting may occur of the u pp er incisors is likely to be better than that of the
where periodontal support is also reduced. low er incisors.

Labial shi t o periodontally involved upper incisors may result ■ Wha t are the treatment options or the upper labial
also rom a displacing occlusal contact on closure producing segment spacing?
a orward mandibular slide. 1. Orthodontic alignment o the upper labial segment teeth
Undue orces may be placed on the incisors, where posterior with space closure. Six months a ter completion o
tooth support is lacking due to tooth loss, and these can lead periodontal therapy, the periodontist should re-evaluate
to upper incisor proclination. the periodontal status. Provided it has not deteriorated
and Iain is not averse to the prospect o wearing an
orthodontic appliance, orthodontic treatment could be
considered.
Key point 2. Extraction o 2 or o 2112 and replacement on a partial
Migration o periodontally involved incisors may be upper denture or adhesive bridgework. Extraction o 112
exacerbated by: may be required also with similar prosthetic replacement
• A orward mandibular displacement on closure. to the extracted upper units.
• Def cient posterior occlusal support. Iain op ted for xed ap p liance therap y to align the u p p er
teeth (Fig. 18.4).
■ Wha t options are there with regard to improving appliance
aesthetics in an adult pa tient?
■ What is the signif cance o the medical history a nd social
Options are aesthetic brackets (p olycarbonate or ceram ic),
history to the diagnosis?
clear aligner therap y (Invisalign ®) and lingu al orthod ontics.
Diabetes affects the host resp onse to p eriod ontal p athogens Each, how ever, is not w ithout its shortcom ings (Table 18.1).
by altering polym orph chemotaxis. Althou gh Iain’s d iabetes
is w ell controlled , p eriod ontal d isease is chronic and gener-
alized d u e to inad equ ate oral hygiene m easu res aggravated
by the host resp onse.
Sm oking contribu tes signi cantly to p eriod ontal d isease
throu gh a variety of m eans. Gingival blood ow is red uced ;
salivary ow is slow ed also, lead ing to p oorer rem oval
of p eriod ontop athic bacteria and encou raging calcu lu s
bu ild -up .

Treatment
■ What treatment would you advise?
Cessation of smoking. The patient should be encouraged to
stop sm oking to p revent any fu rther insu lt to his p eriod on-
tal health im posed by this habit. Fig. 18.4 Upper xed appliance.
• 112
18 DRIFTINGINCISORS
Table 18.1 Shortcomings o aesthetic orthodontic appliances
Treatment option Shortcomings
Aestheticbrackets
Polycarbonate brackets Stain, de orm, poor torque control (improved with metal insert)
Ceramic brackets Cannot bond to composite unless a silane coupling agent is
used, which increases bond strength and risko enamel
damage at debond (overcome bydi erent base designs to
allowmechanical bond)
Enamel damage at debond (problemovercome with di erent
base design but must ollowmanu acturer’s instructions
specifc or di erent bracket designs)
Fig. 18.5 Final occlusion (bonded palatal retainer in place).
Bracket breakage
Wear o opposing enamel
Increased riction compared with metal (overcome byinsertion Regu lar p eriod ontal recall shou ld be u nd ertaken
o metal slot) throu ghou t orthod ontic treatm ent.
Clear aligner therapy Control o root movement and possible intermaxillarycorrection
Due to the loss of p eriod ontal attachm ent and alveolar
limited unless bonded attachments added appropriately bone, p erm anent bond ed retention w ill be required .
The nal occlu sion is show n in Fig. 18.5.
Extraction site space closure di cult
Lingual appliances Tongue discomort
Speech adjustment Key point
Di cult to place (indirect bonding required) and adjust For orthodontic treatment in a periodontally involved
(operator/patient time and cost implications)
dentition:
• Avoid bands.
• Use light orces.
■ Wha t specia l considerations are there with orthodontic • Ensure regular periodontal recall during treatment.
treatment in a periodontally compromised dentition? • Retain permanently.

Band s shou ld be avoid ed d u e to the risk of further com p ro-


m ising p eriod ontal su p p ort by p lacem ent of the band
m argin su bgingivally, creating a nid u s for p laqu e accu m ula-
tion. Bond ed attachm ents, therefore, shou ld be p laced on all
Primary resources and
teeth inclu d ing m olars. It is also p referable to u se stainless recommended reading
steel ligatu res rather than elastom eric m od u les to retain Gu stke CJ 1999 Treatm ent of period ontitis in the d iabetic p atient.
orthod ontic archw ires, as m od u les attract higher levels of A critical review. J Clin Period ontol 26:133–137.
plaqu e m icro-organism s. Joffe L 2003 Invisalign ®: early experiences. J Orthod 30:348–352.
If several p osterior teeth have been lost, anchorage for Joss-Vassalli I, Grebenstein C, Top ou zelis N et al 2010
the tooth m ovem ents required m ay not be su f cient and Orthod ontic therap y and gingival recession: a system atic
reinforcem ent w ith a bond ed transpalatal arch or a tem po- review. Orthod Craniofac Res 13:287–297.
rary anchorage d evice m ay be necessary. Johnson GK, H ill M, 2004 Cigarette sm oking and the p eriod ontal
With red u ced alveolar bone su p p ort, light forces shou ld p atient. J Period ontol 75:196–209.
be applied to the teeth. N attrass C, Sand y JR 1995 Ad u lt orthod ontics – a review.
When gingival recession is evid ent p re-treatm ent, the Br J Orthod 22:331–337.
patient shou ld be w arned as to the p ossibility of this being
aggravated by orthod ontic tooth m ovem ent. For revision, see Mind Map 18, page 238.
19
Appliance related
problems
A

CASE1
SUMMARY
Owen, an 8-year-old boy, presents or a routine
check o his upper removable appliance. You notice
a reddened palate (Fig. 19.1A). What is your
diagnosis and how would you manage this?

History
B
Complaint
Ow en’s only com plaint is that his up per rem ovable ap pli- Fig. 19.1 (A) Appearance o the palate at presentation. (B) Upper
ance is loose. H e is not aw are of any p roblem w ith his removable appliance.
p alate.
■ Describe the appearance o the palate.
History o complaint
The lesion affecting the p alate can be d escribed as follow s:
Ow en has been w earing the upp er rem ovable appliance for
the p ast 8 w eeks to correct a crossbite on 1. The current Site palatal mucosa and attached gingivae.
ap pliance has been getting progressively looser over the Size area a ected relates primarily to that covered by the
p ast m onth. baseplate o the upper removable appliance (Fig. 19.1B).
Shape the outline is de ned by, and posteriorly extends
Medical history slightly beyond, the shape o the baseplate.
Ow en is asthm atic and has used a salbutam ol (Ventolin)
Colour uni ormly reddened appearance o the palate and
inhaler for the p ast 4 years. H is asthm a is w ell controlled .
attached gingiva primarily underlying the baseplate.
Background mucosa and gingivae not covered by the
Examination baseplate appear o normal colour except an area in the
palatal midline beyond the baseplate.
Extraoral ■ Wha t are your observa tions rega rding the upper removable
Ow en has a Class I skeletal pattern w ith average FMPA appliance?
and no facial asym m etry. H is lips are com p etent, w ith
the low er lip ju st covering the incisal third of the u pper The appliance has Adams clasps on 6 6 and D D with a Z
anterior teeth. There are no tem p orom and ibu lar joint signs spring to 1. The distal arrowhead on D clasp and the
or sym p tom s. mesial arrowhead on 6 clasp do not engage the undercuts
optimally.
Intraoral The baseplate has posterior capping over 6ED DE6.
The soft tissu es ap p ear norm al excep t for generalized m ild Appliance hygiene does not appear as it should ood debris
m arginal gingivitis and the p alatal m u cosa, w hich is show n and plaque deposits are visible, through the baseplate, on the
in Fig. 19.1A. Oral hygiene is fair. tting sur ace o the appliance.
• 114
19 APPLIANCE-RELATEDPROBLEMS

■ Wha t is the most likely diagnosis based on the in orma tion Saliva sampling or Candida counts.
you have so a r? Culturing or accurate identi cation and sensitivity testing
Palatal (d entu re) stom atitis, as the p atient is sym p tom - may also be undertaken.
free. ■ What stains identi y Ca ndida?
■ Wha t other condition would produce a similar appea rance?
Gram stain: Candida is strongly Gram positive.
Acrylic allergy. This, how ever, is u nlikely as a ‘bu rning’ Periodic acid Schi (PAS): the magenta stain locates
sensation of the m ucosa und erlying the baseplate w ou ld carbohydrate in ungal cell walls.
have been rep orted early after insertion of the ap p liance and
there w ou ld be erythem a of all of the soft tissu es ad jacent
to the acrylic. Treatment
■ What is the aetiology o ‘denture’ stomatitis?
■ How would you treat this condition?
Candida is the princip al cause. Althou gh a norm al oral com -
m ensal, p roliferation of Candida is facilitated by local envi- Ow en shou ld be ad vised to:
ronm ental or system ic factors, thereby allow ing it to becom e 1. Leave the appliance out at night. Due to the in ection, it
pathogenic (Table 19.1). would be wise to wear the appliance throughout the day
but remove it at night until the palatal mucosa returns to
health.
Key point 2. Improve appliance hygiene brush the tting sur ace
and soak it in a 1% hypochlorite solution.
Candida is the principal cause o palatal stomatitis related
to an upper removable appliance. 3. Improve oral hygiene teeth, gingivae and palatal
mucosa should be brushed thoroughly a ter every
meal.
4. Reduce sugar intake alter diet to low carbohydrate
■ Wha t actors in this case ma y have predisposed Owen to consumption.
‘denture’ stomatitis?
5. Anti ungal agents (nystatin or amphotericin suspension
The u se of a steroid inhaler, p oor app liance hygiene and or miconazole oral gel) should be applied to the tting
fu ll-tim e w ear of the ap p liance are likely to be the m ain sur ace o the appliance our times daily. A 0.2%
contribu tory factors. A d irect relationship has been show n chlorhexidine mouthwash may also be bene cial due to
betw een the p resence of an up p er rem ovable ap p liance, its anti ungal e ect.
Candida and low salivary p H levels. In ad d ition, u p per As the asthm a is w ell controlled , there is no need to refer
removable ap p liance therap y has a p ositive, thou gh tran- Ow en to his general m ed ical p ractitioner.
sient, in u ence on the p revalence of Candida and d ensity of
oral cand id al carriage, su ggesting that the carrier state m ay
be initiated by the appliance.
Key point
Management o palatal stomatitis:
Investigations
• Leave appliance out at night.
■ How would you conf rm the diagnosis? • Improve appliance and oral hygiene.
The ideal investigation is a smear rom the palatal mucosa • Reduce carbohydrate consumption.
underlying the baseplate or microbiological testing. • Anti ungal agents.

Table 19.1 Causes o denture stomatitis


Factor Aetiology ■ What is the prognosis or this condition?
Local In ection with Candida (~90%due to Candida albicans) Provid ed all the above strategies are follow ed , the cond ition
Poor denture/appliance hygiene shou ld resolve com p letely w ithin a few w eeks.
Night time wear o denture/appliance
Possible trauma
Poor salivaryclearance o oral commensals
CASES2 AND3
High sugar intake providing substrate or Candida proli eration SUMMARY

}
Systemic Ironandvitamindefciencies Two common f xed appliance problems are shown.
Steroids What is the cause o each, and what treatment
predispose to
Drugs that cause xerostomia Candida in ection would you provide?
Endocrine abnormalities, e.g. diabetes ■ What problem do you notice in Fig. 19.2A?
Antibiotictherapy
A bracket has becom e d ebond ed from 5 .
115 •
APPLIANCE-RELATEDPROBLEMS 19
Table 19.2 Causes o bracket bond ailure
Factor Aetiology
Operator Insu cient etch time
Poor etch pattern*
Poor moisture control during bonding
Non adherence tomanu acturer’s instructions with the bonding adhesive
chosen
Movement o the bracket a ter initial placement, which inter eres with bond
ormation within the adhesive
Application o high orce to the bracket to accomplish archwire engagement
Patient Eating hard/sticky oods
Possiblyuse o phenolic containing mouthrinses that so ten composite
Occlusal trauma/bruxing habit
A Pen chewing/nail biting

*Etch pattern is poorer on premolars than on canine or incisor teeth. This contributes to the greater
bond ailure rate on premolars (note ailure in Fig. 19.2Ais on 5).

Fig. 19.2 (A) Lower occlusal view. (B) Following removal o ligature
wire and 5 bracket. (C) Following replacement o ligature wire
B
and rebonding o a 5 bracket.
Fig. 19.3 (A) Upper occlusal view. (B) Following shortening and
adjustment o upper archwire.

■ Why ha s this occurred?


case, how ever, this risk is less likely as the bracket rem ains
Bond failu re m ay occu r for variou s reasons (Table 19.2). attached to a ligatu re w ire and elastom etric m od u le.
Should the loose bracket be a source of d iscom fort to the
p atient, as a general d ental p ractitioner, you cou ld rem ove
Treatment the loose bracket and the loose ligatu re. The p atient
■ What trea tment would you provide? Explain why. should be ap pointed w ith their orthod ontist at the earliest
op portu nity.
The p atient shou ld be ad vised rst to contact their ortho-
■ Wha t problem do you notice in Fig. 19.3A?
d ontist for rep air of the ap p liance (Figs 19.2B,C).
If this is not p ossible and the loose bracket is at risk of The cheek m u cosa is ulcerated d u e to trau m a from the over-
being sw allow ed or inhaled , it shou ld be rem oved . In this extend ed rou nd archw ire.
• 116
19 APPLIANCE-RELATEDPROBLEMS

■ How has this problem arisen?


The am ount of projection of the archw ire beyond the m olar
band m ay have been overlooked at the tim e of archw ire
placem ent, or the archw ire m ay have m oved to its current
position follow ing insertion, as the teeth aligned or d u e to
loss/ bond failu re of another ap p liance com p onent.
■ How would you manage this problem?
As a general d ental p ractitioner (GDP), you shou ld , rst,
ad vise the patient to contact their orthod ontist to d eal w ith
this p roblem . If this is not p ossible then you shou ld provid e
emergency care.
A
■ As a GDP, what emergency ca re would you provide?
The d istal end of the archw ire shou ld be cut u sh w ith the
terminal aspect of the m olar tube or the archw ire end tu rned
u nd erneath the m olar tube or angled inw ard aw ay from the
cheek. As a rou nd nickel titaniu m w ire is in p lace, w ith the
terminal end heat-treated (note blackened ap p earance), this
is easily cu t to the d esired length or p ositioned to avoid
cheek trau m a (Fig. 19.3B).
The patient should be ad vised to m aintain a high stand -
ard of oral hygiene and to rinse w ith lu kew arm salty w ater
after m eals u ntil the u lceration heals. Provision of a su p ply
of soft w ax to ap p ly over the m olar tu be and the ad justed
w ire end m ay also be help fu l in the intervening p eriod . The
patient shou ld also sched u le an app ointm ent in the near
fu tu re w ith their orthod ontist.
Fig. 19.4 (A) Lower retainer. (B) Lower occlusal view showing
retainer halves abutted; arrow indicates racture line.
Key point
Emergency treatment or:
teeth are also p ossibilities. A w eakness in the retainer d uring
• Loose bracket: remove i risk o ingestion/inhalation.
m anu factu re is another op tion.
• Overextended archwire: cut wire and/or turn wire end
Accid ental d am age is d if cu lt to p revent, but on issu ing
inward away rom the cheek.
the retainer the p atient shou ld have been instru cted to take
care w hen rinsing or cleaning the retainer to avoid it being
d rop p ed and d am aged ; w ear of the retainer for eating is also
not requ ired . Flexing the retainer is to be avoid ed ; w hen not
CASES4 AND5 in u se, it should be stored in the hard , p lastic retainer box
provid ed . The low er alginate im p ression should have been
checked for any tray contact of the low er incisors or ‘d rag-
SUMMARY ging’ that cou ld com p rom ise the m anufactu ring process.
Two problems with retainers are shown. How might In this case the retainer had been accid ently stood on
each o these have occurred, and what treatment after being d ropp ed d u ring rinsing. It had only been issu ed
would you provide? 2 w eeks prior to the incid ent, w hen the low er xed appli-
ance w as rem oved .
■ What problem do you notice in Fig. 19.4A?
There is a m id line fractu re of a low er vacu u m -form ed
(Essix) retainer.
Treatment
■ How might this problem have occurred? How may it have ■ As a GDP, what treatment would you provide? Expla in why.
been prevented?
The p atient shou ld rstly be inform ed to m ake contact w ith
This is a clean-cu t fractu re rather than a small crack and , as his or her orthod ontist to explain the situation and arrange
su ch, is u nu su al w ith a vacu u m -form ed retainer. Crazing of to be seen as soon as possible for a replacem ent retainer. If
the retainer or sm all cracks are m ore typ ical. Extraorally, this is not possible, you shou ld check each part of the
accid entally d ropp ing the retainer w hile cleaning or rinsing retainer to see if the pieces t together and , if so, rinse and
it, follow ed by an im pact from a sharp or heavy object, is d isinfect them before assessing the t in the m ou th. As the
possible; rep eated exu re of the retainer w ou ld also lead to pieces abutted w ell (Fig. 19.4B), they could be w orn as a
reakage. Intraorally, eating hard food s w ith the retainer in tem p orary m easu re to p revent any relap se u ntil a rep lace-
situ or ‘biting’ the retainer into p lace w ith the op p osing m ent retainer is issued . Until a rep lacem ent retainer is tted ,
117 •
APPLIANCE-RELATEDPROBLEMS 19
d ay-tim e w ear, ap art from eating and d rinking, w ou ld be
p referable to night-tim e w ear, as there is a risk of one or
both parts of the retainer becom ing d islod ged and p erhap s
being sw allow ed . If, how ever, the orthod ontist is agreeable,
an alginate im pression could be taken for a rep lacem ent
retainer, w hich you m ay t w ith his or her gu id ance. A
follow -u p ap p ointm ent w ith the orthod ontist shou ld be
sched u led for the near fu tu re.
■ Could such a problem have been preempted a t debond?
A
At d ebond , it is ad visable to issu e the patient w ith tw o
vacuu m -form ed retainers to allow for inad vertent breakage
or loss. The sp are retainer can be w orn u ntil another ap point-
m ent is possible w ith the orthod ontist, at w hich tim e a new
sp are vacu u m -form ed retainer can be m ad e from the m od els
constru cted w hen the xed ap p liance w as rem oved .
■ What problem (arrowed) do you notice in Fig. 19.5A?
The com p osite overlying the w ire on 2 has been lost exp os-
ing the w ire. (N ote the retainer w ire had previously frac- B
tured in the m id line; the w ire end s had been sm oothed and
covered w ith com p osite.) Fig. 19.5 (A) Lower occlusal view. (B) Anterior view.
■ How has this occurred?
Eating sticky or hard food s or chew ing gu m is the m ost
likely cau se. alignm ent w ith clear aligner therapy and replacem ent of the
■ How will you manage the problem? bond ed retainer w ould be optim al.

Enqu ire from the p atient w hen the exp osed w ire on 2 w as
rst noticed , w hether there are p resently or have been any
problems w ith this, how long the retainer has been in p lace Key point
and if the orthod ontist still sees the p atient for review. Emergency treatment or:
The patient rst noticed this 2 w eeks ago after chew ing
• Broken vacuum- ormed retainer: arrange replacement.
a toffee; it rubs the tongu e and has p rod u ced a sm all u lcer.
• Bonded retainer: composite repair/smooth wire end or
The retainer fractu red in the m id line 2 years ago, at w hich
arrange replacement.
time it had been in p lace for 1 year; the w ire end s w ere
sm oothed and covered w ith comp osite. Since then there
have been no problem s. The p atient is d u e for a review w ith
the orthod ontist next year. As such, the p atient should
contact the orthod ontist to have the retainer assessed w ith Primary resources and
regard to repair or replacem ent as soon as possible.
recommended reading
■ In the meantime, what would you do to improve Arend orf T, Ad d y M 1985 Cand id al carriage and p laqu e
patient com ort? d istribu tion before, d u ring and after rem ovable orthod ontic
ap p liance therap y. J Clin Period ontol 12:360–368.
As 2 is still aligned w ith the ad jacent teeth and com posite
remains attached to the enam el and und ersurface of the H obson RS, Ru gg-Gu nn AJ, Booth TA 2002 Acid -etch p atterns on
the bu ccal su rface of hu m an p erm anent teeth. Arch Oral Biol
w ire, a sm all com p osite ad d ition cou ld be m ad e to the w ire
47:407–412.
on 2 . The w ire su rface should be cleaned gently w ith
Mand all N A, Millett DT, Mattick CR et al 2002 Orthod ontic
pu mice, the com p osite rou ghened slightly on 2 w ith a green
ad hesives: a system atic review. J Orthod 29:205–210.
stone, then resin and com p osite ap p lied and light-cu red . An
Patel A, Sand ler J 2010 First aid for orthod ontic retainers. Dent
ap pointm ent shou ld be arranged w ith the orthod ontist in
Up d ate 37:627–730.
the near future.
Wilson J 1998 The aetiology, d iagnosis and m anagem ent of
■ What urther trea tment do you think will be required? d entu re stom atitis. Br Dent J 185:380–384.
In light of the m inor shifts in alignm ent that are now visible
on several teeth (Fig. 19.5A and B), rem oval of the retainer, For revision, see Mind Map 19, page 239.
20 B

A
A

Tooth movement and


related problems
Fig. 20.1 Diagram illustrating zones o pressure (A) and tension
(B) in the periodontal ligament induced by a tipping orce.

CASE1
SUMMARY
Darren, a 13-year-old boy, has been undergoing Key point
upper removable appliance therapy or 6 months Tipping movement, typically, requires orces o 30 50 g or
to retract and align 3 , ollowing extraction o 4 . 0.3 0.5 N.
Tooth movement has been very slow with no
movement recorded at the last two visits.
■ What are the possible reasons or a slow rate o orthodontic
tooth movement?
These can be d ivid ed broad ly into p atient, ap p liance and Pressure zones The cellu lar response d epend s on w hether
op erator factors. a light or heavy force is ap plied . With a light sustained force,
Patient actors Non-compliance with instructions regarding tooth m ovem ent occu rs w ithin a few second s as p eriod ontal
wear o the appliance. ligam ent uid is squeezed ou t and the vascu lar sup ply is
Incorrect positioning o the spring on appliance insertion or com pressed , setting off a com p lex biochem ical response.
distortion o the spring. Within 2 d ays, osteoclast invasion occu rs and frontal resorp -
tion follow s.
Contact o the tooth with the buccal cortical plate or with a
When a heavy sustained force is app lied , the period ontal
retained root part o 4 .
ligam ent is com p ressed to su ch a d egree that the blood
Occlusal inter erence rom the opposing arch. su p p ly is cu t off com p letely, prod ucing an area of sterile
necrosis (hyalinization). Sm all areas of hyalinization are
Appliance actors Acrylic and / or w ire m ay be interfering
inevitable, even w ith light forces, bu t the area of hyaliniza-
w ith tooth m ovem ent.
tion is extend ed w ith forces of greater m agnitu d e. Osteo-
Operator actors Design aw s, u nd eractivation, overacti- clast d ifferentiation is im p ossible w ithin the necrotic
vation or activation of the sp ring su ch that 3 is d irectly period ontal ligam ent space, bu t after several d ays osteo-
buccally into contact w ith the cortical plate rather than clasts app ear ad jacent to and w ithin the ad jacent cancellou s
through the cancellous corrid or of bone. sp aces. From there they invad e the bone ad jacent to the
■ What orce ra nge is optima l or retraction o 3 by tipping hyalinized area and tooth m ovem ent eventually occurs by
movement? und erm ining resorp tion.

The op tim al force range is 30–50 g or 0.3–0.5 N . Tension zones Follow ing initial ap plication of a light force,
the blood vessels vasod ilate and the p eriod ontal ligam ent
■ What cellula r response is there ollowing activation o the
bres are stretched w hile broblast and preosteoblast p ro-
spring to retract 3 by tipping movement?
liferation occu rs. The stretched bres become em bed d ed in
The consequ ence of activating the spring is to set u p zones osteoid , w hich later m ineralizes. The norm al period ontal
of pressu re and tension w ithin the period ontal ligam ent. ligam ent w id th is eventu ally regained by sim u ltaneous col-
H alf of the p eriod ontal ligam ent is stressed w ith m axim u m lagen bre rem od elling.
pressu re created at the alveolar crest in the d irection With heavy forces, rup tu re of blood vessels and severing
of m ovem ent and at the d iagonally op posite ap ical area of the p eriod ontal ligam ent bres are likely, bu t both are
(Fig. 20.1). restored w ith the rem od elling p rocesses.
119 •
TOOTHMOVEMENTANDRELATEDPROBLEMS 20
Key point
Application o a sustained orce to a tooth creates areas
o pressure and tension within the periodontal ligament
which ultimately lead to bone resorption and deposition,
respectively.

■ What is the mechanism or tooth movement?


Althou gh the histological response to an app lied orthod on- Fig. 20.2 Periapical radiograph.
tic force has been investigated extensively, the m echanism
by w hich a m echanical stim u lu s is transferred to a cellu lar
response is com p lex and is at present u nresolved . It is likely
that vascular changes in the p eriod ontal ligam ent in areas
of p ressu re and tension, electrical signals in response to ■ Wha t treatment would you advise?
alveolar bone exing follow ing force ap p lication, as w ell as
Com p lete retraction of 3 is u nlikely d u e to the p resence of
prostagland ins and cytokine release interact in this p rocess.
this root fragm ent. H ow ever, its rem oval w ou ld requ ire a
■ How would you manage the problem in this case? surgical proced ure, w hich w ould rem ove a consid erable
Managem ent is sp eci c to each cau se. The action taken m ay amou nt of alveolar bone and m ay d am age the roots of the
be one or more of the follow ing: ad jacent teeth. The root fragm ent m ay resorb in tim e and
The need or ull-time wear o the appliance must be becom e con u ent w ith the alveolar bone.
emphasized to the patient/parent. They must be made aware In view of the surgical risks, it w ou ld be w ise to leave
that treatment will stop unless ull cooperation is the root fragm ent in situ, monitor its status rad iographi-
orthcoming. cally and accep t the lim itation this p oses to com p lete retrac-
tion and alignm ent of 3 . The p atient should be ad vised
Show the patient how to insert the appliance with the spring
accord ingly.
positioned correctly and explain the action o the spring.
Ensure that the appliance is not being removed by the spring.
Check the activation o the spring and adjust it appropriately. CASE2
Remove any acrylic or wire obstruction to tooth movement.
This may require a remake o the appliance to an improved SUMMARY
design. Alan is reviewed 3 months a ter completion o a
Add to a f at anterior biteplane or buccal capping to 2-year course o upper and lower f xed appliance
disengage the occlusion, i this is hampering tooth
therapy. He is wearing upper and lower removable
movement.
retainers at night only. On removal o the retainers,
Ask the patient and check his or her case notes or any
you detect grade 2 mobility o the upper incisors
recorded problem with 4 extraction. I indicated, take a
periapical radiograph o 4 area to check or a retained root
and grade 1 mobility o all other teeth anterior to
ragment. I this is detected, seek an oral surgical opinion and including the f rst permanent molars in both
regarding its removal or whether it could be le t in situ and its arches. His oral hygiene is good, and there is no
position monitored radiographically. bleeding on probing. You order a dental panoramic
Check 3 is not ankylosed. This is unlikely in this case as some tomogram.
movement o 3 has occurred since 4 was extracted. ■ Why is the ra diogra ph ordered?
Darren admitted to intermittent wear o the appliance and, in It w ill allow a general screen of alveolar bone height and
particular, to leaving it out at meals. He was advised root length of all teeth.
accordingly.
■ Wha t do you notice on the f lm (Fig. 20.3A)?
Generalized ap ical blunting (root resorp tion) of all teeth,
Key point anterior to and inclu d ing the rst perm anent m olars w ith
the p ossible excep tion of the second p rem olars. The u p per
Full-time wear o an upper removable appliance is required
incisors ap p ear to have the m ost root resorp tion.
or an optimal rate o tooth movement.
■ Which teeth experience most orthodontically induced
root resorption?
The u pper incisors, follow ed by the low er incisors and
■ What do you notice on the peria pical radiograph o
rst p erm anent m olars, have consistently m ore root resorp-
another ca se (Fig. 20.2)?
tion than other teeth irrespective of suggested genetic or
There is a sm all retained ap ical root fragm ent of 4 . treatm ent-related risk factors.
• 120
20 TOOTHMOVEMENTANDRELATEDPROBLEMS
evid ence sup p orts the u se of light forces, esp ecially for
incisor intru sion.
Alan su stained coronal fractu res to 1 1, involving
enam el and d entine, 3 m onths before starting orthod ontic
treatm ent.

Key point
Root resorption:
• Increases in incidence and severity with comprehensive
orthodontic treatment.
• Is promoted by the use o heavy orces.
A

■ Could root resorption ha ve been prevented?


Root resorption is regard ed as an u navoid able sequ ela of
orthod ontic tooth m ovem ent. Althou gh, on average 1 m m
of root length w ill typ ically be lost over a 24-m onth cou rse
of orthod ontic treatm ent, w id e ind ivid u al variation exists.
Determ ining risk factors to id entify those su sceptible to
orthod ontically ind uced root resorp tion, and the m eans by
w hich its severity and prevalence m ay be red uced , requires
B
fu rther w ell-d esigned clinical research stu d ies.
Du ring treatm ent excessive force ap p lication shou ld be
avoid ed . Where early root resorp tion is d etected on progress
rad iograp hs, taken 6–12 m onths into treatm ent (Fig. 20.3B),
fu rther root resorp tion m ay be red u ced by a 2–3-m onth
p ause (w ith a p assive archw ire). Where severe root resorp-
tion is d etected (>4 m m or one-third of the original root
length), the treatm ent goals should be reconsid ered w ith the
p atient and other options explored .
C ■ What must the orthodontist ensure be ore
treatment commences?
Fig. 20.3 (A) Dental panoramic tomogram. (B) Periapical
radiographs o upper incisors taken 6 months into treatment. The risk of apical root resorp tion as a consequ ence of ortho-
Note no apical root resorption apparent on 12, but slight apical d ontic treatm ent m u st be exp lained to the p atient/ p arents
resorption o 21 roots. (C) Periapical radiographs indicating and d iscu ssed w ith them. In ad d ition, an informed consent
marked root resorption o 2112. form signed by p atient/ p arents and the orthod ontist m u st
d escribe, in p articular, the risk of apical root resorption.
Treatm ent shou ld not be offered unless the anticip ated ben-
■ What risk a ctors have been suggested in rela tion to e ts far ou tw eigh the risk of m inor ap ical resorp tion seen
orthodontica lly induced root resorption? in most p atients.
Suggested patient-related risk factors inclu d e patient age Ap prop riate rad iographs shou ld be available, and the
and sex, root shap e, history of p reviou s trau ma, teeth w ith need for progress rad iograp hs, as requ ired , shou ld be
short roots and previous root resorp tion, cortical bone p rox- explained .
imity to the root, genetic factors and system ic factors.
Su ggested treatm ent-related risk factors inclu d e length Key point
of treatm ent, force m agnitu d e and m ethod of ap p lication,
• Root resorption is a common consequence o
d irection of m ovem ent, ap p liance typ e, treatm ent m echan-
orthodontic treatment.
ics and extent of ap ical m ovem ent.
• Explain the risk to the patient be ore treatment starts
■ What does current evidence suggest with regard to
and obtain consent.
orthodontica lly induced root resorption?
• Monitor radiographically as required.
System atic review s ind icate that previous traum a and
u nusu al root m orphology are u nlikely causes; root lled
teeth d o not app ear to be m ore vulnerable than contralateral ■ What would you do in this case?
vital teeth. An increased incid ence and severity occu rs w ith
comp rehensive orthod ontic treatm ent w ith total root resorp - Ask the patient i he is aware o marked mobility o any teeth
tion red u ced by a 2–3-m onth pau se in treatm ent. Bracket and/or any other symptoms.
prescrip tion, self-ligation and archw ire sequ ence d o not Clinically, the mobility o all teeth should be recorded. Alan
affect this. H eavy forces prod u ce m ost root resorp tion, and was aware o upper incisor mobility.
121 •
TOOTHMOVEMENTANDRELATEDPROBLEMS 20
Enquire about bruxism or other habits, such as nail-biting.
None were reported.
Sensibility testing o the incisors and canines should be
undertaken. All teeth were responsive to sensibility testing
and no marked di erences in recordings were detected
between each tooth and its opposite number in each arch.

■ What treatment would you provide?


Alan shou ld continu e w ith night tim e w ear of his retainers.
The retention p lan m ay need to be re-evalu ated d ep end ing
on the resorp tion statu s of the incisors (see below ).
Periap ical rad iograp hs shou ld be taken of the u pp er inci-
A
sors as these exhibit grad e 2 m obility (Fig. 20.3C). Follow -u p
rad iograp hic examinations and sensibility tests are recom -
m end ed at 6 m onths to ascertain if the ap ical root resorp tion
is progressive. This, how ever, shou ld be u nlikely as ‘active’
orthod ontic treatm ent has been conclu d ed . In the u nlikely
event that fu rther resorp tion is observed at follow -u p, rad i-
ograp hic exam ination is recom m end ed u ntil the resorp tion
is stabilized . In the long term (10–25 years) after orthod ontic
treatm ent, teeth w ith root length of ≥10 m m and a healthy
period ontiu m have been show n to rem ain stable.

CASE3 Fig. 20.4 (A) Right buccal occlusion at presentation. (B) Anterior
occlusion at presentation
SUMMARY
Lisa, an 18-year-old girl, had previous extraction o
lower f rst premolars and f xed appliance therapy ■ Why has this occurred?
to success ully correct her Class III malocclusion.
The follow ing factors are likely to d estabilize the nal ortho-
Both upper second premolars were congenitally d ontic resu lt.
absent. On removal o the f xed appliances 1 year
Forces rom the supporting tissues Reorganization of p rin-
ago, a palatal bonded retainer was placed rom 3 cipal p eriod ontal ligam ent bres and su p p orting alveolar
to 3; she was also issued an upper Hawley retainer, bone occu rs w ithin 4–6 months after cessation of active
designed to f t over the bonded retainer, and a tooth m ovem ent. At least 7–8 m onths, how ever, are requ ired
lower vacuum- ormed retainer. Both removable for the su p racrestal bres to reorganize becau se of the
retainers were to be worn at night only. slow tu rnover of the free gingival bres. Rotational correc-
tion and sp ace closure are, therefore, liable to relap se.
■ What do you notice in Figure 20.4 A and B?
Pre-treatm ent view s of Lisa’s m alocclusion are show n in
So t tissues appear healthy; 6 seems to be restored with an Figs 20.5A and B. N ote the rotated teeth and spacing of the
amalgam restoration. arch.
Lower arch is reasonably aligned with a small space between Forces rom the oro acial so t tissues Follow ing ap pliance
5 and 3 . therapy, the teeth shou ld be in a p osition of soft tissu e
Right upper buccal segment and labial segment aligned with balance. The original m and ibu lar arch form shou ld rem ain
the possible exception o 6 . u nchanged as m arked alteration in the inclination of the
low er incisors w ill prom ote relapse. Lim ited p roclination
Spacing between 1 1 and 1 and 2 with slight mesiolabial
rotation o 1 and 2. of the low er labial segm ent, how ever, m ay be stable in
Class II d ivision 2 m alocclu sion, or if the low er incisors
Average overjet; overbite average to slightly reduced but w ere retroclined by a thu m b-sucking habit or by a low er
complete; mild centreline discrepancy (lower appears shi ted lip trap .
to the le t by about 2 mm). If a thu m b- or d igit-su cking habit is not ceased before
Molar relationship is Class I with crossbite a ecting the treatm ent com m ences, its p ersistence w ill prom ote overjet
mesiobuccal cusp o 6 . relapse.
One-third to half of the labial su rface of the u p p er inci-
■ What does this indica te?
sors should be covered by the low er lip to give the best
There has been relap se in the p osition of Lisa’s teeth follow - chance of stable overjet correction. Where the lips are grossly
ing treatm ent. incom p etent post-treatm ent, the u pp er incisor position w ill
• 122
20 TOOTHMOVEMENTANDRELATEDPROBLEMS

Key point
Relapse may be due to:
• Forces rom:
• Supporting tissues.
• Oro acial so t tissues.
• Occlusion.
• Facial growth.
• Inappropriate retainer design or retention regime, and/
or patient compliance.
A
Pre-treatment:
• Discuss relapse and retention as part o in ormed
consent.
• Highlight patient’s responsibility to adhere to long-term
retainer wear.

Lisa ind icated that she had w orn the rem ovable retainers
for a few m onths at night but then had d iscontinu ed w ear.
The u p p er bond ed retainer had com e aw ay from 13 2 w eeks
B ago, and the w ire had fractu red betw een 1 1. She w as now
concerned abou t the sp aces that had d evelop ed and that her
Fig. 20.5 (A) Right buccal occlusion (pre-treatment); note rotations front teeth w ere ‘tw isting’ back to the w ay they u sed to be.
o 621 1 and spacing. (B) Anterior occlusion (pre-treatment); note
■ What designs a re there o f xed and removable retainers?
incisor spacing and rotations.
Several d esigns of xed retainers exist, althou gh in the
low er arch this is m ost com m only either a sm ooth, rou nd
w ire w ith sand blasted end s bond ed to the canines only or
a mu ltistrand / spiral w ire bond ed to each of the incisors and
be inherently u nstable, and the mechanism by w hich an canines ind ivid u ally (Fig. 20.6). In the upp er arch, a m ulti-
anterior oral seal is achieved w ill aggravate this further. strand w ire is com m only bond ed to the incisors only or to
the canines and incisors. For either arch, a 020 in m u lti-
Occlusal actors A poor buccal segm ent interd igitation w ith
strand w ire is recom m end ed . Bond ing is u sually w ith light-
d isp lacing occlu sal contacts and an u nfavou rable interin-
cu red com posite. Rem ovable retainers m ay either be
cisal angu lation w ill encou rage instability.
vacu um -form ed (Fig. 20.7) or have a w ire labial bow
Post-treatment acial growth Facial grow th continues into (u su ally H aw ley or Begg typ e: Fig. 20.7; Fig. 20.8A); the
ad u lt life and , althou gh of lesser m agnitu d e than that latter m ay be acrylated to im prove app earance (Fig. 20.8B).
observed d uring child hood , it varies am ong ind ivid u als. On ■ What does current evidence indicate with regard to
average, fem ales tend to d em onstrate a backw ard m and ibu- post-orthodontic retention?
lar rotation, and this w ill not aid overjet stability. Late facial
grow th also im pacts on the d evelopm ent of late low er A recent upd ate of a Cochrane review found that m ost
incisor crow d ing. stud ies on types of retainers and d u ration of retainer
w ear w ere of low qu ality. Based on a sm all nu m ber of
Retention plan An inap p rop riate d esign of retainer, reten- p articip ants in one w ell-cond ucted short term (6 m onths)
tion regim en and / or inad equ ate com pliance by the patient stud y, mod erate qu ality evid ence ind icates that there is no
w ith the prescribed retainer w ear w ill facilitate relap se. d ifference in stability betw een fu ll-tim e or p art-tim e w ear
Firm ru les, how ever, d o not exist for retention follow ing of therm oplastic retainers. There is, how ever, insu f cient
active tooth m ovem ent; instead the retention plan should high qu ality evid ence w ith regard to retention p rotocols and
be d ecid ed ind ivid u ally for each case. H ow long this shou ld p roced ures follow ing orthod ontic treatm ent to m ake rm
be for is unknow n at present and probably unlikely to be recom m end ations.
ascertained d u e to the m u ltip licity of com p lex factors
■ How ca n relapse be prevented long-term?
involved in m alocclu sions and their treatm ent, as w ell as
the inherent d if culties in u nd ertaking clinical trials on the N ight-tim e w ear of a rem ovable therm oplastic retainer
m any likely retention strategies. It is acknow led ged that the shou ld continu e long-term , althou gh w ear on alternate
only m eans of guaranteeing that the p ost-treatm ent resu lt nights red uced to biw eekly m ay be suf cient w ith the onu s
rem ains u nchanged long-term is to ad vise long-term reten- on the p atient to m onitor occlu sal change.
tion. This should be exp lained to the patient by the ortho- Du e to the u npred ictability, bu t high likelihood , of late
d ontist before treatm ent starts as a p art of inform ed consent; low er incisor crow d ing d eveloping post-treatm ent (see
im portantly, the p atient’s resp onsibility in the retention Chapter 16), instead of ad vising long-term w ear of a rem ov-
process m u st also be em phasized . able retainer, a bond ed lingu al retainer m ay be tted to the
123 •
TOOTHMOVEMENTANDRELATEDPROBLEMS 20

Fig. 20.8 (A) Upper Begg retainer. (B) Upper Hawley retainer with
an acrylated labial bow.

■ What specif c indications are there or a f xed (bonded)


retainer over a remova ble retainer?
A xed bond ed retainer is ind icated in the follow ing cir-
cu mstances: w hen the lip s are incom p etent after correction
of an increased overjet; follow ing sp ace closu re in spaced
d entitions (Figs 20.5A,B; also inclu d es m ed ian d iastem a,
B Chap ter 1); w here the anterop osterior p osition of the low er
labial segment has been rad ically altered ; after alignm ent of
Fig. 20.6 (A) Lower bonded retainer (multistrand wire). (B) Upper severely rotated teeth (Fig. 20.5) or of an im p acted u p p er
bonded retainer (multistrand wire) with Hawley retainer. incisor (Chap ter 2) or p eriod ontally involved d entitions
(Chap ter 18); cleft lip and p alate (Chapter 21) and w hen
m inim al overbite exists follow ing incisor crossbite correc-
tion (Chap ter 10). In ad d ition, w here a com prom ised
occlusion w as the treatm ent goal, albeit w ith im proved
app earance, xed retention shou ld be used .

Key point
A bonded retainer is indicated ollowing correction o :
• Severely rotated teeth and spaced dentitions.

■ Ha s the general dental practitioner (GDP) got a role with


Fig. 20.7 Lower vacuum- ormed retainer and upper Hawley regard to orthodontic retention?
retainer. The GDP has a general inform ative and im portant su pport-
ive role w ith regard to p ost-orthod ontic retention and
low er incisors and canines; increasingly this is left in place retainer w ear by their patients. This is su m m arized in
perm anently to m aintain low er incisor alignm ent. Table 20.1
■ Wha t oral hygiene mea sures should the patient be
Key point instructed to ollow a ter placement o a bonded retainer?
In the short-term, night-time wear o a thermoplastic In ord er to m aintain optimal oral hygiene in relation to the
removable retainer seems su cient to maintain tooth bond ed retainer, the patient shou ld be tau ght how to effec-
alignment a ter f xed appliance therapy (except in cases tively use interd ental cleaning aid s such as Su per oss and
with high relapse potential when a bonded retainer is interd ental bru shes.
required). Instru ctions in relation to a rem ovable (Essix) retainer are
given in Chapter 5.
• 124
20 TOOTHMOVEMENTANDRELATEDPROBLEMS
Table 20.1 Role o the general dental practitioner (GDP) in relation 3. A compromise plan would be to use a sectional clear
to post-orthodontic retention and retainer wear aligner to the upper labial segment teeth only. Then,
When Role Speci c aspect o retention prolonged retention would be required to maintain
Re erral/pre treatment In orm Retention vital component o treatment the result, as outlined in option 2 above, with the
vacuum- ormed retainer con ned solely to the labial
Post treatment at Rein orce Need or retainer wear and howto maintain
segment.
review optimal oral and retainer hygiene
As Lisa is now a u niversity stud ent, she w as not keen for
Removable retainer Confrm Retainer wear as advised
fu rther xed ap p liance therap y and op ted for sectional
Ensure good ft Adjust as required aligner therapy follow ed by prolonged retention.
Repair/replace? *Depends i still under reviewwith
orthodontist Primary resources and
Bonded retainer Check Wire/composite, integrityo bond and
related oral hygiene
recommended reading
Brezniak N , Wasserstein A 2002 Orthod ontically ind u ced
Repair *Seekadvice, as required, romorthodontist in am m atory root resorp tion. Part I. The basic science aspects.
regarding repair o broken/debonded retainer Part II: The clinical asp ects. Angle Orthod 72:180–184.
*It is important when the patient is no longer under reviewbythe orthodontist, but still wearing H arts eld JK Jr, Everett ET, Al-Qaw asm i RA 2004 Genetic factors
retainers, that the orthodontist communicates with the GDPwith regard to the retainer type and in external ap ical root resorp tion and orthod ontic treatm ent.
retention protocol. Crit Rev Oral Biol Med 15:115–122.
H ennem an S, Von d en H off JW, Maltha JC 2008 Mechanobiology
of tooth m ovem ent. Eu r J Orthod 30:299–306.
Johnston CD, Littlew ood SJ 2015 Retention in orthod ontics.
Key point Br Dent J 218:119–122.

Following placement o a bonded retainer, instruction Jonsson A, Malm gren O, Levand er E 2007 Long-term follow -u p of
tooth m obility in m axillary incisors w ith orthod ontically
must be provided in relation to interdental cleaning.
ind u ced ap ical root resorp tion. Eu r J Orthod 29:482–487.
Kotescha S, Gale S, Kham asha–Led ezm a L et al 2015 A
m u lticenter au d it of GDPs know led ge of orthod ontic retention.
Br Dent J 218:645–653.
■ What management options are there or Lisa’s problem?
Littlew ood SJ, Millett DT, Dou bled ay B et al 2016 Retention
The follow ing options exist: p roced u res for stabilising tooth p osition after treatm ent w ith
1. Accept and monitor study models should record the orthod ontic braces. Cochrane Database of Syst Rev Issu e 1.
current tooth position and occlusion to allow or Art N o: CD002283. DOI: 10.1002/ 14651858.CD002283.p u b4.
comparison with the pre- and post-treatment study casts Little RM 2009 Clinical im p lications of the University of
and to act as a baseline rom which to assess any urther Washington p ost-retention stu d ies. J Clin Orthod 43:645–651.
occlusal change. I there is urther relapse, then consider Melrose C, Millett DT 1998 Tow ard a p ersp ective on orthod ontic
retention? Am J Orthod Dentofacial Orthop 113:507–514.
options 2 or 3 below.
Pand is N , Vlahopou los K, Mad ianos P et al 2007 Long-term
2. Re-treatment ull case assessment is required with
p eriod ontal statu s of p atients w ith m and ibu lar lingual xed
study models and photographs; radiographs are not
retention. Eu r J Orthod 29:471–476.
indicated. For comprehensive recorrection, xed
Ren Y, Maltha JC, Ku ijp ers-Jagtm an AM 2003 Op tim u m force
appliance therapy is an option, but clear aligner therapy
m agnitu d e for orthod ontic tooth m ovem ent: a system atic
with Invisalign is also likely to produce a satis actory literatu re review. Angle Orthod 73:86–92.
outcome due to the mild nature o the relapse; the slight Roberts-H arry D, Sand y J 2004 Orthod ontics. Part II: orthod ontic
centreline shi t could be accepted with the latter tooth m ovem ent. Br Dent J 196:391–394.
treatment option. Long-term bonded retention will be Walker SL, Long DT, Flores-Mir C 2013 Rad iograp hic com p arison
required, therea ter, to the upper labial segment; it would of the extent of orthod ontically ind u ced external apical root
also be advisable to provide a vacuum- ormed retainer to resorp tion in vital and root- lled teeth: a system atic review. Eu r
t over the bonded retainer. Although this dual retention J Orthod 35:796–802.
( xed and removable retainer) strategy was adopted Weltm an BJ 2011 External root resorp tion and orthod ontic
ormerly, it must be emphasized to Lisa that should the treatm ent—assessm ent of the evid ence. In: H u ang GJ,
bonded retainer partially debond, then the vacuum- Richm ond S, Vig KWL (ed s), Evid ence-based orthod ontics.
ormed retainer must be worn to maintain tooth Wiley-Blackw ell, Chichester, p p. 63–88.
alignment prior to repair or replacement o the bonded
retainer. For revision, see Mind Map 20, page 240.
21
■ How common is a a mily history with cle t lip and palate? Is
this the same or cle t palate (CP) only?
In arou nd 40% of cases w ith CLP, there is a fam ily history.
For CP alone, how ever, a fam ily history is less (arou nd 20%).
■ Wha t is the preva lence o cle t lip and palate?
It affects abou t 1 in 700 live births am ong Cau casians, but
the prevalence varies betw een racial grou ps as w ell as geo-

Cle t lip and palate


graphically and is increasing. Cleft lip only is fou nd in abou t
9% of all clefts, w hereas a cleft of the lip and alveolus com -
p rises about 3% of all clefts. Com p lete u nilateral CLP is the
m ost com m on type of cleft and rep resents 50% of all clefts.
■ Is there a sex a nd side va riation or CLP?
Females are affected less frequ ently than m ales, and the
right sid e is involved less com m only than the left.
■ How does this ma l ormation occur?
Failu re of fu sion of the m ed ian and lateral nasal p rocesses
SUMMARY and the m axillary process at abou t 4–6 w eeks of intrau terine
life lead s to a cleft of the p rim ary p alate (the u p p er lip and
Karen, a 9-year-old girl, is unhappy about the the alveolu s in the anterior region as far p osteriorly as the
appearance o her teeth (Fig. 21.1). What is the incisive foramen). Cleft of the second ary p alate (hard palate
cause, and how will it be treated? from incisive foram en back and soft p alate) is d u e to failu re
of the palatal shelves to elevate and fuse at abou t 8 w eeks
in utero.
History Genetic and environm ental factors, e.g. steroid therapy,
folic acid d e ciency or anticonvu lsant d rugs, interact in the
Complaint
aetiology.
Karen d oes not like the crooked ness and spacing of her
u p per front teeth. H er m other is also aw are that the bite of
Karen’s sid e teeth is not correct and feels that she m oves her
jaw to the sid e w hen she closes her teeth together.
Key point
CLP:
History o complaint • Prevalence o 1 in 750 live Caucasian births.
Karen’s baby front teeth w ere also crooked and spaced . H er • Aetiology due to genetic and environmental actors.
m other has noticed the p roblem w ith her bite for several • Positive amily history in ~40% o cases.
m onths. • More common in males and on the le t side.

Medical history
Karen w as born w ith a cleft lip and p alate (CLP), w hich has
been repaired . ■ Wha t genetic risk is there o CLP? How does this compare to
CP alone?
Family history As in this case of CLP, w here neither parent has a cleft and
Karen’s parents have no fam ily history of cleft lip and one child is affected , there is a sm all (4%; 1 in 25) risk that
p alate. H er old er brother and sister are also unaffected . the follow ing child w ill be affected ; if the m other or father
is affected , there is a 2% (1 in 50) likelihood of the rst child
having CLP. Due to the lesser genetic involvem ent in clefts
of the second ary palate, for u naffected parents of a child
w ith isolated CP, the risk of a su bsequ ent cleft-affected child
is 1 in 80.

Key point
There is a greater genetic risk with CLP than there is with CP.

Karen has been attend ing a cleft clinic at the regional


Fig. 21.1 Anterior occlusion at presentation. d ental teaching hosp ital since birth.
• 126
21 CLEFTLIPANDPALATE

■ Why is this? What treatment will have been provided to On the cle t side, 2 is either absent, o abnormal size and/or
date, a nd what role have you to play as her general shape, hypoplastic or present as two conical teeth on either
dental practitioner? side o the cle t.

Due to the interd isciplinary care required , treatment is facili- A supernumerary or supplemental tooth may exist on either
tated for the patient and family by coord inating m anagement side o the cle t.
in a specialized centre w ith a team com prising an orthod on- 1 is o ten rotated and tilted toward the cle t and may be
tist, speech therapist, health visitor and clinical psychologist hypoplastic.
as w ell as plastic, EN T and maxillofacial su rgeons. Eruption is delayed.
Treatm ent u ntil now is likely to have been as follow s:
Tooth size elsewhere in the mouth is small.
Neonatal period to 18 months
Class III incisor relationship is common with crossbite o one
Parental counselling by a member o the Cle t Lip and Palate or both buccal segments and a lateral open bite on the
Association (CLAPA) and/or clinical psychologist, as well as cle t side.
reassurance o the uture treatment by an orthodontist and
member o the surgical team.
Key point
Advice and support to the parents by a specialized health
visitor, particularly in relation to eeding. In this case eeding Incisor and buccal segment crossbites are common in
problems are likely to have been modest, as the cle t only repaired CLP.
involves the primary palate.
Planning o lip repair: this usually occurs at 3 months,
although neonatal repair is being assessed. In this case
closure o the alveolar de ect may be undertaken at the same
Examination
time. Where a palatal cle t exists, on average, this is repaired Extraoral
between 6 and 9 months.
■ What do you notice rom Fig. 21.2?
Primary dentition
Karen has a Class I skeletal p attern w ith average FMPA and
First ormal speech and hearing assessment at around 18
no obviou s facial asym m etry. The lips are com petent w ith a
months and then speech therapy as required.
scar on the right sid e of the u p p er lip .
Regular speech and hearing assessments should be N o clicks, locks or crep itu s of the tem porom and ibu lar
undertaken; consider closure o any palatal stulae to assist joints w ere d etected .
speech development. Consider pharyngoplasty to reduce
■ How is lip closure a chieved?
velopharyngeal incompetence at 4 5 years, thereby
attempting to improve any nasal intonation to speech. A Millard rep air w ith or w ithou t its m od i cations is the
Consider lip revision at 4 5 years only i clearly indicated. m ost pop ular, aim ing, after d issection, to p lace the lip
m uscles and alar base in their correct anatom ical location.
As a general d ental p ractitioner you r role is to: Whether subperiosteal or su p raperiosteal d issection and
Liaise with the CP team. skin-lengthening cuts are u sed , to obtain tissue m ovem ent,
Provide dietary advice and oral hygiene instruction to the remains controversial. The extent of alar cartilage d issection
parents, at regular intervals, rom eruption o the primary or the u se of a vom er ap also rem ains u nresolved .
incisors.
Consider the use o f uoride tablets i the level o f uoride in
Intraoral
the local domestic water supply is below 1 ppm. ■ The appea rance o the teeth is shown in Figs 21.1 and 21.3.
Attend to any dental treatment required. What are your observa tions?

Your particular aim in dental care is to promote and maintain Oral hygiene is air. Plaque deposits are visible on several teeth.
excellent dental health or Karen, thereby avoiding the need There is marginal gingival erythema related to most teeth.
or restorative treatment or en orced loss o primary teeth EDCB1 12CDE
visible (6 6 were erupted but not shown).
through dental caries. 6EDC21 12CDE6
E restored; possible caries in E DE; 1 slightly hypoplastic.
Key point
Mild lower labial segment crowding.
Management o CLP requires a team approach.
Spaced upper labial segment with 1 distolabially rotated.
The cle t repair involves the right upper lip and alveolus a
■ Wha t skeletal/dental/occlusal problems a re commonly bony depression is evident in the 2 area.
ound with CLP? Class III incisor relationship with average overbite; B in
Skeletally, there is a tendency or the maxilla and mandible to crossbite; lower centreline shi t to the right.
be retrognathic, the upper acial height to be reduced and Right molar relationship hal -unit Class II with buccal segment
the lower acial height to be increased. A Class III skeletal crossbite involving CDE ; (6 was also involved but not shown);
pattern is common. le t molar relationship is Class I.
127 •
CLEFTLIPANDPALATE 21

B C

Fig. 21.2 (A) Pro le. (B) Full ace. Fig. 21.3 (A) Lower occlusal view. (B) Right buccal occlusion.
(C) Le t buccal occlusion.

■ In view o the unila teral crossbite o the right buccal


segment, what should you check or? How would you
do this?
An upp er anterior occlu sal rad iograph is requ ired to
It w ou ld be im portant to check if there is a m and ibular
d eterm ine the extent of the alveolar cleft and the p osition
d isp lacem ent on the p ath of closu re associated w ith the
of the perm anent m axillary canine on the cleft sid e.
crossbite. As Karen’s m other has alread y noticed a shift of
the low er jaw on closu re, a d isplacem ent is likely. To d etect ■ Ka ren’s dental pa noramic tomogram and upper a nterior
this, Karen shou ld be instru cted to m aintain the tip of her occlusal radiograph are shown in Fig. 21.4. What do you
tongue in contact w ith the back of her palate as she closes notice?
her teeth together. Carefu l observation should be m ad e of The d ental panoram ic tom ogram show s:
rst tooth contact on the path of closu re and the extent and • Normal alveolar bone height except or the alveolar cle t
d irection of any m and ibu lar d isp lacem ent into m axim u m in the 2 area, which extends to involve 3 .
intercuspation shou ld be record ed . • All permanent teeth are present, except or 2 and third
There is a m and ibular d isplacem ent to the right on molars; ED DE are slightly in raoccluded.
C
closu re resu lting from p rem atu re contact of . • 1 is hypoplastic; E E appear to have secondary caries
C
underneath the restorations with possible urcation
radiolucencies.
Investigations • There is carious involvement o D D distally.
• The upper anterior occlusal radiograph con rms
■ What investigations are required? Explain why.
the extent o the alveolar cle t, and when assessed
A d ental p anoram ic tom ogram shou ld be taken to accou nt in combination with the dental panoramic
for the presence and p osition of any u neru p ted teeth and to tomogram, 3 appears to be lying in the line o
ascertain w hether there are any perm anent teeth absent. the arch.
• 128
21 CLEFTLIPANDPALATE

A
Fig. 21.5 Right buccal occlusion ollowing crossbite correction
with quadhelix.

■ What would you do at this stage?


Rein orce oral hygiene practice in preparation or orthcoming
orthodontic treatment.
Restore the lower primary molars.
Fissure seal the rst permanent molars.
B Liaise with the orthodontist on the cle t team regarding the
planned orthodontic treatment.
Fig. 21.4 (A) Dental panoramic tomogram. (B) Upper anterior
occlusal radiograph. ■ What orm do you envisage the orthodontic trea tment
to take?
Upp er arch exp ansion, by a qu ad helix appliance, to correct
the right bu ccal segm ent crossbite is likely to be und ertaken
Diagnosis p rior to alveolar bone grafting.
■ When is secondary alveolar bone gra ting usua lly
■ Wha t is your diagnosis?
undertaken and wha t adva ntages does it con er?
Repaired right unilateral cle t lip and alveolus.
Optim ally, it is u su ally u nd ertaken at arou nd 9–10 years. It
Class III malocclusion on a Class I skeletal base with average p rovid es bone through w hich 3 can eru p t, restores arch
FMPA. integrity, im p roves alar base sup p ort, aid s closure of an
C oronasal stu la and allow s orthod ontic space closu re.
Mandibular displacement to the right on closure on .
C ■ The occlusion prior to bone gra ting is shown in Fig. 21.5.
Generalized mild marginal gingivitis. What may you consider at this stage?
Caries ED DE. Due to the ad vanced state of root resorption of C (see also
Fig. 21.4), it w ould be u sefu l to rem ove this at least 3 w eeks
Mild lower labial segment crowding.
before grafting to allow tim e for socket healing, thereby
Spaced upper labial segment with 1 distolabially rotated; im proving the likelihood of graft success as an access rou te
absent 2 . for oral infection is rem oved .
Lower centreline to the right. ■ What treatment will be required ollowing alveolar
Right molar relationship hal -unit Class II with buccal crossbite bone gra ting?
o the right buccal segments; le t molar relationship is Class I. Once 3 eru pts, consid er space closu re w ith 2 replacem ent
■ Wha t is the IOTN DHC grade (see p. 264)? Explain why. by 3 as the right u pper bu ccal segm ent is brou ght forw ard .
This is a feasible plan and is the p referred op tion, as it obvi-
5p – d u e to CLP. ates the need for any p rosthetic rep lacem ent of 2 .
Consid er relief of crow d ing in the non-cleft qu ad rant and
Treatment in the low er arch. Delay any low er arch extractions if
orthognathic su rgery is p lanned at a later d ate.
■ What are the aims o treatment at this stage? Fixed app liance therap y w ill be necessary for the active
tooth m ovem ents requ ired .
Caries control.
Bond ed retention w ill then be requ ired to m aintain
Elimination o the mandibular displacement with correction up per labial segm ent alignm ent, and restorative treat-
o the right buccal segment crossbite. m ent w ill be requ ired to make 3 sim ulate an u pper lateral
Elimination o the alveolar cle t de ect. incisor.
129 •
CLEFTLIPANDPALATE 21
■ What restorative considerations a re there when 3 Primary resources and
replaces 2 ?
recommended reading
Restorative consid erations d ep end on the colou r, size and Bergland O, Sem b G, Abyholm FE 1986 Elim ination of the
shap e of 3 in 2 p osition and an assessm ent of fu nctional resid ual alveolar cleft by second ary bone grafting and
occlu sion (see Chapter 3). Close assessm ent shou ld be m ad e subsequ ent orthod ontic treatm ent. Cleft Palate J 23:175–205.
of 2, so that insofar as p ossible, 3 is a close match to 2. Gu o J, Li C, Zhang Q et al 2011 Second ary bone grafting for
Bleaching of 3 m ay be requ ired to tone in w ith the incisors. alveolar cleft in child ren w ith cleft lip or cleft lip and palate.
Some reshap ing of the 3 crow n is also likely – in p articu lar, Cochrane Database of Syst Rev Issu e 6. Art N o: CD008050.
DOI: 10.1002/ 14651858.CD008050.p ub2.
red u cing the prominence of the cu sp tip and p ossible com -
posite ad d ition to the m esial su rface so that it m ore closely Mossey P, Little J, Mu nger RG et al 2009 Cleft Lip and Palate.
Lancet 374:1773–1785.
resem bles a 2 .
As 4 w ill then be in the p osition of a canine, occlu sal Rivkin CJ, Keith O, Craw ford PJM et al 2000 Dental care for the
p atient w ith a cleft lip and palate. Part 1: From birth to the
ad ju stm ent of the p alatal cu sp m ay be necessary to rem ove
m ixed d entition stage; Part 2: The m ixed d entition stage
any interference in lateral m and ibu lar excu rsions. Alterna-
throu gh to ad olescence and you ng ad u lthood . Br Dent J
tively, if 4 has been intru d ed orthod ontically to achieve 118:78–83, 131–134.
coincid ence of the gingival m argins of 4 and 1 , com p osite
Thom AR 1990 Mod ern m anagem ent of the cleft lip and p alate
ad d ition m ay be need ed to achieve ‘canine’ gu id ance. p atient. Dent Up d ate 17:402–408.
In the late teenage years, consid eration m ay be given to
fu rther lip revision or orthognathic su rgery w ith rhino-
plasty later, if a m arked anterop osterior and / or vertical
For revision, see Mind Map 21,
skeletal d iscrep ancy exists. page 241.
In Karen’s case, p rovid ed facial grow th is reasonably
favou rable, orthognathic su rgery m ay not be requ ired .

Key point
Secondary alveolar bone gra ting:
• Provides bone or 3 eruption.
• Restores arch integrity.
• Improves alar base support.
• Aids oronasal f stula closure.
• Allows orthodontic space closure.
22
■ What is the cause o this pattern o decay?
‘N ursing caries’, or ‘nursing bottle m ou th’ or ‘bottle m outh
caries’.
■ What can cause this?
Consum ption of a sw eetened d rink or fruit- avoured d rink
from a bottle or d inky feed er, esp ecially if the feed er is
constantly in the m ou th or the child falls asleep w ith it in

Nursing and early


the m ou th. Care shou ld also be taken w ith lactu lose-free
alternatives to d airy m ilk su ch as soya or rice m ilk.
Persistent on-d em and breastfeed ing at night after 12

childhood caries
m onths of age (child is allow ed to sleep on the breast) m ay
cause caries. There are many biological and social variables
that confou nd this com p lex relationship .
As can be seen, the term ‘nu rsing caries’ is p robably the
m ost accu rate as it encom p asses both breastfeed ing and
bottle feed ing.
■ Why are the teeth a ected in this pattern?
SUMMARY Teeth become carious in the ord er in w hich they eru pt (Fig.
22.2), w ith the exception of the low er p rim ary incisors,
Kelly-Ann is only 3 years old. She has been brought
w hich are p rotected by tw o m ajor m echanism s: the p osition
to the dentist by her mother because her upper of the su bmand ibu lar d u cts that op en ad jacent to these teeth
ront teeth are ‘wearing away’ (Fig. 22.1). What has and the position of the tongu e in su ckling, w hich covers the
caused this, and how may it be treated? low er incisors.
■ What additional actors make the upper prima ry incisors
History more predisposed to ca ries?

The teeth apparently never cam e throu gh properly and High bow-shaped upper lip in in ants which does not cover
w ere never w hite like the rest of her teeth. There has been the upper incisors and results in an increased evaporation o
no p ain from the teeth, and Kelly-Ann is eating and d rink- any saliva on these teeth.
ing norm ally. Gravity, which keeps submandibular saliva pooled around the
lower incisors and less likely to reach the upper incisors.
Medical history Any liquid with sugar that is allowed to bathe the teeth on a
Kelly-Ann is a healthy child . She has had all her vaccina- requent basis will cause caries. This is especially so at night
tions and has had no illnesses. She has never taken any when the protective unction o saliva reduces as less saliva is
m ed ication. produced. Even breast milk, ormula milk or cows’milk with
their lowered natural sugars can still be cariogenic on this
basis.
Examination
Extraorally there is no sw elling and no facial asym m etry.
Intraorally she is in the fu ll prim ary d entition w ith the
second p rim ary m olars having ju st eru p ted . There is caries
affecting the u p per incisors and cavitation in all rst p rim ary
m olars.

Fig. 22.2 Classical distribution o a ected teeth in nursing caries


in upper arch. This is a more advanced case than Kelly-Ann’s,
with sign cant caries involving all maxillary primary incisors and
Fig. 22.1 Early cavitation in nursing caries a ecting upper right upper central incisors non-vital and in ected. Occlusal caries is
lateral and central primary incisor and upper le t central incisor. also seen in upper rst primary molars.
131 •
NURSINGANDEARLYCHILDHOODCARIES 22

Fig. 22.3 Hypomineralization and hypoplasia are seen on the


buccal sur aces o DBA ABD.

In some situations maxillary incisors may erupt with


hypoplastic or hypomineralized de ects, thus making the
teeth less resistant to the development o dental caries
(Fig. 22.3).
B

Fig. 22.4 (A&B) Upper and lower dentition o a child with


EB ABE
Key point extensive early childhood caries ( ) with a di ering
ED DE
Nursing caries: pattern to that o nursing caries. Please note the gross caries in
• A ects teeth in order o eruption. the lower second primary molars with an associated sinus E and
exposure o root E.
• Lower incisors are protected by saliva.
• Can be caused by any sugar-containing liquid even i
the sugar is naturally occurring rather than added to
the liquid.

Treatment
Prevention
■ Wha t should be your advice about night-time eeding? ■ Kelly-Ann is at high risk or caries. List all the main actors
you ca n think o or placing someone in the high risk group
Only w ater should be given d uring the night after 12 m onths
or denta l ca ries.
of age.
The term early child hood caries (ECC) is a further term See Table 22.1.
u sed to d escribe caries presenting in one or m ore p rim ary ■ Wha t preventive a dvice would you provide or Kelly-Ann’s
teeth of child ren u nd er the age of 5. Som e child ren p resent mother?
w ith extensive caries that d oes not follow the ‘nu rsing
caries’ pattern and often p resent later, betw een 3 and 5 years Preventive ad vice shou ld follow the DoH Prevention Toolkit
old (Fig. 22.4). gu id ance. This provid es ad vice for child ren of d ifferent ages
(in Kelly-Ann’s case, 0–3 years old ) follow ing placem ent in
■ How could you identi y pre-school children in need o
the high risk for caries category.
denta l ca re?
Home based advice Although m any parents are fully aw are
Encou rage p arents to bring their child ren for a d ental
of the cau ses of d ental caries in their child or child ren,
check-u p as soon as the rst p rim ary tooth eru p ts.
changing family behaviou r to u nd ertake healthier rou tines
Develop good w orking relationships w ith local health
is more com plex. Consequently, carefu l consid eration is
visitors, baby clinics, m other and baby grou p s, nu rseries
need ed in how preventive ad vice is given and w ho d elivers
and their local general m ed ical practice to encou rage w id er
it. All m em bers of the d ental team shou ld have a basic
health and nu rsery staff to p rovid e ap p rop riate oral health
u nd erstand ing of how they can support parents and chil-
ad vice. This ad vice shou ld follow the gu id ance of the
d ren to und ertake these changes, and those d elivering these
Dep artm ent of H ealth (DoH ) Prevention Toolkit. Pu blic
preventive m essages shou ld have a m ore d etailed know l-
health p rogram m es in som e cou ntries encou rage and train
ed ge of behaviou r change strategies.
general health p rofessionals, su ch as health visitors, to ‘lift
the lip’ to id entify early caries affecting the u p p er m axillary Toothbrushing and f uoride toothpaste Kelly Ann’s p arents
incisors. shou ld u nd ertake toothbru shing for her last thing at night
• 132
22 NURSINGANDEARLYCHILDHOODCARIES
Table 22.1 ‘High-risk’ actors or caries scores (w ith or w ithou t a d isclosing agent) allow s the d ental
Risk actor Aetiology professional to m onitor p arental com p liance w ith the tooth-
Clinical evidence Newlesions bru shing ad vice.

Premature extractions ■ What advice should be given i Kelly-Ann does not like
strong, mint- avoured toothpaste?
Anterior caries or restorations
Multiple restorations Some child ren struggle w ith the strong m int avours u sed
in ad ult toothp aste. As a d ental care p rofessional, you
Fixed appliance orthodontics
shou ld be able to ad vise on alternative child -friend ly tooth-
Partial dentures
p astes. These shou ld have a mild , non-m inty avour that
Dietaryhabits Frequent sugar intake still have the required levels of u orid e. Other child ren m ay
Social history Social deprivation struggle w ith the foam ing action of the toothp aste, and
High caries in siblings again you shou ld be able to p rovid e ad vice on the availabil-
ity of toothpastes w ithou t these agents (for exam p le, sod iu m
Lowknowledge o dental disease
lauryl su lp hate-free toothp astes).
Irregular attendance
■ Is there a role or uoride supplements?
Readyavailabilityo snacks
Lowdental aspirations Fluorid e sup plem ents althou gh still recom m end ed are no
longer consid ered a m ainline intervention. The reason for
Use o uoride Drinking water not uoridated
this relates to the p oor com p liance w ith these su p p lem ents
No uoride supplements
in the families w ith child ren at high risk. All uorid e prod -
No uoride toothpaste ucts, inclu d ing toothp aste, shou ld be treated as a m ed icine
Plaque control In requent, ine ective cleaning and kep t ou t of reach of you ng child ren to prevent excessive
Poor manual control ingestion. The DoH Prevention Toolkit provid es fu rther
read ing and ad vice on preventing u orosis, especially for
Lacko parental involvement in brushing
child ren at low caries risk living in areas w ith u orid ated
Saliva Low owrate
w ater su p p lies.
Lowbu ering capacity
Diet advice The only w ay to effectively u nd ertake d ietary
High Streptococcus mutans and Lactobacillus counts
ad vice is to ask p arents to com plete a 4-d ay d iet d iary. From
Medical history Medicallycompromised this a w ritten analysis can be p rod uced . It is im portant to
Physical disability try to obtain 1 d ay’s history from a w eekend , as they are
Intellectual disability invariably d ifferent from w eekd ays. In m od ern society it is
Xerostomia comm on for m ost p arents to w ork and the child to be looked
after by a carer or nu rsery. It is critical to establish w ho is
Long termcariogenic medicine
the carer on w eekd ays and w eekend s. Ad vice need s to be
clear at all tim es, bu t if it has to be relayed from a p arent in
the su rgery to a carer, then it need s to be clear, su ccinct and
w ritten. Frequ ent consu m ption of sugar-containing d rinks
and at least on one other occasion every d ay. On each and food s is the key aetiological featu re in m any pre-school
occasion, a sm ear of 1350–1500 p p m toothp aste shou ld child ren w ith caries. Red u cing the frequency of sugar-
be u sed . containing snacks is an imp ortant m essage. If the child is a
‘poor eater ’, there is need to bu ild u p the am ount of food at
■ Why is parental involvement important?
m ealtim es and , therefore, red u ce the need for frequent
Child ren need help from their p arents if effective oral snacking. Child ren d o not need zzy d rinks or fru it-based
hygiene is to be achieved . This help shou ld extend u p to at d rinks. These d rinks often m ake u p for calories m issed at
least 7 years old . Bru shing need s to start as soon as the rst m ealtim es. Only m ilk and w ater should be taken betw een
tooth eru pts. Stand ing or kneeling behind the child in front m eals. A sm all am ou nt of fru it-based d rink can m ore
of a sink or m irror is often the best w ay to bru sh a you ng safely be taken w ith a m eal. As p reviou sly m entioned , it is
child ’s teeth. Other su ccessfu l w ays inclu d e the child lying critical to stop the night-tim e bottle w ith anything other
on his or her p arents’ legs. Su pervision of bru shing is than w ater.
im portant so that an ap p rop riate am ou nt of p aste is p laced In you r d ietary ad vice you m u st be practical, personal and
on the bru sh to p revent/ red u ce ingestion of p aste. Carefu l positive. Avoid m aking the p arent feel excessively gu ilty, but
qu estioning of p arents and hand s-on d em onstration of concentrate on p ractical strategies. It is p robably unreason-
how to u nd ertake toothbru shing is im p ortant. Qu alitative able to give ou t m ore than fou r pieces of w ritten ad vice.
research has reported a num ber of barriers that parents These should concentrate on d ay-tim e d rinks, night-time
d escribe w hen trying to u nd ertake their child ’s toothbru sh- d rinks, betw een-m eal snacks, and m aking sure the child has
ing. A nu m ber of these barriers relate to p arenting skills, no food or d rink for 1 hou r before going to bed and cleans
such as m anaging tod d lers w ho w ant to bru sh them selves, their teeth just before bed . Furtherm ore, d ietary ad vice
or establishing and maintaining routines. Consequ ently shou ld not be lim ited to su gary snacks bu t also brie y eval-
ad vice and gu id ance need s to ad d ress these w id er p arent- uate the d iet as a w hole and assess if it is in line w ith the
ing issu es. The use of sim p le serial p laqu e and gingivitis eat w ell plate.
133 •
NURSINGANDEARLYCHILDHOODCARIES 22
Medication For child ren on regu lar m ed ication, this shou ld , either a d irect com p osite or a com p osite crow n u sing a
w here p ossible, be su gar free. Often this w ill requ ire liaison p lastic strip crow n. Follow ing the tem p orary GIC restora-
w ith their general m ed ical p ractitioner. A carefu l m ed ical tions, Kelly-Ann’s coop eration im proved signi cantly to
history–taking is essential, as som e p arents m ay not p erm it the placem ent of d irect com posite restorations und er
perceive regu lar nu tritional su p p lements or laxatives as local anaesthetic several m onths later.
m ed ication.
■ How would you restore the early cavita tion in the f rst
Pro essional interventions For Kelly-Ann, cu rrent ad vice is primary mola rs?
that u orid e varnish shou ld be ap plied to her teeth three to
Similar to the incisors, d ifferent options are available:
fou r tim es a year. Site-sp eci c ap p lication of u orid e varnish
can be very valu able in the m anagem ent of early, sm ooth- Prevention only namely to modi y the cavity to permit
su rface and ap p roxim al cariou s lesions. The m ost com - plaque removal using a toothbrush and regular application o
m only u sed varnish – 5% sod iu m u orid e – has 22 600 p pm . f uoride varnish. Regular plaque scores will allow you to
Before ap p lying the varnish the child ’s asthm a statu s and monitor how well parents are complying with your
any history of allergy to colop hony shou ld be checked . In toothbrushing instructions.
cases w here child ren have been hosp italized for their Seal with partial or no caries removal namely to seal the
asthm a or have a history of an allergic reaction to sticky caries with composite and/or ssure seal ollowing limited
plasters (w hich can contain colop hony), the u se of coloph- caries removal with a slow handpiece.
ony containing u orid e varnishes are contraind icated . Complete caries removal using local anaesthetic, rubber
Alternative, non-colophony-containing varnishes are avail- dam and handpieces to remove caries ollowed by composite
able. When ap plying the varnish, ensure the correct d ose restorations.
and instru ctions are given.
The recall interval betw een d ental clinic visits w ill red u ce Initially a p reventive ap p roach w as taken w ith the
to 3 m onths. This w ill enable the m ore frequ ent ap plication cariou s rst p rim ary m olars. As Kelly-Ann’s coop eration
of u orid e varnish, m ore intense preventive su p port and im proved , a com plete caries removal ap p roach w as und er-
the exam ination of Kelly-Ann’s d entition for caries progres- taken, as both her and her m um w anted w hite coloured
sion or new lesions. llings rather than stainless steel crow ns.

Why can Kelly-Ann not have f uoride mouthwash? These ■ What method o caries removal, without a handpiece, may
are contraind icated in child ren less than 6 years of age be applica ble here?
because m ore than half the m ou thw ash w ill be sw allow ed , With the ad vent of the H all techniqu e (see Chapter 24)
increasing the risk of u rosis. This is also the reason w hy chem ical caries rem oval has had lim ited u se. An exam ple of
parents are ad vised to u se only a sm ear-sized (for 0–3-year- chem o-m echanical caries rem oval is the u se of Carisolv. It
old s) or pea-sized (for 4–6-year-old s) am ou nt of toothpaste consists of a p ink gel that contains m ainly the am ino acid s
and w hy the child is encou raged to sp it ou t the excess leucine, lysine, glu tam ic acid and hypochlorite. In ad d ition,
resid ue of the toothpaste as soon as he or she is able to there is cellulose and a colou ring agent, erythrocin. The
d o so. amino acid s and hypochlorite w ork to sep arate carious
d entine from sou nd d entine, and the cariou s d entine is
Treatment removed w ith the aid of special hand instrum ents that have
d ifferent cu tting ed ges and hand actions to excavators. They
Restorative care are used in a w hisking, rotating or u p-and -d ow n m ove-
(Please see Chapter 24, w hich p rovid es greater d etail of m ent. Because the sou nd d entine is not stim u lated by the
treatm ent op tions and treatm ent p lanning.) temperatu re or vibration of a hand piece, or the tem perature
Kelly-Ann has caries involving her u pper incisors (Fig. changes of a three-in-one sp ray, it is a p ainless p roced u re.
22.1) and cavitated occlu sal caries in her rst prim ary The cavity shou ld be d ried by saline-d am pened cotton
m olars. Treatment can be p rovid ed in a nu m ber of w ays and w ool, then d ry cotton w ool, prior to restoring w ith an ad he-
relates to parental expectations and w illingness to com ply sive m aterial. Bond strengths to ad hesive m aterials are the
w ith the hom e-based p reventive ad vice, p arents’ expecta- same as conventionally p repared cavities. The d isad van-
tions and w ishes, child ’s coop eration and any history of tages of the techniqu e inclu d e the taste of Carisolv if it
toothache or other sym p toms. Child coop eration can change should leak ou t of the cavity, the tim e taken to rem ove caries
rap id ly as the child grow s, and consequ ently, over a few and the noises and sensation of the hand instru m ents.
m onths, d ifferent treatm ent op tions m ay be ap prop riate ■ How is pain relie best achieved in the child with nursing
w ith im proving behaviou r and cooperation. In Kelly-Ann’s ca ries in Fig. 22.2?
case, w ith no history of p ain, tim e w as available at rst to
tem porize and seal in the caries w ith glass ionom er cem ent This is a case w here general anaesthesia for tooth rem oval
(GIC) u ntil coop eration im p roved . is ju sti ed . This is covered in Chapter 26. Depend ing on the
type of the general aneasthetic (extraction only or com p re-
■ How would you restore the upper incisors?
hensive care), d ental care w ou ld consist of either extracting
There are a nu m ber of d ifferent op tions and m aterials avail- all cariou s teeth (DBA ABD) or extracting irreversibly
able. These inclu d e the p revention-only ap p roach w ith no in am ed and non-vital p rimary teeth and restoring other
caries rem oval or d isking, tem p orarization w ith GIC or teeth affected by caries together w ith ssu re sealants of
comp lete caries rem oval w ith d e nitive restoration u sing other p rim ary m olars.
• 134
22 NURSINGANDEARLYCHILDHOODCARIES

Primary resources and H ealth England . Available at: http s:/ / w w w.gov.u k/
governm ent/ u p load s/ system / u p load s/ attachm ent
recommended reading _d ata/ le/ 367563/ DBOH v32014OCTMainDocu m ent
Am erican Acad em y of Ped iatric Dentistry 2014 Guid eline on _3.pd f.
caries-risk assessm ent and m anagem ent for infants, child ren, Scotish Dental Clinical Effectiveness Program m e (SDCEP) 2010
and ad olescents. Ped iatr Dent 36 (6):127–134. Prevention and Managem ent of Dental Caries in Child ren:
Am erican Acad em y of Ped iatric Dentistry 2014 Policy on early Dental Clinical Gu id ance. Du nd ee, SDCEP. Available at:
child hood caries (ECC): unique challenges and treatm ent http :/ / w w w.sd cep .org.u k/ w p-content/ u p load s/
op tions. Ped iatr Dent 36 (6):53–55. 2013/ 03/ SDCEP_PM_Dental_Caries_Fu ll_Gu id ance1
.p d f.
Deery C 2013 Caries d etection and d iagnosis, sealants and
m anagem ent of the p ossibly cariou s ssu re. Br Dent J 214 Marshm an Z, Ahern S, McEachan R et al Parents exp eriences of
(11):551–557. tooth bru shing w ith child ren: a qu alitative stu d y. Accepted for
p u blication in Jou rnal of Dentistry Clinical and Translational
H ealthcare Im provem ent Scotland 2014 SIGN 138: Dental
Research, 2016.
Interventions to Prevent Caries in Child ren. Ed inbu rgh, SIGN .
Available at: http:/ / w w w.sign.ac.u k/ p d f/ SIGN 138.p d f.
Public H ealth England 2014 Delivering Better Oral H ealth: An
For revision, see Mind Map 22,
Evid ence-Based Toolkit for Prevention, 3rd ed . Lond on, Public page 242.
23
Dental history
At the age of 5 years he had a num ber of his p rim ary teeth
extracted u nd er general anaesthetic.
H e has su bsequ ently requ ired restorative care and an
extraction of his u p p er right rst p erm anent m olar u nd er
local anaesthetic several years ago. You have not seen Peter
for at least 3 years (Fig. 23.1).
■ Which aspects o his presenta tion and history help to

High caries risk determine his ca ries risk status so a r?


• Social history irregular attendance and low dental

adolescents
aspirations to date (high caries risk).
• Fluoridated water living in an area with no f uoride in
the drinking water (high caries risk).
• Medical history t and well (low caries risk).
• Dental history primary tooth extractions under general
anaesthesia and subsequent permanent tooth extraction
and restorations with local anaesthetic (high caries risk).
SUMMARY
Peter is 13 years old. He is concerned about the Examination
appearance o his teeth, especially the spaces Extraoral
between his ront teeth, and would like this N othing relevant is revealed .
improved (Fig. 23.1). He is not very keen on the
prospect o complex restorative or orthodontic Intraoral
treatment. On assessment he is diagnosed as high Peter ’s oral hygiene is poor. Basic Period ontal Exam ination
caries risk. How would you plan preventive (BPE) scores of one in each sextant are record ed . N orm al
treatment or this patient? saliva levels are noted . Caries is evid ent in the low er left
second p erm anent m olar. H e is in the perm anent d entition
w ith missing both low er central incisors and a retained
History low er left prim ary central incisor. H is u pper right rst p er-
m anent m olar w as extracted p reviou sly and the u p p er and
Complaint low er left rst p erm anent m olars and low er right rst and
Peter attend s your su rgery for the rst tim e in a nu m ber of second m olars restored . N o ssu re sealants are present.
years. H e ad vises you he w ou ld like the sp aces betw een his There is evid ence of m ild buccal crow d ing in the low er arch
front teeth corrected (Fig. 23.1). w ith low er left second perm anent prem olar lingually
p laced . The u pper left second p erm anent p rem olar is
History o complaint u nerupted and palatally d isplaced (Figs 23.2 and 23.3).
Peter is a sp orad ic attend ee. H e rep orts no p ain from any of ■ Which urther a spects o his clinical presentation help
his teeth. H e is now keen on having the app earance of his determine his ca ries risk status?
front teeth im p roved and is eager to learn how this can be
Clinical evidence
achieved .
New caries in the lower le t second permanent molar,
Medical history previous restorations and a missing permanent tooth which

Peter is t and w ell. H e is not taking any m ed ication.

Fig. 23.1 Anterior view o teeth at presentation. Fig. 23.2 Upper occlusal view at presentation, with upper right six
previously extracted.
• 136
23 HIGHCARIESRISKADOLESCENTS
was extracted are all noted (high caries risk). This caries risk separate time to brushing his teeth, or example a ter coming
category will have the largest weighting in scoring overall in rom school or a ter dinner is well remembered by many
caries risk. high caries risk patients.
Plaque control Dietary history
Oral hygiene poor, BPE scores all one. His oral hygiene is Frequency a nd timing o a ll ood a nd drinks including
especially poor in the upper anterior region (high caries risk). milk a nd wa ter? Carbonated drinks are consumed at least
Saliva our times a week with diluted juice consumed on a daily
basis.
Normal saliva levels noted (low caries risk).
Frequency a nd timing o ood. Peter is a grazer and likes to
For a fu ll list of caries risk factors, see Chap ter 22, Table
eat well into the evening. Peter is advised to complete a
22.1. This w ill help bu ild u p a p ictu re of caries risk and
4-day diet diary (at least 1 day should be completed over the
form u late a p revention p lan that is tailored to the p atient’s
weekend; high caries risk).
requ irem ents.
■ At present what ca ries risk status would you assign Peter Peter ’s history, clinical assessm ent and fu rther qu estion-
to? ing com bine to give a nal caries risk assessment, w hich
ind icate a high caries risk statu s (Table 23.1).
H igh caries risk.
■ What urther in ormation would you ask Peter to complete
his ca ries risk assessment? Key point
Fluoride history Caries risk assessment:
Wha t strength/type o f uoride toothpa ste does he use? I • The largest weighted caries risk assessment category is
Peter is not aware o the f uoride strength o the toothpaste clinical evidence.
he uses, he should be shown how to determine this on a • The completion o a caries risk assessment helps
toothpaste tube. He should be using toothpaste strength at ormulate an enhanced prevention plan specif c to this
least 1350 1500 ppm F (children 6 and older). He is presently patient.
using a brand which is the correct strength/type or a low
caries risk child o his age.
How ma ny times a da y does he brush his teeth? He
presently brushes twice daily, but not e ectively. Twice daily
or 2 minutes should be recommended and the technique
Preventive care and treatment
demonstrated. Peter is presently high caries risk. Prior to restorative w ork
Does he presently rinse with wa ter a ter brushing? He being u nd ertaken, the preventive treatm ent p lan shou ld
rinses his teeth with water a ter brushing. The ‘spit but don’t ensure that his caries risk status is red uced and he remains
rinse’with water guidance should be recommended. caries free in the fu tu re.

Is he presently using a ny f uoride mouthwa sh? I so, when


Radiographs
does he do this? At present only toothpaste is used. Fluoride
mouthwash should be recommended 0.05% NaF at a ■ A ter the initial bitewing radiographs are taken (Fig. 23.4),
when should Peter have radiographs ta ken a ga in?
In 6 m onths’ tim e if he rem ains at high caries risk.

Fig. 23.4 Bitewing radiographs. These show dentinal caries in the


le t lower second permanent molar and enamel lesions on upper
le t rst permanent premolar and the lower right rst permanent
Fig. 23.3 Lower occlusal view at presentation. molar.

Table 23.1 Peter’s caries risk assessment


Clinical evidence Dietary habits Social history Fluoride use Plaque control Saliva Medical history Caries risk
LH LH LH LH LH LH LH LH
137 •
HIGHCARIESRISKADOLESCENTS 23
■ What other orms o preventive ca re would he benef t rom? Table 23.2 helps highlight the com plete p ackage of pre-
ventive care that Peter shou ld receive prior to any restora-
Toothbrushing instruction Oral hygiene instru ction w ith
tive w ork is und ertaken.
a toothbru shing d em onstration at each recall visit for
Where m ore extensive caries is seen in an ad olescent,
enhanced prevention.
stabilization of the d entition is often an app ropriate rst
Child ren of Peter ’s age should be show n how to bru sh
step. This can be u nd ertaken w ith simp le hand excavation
their teeth u sing d isclosing tablets or solu tions. Disclosing
and glass ionom er cem ent tem p oraries. This w ill slow d ow n
tablets should be used before going to bed w hen tim e can
the progress of caries w hile the resp onse of the child and
be d evoted to im p roving bru shing techniqu es. Although
family to the preventive ad vice can be evaluated . As gingi-
patients or p arents m ay ask abou t the bene ts of a m anu al
val health im proves, d e nitive caries rem oval and com pos-
or pow ered toothbru sh, clinically it is m ore im p ortant w hat
ite or other restorative options can be u nd ertaken. Fu rther
he d oes w ith the toothbru sh in resp ect to the thorou ghness
bene ts of this ap proach are d iscu ssed in Chapter 24.
of the bru shing and its frequ ency.
Only on comp letion of this initial p hase of prevention
Does he use a nyt hing t o clea n in bet w een his t eet h? H e
and sim ple restorative treatm ent shou ld com p osite build -
w as show n previou sly how to d o this, bu t ‘his m u m forgot
u ps or other ad hesive brid gew ork be consid ered (Figs 23.7
to bu y m ore oss’. Exp lain the im p ortance of interd ental
and 23.8). In Peter ’s case the retained low er left prim ary
cleaning and ask Peter to d em onstrate ossing in his ow n
m ou th. Give m otivational feed back. This is esp ecially
im portant, as enam el lesions are visible on the bitew ing
rad iograp hs.
Strength o f uoride toothpaste In ad d ition to the ad vice on
ad ult toothp aste strength on initial p resentation, d u e to
Peter ’s age and high caries risk statu s, he w ou ld gain ad d i-
tional bene t from a prescription of a higher-strength
2800 p pm F toothp aste.
Fluoride varnish application The evid ence su ggests an
ad d itional bene t of three to fou r ap p lications annu ally of
uorid e varnish for child ren at high caries risk. Even if
Peter ’s caries risk statu s w ere to subsequ ently red u ce to low
caries risk, tw ice-yearly ap p lication of a u orid e varnish,
termed stand ard p revention care (2.2% sod ium u orid e
22 600 pp m F), w ou ld still be recom m end ed .
Fig. 23.5 Fissure sealants in upper premolars. The upper le t
Fluoride supplements Flu orid e m outhw ash at 0.05%
second permanent premolar is unerupted and visible as a budge
sod iu m u orid e, alcohol free, is recom m end ed for d aily use
on the palate.
at a sep arate tim e to toothbru shing. Dem onstrate to the
patient on a m ou thw ash bottle w here the u orid e concen-
tration inform ation is w ritten. This then allow s the p atient
to make an inform ed choice regard ing the m any brand s of
m ou thw ash available for u se on a d aily basis. The m outh-
w ash should be sw ished arou nd the m ou th for 60 second s
before spitting out the resid ue.
Diet analysis As stated in Chap ter 22, a 4-d ay d iet d iary can
highlight frequ ency and tim ing of food s that enable p racti-
cal and p atient-centred ad vice to be given. Ad vice shou ld
be given in line w ith the eat-w ell p late. Child ren of second -
ary school age can be given inform ation that enables them
to make inform ed choices regard ing their eating and d rink-
ing practices. In m any situ ations, sim p le changes can red u ce
their caries risk signi cantly.
Fissure sealants Peter required ssu re sealants on all his
prem olars (Figs 23.5 and 23.6). Fig. 23.6 Fissure sealants in lower premolars.

Table 23.2 Peter’s prevention plan


Radiographs Strength F− F− varnish F− supplements Fissure Sugar- ree
(Frequency) Toothbrushing Instruction toothpaste (F− ppm) ( requency) (dose) Diet analysis sealants medicines
6 months With disclosing tablets and 2800 ppmF 3–4 monthly Dailyuse uoride 4 day ood diary Yes Yes N/A
interdental advice and demonstration application mouth wash (in line with the Yes Yes
0.05%Na F eat well plate)
• 138
23 HIGHCARIESRISKADOLESCENTS

■ What else might you suggest when Peter is older that could
help urther reduce his caries risk status or the uture?
Higher st rengt h uoride t oot hpa st e (5000 ppm F). The
effect of uorid e toothpaste is concentration d ep end ent.
The m axim al over-the-cou nter prod u ct is 1500 p pm uo-
rid e. Prescrip tion-only toothp astes containing 2800 and
5000 p pm u orid e allow s the d ental p rofessional to target
high caries risk ad olescents. The resu lts of a nu m ber of
rand om ized clinical trials su ggest that in the range 1000–
2500 p pm u orid e, every ad d itional 500 p pm u orid e, over
and above 1000 pp m u orid e, w ould provid e a cu m u lative
6% red uction in caries increm ent. This d ose resp onse is
highest in high caries risk child ren and those aged over 11
Fig. 23.7 Final restorative treatment o the upper incisors. years. H ow ever, this w ou ld only be p rescribed after assess-
ing su itability and com p liance w ith instru ctions for the u se
of higher strength u orid e toothp aste. It shou ld be em p ha-
sized that such high-strength u orid e toothpastes should be
kept ou t of reach of you nger child ren. Ind ivid uals for w hom
this toothp aste is p rescribed should be encouraged to expec-
torate after bru shing. The 5000 pp m u orid e toothp aste is
u sed for ad olescents over 16 years of age and ad u lts.
Toot h mousse or t oot h mousse plus (CP P -ACP or CP P -
ACFP ). Tooth m ou sse is a w ater-based cream containing
Recald ent (casein p hosp hop ep tid e-am orp hou s calciu m
p hosphate or CPP-ACP). Tooth m ou sse plus, a stronger
tooth m ou sse, is recom m end ed at night only for p atients
w ho either have m arked salivary d ysfu nction or increase
risk of m ineral loss from d ental caries or erosion of teeth.
Child ren should be at least 6 years of age before u sing tooth
m ousse plu s. The p rop osed anticariogenic m echanism of
CPP-ACP involves the enhancem ent of rem ineralization
Fig. 23.8 Final restorative care o the anteriors. throu gh the localization of bioavailable calcium and phos-
p hate ions at the tooth surface. Casein p hosp hop eptid es
(CPPs) stabilize high concentrations of calciu m and p hos-
p hate ions as em bryonic ACP nanoclusters together w ith
central incisor w as assessed and d u e to lim ited root length, u orid e ions at the tooth su rface by bind ing to p ellicle and
he w as ad vised of its p oor long-term p rognosis. The p rim ary p laque and act as a d elivery vehicle to the tooth su rface. The
incisor w as extracted and a one-u nit Maryland brid ge p ro- ions are freely bioavailable to d iffu se d ow n concentration
vid ed w ith fu rther related oral hygiene instru ction. This grad ients into enamel su bsurface lesions, thereby effectively
includ ed the u se of su p er oss to clean beneath the p ontic. p rom oting rem ineralization. Tooth m ou sse m ay not be
At a later stage, shou ld Peter d ecid e he w ou ld like m ore ap propriate for all p atients ow ing to its cost. This is ap proxi-
com p lex treatm ent su ch as xed orthod ontics or im plants, m ately 5–10 tim es as expensive as a stand ard tu be of uo-
this can still be provid ed . rid e toothpaste.
Suga r-free chew ing gum. Many chew ing gum s are now
available as sugar-free w ith 50% sw eetened w ith su gar su b-
stitutes. Oral bacteria d o not u se these su gar substitutes to
prod u ce the acid s that d em ineralize enam el and d entine.
Key point
Fu rtherm ore, the act of chew ing stim u lates saliva ow,
Preventive care: w hich increases bu ffering cap acity and enhances clearance
• The evidence to date suggests an additional benef t of food d ebris and m icro-organism s from the oral cavity.
rom application o uoride varnish 3 4 yearly in high Chew ing gu m containing xylitol, a p olyol 5 carbon sw eet-
caries risk children. Apply twice yearly in low caries risk ener, red uces p laqu e salivary Streptococcus mutans levels and
children. tooth d ecay, as w ell as enhancing rem ineralization.
• Fluoride mouthwash at 0.05% sodium uoride is
recommended or daily use at a separate time to Primary resources and
toothbrushing.
• For high caries risk children (age 10 and over) and
recommended reading
Cochrane N J, Saranathan S, Cai F et al 2008 Enam el su bsu rface
adolescents, higher strength prescription uoride
lesion rem ineralisation w ith casein phosphopep tid e stabilised
toothpaste (2800 ppm F) is recommended. solu tions of calciu m , p hosp hate and uorid e. Caries Res
42:88–97.
139 •
HIGHCARIESRISKADOLESCENTS 23
Facu lty of General Dental Practice (FGDP) 2013 Selection Criteria Walsh T, Worthington H V, Glenny A-M et al 2010 Flu orid e
for Dental Rad iograp hy, third ed . Lond on, FGDP. toothpastes of d ifferent concentrations for p reventing d ental
H ealthcare Im provem ent Scotland , 2014. SIGN 138: Dental caries in child ren and ad olescents. Cochrane Database of
Interventions to Prevent Caries in Child ren. Ed inburgh, SIGN , System atic Review s 2010, Issu e 1. Art. N o.: CD007868. DOI:
sections 2–8. Available at: http :/ / w w w.sign.ac.uk/ p d f/ 10.1002/ 14651858.CD007868.pu b2.
SIGN 138.p d f.
Kiet AL, Milgrom P, Rothen M 2008 The potential of d ental- For revision, see Mind Map 23, page 243.
p rotective chew ing gu m in oral health interventions. J Am Dent
Assoc 139:553–563.
Public H ealth England 2014 Delivering Better Oral H ealth: An
Evid ence-Based Toolkit for Prevention, third ed . Lond on, Pu blic
H ealth England , sections 2–5.
24
Place a glass ionomer cement (GIC) or other temporary
restoration (Kalzinol or Intermediate Restorative Material.). I
the pain settles, a more de nitive restoration will be required
when time permits and as part o a comprehensive treatment
plan (see later).
Irreversible pulpitis i possible, gently excavate the so test
layer o coronal caries, then, place a poly-antibiotic and
steroid paste (e.g. Ledermix or Odontopaste) beneath the GIC.

Pain control and


These pastes are e ective at reducing the symptoms rom the
tooth. Again, more de nitive treatment (extraction or
pulpectomy) will be required or this tooth when time

treatment planning or
permits and as part o a comprehensive treatment plan.
Acute apical periodontitis i a tooth is abscessed, there is
o ten signi cant coronal destruction. The pulp chamber o

carious primaryteeth
such teeth can o ten be accessed, and a dressing o Ledermix
or Odontopaste on some cotton wool placed within the
chamber and sealed with a GIC will o ten lead to temporary
resolution o symptoms and swelling. Again, more de nitive
treatment (extraction or pulpectomy) will be required or this
tooth when time permits and as part o a comprehensive
treatment plan.
Where tim e and p atient cooperation p erm it, d ress-
SUMMARY
ing open cavities has a nu m ber of ad vantages (see Key
Paul is 5 years old. He is in pain rom one o his upper Point box and Fig. 24.1):
right back teeth. He has never had any treatment Simple introduction to dental procedures.
be ore. How would you manage Paul’s problem? Oral mutans streptococci count is reduced when excavation
■ What questions do you need to ask rega rding the pain? o gross caries is accomplished. I the cavity is then
completely sealed by a GIC, there is evidence that the viability
Site ask Paul to point to the tooth.
o the remaining organisms decreases and caries progression
Severity does the pain stop him rom playing, eating or is greatly reduced. This buys the dentist time to institute
sleeping? This question you o ten have to ask both Paul and preventive and behaviour management programmes be ore
his parents. reassessing teeth with temporary restorations.
Onset what makes the pain worse? Is it in response to hot, GICs act as a f uoride reservoir.
cold or sweet stimuli, or does it occur spontaneously? Does it
Makes toothbrushing and eating more com ortable.
wake him up rom sleep?
Character is it a sharp pain or is it dull and throbbing?
Table 24.1 Pain characteristics
Duration how long does it last when the pain is present?
Reversible Irreversible
How long has Paul su ered rom pain? What makes the pain
worse or better (e.g. painkillers) Transient or short duration (minutes) Long duration
Response to hot, cold, sweet Response to pressure (chewing)
A sim ple m nem onic can help you to rem em ber the d if-
ferent questions, SOCRATES (Site, Onset, Character, Rad ia- Sharp Spontaneous
tion, Association, Tim e cou rse, Exacerbating and relieving Does not stop playor sleep Throbbing
factors, Severity). You ng child ren m ay how ever nd som e Stops playor sleep
of these qu estions d if cu lt to answ er, su ch as rad iation and
association. Moreover, care need s to be taken to u se the m ost
approp riate langu age. For exam ple, even thou gh child ren
m ay not u nd erstand sensitivity, they or their p arents are
likely to rep ort if they avoid cold d rinks or ice cream .
The d ifferent characteristics of reversible and irrevers-
ible p u lp itis are show n in Table 24.1. The initial m anage-
m ent of a child attend ing in p ain is often constrained by
the lack of sleep on behalf of the child or tim e available
to the d entist to treat this extra ‘em ergency p atient’. An
accu rate d iagnosis is essential before any treatm ent can be
provid ed .
■ Wha t dressings ca n help mana ge pulpitis initia lly?
Reversible pulpitis i possible, gently excavate the so test Fig. 24.1 Temporary GIC restoration placed, the bene ts o which
layer o coronal caries: are described in the text.
141 •
PAINCONTROLANDTREATMENTPLANNINGFORCARIOUSPRIMARYTEETH 24
Key point
Advantages o dressing (stabilization) open cavities are:
• Introduction to dental procedures and usually does not
involve local analgesia or complete caries removal.
• Reduction o Streptococcus mutans count. The
temporary restoration deprives the bacteria in the
active lesion o sugar and oxygen.
• GIC acts as a uoride reservoir.
• Eating and toothbrushing are more com ortable.
A
An acute and / or spread ing infection or sw elling m ay
requ ire the p rescrip tion of antibiotics. This is d iscu ssed in
Chap ter 25. Antibiotics shou ld only be p rescribed for pain
in the absence of sw elling for im m u nosu ppressed p atients.
Ad vice abou t analgesics for pain w ill be necessary to
su p p ort the em ergency treatm ent (Table 24.2).

Table 24.2 Dosages or common paediatric analgesics ( or children


12 and older, re er to adult doses)
Drug Dosage
Paracetamol 20 mg/kg initiallythen 10–15 mg/kg every4–6 hours B

Maximumo our doses in a 24 hour period Fig. 24.2 (A&B) Paul’s upper and lower arches.
Ensure adequate hydration
Ibupro en (non steroidal 10 mg/kg every8 hours
anti in ammatorydrug (NSAID)) Can be used in conjunction with paracetamol
Best given with ood and drink

History
Questioning su ggested an irreversible p ulpitis in the u pper
right qu ad rant.

Examination
Pau l has no extraoral sw elling or asym m etry. Intraorally he
has all his prim ary teeth. There is occlu sal caries of E and A
m esial caries of A A. The E is grossly cariou s (Fig. 24.2A).
There is no associated soft tissu e sw elling.
■ What investiga tion is essential to allow you to ormula te a
treatment plan?
Bitew ing rad iograp hs are necessary to d iagnose ap p roxi-
m ate caries in p rim ary m olars w ith their w id e contact areas
and to con rm the d ep th of d entine caries. Rad iograp hs w ill
increase the d iagnosed yield of caries by 50%. Frequ ency of
su bsequ ent bitew ing rad iograp hs w ill d ep end on the clas-
si cation of caries risk: high risk p atients shou ld have rad io-
grap hs taken every 6–12 m onths; m ed iu m risk 12–18
m onths; low risk 18–24 months. In Pau l’s case, the low er B
arch (Fig. 24.2B) and bitew ing rad iograp hs (Figs 24.3A and
Fig. 24.3 (A&B) Paul’s bitewing radiographs.
B) show extensive caries of E involving the p u lp. There is
E
also caries extend ing into the m id -d entine for .
E DE ■ Wha t is your def nitive trea tment plan or E ?
The E w as u nrestorable w ith the d istal m argin of the
Treatment caries extend ing su bgingivally. Therefore the only option
Initial tem porization of Pau l’s E w as w ith w ith Od onto- for the E w as extraction. The tem porary d ressing alleviated
p aste and GIC. fu rther p ain from this tooth and allow ed for behaviou r
• 142
24 PAINCONTROLANDTREATMENTPLANNINGFORCARIOUSPRIMARYTEETH
m anagem ent and acclim atization to the d ental su rgery to be Table 24.3 Paul’s proposed treatment plan
u nd ertaken. Extracting p rim ary teeth on a child ’s rst visit Teeth af ected
to a d entist should be avoid ed w here possible. If an extrac- by caries Extent o caries Restorative plan
tion is u nd ertaken on the rst visit, it is likely to lead to E Into pulp Extraction
signi cant d ental anxiety and relu ctance to attend further
AA Outer 1/3rd dentine Disk
d ental visits.
E Mid 1/3rd dentine Hall crown
■ Will local a naesthetic be needed or extracting E ?
EE
Where the d ecision has been m ad e to extract E , local anaes- Outer 1/3rd dentine Partial caries removal, ollowed bycomposite
D
thetic is essential. It is tim e consum ing to give local anaes- restoration and fssure sealant
thetic in a p ain free and child -friend ly w ay and ensu re the
entire tooth is anaesthetized p rior to the extraction. Often
Table 24.4 How preventive and restorative treatment was provided
child ren and p arents are su rp rised how qu ickly the tooth is
or Paul on a visit by visit basis
extracted in com parison to the tim e taken for the local
anaesthetic stage. Visit Preventive plan Restorative plan
1 Pain control E temporary
■ Wha t are the consequences o extracting E ?
2 Bitewing radiographs
Extracting the E is likely to lead to localized sp ace loss, w ith
Toothbrushing instruction
the 6 d rifting m esially into this sp ace. This esp ecially is the
case w hen the extraction is u nd ertaken p rior to the eru p tion Fluoride advice
of the rst p erm anent m olar. The d rifting w ill exacerbate Hand out diet diary
any tend ency for crow d ing and can frequently lead to the Plaque score
palatal exclu sion of the m axillary p erm anent second prem o- Apply uoride varnish
lar in ad olescence. Althou gh sp ace m aintainers have been
DD
ad vocated to p revent sp ace loss follow ing p rem atu re extrac- 3 Collect in diet diaryand give advice Fissure seal
DD
tion of primary m olars, there is a lack of robu st evid ence to
support their longevity and cost effectiveness in you ng chil- DiskAA
d ren w ith extensive d ental caries. 4 Toothbrushing instruction Plaque score
■ I the E was restorable, what other treatment options Restore D
would be available? Place separators E
The only other option for a tooth d iagnosed w ith irrevers- 5 Rein orce diet advice Hall crown E
ible p u lp itis or w here it is necrotic and non-vital is a p u lp ec-
Place separators EE
tomy. A p u lp ectom y is sim ilar to a root canal treatm ent in
6 Toothbrushing instruction Plaque score
perm anent teeth. It is m ad e m ore com p lex by the ared and
irregu lar shap e of the root canals, as w ell as the need for the Hall crowns EE
roots to resorb, thu s allow ing natu ral exfoliation of the 7 Post extraction pain control Extract E
primary tooth. Althou gh hand les are u sed for gentle
shap ing, the m ost im p ortant stage of the proced u re is d is-
penetration. Table 24.3 shows such a list or Paul based
infection of the root canal w ith carefu l u se of hyp ochlorite.
on his clinical examination and bitewings.
Like in the perm anent d entition, a ru bber d am is essential
to prevent salivary ingress into the root canal and the escap e C. Develop a robust and rigorous preventive plan as
of the hypochlorite into the oral cavity. Once the w orking described in Chapters 22 and 23 and shown in Table 24.4.
length is established and the root canals d isinfected , canals D. Evaluate Paul’s cooperation to treatment ollowing the
are lled w ith either zinc oxid e cem ent or a calciu m delivery o behaviour management strategies described
hyd roxid e-iod oform p aste w hich w ill resorb w ith the root in Chapter 26. Although several visits may be required to
as part of the p hysiological root resorp tion. develop the child’s cooperation, it is not acceptable to
leave an in ected tooth untreated or longer than 3
months.
Treatment planning E. There are multiple options available or treating primary
■ How do we underta ke treatment pla nning and devise a teeth, and f exibility in approach is needed as treatment
plan which is appropriate or ea ch and every pa tient? options may change in response to parental expectations
and willingness to comply with the home-based
There are several im p ortant stages to treatm ent p lanning: preventive advice, the child’s cooperation and the
A. The rst priority is to relieve pain. Where pain relie can development o urther toothache or other symptoms. In
be provided, even over a short time rame, this permits addition, the choice o how the treatment is undertaken,
time to develop a holistic treatment plan described in e.g. local anaesthetic, sedation or general anaesthetic,
steps B E. described in Chapter 26, may dictate what treatment is
B. Identi y all pathology. Bitewings (Figs 24.3A and B) and available and appropriate. For example, all carious teeth
other radiographs are essential to support the detection may be extracted even when some are restorable where
o caries. A list should be drawn up o each tooth an ‘extraction-only general anaesthetic’is the only
a ected by caries together with the depth o caries method available or an uncooperative child in pain.
143 •
PAINCONTROLANDTREATMENTPLANNINGFORCARIOUSPRIMARYTEETH 24
E was extracted. This was provided towards the end o
Key point his course o treatment. Earlier, simpler visits (Table 24.4)
The ollowing considerations will in uence the treatment had developed his cooperation and con dence, thus
provided: permitting this more di cult procedure to be
• Parental expectations and willingness to comply with undertaken without the need to use sedation or general
home-based preventive advice. anaesthesia (see Chapter 26).
• Child’s cooperation.
• Development o urther pain and symptoms.
• Dental care pro essional’s expertise and training in
Key point
di erent treatment procedures.
Advantages and disadvantages o di erent restorative
approaches:
• Preventive
Di erent restorative philosophies These d ifferent ap - Advantages: limited child cooperation needed, some
proaches can be used at a p atient level or at a tooth level cooperation is needed to modi y carious cavities to
w ith several em p loyed w ithin one p atient. Table 24.4 show s improve toothbrushing access.
Paul’s treatm ent plan and includ es several d ifferent ap- Disadvantages: high levels o parental cooperation, as
proaches to m anage his cariou s teeth. they will need to undertake a rigorous preventive plan,
• Prevention only. This approach requires a high level o more requent clinic visits, accurate diagnosis o the
engagement and cooperation rom the parents as extent o caries to ensure these teeth are at low risk o
treatment is predicated on their compliance with the pain or in ection.
preventive advice provided. Preventive advice as • Sealing in caries with partial or no caries removal
discussed in Chapters 22 and 23 is provided with regular Advantages: moderate child cooperation, no local
clinic visits or f uoride varnish application. To support anaesthetic and minimal or no drilling.
daily plaque removal, carious cavities can be adjusted to
Disadvantages: more requent clinic visits to monitor
acilitate toothbrush access to remove plaque. For Paul,
seal, more radiographs to monitor any caries
early dentinal caries was identi ed on the mesial aspects
progression, accurate diagnosis o the extent o caries
o A A. The mesial aspects o these teeth were ‘disked’to
with a low risk o irreversible pulpitis, short term
allow or better toothbrush access. This decision was
occlusal disturbance ollowing Hall crown placement.
made based on the short time these teeth had to be
• Complete caries removal
retained be ore they ex oliated, the dark appearance o
Advantages: high levels o child cooperation, can
the caries suggesting it was arrested, the low likelihood
directly assess pulpal health at time o pulpotomy,
o caries progression or pain and parent satis action with
permits treatment o more extensive caries, can modi y
this approach.
crown to ensure f t o stainless steel crown, permits
• Sealing in caries with no or partial caries removal. This
tooth coloured restorations or caries involving the
approach is based on the biological principle that by
outer and mid third o dentine.
sealing the cavity, the carious process cannot progress
Disadvantages: risk o iatrogenic damage to adjacent
without nutrients and will there ore either slow down or
teeth, requires high levels o clinical expertise and
arrest completely. There ore ensuring a good seal is
e ciency, local anaesthetic and rubber dam needed.
essential. This can be achieved using a number o
di erent materials and with or without partial caries
removal. Best examples o these techniques in the
primary dentition are the Hall technique, sealing in
dentinal caries and indirect pulp therapy. For Paul, his
cooperation was limited and there was little likelihood o Hall crowns and indirect pulp therapy
him cooperating or our separate visits o complete
caries removal under local anaesthetic. There ore Hall ■ Wha t are Hall crowns?
crowns were provided on the three other a ected Stainless steel crow ns are a very effective m ethod for restor-
E ing cariou s p rim ary m olars. H istorically, it w as only felt that
second primary molars ( ) and a composite restoration
EE these crow ns cou ld be placed u sing a com plete caries
on D. The D used simple hand and slow handpiece removal approach to ensure a good t w as achieved thereby
excavation to remove only the in ected dentine. optim izing the coronal seal. Furtherm ore, this approach
• Complete caries removal. This approach involves complete m inim ized any occlu sal d istu rbance. Using this conven-
caries removal and necessitates the need or local tional approach (inclu d ing occlusal red uction and m esial
anaesthetic, rubber dam and ast and slow handpieces. and d istal slices) excellent long-term outcom es w ere
Consequently it requires signi cant levels o cooperation achieved .
rom the child. All in ected carious dentine is removed. In the last ten years, an alternative (H all crow n) ap proach
Only once this is completed is the tooth restored. Where to their application has em erged . This involves no caries
caries extends into the pulp, a pulpotomy is undertaken. removal or local anaesthetic. The correct size of crow n is
Paul coped with one visit o local anaesthetic where the chosen and lled w ith GIC. The crow n is then p u shed onto
• 144
24 PAINCONTROLANDTREATMENTPLANNINGFORCARIOUSPRIMARYTEETH
the tooth using the child to bite it d ow n into p lace. The sym p tom s su ggesting irreversible p u lp itis (see Table 24.1)
coronal seal generated by the crow n and cem ent lead s to the or p ulp al necrosis. This includ es the absence of a sinu s,
arrest of the u nd erlying caries. Fu rther inform ation on sw elling, peri-furcation rad iolucency, internal resorption,
the H all techniqu e is provid ed in a sp eci c m anu al w ith the tend erness to p ercu ssion or a clinical history of irreversible
w eblink p rovid ed in the Prim ary Resou rces and Recom - pu lpitis.
m end ed Read ing section. The in am ed coronal pulp is rem oved u nd er local anaes-
■ When ca n I use Hall crowns? thesia (LA) and ru bber d am w ith the healthy non-in am m ed
pu lp stum ps d ressed for 15 second s w ith 15.5% ferric su l-
There are several im p ortant caveats to H all crow ns, and like phate. Follow ing this ap plication, bleed ing from the pu lp
all restorative techniqu es, they have ad vantages and d isad - stu m p s shou ld stop . The technique allow s one fu rther
vantages to their u se. Most im p ortantly, a carefu l pain ap plication of ferric su lphate if the bleed ing has not stop ped
history and bitew ing rad iograp hs are im p ortant stages in entirely after the rst ap plication. The cham ber is then lled
id entifying p u lpal health. Where caries is w ithin the inner w ith zinc oxid e eu genol cem ent or Mineral Trioxid e Aggre-
third of d entine or there is history of pain or signs and gate. The d efau lt restoration for a pu lpotom ised p rim ary
symp tom s of p u lpitis or p u lp al necrosis, it is u nw ise to m olar is a stainless steel crow n to facilitate an excellent
u nd ertake a H all crow n. For Pau l, the caries w as lim ited to coronal seal. If the pu lp stu m p s d o not stop after tw o appli-
the outer half of the d entine and there w ere no sym p toms cations of ferric su lp hate, treatm ent shou ld p roceed to either
E a vital p ulpectom y (a variation of the p u lpectom y techniqu e
from .
E E d escribed earlier in the chap ter) or the tooth shou ld be
Ow ing to the tight contacts seen in the p rim ary d entition, extracted . This continu ed bleed ing d em onstrates in am m a-
pu shing the crow n throu gh these contacts can take signi - tion extend ing into the p u lp stu m p s and early tooth loss is
cant force. Where tight contacts are seen, an alternative likely if a p ulp otom y is u nd ertaken (as a resu lt of internal
approach is to place orthod ontic sep arators for u p to 2 resorp tion).
w eeks to p rovid e sp ace for p lacem ent of the crow n. Pu lpotom ies are an area of active research as show n by
■ Wha t will happen to the child’s occlusion ollowing the the recent Cochrane review on the su bject. This id enti ed
placement o a Hall crown? 47 rand om ized controlled trials on the su bject. Current areas
of active research inclu d ed how haem ostasis is achieved
As there is no occlu sal red uction as p art of the H all tech-
and alternatives to ferric su lphate and old er agents such as
niqu e, the crow n w ill be p rou d and cau se occlu sal interfer-
form ocresol. Althou gh form ocresol has show n long-term
ence. Research has show n that after a few w eeks, there is
su ccess and is still u sed in som e parts of the w orld , there
occlu sal ad ju stm ent w ith the crow n no longer sitting p rou d .
have been international concerns over its toxicity both
A post-op erative bitew ing is ad visory once all H all
locally and system ically. These concerns have grow n in the
crow ns have been p laced to ensu re the crow ns are fu lly
past 15 years w ith form ald ehyd e, one of the im portant com -
seated and have sealed the cariou s cavity. This esp ecially is
ponents of form ocresol being associated w ith nasop haryn-
tru e for proxim al caries.
geal cancer. Form ocresol is no longer ad vocated as a
■ How does indirect pulp therapy (IPT) di er rom front-line m ed icam ent for p u lp otom ies and is no longer
a Hall crown? tau ght in UK u nd ergrad u ate cu rricu la.
IPT is a proced ure requ iring local anaesthetic and ru bber
d am. Infected caries is rem oved from the m argins of the
cavity. At the cavity base, carefu l caries rem oval is u nd er-
Primary resources and
taken to rem ove as m u ch soft infected d entine as possible recommended reading
bu t to ensure there is no pu lp al exp osu re. Calciu m hyd rox- Am erican Aced em y of Ped iatric Dentistry 2014 Policy on
id e is then p laced over the d eep caries before a d e nitive p ed iatric p ain m anagem ent. Reference Manu al 36 (6):78–79.
restoration is placed . It is essential that the restoration Pain control for child ren 2013 In: Cam eron A, Wid m er R (ed s),
provid e an excellent coronal seal. H and book of Ped iatric Dentistry, 4th ed n. Mosby, Ed inbu rgh,
IPT and H all crow ns are sim ilar in that they both requ ire p p . 25–46.
an excellent d iagnosis of p u lp al health. In both techniqu es Deery C 2013 Caries d etection and d iagnosis, sealants and
infected caries is left, and both rely on an op tim al coronal m anagem ent of the p ossibly carious ssu re. Br Dent J 214
(11):551–557.
seal to p revent caries p rogression. The d efau lt restoration
m aterial for both techniqu es is a stainless steel crow n. In the Du ggal MS, Day PF 2005 Op erative treatm ent of d ental caries in
the p rim ary d entition. In: Welbu ry RR, Du ggal MS, H osey MT
perm anent d entition, step w ise caries rem oval (of w hich IPT
(ed s), Paed iatric Dentistry, third ed . Oxford University Press,
is a form of) has show n very p ositive resu lts in red u cing the Oxford , p p . 149–174.
need for root canal treatm ent.
Evans D, Innes N 2010 The H all Technique: A Minim al
Intervention, Child Centred Ap proach to Managing the Cariou s
Prim ary Molar. University of Du nd ee. Available at: http s:/ /
Pulpotomies d entistry.d u nd ee.ac.u k/ sites/ d entistry.d u nd ee.ac.u k/ les/
3M_93C%20H allTechGu id e2191110.p d f.
■ Wha t is a pulpotomy?
Facu lty of General Dental Practice (FGDP) 2013 Selection Criteria
Pu lpotom y is a proced u re for d eep caries involving or for Dental Rad iograp hy, third ed . Lond on, FGDP.
in close ap p roxim ation to the p u lp . Carefu l p re-op erative Franzon R, Gu im araes LF, Magalhaes CE et al 2014 Ou tcom es of
d iagnosis is essential to ensu re the absence of signs and one-step incom plete and com plete excavation in prim ary teeth:
145 •
PAINCONTROLANDTREATMENTPLANNINGFORCARIOUSPRIMARYTEETH 24
a 24-m onth rand om ized controlled trial. Caries Res 48 http :/ / w w w.sd cep .org.u k/ w p -content/ u p load s/
(5):376–383. 2013/ 03/ SDCEP_PM_Dental_Caries_Fu ll_Gu id ance1
Ricketts D, Lam ont T, Innes N P, et al 2013 Op erative caries .p d f.
m anagem ent in ad u lts and child ren. Cochrane Database of Sm ail-Fau geron V, Courson F, Du rieu x P et al 2014 Pu lp treatm ent
Syst Rev Issue 3. Art N o: CD003808. DOI: 10.1002/ 14651858. for extensive d ecay in p rim ary teeth. Cochrane Database of Syst
CD003808.pub3. Rev Issu e 8. Art N o: CD003220. DOI: 10.1002/ 14651858.
Scotish Dental Clinical Effectiveness Program m e (SDCEP) 2010 CD003220.p u b2.
Prevention and Managem ent of Dental Caries in Child ren:
Dental Clinical Gu id ance. Du nd ee, SDCEP. Available at: For revision, see Mind Map 24, page 244.
25
Acu te infections tend to present w ith facial cellulitis
rather than a facial abscess w ith p us. Danny w as febrile,
althou gh he w as not in any signi cant pain, because the
infection had perforated the cortical plate. The m ainstay of
treatm ent is rem oval of the cause – either p ulpal extirpation
or extraction of the tooth.

History
Facial swelling and Danny traum atized his u pper right lateral incisor 3 w eeks
ago, su staining a d eep enam el d entine fractu re. The d entine

dental abscess
w as d ressed w ith calciu m hyd roxid e and a glass ionom er
cem ent w as p laced over the exp osed d entine and enam el.
H e had a review appointm ent w ith his d entist the follow ing
w eek.
On Saturd ay m orning his m other noticed that his cheek
w as sw ollen and the tissu es arou nd his right eye w ere ‘p u ffy
and red ’. H e attend ed the accid ent and em ergency d epart-
m ent of the local hosp ital, w here he w as prescribed Am oxi-
SUMMARY cillin 250 m g tablets to be taken three tim es d aily for 5 d ays.
Danny is 12 years old. He attended with a large Unfortu nately, by Sund ay evening Danny had becom e list-
less and his sw elling had increased . H e felt hot.
acial swelling a ter an episode o trauma 3 weeks
previously. He eels unwell, and his right eye is
closing (Fig. 25.1). Examination
The presentation o acute in ection, as Extraorally there w as facial asym m etry w ith a sw elling of
demonstrated by Da nny, is very di erent rom the right m axillary canine fossa. The overlying skin w as red
chronic in ection. and hot. The right eye ssure w as partially closed . Danny’s
■ List our symptoms and signs specif c to each type o temp erature w as 39°C. Maxillary canine fossa infections can
in ection sp read via em issary veins, w hich have no valves, to the
intracranial venou s system causing either a cavernou s sinus
Acu te: throm bosis or a brain abscess. The pathw ays of the IIIrd
• Sick, upset child. and VIth cranial nerves lie in the w alls of the cavernou s
• Raised temperature. sinu s. Throm bosis in the cavernou s sinu s can present w ith
• Red, swollen ace. a squ int d u e to involvem ent of the IIIrd and VIth cranial
• Reduced intake o ood and drink nerves, w hich are involved in control of the extraocu lar
Chronic: m u scles.
• Buccal sinus may be present. ■ What is the major problem with mandibula r in ections?
• Mobile tooth. Spread alongside the ascial planes that surround the airway
• Halitosis. with subsequent narrowing o the airway and stridor.
• Discoloured tooth.
Spread via the ascial planes to the mediastinum to cause a
Other signs and sym ptom s can be seen in both acu te and mediastinitis.
chronic infection, includ ing pain and lym p had enop athy.
■ What is the basic mana gement o a ny in ection?
Removal o the cause extraction or root canal therapy.
Local drainage and debridement via root canal or incision
and drainage.
Oral antibiotics i systemic involvement (see Table 25.1)
Amoxicillin or Penicillin V are usually the drugs o rst choice.
Amoxicillin has the advantage that it is given with ood and
only needs to be taken three times per day. Metronidazole,
which is active against anaerobes, can be added to either
Amoxicillin or Penicillin i the in ection is severe. O ten the
extraction o the abscessed tooth alone will bring about
resolution without antibiotic therapy. It is important that
antibiotics alone should not be considered as a rst line o
treatment unless there is systemic involvement. In a child a
temperature o 39°C or higher can be considered signi cant
Fig. 25.1 Severe in ection o canine ossa. (normal 37°C). Immunosuppressed patients and those with
147 •
FACIALSWELLINGANDDENTALABSCESS 25
Table 25.1 Common antibiotics used in paediatric dentistry
Drug Route Dose Frequency Notes
Antibiotics
Amoxicillin PO 25–50 mg/kg/day tds Syrup or chewable tablets or young children
IV 100–400 mg/kg/day tds
Amoxicillin plus clavulanic acid PO 20–40 mg/kg/day tds For beta lactam resistant organisms only
Ampicillin IV 50–100 mg/kg/day qds
IV 50 mg/kg stat
Benzyl penicillin IV 15–350 mg/kg/day
20000–500000 U/kg/day qds First IVdrug o choice or odontogenic in ections
PenicillinV PO < 5 years 500 mg/day
> 5 years 1–2 g/day qds Give 1 hour be ore meals
Cephalexin PO 25–50 mg/kg/day qds
Cephazolin IV 25–50 mg/kg/day
Erythromycin PO 25–40 mg/kg/day qds Ethylsuccinate is readilyabsorbed
Metronidazole IV 22.5 mg/kg/day tds Not in pregnancy
Gentamicin PO 10–15 mg/kg/day tds
Clindamycin PO,IV 15–40 mg/kg/day qds Risko pseudomembranous colitis

Analgesics see Chapter 24, Table 24.2


PO, per oral; IV, intravenous; IM,intramuscular; PR, per rectum; tds; three times daily; qds, our times daily; stat, at once
(Frontline antibiotics used or acial swellings and dental abcesses are Amoxycillin, Penicillin Vand Metronidazol)

cardiac disease should receive antibiotics immediately i any d azole shou ld be ad d ed as anaerobic organism s p lay a sig-
in ection is suspected. ni cant role.
Maintenance uid s w ill be given until the child is d rink-
■ What are the criteria or hospital a dmission with oro acial
ing norm ally again.
in ection?
Warm saline m ou thw ashes.
Dehydration. Ask whether the child has had a decreased Ad equ ate pain control com m only w ith paracetam ol or
requency o micturition (urine output) in previous 12 hours. ibu p rofen (Table 24.2).
Signi cant in ection or temperature greater than 39°C. If the eye is shu t d u e to a sw elling in the canine fossa,
it may be necessary to give chloram p henicol eye d rop s
Floor o mouth swelling.
0.5% or chloram p henicol ointm ent 1.0% to prevent
■ What will the hospital mana gement o a severe in ection conju nctivitis.
involve?
Extraction of involved teeth. It is im p ossible to d rain a sig-
ni cant infection solely throu gh the root canals of a tooth. Key point
Drainage of any p u s. In ad d ition to extractions, there m ay Hospital admission in oro acial in ection is necessary with:
be a need to incise and d rain p u s, often leaving a d rain in
• Dehydration.
situ for a few d ays to enhance d rainage. With severe m an-
• Temperature >39°C.
d ibular sw ellings or w here the oor of the m ou th is raised ,
it may be necessary to have an extraoral d rain throu gh a • Floor o mouth swelling.
skin incision or a ‘throu gh and throu gh’ d rain, w hich p asses • Trismus or signif cant swelling around the eye leading
comp letely throu gh the area of infection. Extraoral incisions to complete closure.
are to be avoid ed if at all p ossible d u e to p ost-op erative • Breathing and swallowing di culties.
scarring. Sw abs of p u s for laboratory cu lture to establish
accu rate sensitivities of the organism s concerned to com mon
antibiotics.
Intravenou s antibiotics. Benzyl p enicillin is the d ru g of Treatment
rst choice, or Am oxicillin. Cep halosp orins are effective if
there is a p enicillin allergy, bu t there is som e cross-reactivity Danny w as treated by the follow ing regim en:
in those patients allergic to penicillin and so cephalosporins 1. Extirpation o 2 . The tooth was immature and good
shou ld be u sed w ith care, esp ecially w hen there w as a drainage o pus was achieved through the root canal.
severe reaction to p enicillin. In severe infections, m etroni- 2. Open drainage o the tooth or 2 days.
• 148
25 FACIALSWELLINGANDDENTALABSCESS
3. Amoxicillin 250 mg three times daily, metronidazole Primary resources and
200 mg twice daily or 5 days each.
4. Hot saline mouthwashes.
recommended reading
Cam eron A, Wid m er R (ed s), 2013 Paed iatric oral m ed icine and
After 2 d ays Danny w as review ed . The sw elling w as
p athology. In: H and book of Ped iatric Dentistry, 4th ed n. Mosby,
red u ced around his right cheek, and his right eye w as Ed inbu rgh, p p . 209–268.
norm al. The 2 w as cleaned and led and non-setting
Am erican Acad em y of Ped iatric Dentists 2015 Guid eline on u se
calcium hyd roxid e placed into the canal. The access cavity of antibiotic therap y for p ed iatric d ental p atients. Ref Man 36
w as sealed w ith cotton w ool and glass ionom er cem ent. The (6):284–286.
non-setting calciu m hyd roxid e w as rep laced once after a
m onth. Once the root canal w as free from any signs or
symp tom s of infection, the tooth w as obu rated w ith a For revision, see Mind Map 25,
Mineral Trioxid e Aggregate p lu g and w arm gu tta p ercha. page 245.
26
be m ore d if cu lt to treat. They often give up becau se they
‘never succeed ’ and are often called ‘failures’ by their
p arents or peers.
Communicative management. This is the m ost fu nd am ental
form of behaviour m anagem ent. It is the basis for establish-
ing a relationship w ith a child that w ill allow you to su c-
cessfu lly com p lete d ental p roced u res and help the child
d evelop a p ositive attitu d e tow ard d ental health.

The uncooperative child


■ What main orms o communica tive mana gement
are there?
Non-verbal communication.

and adolescent Tell-show-do.


Positive rein orcement.
Distraction.
Voice control.
Parental presence/absence.

CASE1 Non-verbal communication is the reinforcem ent and gu id -


ance of behaviou r throu gh ap prop riate contact, p osture and
facial exp ression.
SUMMARY Objectives:
Liam is 5 years old. He is shaking and tear ul as he To enhance the e ectiveness o other communicative
is brought into the surgery. His mother says he has management techniques.
been in pain rom his teeth or a long time. How To gain or maintain the patient’s attention and compliance.
would you manage Liam and his dental treatment? Ind ications:
■ What do you understand by the term behaviour
May be used with any patient.
management?
Contraind ications:
Behaviou r m anagem ent inclu d es a nu m ber of skills:
None.
emp athy, com m u nication, coaching and listening. These
skills need to be com bined w ith an u nd erstand ing of child Tell-show-do is a techniqu e of behaviou r shap ing u sed
d evelop m ent and p sychology. by m any p aed iatric p rofessionals. The technique involves
The goals are to establish com m u nication, alleviate fear verbal explanations of proced u res in phrases approp riate
and anxiety, d eliver qu ality d ental care, build a tru sting to the d evelop mental level of the patient (tell); d em on-
relationship betw een d entist and child and prom ote the strations for the p atient of the visu al, aud itory, olfactory
child ’s p ositive attitu d e tow ard s oral/ d ental health. and tactile asp ects of the p roced ure in a carefully d e ned ,
All d ecisions m u st be based on a bene t versu s risk eval- non-threatening setting (show ); and then, w ithou t d evi-
uation. Parents/ legal gu ard ians share in the d ecision- ating from the exp lanation and d em onstration, com ple-
m aking p rocess regard ing treatment of their child ren. They tion of the p roced u re (d o). The tell-show -d o techniqu e is
are also resp onsible for d ental attend ance, Liam ’s oral u sed w ith com m u nication skills (verbal and non-verbal)
health regim e (inclu d ing p u rchase of toothp aste and u nd er- and positive reinforcem ent. The langu age chosen m ust
taking toothbru shing) and control w hat Liam eats and be appropriate to the child ’s level of u nd erstand ing and
d rinks. It is therefore essential to establish a good relation- exp erience.
ship w ith p arents to ensu re they are su p p ortive and com p ly Objectives:
w ith the p reventive ad vice p rovid ed .
To teach the patient important aspects o the dental visit and
■ What history is important in Liam’s case?
amiliarize the patient with the dental setting.
Liam’s dental history. It is cru cial to id entify any p reviou s To shape the patient’s response to procedures through
ep isod es at the d entist, d octor or hosp ital, u su ally involving desensitization and well-described expectations.
need les, that m ay have frightened him . If there are no previ-
Ind ications:
ou s p recip itating factors he m ay have been frightened by
stories or com m ents from his p eers or fam ily. May be used with any patient.
Family dental history. Parental fear, and a negative attitud e Contraind ications:
tow ard d ental and oral health, can signi cantly affect the None.
coop eration of a child .
Liam’s development. Delayed d evelop m ent and p oor cog- Positive rein orcement In the p rocess of establishing d esir-
nition can affect the ability of a child to u nd erstand w hat able patient behaviou r, it is essential to give appropriate
you are trying to d o to help. Child ren w ith a negative im age feed back. Positive reinforcem ent is an effective technique
of them selves w ho have never su cceed ed at anything w ill to rew ard d esired behaviou rs and thus strengthen the
• 150
26 THEUNCOOPERATIVECHILDANDADOLESCENT
recu rrence of those behaviou rs. Social reinforcers includ e Objectives:
positive voice m od u lation, facial exp ression, verbal praise To gain the patient’s attention and compliance.
and ap p ropriate physical d em onstrations of affection by all
To avert negative or avoidance behaviours.
m em bers of the d ental team. N on-social reinforcers inclu d e
tokens and toys. To establish appropriate adult-child roles.
The d elivery of p ositive reinforcem ent shou ld follow To enhance the communication environment.
these p rinciples. When the d esired behaviou r is show n it
Ind ications:
should be immediately rew ard ed . Each tim e the behaviou r is
show n, consistency is need ed in rew ard ing this behaviou r. It May be used with any patient.
should be clear w hat behaviou r is d esired . The p ositive rein- Contraind ications:
forcer is only u sed contingent on that behaviou r being
None.
d isp layed .
Objective: All these com m unication techniqu es m ay be need ed to
enhance the evolution of a comp liant and relaxed patient. It
To rein orce desired behaviour.
is an ongoing subjective process rather than a singular tech-
Ind ications: nique and is often the extension of the personality of the
May be use ul or any patient. d entist.
Contraind ications:
None.
Key point
Positive rein orcement should be:
Distraction is the techniqu e of d iverting the p atient’s atten-
• Immediate.
tion from w hat m ay be perceived as an unpleasant
• Clear.
proced u re.
Objectives: • Consistent.
• Contingent.
To decrease the perception o unpleasantness.
To avert negative or avoidance behaviour.
Ind ications:
May be used with any patient.
Examination
After sp end ing som e tim e talking to Liam and show ing him
Contraind ications:
that you are genuine in w anting to help , he allow s you to
None. look at his teeth. Both low er rst p rim ary m olars are cariou s.
All the other teeth are sou nd . Liam has been frightened by
Voice control is a controlled alteration of voice volu me,
stories from his friend s. H is fam ily are very su p portive and
tone or pace to in uence and d irect the patient’s
are regu lar attend ers.
behaviou r.
Liam resp ond s w ell to com m u nicative m anagem ent, but
Objectives:
although he w ants to have his treatm ent carried ou t, he ju st
To gain the patient’s attention and compliance. cannot overrid e his fear of the u nknow n.
To avert negative or avoidance behaviour. ■ What a dditional help might you consider giving Lia m?
To establish appropriate adult-child roles. Inhalational sed ation has been an effective and safe m ethod
Ind ications: of red u cing anxiety and enhancing effective comm u nication
for the past 30 years. Its onset of action is rap id , the d ep th
May be used with any patient.
of sed ation is easily titrated and reversible and recovery
Contraind ications: is rap id and com p lete. Ad d itionally, nitrou s oxid e m ed iates
None. a variable d egree of analgesia, am nesia and gag re ex
red uction.
Parental presence/absence This techniqu e involves u sing The need to d iagnose and treat, as w ell as the safety of
the presence or absence of the parent to gain cooperation the patient and practitioner, shou ld be consid ered before the
for treatm ent. A w id e d iversity exists in p ractitioner p hi- use of nitrou s oxid e. The d ecision to u se nitrou s oxid e m u st
losop hy and p arental attitu d e regard ing p arents’ p resence take into consid eration the follow ing p oints:
or absence d u ring p aed iatric d ental treatm ent. Practitioners
Alternative behaviour management modalities. (Although
are u nited in the fact that com m u nication betw een d entist
nitrous oxide sedation is e ective or mild to moderately
and child is p aram ou nt and that this com m u nication
anxious patients, it is not e ective or children who cannot or
d em and s focu s on the p art of both p arties. Child ren’s
will not breath through their nose some young children
responses to their parents’ presence or absence can range
have not developed the level o maturity and understanding
from very bene cial to very d etrim ental. It is the resp onsi-
to be able to ollow this requirement.)
bility of p ractitioners to d eterm ine the com m unication
m ethod s that best op tim ize the treatm ent setting; recogniz- Dental needs o the patient.
ing their ow n skills, the abilities of the p articu lar child and The e ect on the quality o dental care.
the d esires of the sp eci c parent involved . The patient’s emotional development.
151 •
THEUNCOOPERATIVECHILDANDADOLESCENT 26
The patient’s physical considerations.
Written in ormed consent must be obtained rom a legal CASE2
guardian and documented in the patient’s record prior to use
o nitrous oxide. SUMMARY
The p atient’s record shou ld inclu d e: Maria is a 12-year-old girl with early ca ries in her
In ormed consent. mandibular f rst permanent molars. She has an
Indication or use. increased overjet, and the orthodontic plan
N itrou s oxid e d osage:
includes our premolar extractions, provided her
ca ries risk is reduced and the dental disease is
• Percent nitrous oxide/oxygen and/or f ow rate.
treated.
• Duration o the procedure.
• Post-treatment oxygenation procedure.
■ How would you motivate Ma ria to reduce her ca ries risk
and assess and treat her dental anxiety?
Objectives:
To reduce or eliminate anxiety.
To reduce untoward movement and reaction to dental History
treatment.
Complaint
To enhance communication and patient cooperation.
Maria says she d oes not like to sm ile and is getting teased
To raise the pain reaction threshold. at school.
To increase tolerance or longer or more di cult treatment.
To aid in treatment o the mentally/physically disabled or
History o complaint
medically compromised patient. Maria w ent to her local d entist recently and received pre-
ventive ad vice bu t d id not m ange to accept local analgesia.
To reduce gagging.
Fu rther qu estioning revealed that she rem em bered being
Ind ications: ‘forced ’ to have a p rim ary tooth extracted at age 6 years and
A mild to moderately ear ul, anxious or obstreperous patient. has been fearfu l of d ental treatm ent since then.
Certain mentally, physically or medically compromised
patients. Medical history
There is no relevant m ed ical history.
A patient whose gag ref ex inter eres with dental care.
A patient in whom pro ound local anaesthesia cannot be Family history
obtained.
Maria’s m other says she is also afraid of going to the d entist.
Contraind ications:
May be contraindicated in some chronic obstructive
pulmonary diseases. Examination
Where the child has a blocked nose, common cold or Extraoral examination
tonsillitis, their appointment should be postponed until they
Maria has a slightly p rom inent increased overjet, bu t a Class
have recovered.
I skeletal base and norm al face height w ith no asym m etry.
May be contraindicated in certain patients with severe
emotional disturbances or drug-related dependencies. Intraoral examination
Patients in the rst trimester o pregnancy. Maria has good oral hygiene. She has stained ssu res on the
May be contraindicated in patients with sickle cell disease. m and ibu lar m olars. Bitew ing rad iograp hs su ggest that the
Patients treated with bleomycin sulphate. caries is into the enam el only.
Neuromuscular disease, e.g. myasthenia gravis, multiple ■ Wha t recommenda tions are used regarding diet, tooth-
sclerosis. brushing and uoride or adolescents?

Liam had inhalational sed ation, w hich enabled com p le- Id eally, w e w ou ld all hop e that by ad olescence a child is
tion of his treatm ent and helped him overcom e his fear of d isease free and therefore w ill need little op erative treat-
local anaesthesia. m ent. H ow ever, there are ad olescents w ho have caries or
other d ental problems, su ch as non-caries tooth tissu e loss;
Key point perhap s d ue to acid erosion, or period ontal d isease; or w ho
have aesthetic need s that require orthod ontic treatm ent,
The patient’s record in inhalational sedation should w hich p erhap s inclu d e p rem olar extractions as p art of the
include: treatm ent plan. Ad olescents w ho need orthod ontic extrac-
• In ormed consent. tions, in p articu lar, may not have had very m uch d ental
• Indications or use. treatm ent in the past, esp ecially if they are low caries
• Nitrous oxide dosage. risk. On the other hand the ad olescent w ho has high caries
risk (ap p roxim ately 14% of the ad olescent p op ulation,
• 152
26 THEUNCOOPERATIVECHILDANDADOLESCENT
Child Dental H ealth Su rvey 2013) often has a heavy bu rd en shou ld also includ e habits su ch as sm oking, experim ental
of d isease requiring m any visits to com p lete their restora- d ru g and alcohol frequ ency; e.g. ‘alco-p ops’ and also ‘rec-
tive treatm ent. reational’ d ru gs that cau se thirst lead ing to increased acid ic/
To overcom e the bu rd en of d isease, p reventive action is su gary d rink consu m p tion.
need ed . This is a tw o-pronged ap p roach that involves good ,
clear, evid ence-based p reventive ad vice from the d entist
Key point
and the active agreem ent to im p lem ent the ad vised behav-
iour changes from the ad olescent. The Dep artm ent of H ealth • Adolescents are sel -conscious and can have a ragile
(DoH ) Oral H ealth Preventive Toolkit contains evid ence- sel -esteem.
based prevention ad vice inclu d ing tw ice d aily tooth- • They want to be able to determine and make their own
bru shing w ith 1450 p p m u orid e toothp aste, the need for choices in li e.
regu lar d ental check-u ps and red u cing su gar frequency. • They have concerns about their looks.
In Maria’s case, 2800 p p m u orid e toothp aste shou ld be • They want to f t in with their peers.
recomm end ed . This is su itable for child ren over 10 years of
• Question or habits such as smoking, experimental
age w ho m ay be at risk for tooth d ecay, inclu d ing p atients
drugs and alcohol.
w ith active tooth d ecay or a history of tooth d ecay, exp osed
• Be alert to bullying and sa eguarding issues.
root surfaces, high-su gar d iets, orthod ontic ap pliances or
d ry m ou th.
Ad vice given should be evid ence based and given in a
know led geable context of ad olescent m ind -set and behav- ■ Ca n a nxiety be measured, and what questions should
iour change p sychological theory, su ch as u sing m otiva- be asked?
tional interview ing and cognitive behaviou r therap y. These There are variou s anxiety scales that help alert a d entist to
theories are generally based on ad olescents und erstand ing d ental anxieties. Thou gh these are not sensitive enou gh to
their ow n d ental need s and the challenges that they face in tease ou t the d ifferences betw een these tw o possible aetiolo-
imp lem enting recom m end ations and then taking action, gies, they m ay be of valu e. The com m on ones are the Mod i-
and ‘rew ard ing’ them selves for d oing so. ed Child Dental Anxiety Scale (MCDAS) and the Child
This ap p roach to p revention w ill also help in Maria’s Fear Su rvey Sched u le (CFSS). A sim p ler m easure, w hich can
anxiety m anagem ent becau se she w ill feel that she has a role also be valu able in younger child ren, is the Facial Im age
and a say in her ow n d ental m anagem ent. In this w ay, the Scale (FIS); this is sometim es com bined w ith the MCDAS to
d entist and Maria w ill alread y be w orking together; ‘on the assist you nger child ren’s com prehension. An exam ple is
sam e sid e’ against the d ental d isease. show n in Fig. 26.1.
H ow ever, these also need to be au gm ented by carefu l
Key point and thou ghtful qu estioning, especially relating to p ast
d ental history and p ossible p erceived traum atic events.
• Prevention o urther dental caries is essential, provided Those w ho report that they have a need le phobia should be
the caries that is already present is managed. asked if they m anaged to accept their vaccinations at school;
• Adherence to preventive advice is key to progression to e.g. the tetanu s/ polio booster is usu ally given at arou nd 13
orthodontic treatment a terwards. years of age in the UK.
• The DoH toolkit gives clear, evidence-based preventive
recommendations.
Treatment
■ What are the anxiety management options?
■ Wha t is the aetiology o dental a nxiety? N on-pharm acological behaviou r m anagem ent is just as
There are tw o reasons behind ad olescent anxiety. The rst effective w ith ad olescents as it is w ith you nger child ren, so
is d ental anxiety that is learnt over the p reced ing years and positive reinforcem ent, tell-show -d o and enabling a hand
cau sed by real or p erceived trau m a of p reviou s d ental or signal to ‘stop ’ to enhance Maria’s sense of control shou ld
m ed ical treatm ent. The second is d ental anxiety that is part all be u sed . Giving information abou t w hat w ill hap pen,
of a d eep er p sychological d istu rbance that is beginning to w hat it w ill feel like and how to cop e beforehand and d u ring
emerge. the p roced u re are all im p ortant. Psychological interventions
su ch as cognitive behaviou ral therap y can also p lay a key
■ Why should a n a dolescent be ma naged di erently orm a
role. The id ea behind this technique is to help Maria recon-
younger child?
sid er her belief that a fu tu re d ental extraction w ill be as
There are som e fu rther d etails that a d entist cannot overlook trau m atic as that rem em bered from her child hood .
w hen m anaging an anxiou s ad olescent. These relate to the Maria has alread y u nd ergone a d ental extraction at an
m ind -set and to the em erging character of the ad olescent earlier age, and in her m emory this w as traum atic. It is not
them selves. Ad olescents have a rather fragile self-esteem uncom m on for a child to perceive a p ast experience as ‘trau-
yet w ant to be able to d eterm ine and m ake their ow n choices m atic’, even if in reality it w as not. The fact is that in Maria’s
in life. They have concerns abou t their looks, especially ow n m ind the event w as d am aging to her, so this has to be
d ental aesthetics, and w ant to t in w ith their p eers. A taken into accou nt. In Maria’s case, she w ill need to u nd ergo
d etailed history, perhap s based on som e subtle qu estioning, extractions as part of her orthod ontic treatm ent p lan. It is
153 •
THEUNCOOPERATIVECHILDANDADOLESCENT 26
Modified Child Dental Anxiety Scale – MCDAS

For the next eight questions I would like you to show me how relaxed or worried you get about the dentist and what
happens at the dentist. To show me how relaxed or worried you feel, please use the simple scale below. The scale is
just like a ruler going from 1, which would show that you are relaxed, to 5, which would show that you are very worried.

1 would mean: relaxed/not worried


2 would mean: very slightly worried
3 would mean: fairly worried
4 would mean: worried a lot
5 would mean: very worried.

How do you feel about …

… going to the dentist generally? 1 2 3 4 5

… having your teeth looked at? 1 2 3 4 5

… having your teeth scraped and


1 2 3 4 5
polished?

… having an injection in the gum? 1 2 3 4 5

… having a filling? 1 2 3 4 5

… having a tooth taken out? 1 2 3 4 5

… being put to sleep to have


1 2 3 4 5
treatment?

… having a mixture of ‘gas and air’ which will help you feel
1 2 3 4 5
comfortable for treatment but cannot put you to sleep?

Fig. 26.1 MCDAS with FIS images.


(FromWong(1998), with permission.)

likely that d iscu ssing Maria’s fears w ith her and p u tting to assess treatm ent progression and to evalu ate w hither the
them into a d ifferent context, in line w ith a cognitive p lan is likely to su cceed .
behaviou ral therap y ap proach, and u sing excellent behav-
iou ral m anagem ent skills, giving her a locu s of control, and ■ Wha t treatment plan would you propose?
ensu ring ad equ ate analgesia w ill enable her to proceed to The treatm ent plan:
accep t her p lanned orthod ontic extractions. Resou rces and
fu rther gu id ance on the u se of cognitive behaviou ral therap y VISIT 1
are listed in the fu rther read ing section. 1. Preventive advice (this is discussed in greater detail in
■ What is the commonest conscious sedative in children Chapter 23) ollowing the DoH toolkit, so including:
and adolescents? a. Prescription o twice daily brushing with 2800 ppm
f uoride toothpaste.
For m od erately anxiou s child ren, nitrou s oxid e inhalation
b. Handing out a 4-day diet diary.
sed ation is the m ost com m only u sed sed ative and is effec-
2. Ask Maria to sign a contract to:
tive in ad olescents w ho requ ire p rem olar extractions. Maria
m ay need this in ad d ition to the behaviou ral techniqu es a. Reduce dietary sugar intake requency based on
alread y d escribed . The key is to carry ou t a thorou gh assess- tailored advice derived rom her diet diary.
m ent based on a frank d iscu ssion w ith her and her p arent(s) b. Implement twice daily toothbrushing with a
before the treatm ent p lan is d evelop ed , to inclu d e inhalation 2800 ppm f uoride toothpaste.
sed ation from the start if it is need ed , rather than have a c. Attend pre-agreed and booked dental
‘failed ’ behavioural m anagem ent visit before consid ering appointments.
consciou s sed ation as an option. It is also w ise to bu ild in a d. Conduct a discussion between dentist, parent(s) and
reassessment of the agreed plan after the rst or second visit patient that leads to written parental consent and
• 154
26 THEUNCOOPERATIVECHILDANDADOLESCENT
adolescent assent o sedative choice and operative ■ What other sedation techniques are availa ble?
treatment.
Other sed ation techniqu es su ch as oral or intravenou s m id a-
e. Document the consent and assent process and
zolam are effective. Dental sed ationists w ho offer this type
document written in ormed consent.
of treatm ent have to show that they have u nd ergone fu rther
. Disclose dental plaque and modi y toothbrushing training in the technique, as w ell as continu ed com petency
technique and choice o toothpaste regarding and experience in line w ith contem porary national gu id e-
f uoride dose; consider 2800 ppm f uoride lines and recom m end ations.
toothpaste. For young child ren oral sed ation is a viable option u sing
g. Introduce her to the inhalation sedation nasal mask m id azolam . A num ber of recent stud ies have show n that its
and technique. use in a d ose of 0.3–0.5 m g/ kg, d epend ing on age, has been
h. Fissure seal a maxillary rst permanent molar and therap eu tically effective in p rod u cing sed ation that has
introduce topical analgesia. allow ed su bsequ ent d ental treatm ent.
For ad olescents w ho require m ore invasive p roced ures
VISIT 2 (preventive advice will continue and expand on 2a,
or w ho are m ore severely p hobic, m id azolam intravenou s
2b, 2 )
sed ation is an op tion becau se it has the ad d ed bene t of
3. Use inhalation sedation or the rst time: causing am nesia. Am nesia also occu rs w ith oral m id azolam .
a. Fissure seal the remaining rst permanent molars. The p atient cannot rem em ber u nd ergoing the proced u re.
b. Introduce topical analgesia. They m ay rem em ber the start of the d ental visit and the
c. Reassess Maria’s caries risk by checking compliance beginning of the sed ation, bu t nothing u ntil after the sed a-
with dietary changes and toothbrushing, and tive effect has begu n to w ear off.
con rm premolar extraction plan with the
■ What a re the indications or general anaesthesia?
orthodontist. Reassess the use o nitrous oxide
inhalation sedation. Does Maria still need it? Patients who are unable to cooperate due to a lack o
psychological or emotional maturity and/or mental, physical
VISIT 3 AND 4 (preventive advice will continue and expand
or medical disability.
on 2a, 2b, 2 )
Patients or whom local anaesthesia is ine ective because o
4. Begin orthodontic premolar extractions perhaps in two
acute in ection, anatomical variations or allergy.
quadrants per visit. O ten an extraction in maxillary and
mandibular quadrant is undertaken rather than The extremely uncooperative, ear ul, anxious or
extracting both mandibular premolars on the same visit. uncommunicative child or adolescent.
For mandibular extractions, in ltrations using Articaine Extensive dental caries involving dental treatment in multiple
local anaesthetic rather than in erior dental blocks with quadrants.
Lidocaine is an e ective but less invasive method. Patients requiring signi cant surgical procedures ( or example,
■ Wha t ollow-up does Maria need now? the extraction o all rst permanent molars or a severely
in raoccluded primary molar).
Maria has su ccessfu lly u nd ergone the p rem olar extractions
and has red u ced her su gar frequ ency and is toothbrushing Patients or whom the use o general anaesthesia (GA)
w ith 2800 p pm u orid e tooth p aste. Therefore she is pro- may protect the developing psyche and/or reduce medical
ceed ing to have orthod ontic xed ap p liance treatm ent. As risks.
such, her caries risk is still high and she w ill need regular, Patients requiring immediate, comprehensive oral/
four-m onthly follow -u p s d u ring her orthod ontic treatm ent. dental care.
She w ill also need fu rther bitew ings to assess for new d ental
caries once yearly (once her orthod ontic ap p liance is ■ What a re the contra indications or general anaesthesia?
removed , u ntil no fu rther caries is seen and she is d eem ed
A healthy, cooperative patient with minimal dental need.
to be at low caries risk).
Maria feels hap py and p roud to have u nd ergone the Predisposing medical conditions that would make GA
extractions. She now enjoys visiting the d entist for check- inadvisable.
u ps and is excited abou t her new sm ile. General anaesthesia is a controlled state o unconsciousness
accompanied by a loss o protective ref exes, including the
ability to maintain an airway independently and respond
CASE3 purpose ully to physical stimulation or verbal command. It
should only be provided in premises with resuscitation
SUMMARY capability and intensive care back-up (e.g. an acute hospital
setting). All equipment must ollow current guidelines (please
Un ortunately, there are some children and see re erence section).
adolescents or whom inhalational sedation is Parental or legal guard ian inform ed consent m ust be
unable to overcome their anxiety, and alternative obtained and d ocu m ented prior to the u se of general anaes-
sedation agents or general anaesthesia is the only thesia. The p atient’s record should inclu d e:
option that will allow relie o pain and completion • In ormed consent.
o dental treatment. • Indications or the use o GA.
155 •
THEUNCOOPERATIVECHILDANDADOLESCENT 26
Therefore agreeing to a contract w ith an ad olescent
In ormed consent p atient, w here they sign-u p to agree to d ietary behaviou r
Regard less of the behaviour m anagem ent techniques u ti- changes and regu lar toothbru shing w ith the correct d osage
lized by the ind ivid u al p ractitioner, all m anagem ent d eci- of u orid e toothp aste, not only p rovid es them w ith a sense
sions m u st be based on a su bjective evalu ation w eighing of self d eterm ination and esteem but also tailors and clari-
bene t and risk to the child . Consid erations regard ing es the preventive practices that und erp in the d ental treat-
need of treatm ent, consequ ences of d eferred treatm ent and m ent p lan.
p otential p hysical/ em otional trau m a m u st be entered into It is alw ays p ru d ent to involve both the ad olescent and
the d ecision-m aking equ ation. the ad u lt w ith parental responsibility. Child ren u nd er 16
Delivery of d ental treatm ent is often a com p lex d ecision. years of age can consent for treatm ent if they u nd erstand
Decisions regard ing the u se of behaviou r m anagem ent tech- w hat is proposed . ‘It is u p to the d entist to d ecid e w hether
niqu es other than com m u nicative m anagem ent cannot be the child has the m atu rity and intelligence to fu lly und er-
m ad e solely by the d entist. Decisions m u st involve a legal stand the natu re of the treatm ent, the op tions, the risks
gu ard ian and , if app rop riate, the child . The d entist serves involved and the bene ts. A child w ho has su ch u nd er-
as the exp ert abou t d ental care, i.e. the need for treatm ent stand ing is consid ered Gillick com petent.
and the techniqu es by w hich treatm ent can be d elivered .
The legal gu ard ian shares w ith the p ractitioner the d ecision
w hether to treat or not to treat and m u st be consu lted
regard ing treatm ent strategies and p otential risks. Therefore Key point
the successful comp letion of d iagnostic and therapeutic • It is good practice to gain consent rom the adult with
services is view ed as a p artnership of d entist, legal gu ard ian parental responsibility and assent rom the child up to
and child . 16 years o age.
Althou gh the behaviou r m anagem ent techniqu es
• Where a child is considered Gillick competent, they can
inclu d ed in this chap ter are u sed frequ ently, p arents m ay
consent or their own dental treatment.
not be entirely fam iliar w ith them . It is im p ortant that the
• Di erent ages and rules apply in di erent parts o
d entist inform the legal gu ard ian abou t the natu re of the
the UK.
technique to be u sed , its risks, bene ts and any alternative
techniques. All qu estions m u st be answ ered . This is the
essence of inform ed consent.

■ Who can consent or a child? Primary resources and


Mother all mothers automatically have parental recommended reading
responsibility. Am erican Acad em y of Ped iatric Dentistry 2015 Gu id eline on
Fathers i married at the time o the child’s conception, behavior guid ance for the p ed iatric d ental patient. Reference
birth or sometime a ter this, this responsibility is not lost i the Manu al 37 (6):180–193.
mother and ather divorce. Am erican Acad em y of Ped iatric Dentistry 2015 Gu id eline on use
of local anesthesia for p ed iatric d ental p atients. Reference
Unmarried athers only have parental responsibility i Manu al 37 (6):199–205.
either given parental responsibility by a court order or in
Am erican Acad em y of Ped iatric Dentistry 2015 Gu id eline on use
agreement with the mother that is registered with the High of nitrou s oxid e for p ed iatric d ental p atients. Reference Manual
Court or the child was born a ter 1/12/03 ( or England and 37 (6):206–210.
Wales) and is named on the birth certi cate. Cam pbell C, Sold ani F, Bu su ttil-N au d i A et al 2011 N on-
Step parents only have parental responsibility i given by a Pharm acological Behaviou r Managem ent Gu id eline. Available
court order. at: http :/ / bsp d .co.u k/ Portals/ 0/ Pu blic/ Files/ Gu id elines/
N on-p harm acological%20behaviou r%20m anagem ent%20.p d f.
Grandparents, relatives, riends other people may be
Davies C, H arrison M, Roberts G 2008 Gu id eline for the Use of
given responsibility by a court order or by being appointed General Anaesthesia (GA) in Paed iatric Dentistry. Lond on:
guardian upon the death o the parents. Royal College of Su rgeons of England . Available at: http :/ /
Social services social care may have, or share, parental w w w.rcseng.ac.u k/ fd s/ p u blications-clinical-gu id elines/
clinical_gu id elines/ d ocu m ents/ Gu id eline%20for%20the%
responsibility i a child is under a care order or is a ward o
20u se%20of%20GA%20in%20Paed iatric%20Dentistry%
court. (Reproduced with kind permission o Brad ord District 20May%202008%20Final.p d f.
Care NHS Trust.)
H osey MT 2002 Managing anxiou s child ren: the u se of conscious
sed ation in p aed iatric d entistry. UK N ational Clinical Gu id eline.
■ Ca n an adolescent consent or themselves? Int J Paed iatr Dent 12:359–372.
Intercollegiate Ad visory Com m ittee for Sed ation in Dentistry
Ad olescents w ant to be in charge of their d estiny and feel
2015 Stand ard s for Consciou s Sed ation in the Provision of
that they are com p etent to m ake their ow n d ecisions. Dental Care. Lond on: Royal College of Surgeons of England .
Legally, w ritten consent for treatm ent, su ch as sed ation, Available at: http s:/ / w w w.rcseng.ac.u k/ fd s/ Docu m ents/
d oes still requ ire w ritten p arental agreem ent, bu t the d iscu s- d ental-sed ation-rep ort-2015-w eb-v2.p d f.
sions that lead u p to this shou ld inclu d e the ad olescent’s Marshm an Z, Baker S, Cresw ell C et al 2016 Cognitive behaviou r
w ishes and incorp orate their assent. therap y for d ental anxiety. Resou rces are available for the d ental
• 156
26 THEUNCOOPERATIVECHILDANDADOLESCENT
team , child ren and their p arents. Available at: http:/ / d ental.llttf England , Lond on. Available at: http s:/ / w w w.gov.u k/
.com . governm ent/ u p load s/ system / u p load s/ attachm ent_d ata/
Med ical Protection Society 2014 Consent – child ren and you ng le/ 367563/ DBOH v32014OCTMainDocu m ent_3.p d f.
p eop le. Available at: http :/ / w w w.m ed icalprotection.org. Wong H M, H um p hris GM, Lee GTR 1998 Prelim inary valid ation
N u nn J, Foster M, Master S et al 2008 Consent and the and reliability of the m od i ed child d ental anxiety scale.
Use of Physical Intervention in the Dental Care of Child ren. Psychol Rep 83:1179–1186.
Lond on: Royal College of Su rgeons of England . Available at:
https:/ / w w w.rcseng.ac.uk/ fd s/ p ublications-clinical-gu id elines/
clinical_gu id elines/ d ocu m ents/ p aed _d ent_intervention.p d f.
Public H ealth England 2014 Delivering Better Oral H ealth: An
For revision, see Mind Map 26,
Evid ence-Based Toolkit for Prevention, third ed . Pu blic H ealth pages 246 247.
27
d iagnosis w as nally m ad e. Sanjeev’s m other has only
recently retu rned to p art-tim e w ork, now that he seem s
m ore settled in school.
■ What challenges ha s Sanjeev’s a mily aced?
The fam ily m ay have faced genuine d if cu lties relating to
access to m ed ical and d ental care. A key focu s for San-
jeev’s parents w as to get a thorou gh assessm ent of his
m ed ical cond ition. They also had to w ork w ith his school

Children with disabilities and w ith social services to have his ed u cational and
social need s assessed . As a resu lt of these assessm ents,
Sanjeev now has better access to health care, social and

and learning di culties ed ucational sup port, and his p arents have been in touch
w ith other parents w ho have child ren w ith Asperger ’s
synd rom e.
Sanjeev has tw o healthy brothers w ho also need su p p ort
and attention in their ow n right. Sanjeev’s m other gave
u p her p art-tim e job to look after him . Although she is
p leased that she has achieved stability, su pport and
m ed ical care for her child , she is w orried abou t his teeth.
SUMMARY She has previously p rioritized the m any ed u cational and
Sa njeev is an 8-year-old boy with Asperger’s school assessm ents, w hich led to Sanjeev’s irregu lar d ental
syndrome. You receive a letter rom a community attend ance.
dentist who has undertaken a routine school
inspection. The letter reports that Sanjeev has
caries in his f rst and second primary molars. On
entering the clinic, Sa njeev’s mother gives the
receptionist a communication passport (http://
www.autism.org.uk/living-with-autism/out-and
-about/my-hospital-passport.aspx). She asks the
dentist to read the passport be ore seeing her son
(Fig. 27.1).
■ How will you ma nage him?

History
Complaint
Sanjeev’s m other reports that she d oes not think
he is in p ain. H e has been eating and sleep ing normally.
Sanjeev says, bu t only w hen he is p rom p ted , that he has
occasional p ain w hen he eats sw eets.

History o complaint
There is no relevant history.

Medical history
Sanjeev is t and w ell. H e w as d iagnosed w ith Asp erg-
er ’s synd rom e last year. H e d oes not take any m ed ica-
tion. Asp erger ’s synd rom e is a typ e of high-fu nctioning
au tism .

Social history
Sanjeev is in a m ainstream school and receives ad d itional Fig. 27.1 Sanjeev’s communication passport where he describes
su p p ort d u ring his lessons from a teaching assistant. how best to manage his dental visit. Passport originated by
H is m other rep orts that she had to ght very hard for National Autism Society. See http://www.autism.org.uk/about/
this su pport and w ent through nu m erou s ed ucation health/hospital-passport.aspx.
assessments – esp ecially before the Asp erger ’s synd rom e Reproduced with permission.
• 158
27 CHILDRENWITHDISABILITIESANDLEARNINGDIFFICULTIES
and level of tru st. Parents w ill be grateful for ap pointm ents
Key point and treatm ents that m inim ize the am ou nt of tim e out of
Key eatures o Autism, Asperger’s syndrome and Attention school and aw ay from w ork.
Def cit Hyperactivity Disorder (ADHD):
• Please see http://www.autism.org.uk or urther Examination
in ormation.
Autism is a spectrum o disorders with children ■ How will you examine Sanjeev?
showing varying levels o severity in three key categories: Sanjeev has provid ed consid erable help and guid ance on
• Communication children show delayed language how best he can cope w ith the d ental exam ination. Follow -
development and limited comprehension o non-verbal ing his ad vice w ill only help w ith the appointm ent. Many
communication. Severely a ected children may have child ren w ith d isabilities and learning d if culties can read ily
rudimentary or no speech. accep t a d ental examination. H ow ever, some can have more
• Social interaction children o ten live in their own challenging behaviour and even a d ental examination can be
world and have limited interest and interaction with d if cult to perform. Some of these d if culties can be over-
other children and adults. come by a sequence of short introd uctory visits, each one set
• Social imagination children may struggle to interpret up in exactly the same w ay as the previou s but just build ing
other people’s eelings and actions. on the examination; this can w ork w ell w ith a child w ith
Asperger’s syndrome, as already described, is at the autistic spectrum d isord er. Other d istractions arou nd the
high- unctioning end o the autistic spectrum. It is surgery should be minimized , e.g. noise or interruption.
characterized by normal spoken language development, Keep ing instru ction sim p le and d irect help ed Sanjeev
and the child can have average or above average sit in the d ental chair and allow ed for a qu ick d ental
intelligence. He or she o ten does not have learning delay exam ination.
but can have specif c learning di culties. There are three
key eatures o the diagnosis:
Extraoral
• Love o routines. There w ere no relevant nd ings.
• Intense interests.
Intraoral
• Sensory impairment which can be either under or over
Caries w as id enti ed in the occlusal surfaces of all fou r
unctioning, or example taste, sound, sensation,
second p rim ary molars. There w ere d istal cavities on both
hearing or sight.
m and ibu lar rst p rim ary m olars.
ADHD is a diagnosis made be ore the age o 7 with
children demonstrating a triad o symptoms, which can be
treated with medication or therapeutic interventions.
Radiographic investigation
• Inattention. ■ What is the best radiographic investigation?
• Impulsivity. With carefu l and sim p le explanation, Sanjeev tolerated
• Hyperactivity. bitew ing rad iographs.
Although many children are diagnosed with high- If this was not possible, a ‘teeth-only dental panoramic
unctioning autism and ADHD, they are two separate tomogram (DPT)’ or a standard DPT (depending on what
conditions. settings are available) is an alternative. A trial run to familiar-
ize Sanjeev with the equipment and what is expected is a very
helpful halfway step. This w ill increase the likelihood of
Sanjeev staying still for the DPT. Although the radiation expo-
■ Wha t ca n dentists do to a ssist the a mily and help them sure is higher for a DPT, w here a radiograph of the carious
overcome ba rriers o access to dental care? primary teeth w ill in uence the treatment provided , this DPT
Managing child ren w ith d isabilities and learning d if cu lties radiograph is still justi ed if bitewings are not possible.
can som etim es seem to be a challenge, bu t it is alw ays The bitew ings show ed that the:
rew ard ing. For m any of these fam ilies, being able to access • First permanent molars have no caries.
d ental care in a p rim ary setting is a w elcom e resp ite from • Occlusal caries in the second primary molars was into
the other specialist services that they attend . Many d entists the inner hal o the dentine.
w ho begin to treat a child w ith a d isability often continu e • Caries in the mandibular rst primary molars appears
to d eliver care for the w hole fam ily and provid e a continu ed close to the pulp with the roots almost completely
source of su pport. For m any child ren w ith Asp erger ’s syn- resorbed by the permanent successors.
d rom e, rou tine is very im p ortant and being seen by the
■ What is Sa njeev’s dental dia gnosis?
sam e d entist in the sam e su rgery help s w ith fam iliarization
and their cooperation. Sanjeev appears to have reversible pulpitis rom his
From a d ental p oint of view, it is not u ncom m on to have carious and restorable second primary molars.
a ‘w arrior-m u m ’ w ho has becom e cond itioned to ght for The mandibular rst primary molars may also be causing
her child ’s care and so can ap p ear overly d em and ing or him some annoyance, especially as they become more
imp atient. H ow ever, being aw are of these other challenges mobile. In common with other children his age,
that the fam ily is facing can help d evelop a strong af nity he may be xated by his ex oliating teeth.
159 •
CHILDRENWITHDISABILITIESANDLEARNINGDIFFICULTIES 27
Guid ance for the u se of H all crow ns (see Chapter 24) w ou ld
Key point ad vocate their u se at an earlier stage of the caries process,
• Parents o a child with a learning disability will have before sym ptom s d evelop. H ow ever, in som e situ ations,
many competing demands on their time. and after carefu l d iscu ssion w ith the parents, treatment m ay
• Routine is key and there ore it is important that the be u nd ertaken if there is a clear und erstand ing of the poor
child sees the same sta , in the same surgery to p rognosis and the need for one or m ore teeth to be extracted
develop amiliarity. if fu rther signs and sym p tom s show irreversible p u lp itis or
• Keep instructions simple and direct. ap ical p eriod ontitis (e.g. non-vital).
For som e child ren w ith d isabilities, especially w here
• Children may not always sit in the dental chair and
there are signs of bruxism and tooth w ear, great thou ght
there ore take whatever opportunity arises to examine
shou ld be taken before ap plying H all crow ns. The p ropp ing
their teeth (e.g. sitting on their parents’ lap, in the
op en of the occlusion m ay exacerbate their grind ing and
waiting room, on the surgery oor).
increase the risk of a p erforated crow n.
• Clear communication with a parent is essential to
identi y how much support and/or restraint they are ■ When will these prima ry teeth exo oliate?
happy to provide to permit a dental examination. Any This is an im portant consid eration w hen treatm ent p lan-
restraint must be proportionate with the history and ning. For Sanjeev, the m and ibu lar prim ary rst m olars are
the need or a dental examination. likely to exfoliate very shortly and therefore, u nless sym p-
tomatic, can be left to exfoliate. For the second prim ary
m olars, these teeth exfoliate arou nd the age of 12. Therefore
any restorative plan w ill requ ire m aterials w ith good evi-
■ What are the trea tment options or Sanjeev? d ence of longevity over this 4-year tim e p eriod . Such m ate-
The m anagem ent of the d ental d isease is the sam e as rials inclu d e com posite, am algam or stainless steel crow ns.
d escribed in Chap ter 24. bu t it is the m ethod of d elivery The longer the restoration need s to survive the m ore likely
than can be d ifferent. Prevention is the key. The DoH Preven- a stainless steel crow n w ill be the restorative op tion as these
tion Toolkit is a valu able sou rce of gu id ance. Every visit have the greatest longevity.
presents an op p ortu nity to p revent caries: w hether this is ■ Does beha viour management work with children with
ad vice to red u ce sugar frequ ency, to u se 1450 p p m u orid e Asperger’s syndrome?
toothp aste or to ap ply top ical u orid e. There are a nu m ber
N on-p harm acological behaviou r m anagem ent can w ork
of w ays p arents can be su p p orted to ensu re they are u nd er-
w ell in child ren w ith Asp erger ’s synd rom e. H ow ever, this
taking toothbru shing for their child . This includ es m od i ed
m ay need to be au gm ented w ith voice control to gain San-
toothbru shes (three-sid ed toothbru sh), nger-gu ard s or
jeev’s attention if he becomes d istracted , so p ositive rein-
u sing a tw o toothbru sh techniqu e. The second toothbru sh
forcement, tell-show -d o and enabling a hand signal to ‘stop’
can be u sed to keep the teeth ap art, thus p erm itting the
should all be u sed . Giving sim ple inform ation abou t w hat
other to bru sh the occlu sal su rfaces of the teeth.
w ill happ en, w hat it w ill feel like and how to cope before-
■ Ca n Sa njeev be managed in a primary ca re setting? hand and d u ring the proced u re are all im p ortant. The lan-
The answ er to this qu estion d ep end s on the d e nitive treat- gu age u sed should be ap propriate for his level of
m ent p lan and the p atient’s level of coop eration. Referral to u nd erstand ing and experience. A constant sim ple and rep -
a specialist (com m u nity) or hosp ital-based paed iatric d en- etitious d ialogu e need s to be maintained , otherw ise Sanjeev
tistry u nit m ay be requ ired , esp ecially if general anaesthesia w ill get d istracted .
is need ed .
■ What are the dental management options?
Key point
Sanjeev m ight m anage to have the carious second prim ary
• Behaviour management techniques should be used.
m olars restored w ith com p lete caries rem oval, bu t he w ill
• A constant dialogue should be maintained to keep the
need to accep t local analgesia ow ing to the d ep th of caries
child engaged and ocused on the dentist.
in all fou r of these teeth and their sym p tomatic history.
Conventional caries rem oval is likely to take at least a • Voice control may sometimes be required to ocus the
fu rther ve visits. Therefore, in the rst instance, stabilizing child’s attention.
the caries using glass ionom er cem ent (GIC) tem p orary res-
torations is the best op tion. It also p rovid es Sanjeev w ith
fu rther acclim ation to the d ental su rgery and gives an ind i-
cation of how he w ill cop e w ith m ore invasive treatm ent. In ■ Does nitrous oxide inhalation sedation work or children
these earlier visits, ad d ressing the causes of caries is essen- with Asperger’s syndrome?
tial as d escribed in Chap ters 22 and 23. Many au tistic chil-
d ren can stru ggle w ith som e of the sensations of d entistry N itrou s oxid e inhalation sed ation is the m ost com m only
su ch as the tastes, sm ells, noises and textu res. u sed sed ative in child ren. The key is to carry out a thorou gh
assessm ent based on a frank d iscu ssion w ith Sanjeev and
■ Would Hall crowns be an a ppropriate treatment option?
his m other. The su ccess of the technique is d epend ent on
The extent of the caries and the history of p ain w ou ld red u ce Sanjeev’s ability and w illingness to breathe in and ou t
the chances that the H all ap p roach w ou ld be effective. through his nose. Many child ren w ith Asp erger ’s synd rom e
• 160
27 CHILDRENWITHDISABILITIESANDLEARNINGDIFFICULTIES
enjoy the active engagem ent of this activity. It is also w ise a non-vital central incisor and a general anaesthetic m ight
to assess progress after the rst or second visit and to evalu - be requ ired to m anage the trau m atized tooth. While asleep,
ate w hether Sanjeev w ill m anage to accep t local analgesia. any other d ental care w ould also be carried ou t, su ch as
■ Does Sa njeev need a general anaesthetic? ssu re sealants, extractions or restorations.
■ Are there more complica tions to genera l anaesthesia or
This d ecision is a balance as there are a nu m ber of d isad -
children with a disa bility?
vantages of this ap p roach. The w hole general anaesthetic
event can be trau m atic for the child , and research has Many child ren w ith a d isability or learning d if cu lty are
show n that child ren are m ore d entally anxious afterw ard s. otherw ise p hysically healthy and can u nd ergo d aycase
H ow ever, for som e parents the ability to treat the d ental general anaesthesia w ithou t fu rther concern. N evertheless,
caries ef ciently (e.g. all d ental care is p rovid ed in one go) the w hole general anaesthesia event can be anxiety provok-
and their child ’s lim ited ability to rep ort any p ain and su f- ing, and so p rep aring the child and fam ily can help red uce
fering can m ake a general anaesthetic an attractive option. the psychological anxiety that su rrou nd s the event. Psycho-
Other consid erations w hich in uence the d ecision includ e logical su p p ort inform ation m ay help ; one su ch online and
the likelihood of the child coop erating d u ring treatm ent, the freely available p ackage that has increased fam ily satisfac-
pain history and the w aiting tim e for this treatm ent. tion can be fou nd at w w w.scottga.org.
Sanjeev d oes not need u rgent treatm ent, and he is not in Som e m ay not have the cop ing skills to coop erate w ith
severe p ain. Therefore a general anaesthetic is not cu rrently the anaesthetic ind uction and m ay need pre-m ed ication
ju sti ed esp ecially as in Sanjeev’s case there is tim e to stab- beforehand . The com m onest pre-m ed ication is m id azolam
lise his caries and see how he and his sym ptom s resp ond to 0.5 m g/ kg given app roxim ately 30 m inutes before the
this initial treatm ent. general anaesthetic ind uction.
■ Wha t treatment plan would you propose? ■ What medical comorbidities can children with a disability
commonly present with?
Using the principles d iscussed in Chap ter 24, a treatm ent
plan should be d raw n u p together w ith a visit-by-visit p lan. Apart from his Asperger ’s synd rom e, Sanjeev is otherw ise
As for all child ren, it is essential to ensu re a rigorou s p reven- healthy. H ow ever, som etim es d isabilities are linked w ith a
tive plan. Sanjeev may alw ays need extra su p p ort w hen m ed ical com orbid ity. An exam p le of this is a child w ith
attend ing the d entist and therefore keep ing him caries free Dow n synd rom e, w ho m ight not only have a card iac d efect
is a key p riority. bu t also be p rone to acu te lym phoblastic leu kaem ia, espe-
By follow ing Sanjeev’s instru ctions, he cop ed w ith the cially if the Philad elp hia chrom osom e is carried together
temporary restorations and both his m um and he engaged w ith trisom y 21.
in the p reventive regim e (d iscu ssed in Chap ters 22 and 23). The m anagem ent of these child ren w ith com p lex cond i-
Sanjeev’s cooperation w as good bu t limited . H ow ever, he tions need s liaison w ith paed iatricians and is best referred
respond ed w ell to nitrou s oxid e sed ation. Conventional to a hospital p aed iatric d entistry team .
caries rem oval w as u nd ertaken u nd er local anaesthetic and
nitrou s oxid e sed ation. This w as very effective, and Sanjeev
Key point
comp leted treatm ent. While the m and ibu lar second p rim ary
m olars w ere anaesthetized , the alm ost exfoliated m and ibu- • Learning disabilities can be linked to epilepsy, cardiac
lar p rim ary m olars w ere extracted . de ects and other medical problems.
• There needs to be liaison with the child’s paediatrician.
Common disabilities • Re erral to a community or hospital based paediatric
■ Wha t are the common disabilities? dentistry specialist team may be needed, especially i
general anaesthesia treatment is required and their
There are m any other physical d isabilities and learning d if- medical comorbidity will complicate the general
culties. These can be grou p ed into sensory, p hysical, anaesthesia.
m ental and m ed ical d isabilities. The com m onest of these are
show n in Table 27.1. Som etim es these can be intertw ined .
The im p act of m ed ical cond itions on d ental care is d iscu ssed ■ How does a dentist communicate with children with
in Chap ter 28. a disa bility?
■ How would this plan be altered i Sa njeev’s disability had
Com m u nication w ith child ren w ith com p lex need s inclu d es
been more severe?
a mixtu re of m ethod s and technologies that can sup plem ent
Sanjeev coped w ell w ith a d ental p lan that w as based on the spoken and w ritten w ord and gestu re. For som e, ges-
prevention, stabilization and then d e nitive care. As su ch, tu res, facial exp ressions, bod y p ostu res, eye gaze and m ime
it d id not challenge him too m u ch and it also incorp orated is need ed . For others, sign langu age, e.g. British sign lan-
very good behaviou ral management. H ow ever, for som e gu age, or other hand signals and pictures and sym bols or
child ren, their p hysical and learning d isability is m ore other m ed ia su ch as a sou nd board m ay be u sed . The best-
severe and their d ental need s m ore u rgent. A typical know n technique is MAKATON , d evelop ed in the 1970s to
exam ple is a child w ith epilep sy and cerebral p alsy w ho is help p eop le w ith learning d isabilities to com m unicate. It
u nable to control his or her m ovem ents. This child m ay have u ses som e British sign langu age signs com bined w ith
had a seizu re and fallen, thu s trau m atizing a tooth. In su ch sp eech, facial expression, eye contact and bod y langu age
a case, the child m ay be u nable to cop e w ith treatm ent for along w ith gestu res and sym bols. N ow ad ays, MAKATON
161 •
CHILDRENWITHDISABILITIESANDLEARNINGDIFFICULTIES 27
Table 27.1 Types o disability
Type Example(s) Dental implications Solutions
Physical Cerebral palsy Bruxism Oral hygiene support
Varying degrees o Reduced vertical dimension Modifed brushes
paralysis/muscular Anterior open bite Teaching carers howto brush
dys unction Drooling 2800 ppm uoride toothpaste or children aged 10 and over
Epilepsy Inabilityto per ormadequate oral hygiene Regular check ups and scaling
Behavioural problems Phenytoin induced gingival hyperplasia Corsodyl gel applied via a toothbrush daily, separate rom uoride toothbrushing
Visual and/or hearing Wheel chair access Reduce sugar requency
impairments Wherever possible consider re erral to interdisciplinaryteam or assessment o
Compromised speech and drooling to avoid surgeryto relocate submandibular salivarygland outlet, as
learning skills this will signifcantlyincrease caries riskespeciallylower anterior teeth
Augmentive and alternative communication skills
Learning Down syndrome Class IIIocclusion Oral hygiene support
Macroglossia Modifed brushes
Hypodontia Teaching carers howto brush
Microdontia 2800 ppm uoride toothpaste or children aged 10 and over
Delayed exoliation o primaryteeth Regular check ups and scaling
Predisposition to periodontal disease Corsodyl gel applied via a toothbrush daily, separate rom uoride toothbrushing
Reduce sugar requency
Medical problems:
Cardiac de ect
Leukaemia
Recurrent chest in ection
Severe developmental delay Di cult to examine or to provide dental care Consider gentle holding to prevent harm– known as‘clinical holding’
Mouth props (thimble)
Towels
Pillows
Sa etybelts
Pharmacological restraint
Pre medication
General anaesthesia
Sensory Hearing Di cultygiving a history Lip reading
disabilities Di culty ollowing instructions while in the dental chair, Do not over emphasize mouth and lips when speaking – it causes di culties
especiallyi the dentist is behind with lip reading
Hearing aids and cochlear implants maydistort sounds and Find out the best means o communication
enhance the volume o dental drills and scalers so much Take time to askthemto showit to you and agree howyou will use it to
that it is pain ul and distressing collaborate together to communicate
Uses other means o communication Give child a hand mirror so that theycan see you
Sign language Warn to switch o hearing aid or turn down volume be ore drilling or scaling
Lip reading Agree on hand signals or gestures to allowcontinued communication while
Other communcation aids or media such asVOCA hearing aids are o
Cochlear implants
Avoid monopolar electrosurgery
Vision Needs toothbrushing instruction that is not based on visual
cues such as disclosing
Bright lights can hurt Use sunglasses and avoid shining light into eyes
No visualization o surroundings or gestures o dentists or sta Give verbal cues
Access challenges Use and allowtouch shake hands/touch hands and shoulder
Stairs Use Braille in li ts and in signage
Guide dog Thinkabout where the guide dog can be during treatment
Directional signage

has been au gm ented by new technologies and m ed ia tressing m e’. Other lea ets and storybooks can help chil-
(for exam p le http :/ / w id git-health.com / easy-read -sheets/ d ren p rep are for their d ental visit and w hat to exp ect.
p d fs/ H ealthy%20teeth.p d f). Speech and language thera-
p ists u se all of these techniqu es and tailor them to each Key Point
child ’s need s so that every child can achieve som e level of
com m u nication. As can be seen from Sanjeev’s case, the • Children with disabilities communicate by gestures,
com m u nication p assp ort can w ork very w ell. The fu ll pass- symbols, signing and multimedia.
p ort contains other help fu l sections su ch as ‘things I can’t • Many have their own ‘communication passport’.
cop e w ith and m ake m e d istressed ’ and ‘how to avoid d is-
• 162
27 CHILDRENWITHDISABILITIESANDLEARNINGDIFFICULTIES

Primary resources and Peninsu la Cerebra Research Unit 2013 Dentistry for child ren and
you ng p eop le w ith learning d isabilities and challenging
recommended reading behaviou r. Available at: http:/ / w w w.p encru .org/ evid ence/
Facu lty of Dental Su rgery 2012 Clinical Guid elines and Integrated d entistry/ .
Care Pathw ays for the Oral H ealth Care of Peop le w ith Pu blic H ealth England 2014 Delivering Better Oral H ealth: An
Learning Disabilities. Lond on: Facu lty of Dental Su rgery, The Evid ence-Based Toolkit for Prevention, 3rd ed n. Lond on: Pu blic
Royal College of Su rgeons of England . H ealth England . Available at: http s:/ / w w w.gov.u k/
Makaton®. Available at: w w w.m akaton.org. governm ent/ u p load s/ system / u p load s/ attachm ent_d ata/
le/ 367563/ DBOH v32014OCTMainDocu m ent_3.p d f.
Mencap . Available at: https:/ / w w w.m encap.org.u k/ cqcrep orts.
Research Au tism . Available at: http :/ / researchautism .net
The N ational Au tistic Society. Available at: http :/ / w w w
.au tism .org.uk. RN IB Know led ge and Research H u b. Available at: http :/ / w w w
.rnib.org.u k/ know led ge-and -research-hu b.
N u nn J, Foster M, Master S et al 2008 Consent and the Use of
Physical Intervention in the Dental Care of Child ren. Lond on: Scott’s H ospital Dental Visit. Available at: w w w.scottga.org.
Royal College of Su rgeons of England . Available at: http s:/ /
w w w.rcseng.ac.u k/ fd s/ p ublications-clinical-gu id elines/
clinical_gu id elines/ d ocu m ents/ p aed _d ent_intervention.p d f. For revision, see Mind Map 27, page 248.
28
Dental history
She had a p reviou s lling in the E w ith no local anaesthetic
abou t a year ago. N o bitew ing rad iograp hs w ere taken.

Examination
Extraorally there is no sw elling and no facial asym m e-
try. There is left-sid ed su bm and ibu lar lym phad opathy.

Common medical Intraorally she is in the m ixed d entition. Intraorally there


is a u ctu ant sw elling associated w ith DE.
■ Why a re you concerned with this dental history?

problems in children At the tim e of the lling, no bitew ing rad iograp hs w ere
taken to id entify the d epth of the caries, the p roxim ity of
the caries to the pu lp and the p ulpal resp onse to the caries.
Moreover, the lack of local anaesthetic w hen u nd ertaking
conventional caries rem oval often resu lts in incom p lete
removal, as the child nd s the d rilling uncom fortable. The

CASE1 temporary cem ent in the m ed iu m term w ill d eteriorate,


lead ing to m icroleakage and caries p rogression.
■ What a re the e ects o chemotherapy on cells with
SUMMARY a ra pid turnover?
Hannah is 9 years old. She was diagnosed with Chemotherapy targets different stages of cell mitosis and is
acute lymphoblastic leukaemia (ALL) 3 months toxic to all differentiating cells. Therefore hair, skin, mucosal
ago. She has toothache, and her medical team has cells lining the gastro-intestinal tract, red blood cells, w hite
asked you to see her. blood cells and platelets will be affected in addition to the
cancer cells. This results in loss of hair, dry and fragile skin,
mucositis, anaemia, leukopenia and thrombocytopenia. Fig.
History 28.2 illustrates how chemotherapy works. Rapidly replicating
H annah has been su ffering from toothache for the last 3 cells (e.g. hair, mucosa, skin, red blood cells, white blood cells
d ays. She rep orts that the pain is com ing from the u pper left and platelets) are able to repair and recover quicker following
qu ad rant and feels her gu m is sw ollen (Fig. 28.1). chemotherapy than the cancerous cells. Each tumour w ill
have a different chemotherapy regimen and timeline. This
Medical history schematic is for a solid tumour with the bone marrow recov-
Over the 6-m onth p eriod p rior to her ALL d iagnosis, ery taking about 3 weeks before the next dose of chemother-
H annah started to feel tired and lethargic. A blood test at apy. Providing care for children on chemotherapy requires
her general m ed ical p ractitioner id enti ed an excessive liaison with their oncologist. Acute dental care is often pro-
num ber of im m atu re and p oorly d ifferentiated lym p hocytes. vided once the bone marrow counts have recovered and
A bone m arrow asp iration con rm ed the d iagnosis of ALL. shortly before their next dose of chemotherapy.
She w as ad m itted to hospital and started on a stand ard ■ What urther in ormation do you need?
chem otherap y regim e. For girls, treatm ent involves several
Dental in ormation A full oral assessm ent is need ed . At
stages of chemotherapy over a 2 year p eriod w ith the aim
d ifferent stages of the chem otherap y cycle, patients w ill
of eliminating the leu kaem ic cells and p reventing their
have very low w hite cell cou nts (in particu lar, low neu -
recu rrence.
trophil counts), w hich w ill lim it their ability to ght infec-
tions. A lack of facial sw elling or lym phad enopathy d oes
not necessarily m ean there is no infection related to a
necrotic tooth, especially if a patient is pyrexic. Therefore

White cell count


Cancerous cell count
Normal levels

Cell number
bone marrow recovery

0 5 21 28 42 47 63 68
Chemo Chemo Chemo Chemo
Day
Fig. 28.1 E with a temporary restoration, D distal caries with Fig. 28.2 A schematic and simple diagram to explain the e ect o
buccal swelling. chemotherapy on bone marrow and cancer cell numbers.
• 164
28 COMMONMEDICALPROBLEMSINCHILDREN
taking H annah’s tem p eratu re is essential. H er tem peratu re w ill u nd ergo a lu m bar p u nctu re and bone m arrow asp ira-
is 38.5°C. tion. These are invariably carried ou t u nd er general anaes-
The oral assessment should include bitewings and a d ental thetic and m ay therefore p rovid e an op p ortu nity to
panoramic tomogram, when the patient can tolerate these und ertake any acu te extractions.
radiographs (Fig. 28.3). Follow ing the clinical and radio-
graphic examination, a complete problem list and treatment
plan (see Chapter 24, Table 24.3) should be drawn up. Any
Treatment
primary teeth with a poor prognosis, including those with ■ What acute treatment does Hannah need?
deep caries, should be extracted rather than restored. For
necrotic and infected permanent teeth, either they should be The presence of a tem peratu re, together w ith the bu ccal
extracted or rst stage end odontics should be und ertaken sw elling associated w ith the E and / or D, need s to be d is-
w here there is con d ence that the infection can be eliminated. cu ssed w ith the m ed ical team im m ed iately. Treatm ent w ill
inclu d e the u se of approp riate antibiotics (for exam ple,
Medical in ormation Close collaboration is required w ith amoxicillin and m etronid azole or sim ilar antibiotics given
the m ed ical team coord inating H annah’s chemotherapy. intravenously or orally d ep end ing on the severity of the
There are only a few short w ind ow s in w hich invasive infection) and painkillers (these are frequ ently paracetam ol
d ental treatm ent can be p rovid ed as a resu lt of the anaem ia, and opioid s, as non-steroid al anti-in am m atories such as
leukopenia and throm bocytopenia associated w ith her ibuprofen are avoid ed ow ing to the low p latelet cou nt and
treatm ent. Therefore it is essential to nd ou t the cu rrent fu ll the risk of bleed ing). H annah w as ad m itted to the oncology
blood cou nt (red cell cou nt, w hite cell cou nt, neu trophil w ard and started on intravenou s antibiotics.
cou nt and p latelet cou nt) and w hether the cou nt w ill At the m ost ap prop riate tim e, in relation to the chem o-
im p rove or get w orse in the im m ed iate fu tu re. Moreover, at therap y regime and fu ll blood cou nt, the DE w ill need to
d ifferent points of ALL chem otherap y regim ens, child ren be extracted . Althou gh sw elling is m ost likely to arise from
the E, for H annah the safest option w ou ld be to extract both
DE to elim inate all potential sou rces of infection. In id enti-
fying the m ost ap p rop riate tim e for this treatm ent, consid -
eration shou ld be given to how long it w ill take the socket
to heal p ost-extraction becau se her cou nts w ill start to d rop
again w ith the next cou rse of chem otherap y.
■ What other denta l care does Hannah require?
H annah requires an intense preventative regim e against any
fu rther oral d isease. For som e child ren u nd ergoing chem o-
therap y, their d iet w ill change signi cantly d u ring treat-
m ent. This is fu rther exacerbated by the sid e effects of
chem otherapy, such as m ucositis and the need to m aintain
ad equ ate calori c oral intake. On occasions w here child ren
A
are failing to eat su f cient calories, they m ay be fed via a
nasogastric tu be, gastrostom y or intravenou sly.
■ What are the oral implica tions o childhood
ca ncer treatment?
Treatm ent for leukaem ia and solid child hood cancers can
includ e chem otherapy, rad iotherap y and su rgery. Boxes
28.1 and 28.2 list the short- and long-term oral com plica-
tions of chem otherapy and rad iotherapy.

Box 28.1 Short and long-termoral side e ects o chemotherapy


Short-term oral side ef ects – chemotherapy
B
At risk o peri- and intra-oral in ections – bacterial, viral and ungal, especiallyas
chemotherapyand associated medication will lead to changes in oral ora
Ulcers and mucositis
Bleeding and gingivitis
Xerostomia and dysgeusia
Dysphasia
Trismus
Phantompain in jaw – o ten associated with vincristine or romleukemic infltration
into the jaw or dental pulp
C Long-term oral side ef ects – chemotherapy
Anomalieso dental development – e.g. aplasia, hypoplasia or hypomineralizedcrowns,
Fig. 28.3 Hannah’s bitewings (A, B) and dental panoramic
diminutive or rudimentaryrootsandor crown, delayed eruption andex oliation
tomogram (C).
165 •
COMMONMEDICALPROBLEMSINCHILDREN 28
Box 28.2 Short and long-term oral side e ects o radiotherapy ■ What is Haemophilia A?
involving the oral cavity H aem op hilia is an X-linked recessive d e ciency of factor
Short-term oral side ef ects – radiotherapy VIII. The d efective gene is p assed d ow n throu gh his m other,
Trismus w ho is a carrier for this cond ition, w hich only affects
Osteomyelitis boys. Factor VIII is an essential part of the clotting cascad e
Sialadenitis
and enables the platelet p lu g to be stabilized w ith brin.
Primary haem ostasis is not affected . The cond ition is classi-
In ections
ed by the p ercentage of factor VIII activity, w ith severe
Erythema classi ed as less than 1%, m od erate 1–5% and m ild less
Pulp pain or hypersensitivity than 5%.
Drymouth, impaired salivary unction
■ Wha t dental treatment does Hugo require?
Long-term oral side ef ects – radiotherapy
A full clinical and rad iograp hic exam ination id enti ed m id -
Increase risk o caries
d entine caries in 6 E. The absence of the 5 5 w ill in u ence
Drymouth, xerostomia
the treatm ent op tions available. Treatm ent p lanning for
Change in taste p oor prognosis rst perm anent m olars is d iscussed in m ore
Anomalies o dental development d etail in Chap ter 34. Follow ing an orthod ontic consu ltation,
At risk o osteoradionecrosis it is felt that restoration of both teeth is the most ap p rop riate
option. Conventional caries rem oval is the treatm ent choice
for both 6 E.
H u go is cooperative and happy to have the treatm ent
p rovid ed u nd er local anaesthetic. The qu antity of treatm ent
and the num ber of visits this w ill take m ay in uence
w hether treatm ent is provid ed u nd er local or general anaes-
thetic. For E the m ost ap p rop riate restoration w ou ld be a
stainless steel crow n, ow ing to its enhanced longevity in
comparison to other restoration m aterials. Long-term ou t-
comes for the H all techniqu e are only available for 5 years.
Realistically the E m ay su rvive for 15–30 years, and there-
fore, w hen the child is cooperative, conventional caries
w ould be the m ost ap propriate op tion. For the 6 a single
su rface restoration w ith either an am algam or com p osite
w ith ssure sealants of the rem aining occlu sal surfaces
w ould be the m ost app ropriate option. If a com p osite is
chosen, treatm ent und er ru bber d am is essential to optim ize
m oistu re control and hence longevity.
Fig. 28.4 caries. As im p ortant as the restorative p lan is the p reventative
6E p lan to m inim ize fu rther d ental caries. H ugo w ill requ ire
enhanced p revention as d iscu ssed in Chap ters 22 and 23.
■ Wha t are the ora l implica tions o Haemophilia A on Hugo’s

CASE2 restorative treatment?


Carefu l liaison w ith the haem atology team is requ ired to
SUMMARY d iscu ss the treatm ent need ed and a p lan to system ically
m inim ize any bleed ing. For H ugo, the follow ing m ed ical
Hugo is 8 years old and presents or a routine plan is d raw n up : Firstly, transexam ic acid m outhw ash is
check-up. He has Haemophilia A. Clinical and prescribed at least one d ose prior to his treatm ent and then
radiographic examination reveal mid-dentine continu ed three tim es a d ay for 3–7 d ays d ep end ing on the
caries in E and 6 (Fig. 28.4). A dental panoramic d ental treatm ent requ ired . H ugo need s to gargle the m ou th-
w ash and then sw allow it to optim ize its effectiveness.
tomogram identif es both mandibular second
Transexam ic acid is an anti brinolytic and help s p revent
permanent premolars to be missing (5 5). clot breakd ow n. Second ly, he is ad vised to take his factor
VIII on the m orning of his d ental treatm ent and to take
History a further d ose the next m orning if there are any signs of
post-op erative bleed ing from the gingivae or su bm ucosal
H u go com p lains of no p ain or p roblem s from his teeth.
haem atom a. If not, he should continu e w ith his regular
alternate d ay p attern for taking factor VIII.
Medical history
■ Wha t local mea sures ca n you take to minimize bleeding?
H u go has severe H aemop hili A and is u nd er the care of the
p aed iatric haem atologist. H e requ ires alternate d ay injec- There are a num ber of su rgical principles to m inim ize bleed -
tions of factor VIII, w hich are ad m inistered at hom e by his ing. These inclu d e taking great care w hen m anipu lating the
m other. soft tissues, for exam ple, w hen p lacing ru bber d am clam ps
• 166
28 COMMONMEDICALPROBLEMSINCHILDREN
and m atrix band and w hen u sing high-volu me suction. vegetation to become established , ow ing to im p aired blood
Where teeth have been extracted , the u se of su tu res, Su rgicel ow in the heart. These colonies w ill then cau se p yrexia,
and pressu re is com m only u sed . On rarer occasions bone new or changing heart m u rm urs and em bolic events. Infec-
w ax, Coe-Pak d ressings and bip olar d iatherm y m ay be tive end ocard itis carries w ith it signi cant m orbid ity and
need ed d ep end ing on the oral su rgery being u nd ertaken. m ortality, and consequ ently its p revention is a high p riority
Care shou ld be taken to carefu lly exp lain the p ost- for at risk child ren and ad u lts. Bacteraem ias from the oral
operative instru ctions to p revent d istu rbance or loss of the cavity are com m on follow ing d ental extractions or other
blood clot, for exam p le, avoid ing p oking the clot, eating treatm ents involving gingival m anipu lation. H ow ever,
hard food s, d rinking hot d rinks or exercise. N on-steroid al bacteraem ia is also seen w hen child ren bru sh their teeth
anti-in am m atory p ainkillers shou ld be avoid ed in all or chew, esp ecially if there is m arked gingivitis and poor
patients w ith a bleed ing d iathesis. oral hygiene. Consequ ently, good oral hygiene is a key pre-
■ Inf ltra tions versus in erior alveolar block injection ventative behaviou r in m inim izing the risk of infective
end ocard itis.
Althou gh the factor VIII prophylaxis w ill cover inferior
alveolar block injection, w hen in ltration injections w ill ■ Is antibiotic prophylaxis required or denta l procedures?
achieve sim ilar p ain control, they shou ld be u sed instead .
In the UK, N ational Institu te of Clinical Excellence (N ICE)
Therefore, for both the E and 6 in ltration injections, u sing
guid elines ad vocate that no antibiotic cover is requ ired for
Articaine w ou ld be the m ost ap p ropriate op tion. The ad ren-
d ental proced u res. This is d ifferent in other countries, su ch
aline contained w ithin both Articaine and Lignocaine w ill
as Au stralia and the USA, w here antibiotics are still ad vo-
again help achieve haem ostasis.
cated for d ental p roced ures that cau se bacteria. The logic
behind the N ICE gu id ances w as that since no antibiotic
CASE3 cover w as ad vised for the frequ ent oral bacteraem ia associ-
ated w ith everyd ay behaviours (such as chew ing and brush-
ing) it therefore d id not m ake sense for antibiotics to be u sed
SUMMARY to cover infrequ ent d ental proced u res. H ow ever, a recent
Harriet is 3 years old. Her mum is concerned with article has id enti ed an increase in infective end ocard itis
the appearance o the E . She reports that the tooth since this gu id ance w as p u blished w ith the consequ ent fall
erupted with brown staining and that it appears to in prescribing of antibiotic prop hylaxis in the UK. In light
of these nd ings, N ICE has review ed their gu id elines bu t
be getting worse (Fig. 28.5).
has conclud ed that this evid ence is not suf cient to change
their ad vice.
History
■ What treatment is appropriate or Harriet?
H arriet sometim es com p lains of p ain from the E w hen
eating ice cream . Preventive plan It is essential that a robust p reventive p lan
be d raw n u p for H arriet. Ow ing to her card iac cond ition,
Medical history H arriet shou ld be p laced on an enhanced p reventative plan,
as the consequ ences of d ental caries are potentially signi -
H arriet has had a su rgical correction for a tetralogy of Fallot
cant. Furtherm ore, very careful attention should be paid to
at the age of 4 months. The heart cond ition w as d iagnosed
her oral hygiene. H er parents shou ld be encou raged to
in pregnancy at the 20-w eek scan. H er rst m onths of life
actively u nd ertake her tooth bru shing and ensu re that both
w ere sp ent in and ou t of hosp ital p rior to her card iac su rgery.
the teeth and gingivae are bru shed to m inim ize gingivitis
Cu rrently there are no p lans for fu rther su rgery, althou gh
and hence bacteraem ias of oral origin.
she is still seen on a regu lar basis by the card iologists.
■ What are the oral implications o Harriet’s ca rdiac history? Restorative care A carefu l clinical exam ination and bitew -
ings are essential. The exam ination id enti es that the E is
H arriet’s congenital heart d efect p u ts her at an increased hypom ineralized w ith p ost-eruptive breakd ow n and is sen-
risk of infective end ocard itis. Infective end ocard itis is a bac- sitive to cold air. There are no signs or sym ptom s that the
terial infection in w hich there is potential for bacterial E is non-vital and infected . The rest of the d entition is caries
free. Bitew ing rad iographs show no rad iograp hic signs of a
non-vital E . In d iscu ssion w ith H arriet’s p arents, they feel
she w ou ld stru ggle to coop erate for a conventional stainless
steel crow n (e.g. com p lete caries and hyp om ineralized
enamel rem oval) but m ay cope w ith a stainless steel crow n
placed u sing the H all crow n techniqu e (e.g. no caries or
hypom ineralized enam el rem oved ). Therefore a d if cult
choice exists becau se a conventional stainless steel crow n
w ou ld requ ire a general anaesthetic and w ou ld involve
risks of m orbid ity and m ortality. A carefu l d iscu ssion w ith
H arriet’s parents is requ ired to d iscu ss the d ifferent treat-
ment options. Certainly if a H all crow n w as chosen, very
Fig. 28.5 E hypomineralized with post-eruptive breakdown. carefu l follow -up , both clinically and rad iographically, is
167 •
COMMONMEDICALPROBLEMSINCHILDREN 28
Table 28.1 Common medical problems and their impact on the provision o dental care
Type Example(s) Dental implications Solutions
Medical Cardiac – cyanotic Increased risko bleeding Close liaison with cardiologist; treat in a specialist cardiac centre
Complicated and potentiallyhigh riskgeneral anaesthetic
Avoid stress at the dental visit
Some cyanoticpatients are on war arin and there ore at
increased risko bleeding
Bleeding AV– mal ormation – vascular anomalyseen on perioral skin Close liaison with surgeon/radiologist to identi ythe extent o the lesion and involvement o the
or intra mucosa maxilla or mandible – risko pro use bleeding
Inherited bleeding disorder Close liaison with haematologist – identi yseverityo disorder and medical management; local
surgical management as described is essential
Diabetes Discussed in Chapter 41
Transplants Depends on the type o transplant theyhave had, however, all At increased risko bacterial, viral or ungal in ections; some anti rejection drugs associated
will be on immunosuppressants toprevent organ rejection with gingival enlargement (e.g. cyclosporine); liaise with paediatrician i oral in ections
(bacterial, ungal, or viral) are identifed, as these need to be aggressivelymanaged; care ul
vigilance as long termimmunosuppression is associated with development o cancers including
oral cancers
Asthma Identi yseverityo asthma, howit is managed, what triggers Salbutamol and inhaled steroids each have direct e ects on the oral cavity(e.g. reduced salivary
it, what medication theyare takingandlast hospital admission ow, increase risko oral candidiasis)
Ensure salbutamol is available in emergencydrug box
Care with some topical uorides, as theycan veryrarelycause breathlessness
Epilepsy Identi ythe type/s o seizures and their duration, when was Some epilepticmedication can increase the risko gingival overgrowth (phenytoin); risko
the last seizure, howwas it managed and did it require rescue trauma to the dentition during seizure; parents are normallyveryaware and perceptive o an
medication or hospitalisation tostopthe seizure impending seizure; ensure anti epileptic medication has been taken prior to attendance
Cystic fbrosis Multi systemdisorder predominantlyafectinglungs and Studies showthat children with cystic fbrosis have a lower risko developing dental caries;
digestive system; need toidenti ycurrent respiratorystatus, however, i caries develops, treatment is complicated byhigh risk or general anaesthetic; may
have theybeen admitted or intravenous antibiotics toclear have nutrition supplemented bygastrostomy eeding; long termtreatment options mayinclude
chest in ections or have theybeen colonized bypseudomonas lung transplant
Metabolic Multiple diferent disorders afectingdiferent biochemical Acare ul historyis needed to identi ytheir diet restrictions and eeding; or some children they
pathways; treatment o ten involves strict diets that avoid are exclusivelyor predominantlygastrostomy ed; in ections (including systemic in ections o
aminoacids, which cannot be broken down, or medication to oral origin) can lead to decompensation and hospital admission; admission or general
circumvent these pathways anaestheticrequires care ul liaison to identi yhowbest to manage pre operative starving

required to m onitor any signs of infection as a result of the Primary resources and
tooth becom ing non-vital. Avoid ance of non-vital and
infected teeth is essential to m inim ize the increased risk of recommended reading
infective end ocard itis. H arriet’s p arents, follow ing this d is- RCS Guid eline for Oral Managem ent of Oncology Patients
Requ iring Rad iotherap y, Chem otherap y and / or Bone Marrow
cu ssion, op ted for the H all crow n and very carefu l follow -
Transp lantation 2012 Available at: http s:/ / w w w.rcseng.ac.u k/
u p . They felt that if she d id have a general anaesthetic, they fd s/ p u blications-clinical-gu id elines/ clinical_gu id elines/
w ou ld elect to have E extracted . d ocu m ents/ clinical-gu id elines-for-the-oral-m anagem ent-of
Where a general anaesthetic is requ ired for d ental care, chil- -oncology-p atients-requ iring-rad iotherap y-chem otherap y-and
d ren w ith com p lex congenital heart d efects (rep aired or -or-bone-m arrow -transp lantation.
u nrep aired ) or w ith rhythm ic d isord ers (for examp le Wolf Dayer MJ, Jones S, Prend ergast B et al 2015 Incid ence of infective
Parkinson White synd rom e) should be referred to special- end ocard itis in England , 2000–13: a secu lar trend , interru p ted
ized card iac hospitals. tim e-series analysis. Lancet 385 (9974):1219–1228.
Exam p les of other com m on m ed ical p roblem s and how they N ICE Guid eline (CG 64) 2008 Prop hylaxis against Infective
im pact d ental care is provid ed in Table 28.1. End ocard itis: Antim icrobial Prop hylaxis against Infective
End ocard itis in Ad u lts and Child ren Und ergoing Interventional
Proced u res. Available at: https:/ / w w w.nice.org.u k/ gu id ance/
Key point cg64.
AAPD gu id elines 2015 Gu id eline on d ental m anagem ent of
With all medical conditions: p ed iatric p atients receiving chem otherap y, hem atop oietic cell
• Enhanced prevention is essential to prevent or minimize transp lantation, and / or rad iation therap y. Ped iatr Dent 37 (6):
caries 298–306.
• Care ul liaison with the medical team leading the
patient’s care For revision, see Mind Map 28,
page 249.
29
■ What specif c questions would you ask a nd why?
Was there any loss o consciousness? I so how long? I there was,
then this signi es intracranial trauma and the child should be
re erred to an accident and emergency department. The
duration o loss o consciousness can provide an indication to
the severity o the head injury.
When did the accident occur? Delay in seeking help might
arouse suspicions o a non-accidental injury.

The displaced Where did the accident occur? An accident outside raises the
additional problem o potential wound contamination. Any

primaryincisor
child brought up in the UK should be immunized against
tetanus. I a child has not been immunized, seek the advice o
a local general practitioner or accident and emergency
department.
What was the sur ace on which the accident occurred? Newly
constructed playgrounds have to con orm to British Standards
and should be either o an energy-absorbing polymer or bark
chippings. Older playgrounds and normal pathways will have
SUMMARY non-yielding sur aces and are likely to produce greater
James, who is 3 years old, tripped over while damage and potentially greater risk o more underlying
playing outside. He hit his ront teeth on the injuries. Dirt or gravel can contaminate extra- and intra-oral
ground. How do you manage the immediate wounds, and these oreign bodies will need to be care ully
removed to prevent in ection and reduce scarring.
problem, and what do you advise his parents about
potential damage to the permanent teeth? How did the accident occur? This gives an indication o the
orce that produced the injury. The clinician needs to be
highly suspicious o the high-impact injury that looks simple.
History Always suspect a deeper underlying injury until proven
otherwise.
Complaint
Fragments, where are they? Where a tooth or teeth have been
Jam es has been brou ght to you r su rgery straight from hom e
ractured, these ragments need to be identi ed. Although in
by his mother. She says his u p p er front tooth has been
many cases they will have been spat out, there is a risk o
pu shed back (Fig. 29.1).
these being swallowed, inhaled or incorporated into oral so t
tissues (e.g. lacerated lips or tongue). Suspicion o inhalation
History o complaint o ragments is increased i there was any history o loss o
Jam es trip p ed and fell forw ard hitting his teeth on the consciousness, where the accident was not witnessed by an
grou nd . One tooth is ‘p u shed backw ard s’. adult or i there was coughing or choking ollowing the injury.
I in any doubt, re erral to an accident and emergency
Medical history department to investigate this urther is needed and may
Jam es is a healthy boy w ho has had no signi cant child hood necessitate a chest x-ray.
illnesses and w ho is u p to d ate w ith all his vaccinations. What other parts o the body were injured? Frequently other
parts o the body will also be injured such as abrasions to
Dental history hands, elbows or knees. I visible they should be brief y
Jam es has been a regu lar attend er at his d entist since the examined to establish the extent o these injuries. Again,
age of 2 years. H e has had his teeth p olished and has no concerns o more signi cant injury, or example bony
caries. ractures, should be assessed at an accident and emergency
department.

Examination
Extraoral
James is d istressed , bu t there is no obviou s extraoral sw ell-
ing or facial asym m etry.

Intraoral
■ The a ppea rance o the upper anterior teeth is shown in
Fig. 29.1. What ca n you see?
Fig. 29.1 Palatal luxation o A. Palatal d isp lacem ent of A and associated gingival traum a.
169 •
THEDISPLACEDPRIMARYINCISOR 29
■ What specif c signs will you look or in your exa mina tion? an intruded tooth may occur, and close review is necessary. I
re-eruption has not occurred within 4 6 months, an intruded
The mobility o the teeth. Are they a danger to the airway?
primary incisor should be removed to minimize eruptive
The occlusion. Do the injured teeth prevent normal problems in the permanent dentition.
occlusion?
An adult periapical lm used as an anterior occlusal is the
Mobility o a segment o teeth, e.g. the injured teeth move easiest way to obtain a periapical view o the upper anterior
together rather than individually. This indicates a region in a young child. James’s radiograph did not reveal any
dentoalveolar racture. root ractures or dentoalveolar ractures.
■ What question should dentists keep at the back o their Vitality testing o primary teeth is not indicated, as young
minds when examining children? children are o ten unreliable in reporting any sensation elt.

Are the injuries consistent with the history, and i you eel Direction o displacement provides important in ormation on
they are, is this normal behaviour? the likelihood o any damage to the underlying permanent
successor. Displacement o the crown labial indicates the root
Child physical abuse presents with oro acial signs o bruising,
has moved palatally towards the permanent successor and
abrasions and lacerations, burns, bites and ractures in
vice versa.
approximately 65% o cases.
Radiographic assessment with periapical and/or occlusal lms
Dentists should have a copy o their local area Child
is essential: a palatal intrusion o the root toward the
Protection Committee guidelines. This will tell them who they
successor moves away rom the x-ray source and yields an
should contact or advice.
elongated image.
■ What ea tures in the history and examination would lead to A labial intrusion o the root away rom the successor moves
suspicions o child physical abuse? near the x-ray source, yielding a oreshortened image and a
There are 10 item s to consid er. Five are qu estions to ask gap between the apex o the primary incisor and crown o its
you rself, and ve are observations abou t the behaviou r of successor. Extraoral lateral radiographs have been shown to
the child and the p arent(s): have a limited value in showing labial positioning, especially
in cases o intruded lateral incisors or multiple intrusions.
Could the injury have been caused accidentally and i
so, how?
Does the explanation or the injury t the age and the clinical Diagnosis
ndings?
■ Wha t is your diagnosis?
I the explanation o the cause is consistent with the injury,
James has a palatal luxation injury to his upper le t primary
is this itsel within the normally acceptable limits o
behaviour? central incisor.

I there has been delay in seeking advice, are there good


reasons or this? Treatment
Does the story o the accident vary? ■ Wha t are the three key components o the history and
The nature o the relationship between parent and child. examination in prima ry tooth trauma tha t will dictate i
The child’s reactions to other people. active treatment is required?

The child’s reactions to any medical or dental examinations. Pain. Either spontaneous or on eating suggests pathosis.
The general demeanour o the child. Mobility. Is the tooth a danger to the airway?
Any comments made by the child and/or parent that Occlusal inter erence. A luxation injury that has prevented
give concern about the child’s upbringing or li estyle. For normal intercuspal occlusion will prevent normal eating.
example, lack o parental supervision or a history o repeated
■ Wha t treatments are usua lly required or displaced
trauma.
prima ry incisors?
Concussion and subluxation: observation.
Investigations
Lateral luxation: i no occlusal inter erence, the tooth is
■ What investiga tions would you per orm or James? Expla in allowed to reposition spontaneously; i occlusal inter erence,
why or each. extract.

Radiographs are required to visualize the traumatized area and Intrusion: i the apex is displaced toward the labial bone plate,
assess whether there are any root ractures to either then leave or spontaneous repositioning. I no movement
the traumatized or adjacent teeth. In addition, is a within 4 6 months, extract. In addition, i the root has
dentoalveolar racture evident? Are permanent successor per orated the buccal plate o bone, as identi ed by palpating
teeth present? the area, extract. I the apex is displaced into the developing
tooth germ, extract.
In an intrusive injury a child may have been re erred as an
‘avulsed’incisor. It is imperative always to check in these Extrusion: extract or reposition i only a minor extrusion.
circumstances that the tooth is not intruded. Re-eruption o Avulsion: replantation is not indicated.
• 170
29 THEDISPLACEDPRIMARYINCISOR

Key point
Traumatized primary teeth may need to be extracted i :
• Pain interrupts eating or sleeping.
• Excessive mobility causes a danger to the airway.
• There is occlusal inter erence.
• Tooth becomes non-vital and child is uncooperative or
a pulpectomy. I the tooth is le t in situ, urther damage
to the developing permanent successor can result.

■ Wha t ra diogra phs would you take or these displa cement


injuries?
A periap ical and / or occlu sal view is ind icated for all p eri-
od ontal injuries includ ing concu ssion, su blu xation, lateral
lu xation, intru sion, extru sion and avu lsion. Even in avul-
Fig. 29.2 Endodontically treated primary incisor.
sion cases a rad iograp h is ind icated to ensu re the tooth has
not been intru d ed , u nless the p arent attend s w ith the
avulsed tooth. ■ How would you review it?
■ What are you going to tell James’s mother about the risk to Historically: symptoms.
the permanent teeth?
Clinically: colour, sinus, tenderness, swelling, mobility.
The rep orted incid ence of d am age to the d evelop ing p erm a- Radiographically: 6-monthly or 1 year and then when
nent teeth as a resu lt of trau m a to p rim ary incisors ranges clinically indicated.
from 17–64%. An easily rem em bered gu re to tell all parents
at the tim e of the initial p resentation w ou ld be 50%. It is
better to be p essim istic and then be pleasantly su rprised on Key point
eruption of the perm anent teeth rather than the op posite.
The you nger the child at the tim e of inju ry the greater the A ter primary tooth trauma:
risk of d am age to the p erm anent tooth. Other factors • Damage to permanent teeth may occur in 50% o cases.
increasing the risk of d am age inclu d e the typ e of inju ry • Intrusive and avulsion injuries cause most permanent
(w ith avu lsion and intru sion m ost likely to cau se d am age), tooth damage.
the severity of d isplacem ent (greater d isplacem ent lead s to
an increased likelihood ) and p u lp necrosis.
■ Does a discoloured primary incisor alwa ys need trea tment?
■ Wha t a re the possible e ects on the permanent
successor teeth? When there is progressive or persistent (greater than 3
m onths) greying of the inju red tooth, it is highly likely to
White or yellow-brown discoloration o enamel
be non-vital. When there is initial grey d iscoloration that
(hypomineralization).
im proves then vitality is likely to be m aintained . In the
Enamel hypoplasia. absence of infection a d iscolou red p rim ary incisor can
Crown dilaceration. be review ed . In the presence of infection, extraction or
root canal therapy w ith zinc oxid e cement or a calciu m
Crown-root dilaceration.
hyd roxid e-iod oform paste is ind icated (Fig. 29.2).
Root dilaceration.
Odontome ormation. Primary resources and
Partial/complete arrest o root ormation. recommended reading
Sequestration o permanent tooth germ. And reasen JO, Lau rid sen E, Bakland L et al 2014 Dental Trau m a
Gu id e. Available at: http :/ / w w w.d entaltrau m agu id e.org.
Disturbance in eruption.
Day P, Bu ssel R, Clou gh S 2011 Managem ent of d ental trau m a in
■ Can you tell all o these sequelae on a periapical the p rim ary d entition. e-Den. http :/ / w w w.e-lfh.org.u k/
ra diograph? p rogram m es/ d entistry/ .
Malm gren B, And reasen JO, Flores MT et al 2012 International
N o. Only stru ctu ral abnorm alities and abnorm al root Association of Dental Trau m atology gu id elines for the
grow th w ill be visible. White and brow n areas of hypom in- m anagem ent of trau m atic d ental inju ries: 3. Inju ries in the
eralization w ill only be evid ent on eruption of the p erm a- p rim ary d entition. Dent Trau m atol 28 (3):174–182.
nent teeth. UK Com m ittee of Postgrad u ate Dental Deans and Directors 2013
■ I you retain a luxated primary tooth how o ten would you Child p rotection for the d ental team . Available at: http :/ /
w w w.cp d t.org.u k.
review it?
One w eek after the inju ry, 1 m onth, then 3-m onthly. For revision, see Mind Map 29, page 250.
30
Dental history
Shay is a regular attend er at his d entist and has had local
anaesthetic for a restoration.
■ What specif c questions would you a sk and why?
Was there any loss o consciousness? When unconscious,
patients lose their protective ref exes. Moreover, the duration
o loss o consciousness together with the length o post-

The ractured
traumatic amnesia are key indicators or the severity o the
head injury.
Was the ractured piece o tooth located?

immature permanent A history o loss o consciousness together with a missing


tooth ragment is an indication or a chest radiograph to
check that the tooth ragment has not been inhaled.

incisor crown When, where and how did the injury occur? A clear and
detailed description o the injury is essential.
What other injuries were sustained at the time o the injury?
The time rom the injury to presentation may a ect the
treatment options.
Did Shay cope well with his previous experience o local
SUMMARY anaesthetic? The answer to this will dictate what behaviour
management techniques (see Chapter 26) may be required to
Shay is 8 years old. While saving a penalty or his
acilitate treatment.
school team, he collided with the goalpost and
sustained enamel-dentine-pulp and enamel-
dentine ractures to his upper central incisors. How Examination
would you manage the injuries? Outline a
Extraoral
ollow-up treatment plan.
■ Why is the presence o lip swelling together with a mucosal
lacera tion important?
History
This cou ld ind icate that the m issing tooth fragm ent is
Complaint retained in the lip.
The u p p er right and left p erm anent central incisors are frac- ■ How would you demonstrate there was a ragment o tooth
tured (Fig. 30.1). in the lip?
Take soft tissu e rad iograp hy u sing tw o view s at right angles
History o complaint to each other. A sim p le anterop osterior view using a p eriapi-
The inju ry w as su stained d u ring a gam e of soccer. There cal lm p laced behind the lip and in front of the teeth, fol-
w ere no other inju ries. low ed by a lateral soft tissu e view u sing a lateral occlu sal
lm (Fig. 30.2). Unless exp erienced and trained in nd ing
Medical history these fragments, it is sensible to refer to a su rgical colleague
Shay is a healthy boy w ith no history of illness. H e has had to retrieve them .
all his vaccinations, inclu d ing a pre-school booster for
tetanu s.
Key point
Missing tooth ragments could:
• Be within the so t tissues i there is a laceration.
• Have been inhaled i there was loss o consciousness.
• Have been spat out at the site o the accident.
• Have been swallowed.

Intraoral
■ Wha t injuries are visible in Fig. 30.1?
Fig. 30.1 Trauma to the central incisors. Pulp exposure can be There is an enam el-d entine fractu re of 1 and an enam el-
seen a ecting 1 . d entine-p u lp fractu re of 1 of greater than 1 m m in d iam eter.
• 172
30 THEFRACTUREDIMMATUREPERMANENTINCISORCROWN
also approp riate to brie y check the posterior teeth as they
can on rare occasions be inju red as w ell.

Investigations
• Radiographs (previously mentioned) or:
— Foreign body in so t tissues i applicable.
— Root maturity and apical status o teeth.
— Presence or absence o root or alveolar ractures.
• Vitality testing o all upper and lower incisors.

Treatment
■ What is the prime consideration or both the upper
centra l incisors?
Fig. 30.2 Fragments o tooth in lower lip (di erent case). To m aintain a vital p ulp w ithin the root, w hich w ill allow
ongoing p hysiological d entine d eposition. This w ill resu lt
It is essential that p eriod ontal inju ries are also d iagnosed . in fu ll root grow th w ith norm al d entinal w all thickness,
Unless special tests su ch as m obility or tend erness to per- w hich w ill red u ce the chance of late stage crow n root
cu ssion are u nd ertaken, d iagnoses su ch as concu ssion or fractu re.
sublu xation w ill be m issed . Crow n fractu res com bined w ith ■ What is the appropria te immedia te treatment or 1 (tha t
a period ontal injury have p oorer p u lp su rvival ou tcomes ha s an enamel-dentine racture)?
than those w ith no associated period ontal inju ry.
Reattachment o the ragment.
■ Are the roots o 1 and 1 likely to have open or closed apices?
or
Open. Apices are not u su ally closed on u p per p erm anent
central incisors before the age of 11 years. A bonded restoration/‘bandage’, which will produce a
hermetic seal. Glass ionomer cement is not an adequate
■ How would you conf rm apical status?
material or a ‘bandage’and will be lost shortly a ter
Periapical rad iograph. placement resulting in bacterial ingress and thermal damage
■ Wha t other injuries must you exclude on the periapical to the pulp rom hot and cold stimuli. A layer o setting
ra diograph? calcium hydroxide cement should be placed over dentine
where a pulpal shadow is visible prior to placement o an
Root fractu res. adhesive (composite) bandage.
■ What other eatures o the anterior teeth are important at
■ What are the treatment options or 1 (that ha s
examination?
a pulpal exposure)?
Mobility. In a buccopalatal direction. Excessive mobility
• Direct pulp capping.
suggests either a periodontal ligament injury or a root
racture. • Pulpotomy Cvek, coronal or radicular (depth
determined by level o inf ammation).
Colour. This will indicate whether any direct pulpal damage
Direct pu lp capp ing, the placem ent of w ou nd d ressings
causing haemorrhage into the dentinal tubules has occurred.
on an exp osed p u lp, is consid ered less unp red ictable by
Percussion. Tenderness suggests periapical damage and m any au thors. Partial p ulp otom y (Cvek) is the rem oval of
oedema. A dull note may suggest a clinically undiagnosed only the ou ter layer of d am aged and hyp eraem ic tissu e in
vertical crown racture or root racture. the exposed p ulps. This w ill leave healthy p ulp , w hich w ill
Vitality. Following trauma there may be a period o apparent perm it continu ed fu ll root grow th. Partial pu lpotom y is a
loss o vitality on testing with hot and cold stimuli or the highly su ccessfu l techniqu e. Where d am age to the pu lp has
electric pulp tester even in teeth without obvious crown been more extensive (e.g. size of exp osu re or increased
ractures. Nevertheless, the readings serve as a baseline d u ration betw een inju ry and treatm ent), fu rther in am ed
against which subsequent tests can be compared. Teeth, pu lp al tissue w ill have to be rem oved . Where this extend s
which respond to vitality testing at the time o injury, have an to the root canal ori ce, this is called a cervical pulp otom y
excellent chance o maintaining pulp vitality. For teeth that or a rad icu lar p u lp otom y w hen extend ing d ow n the root
do not respond at this time point, the pulp may be still be canal. The im portant asp ect of this treatm ent is to ensu re
vital and respond at subsequent clinic visits. that all in am ed p u lp is rem oved leaving healthy (u nin-
amm ed ) tissue behind .
■ Wha t teeth should be examined a ter trauma a ecting only
■ What a re the indica tions or permanent tooth pulpotomy?
the upper centrals?
No history o spontaneous pain.
All u pper and low er incisors should be includ ed in an
exam ination after any trau m a to the anterior region. It is Acute minor pain that subsides with analgesics.
173 •
THEFRACTUREDIMMATUREPERMANENTINCISORCROWN 30
No discom ort to percussion, no sulcus swelling, no
mobility.
Radiographic examination shows normal periodontal
ligament.
Tissue appears vital.
Bleeding rom the pulp excision site stops within 2 5
minutes.

■ How would you carry out a pulpotomy?


Local analgesia and ru bber d am . Flu sh exp osed pu lp tissue Fig. 30.3 Fragments ound at scene o incident.
initially w ith isotonic saline. Then excise a 2 m m su per cial
layer of exposed p ulp and su rrou nd ing d entine w ith a high
sp eed d iam ond bu r u sing a light tou ch u nd er w ater sp ray
cooling (partial p u lpotom y). Irrigate the su rface of rem ain-
ing p ulp w ith isotonic saline. If bleed ing ceases w ith p res-
su re ap p lied to the p u lp stu m p u sing a d am p (isotonic
saline soaked ) cotton w ool ball for 2–5 m inu tes, apply a
pu lpal m ed icam ent w ith biologically available calciu m
hyd roxid e and seal coronal cavity w ith a bond ed restora-
tion. H ow ever, if the bleed ing d oes not cease, fu rther p ulp
tissue shou ld be rem oved . Ap p ly a cotton w ool ball for a
fu rther 2–5 m inu tes and reassess. If bleed ing has stop ped ,
then cover w ith a m ed icam ent as p reviou sly stated .
H ow ever, if after removing the p u lp to a coronal level (a
coronal p u lp otom y) there is still bleed ing, continu e d eeper
into the root canal w ith care (rad icu lar p u lp otom y). If bleed -
ing p ersists d eep into the root canal, then the rem aining Fig. 30.4 Fragments reattached.
pu lp tissue shou ld be rem oved and root canal treatm ent
instigated (p ulp ectom y). If on accessing the p ulp cham ber
there is no bleed ing or there is an emp ty infected canal, then
a pu lp ectom y is ind icated . The tim e from inju ry to p resenta-
tion, contam ination and the size of p u lp exp osu re w ill all Key point
in u ence the d ep th of in am m ation and necessitate greater Partial or complete pulpotomy:
am ou nts of in am ed p u lp tissu e to be rem oved . • Has a high success rate.
■ How should the crown o 1 be restored? • Size o exposure and length o time between injury and
treatment will in uence the extent o the pulpal
If the crow n fragm ent has been retrieved , then this can be
in ammation that will need to be removed during the
stored in norm al saline w hile the p u lpotom y is com pleted .
pulpotomy procedure.
The fragm ent can then be reattached .
If the crow n fragm ent is not available or the fractu re • Allows ull root growth with a vital radicular pulp. This
extend s signi cantly su bgingivally, a bond ed com p osite res- will reduce the risk o long-term crown root ractures
toration shou ld be p rovid ed . The carefu l u se of retraction which is a common occurrence i an immature tooth
cord or electrosu rgery w ill ensu re good bond ing of the com - was to become non-vital.
posite to the fractu re m argins.
Figs 30.3 and 30.4 show the crow n fragm ents before and
after reattachm ent in Shay’s case. 1 had a p artial p u lp otom y
as d escribed .
■ How should the upper centrals be reviewed a nd how o ten? Primary resources and
De nitive crow n m orp hology shou ld be restored as soon as recommended reading
possible after em ergency treatm ent to re-establish norm al And reasen JO, Lau rid sen E, Bakland L et al 2014 Dental Trau m a
Gu id e. Available at: http :/ / w w w.d entaltrau m agu id e.org.
sagittal relations w ith the low er incisors.
One-, tw o-, and then six-m onthly clinical and rad io- Diangelis AJ, And reasen JO, Ebelesed er KA et al 2012
International Association of Dental Trau m atology gu id elines
graphic exam inations shou ld be carried ou t to check for
for the m anagem ent of trau m atic d ental inju ries: 1. Fractures
continu ed vitality and norm al root grow th. If there is evi-
and lu xations of p erm anent teeth. Dent Trau m atol 28 (1):
d ence of non-vitality, then the im m atu re tooth m u st be extir- 2–12.
pated and non-setting calciu m hyd roxid e u sed to d isinfect Lau rid sen E, H erm ann N V, Gerd s TA et al 2012 Com bination
the root canal. Root end closu re can then be achieved u sing inju ries 1. The risk of p ulp necrosis in p erm anent teeth w ith
Mineral Trioxid e Aggregate p rior to obtu ration w ith w arm concu ssion injuries and concom itant crow n fractures. Dent
gu tta percha. Trau m atol 28 (5):364–370.
• 174
30 THEFRACTUREDIMMATUREPERMANENTINCISORCROWN
Laurid sen E, H erm ann N V, Gerd s TA et al 2012 Com bination the m anagem ent of trau m atic d ental inju ries: 1. Fractu res and
inju ries 2. The risk of p ulp necrosis in perm anent teeth w ith lu xations of perm anent teeth. Dent Trau m atol 28 (1):2–12.
su blu xation inju ries and concom itant crow n fractu res. Dent
Traum atol 28:371–378.
Diangelis AJ, And reasen JO, Ebelesed er KA et al 2012
For revision, see Mind Map 30,
International Association of Dental Trau m atology gu id elines for page 251.
31
Is there any p ain or d iscom fort w hile opening and closing
the jaw ? Absence of sym ptom s should rule out any cond ylar
injury/ fractu re. When the force that p rod u ces an inju ry is
signi cant, it shou ld raise su spicion of a d eeper u nd erlying
bony inju ry.

Examination
The root ractured Extraoral
There is som e sw elling of the up per lip and som e bru ising

permanent incisor and sw elling u nd er the right eye.


■ Wha t questions and exa mination would you complete
rega rding the swelling and bruising under the right eye?
Is there any double vision? Palpate the in raorbital margin or
‘stepping’and then check or altered sensation over the
distribution o the in raorbital (V2) nerve. Check that there is a
SUMMARY ull range o eye movements especially upward gaze.
Fracture o the in raorbital margin and in raorbital f oor could
Andrea is 12 years old. She was trampolining at
lead to entrapment o the in erior oblique extraocular muscle
school when she ell and sustained middle third preventing upward and outward movement o the globe o
root ractures o 1 and 1. How do you manage this the eye.
type o injury, and what do you advise her about Is there any altered sensation on the cheek? Oedema
the long-term prognosis or these teeth? surrounding the in raorbital nerve or entrapment o the nerve
in a racture can result in paraesthesia.
History I there is any doubt about a racture, then posteroanterior
and occipitomental views will detect displacement o the
Complaint in raorbital margin and tomograms will detect orbital f oor
And rea is brou ght to you r surgery by a schoolteacher. She ‘blow-out’ ractures.
is com plaining that her up per perm anent central incisors
are loose and feel ‘fu nny’ w hen she bites together.
Intraoral
History o complaint There is d ow nw ard and palatal d isplacem ent of the crow ns
And rea fell forw ard w hile on the tram poline at school and of 1 and 1, w hich are m obile. Centric occlu sion is not p os-
hit her teeth. H er m other arrives at the su rgery soon after sible because of the slightly palatal position of 1 and 1.
And rea and her teacher. It app ears And rea w as not being ■ What would be the dia gnosis based on the clinical
su p ervised on the tram p oline and the foam p rotection w as f ndings alone?
not in the correct p osition on the m etal fram e of the tram -
p oline. H er m other is not happ y w ith the exp lanation by the It is im portant to d ifferentiate betw een lateral lu xations and
teacher. an extru sion. On exam ination both d iagnoses can have a
similar ap pearance in the p osition the tooth or teeth are
■ What does this alert you to?
d isp laced to. The im p ortant d ifference is that the tooth w ill
The p ossibility of legal action against the school. It is espe- be m obile for an extrusion and im m obile for a lateral lu xa-
cially im p ortant to m ake d raw ings of any external inju ries tion. With a lateral luxation there is fractu re w ithin the bony
on the face and keep accu rate record s of intraoral inju ries socket w ith the tooth locked into position.
and su bsequ ent treatm ent. A p hotographic record w ill be an ■ Wha t tests would you do prior to repositioning the teeth?
ad vantage. A stru ctu red history form (Append ix 6) can help
clinicians ensu re that they u nd ertake a com p rehensive Radiographs. Intraoral periapicals (Fig. 31.1) or an anterior
history and exam ination by p rom p ting them to record d if- occlusal view are needed to diagnose root ractures
ferent p ieces of inform ation. compared with luxation injuries. The upper lateral incisors
should also be checked or injury. Consideration should also
Medical history be given to taking radiographs o the lower incisors, which
may also have received either direct or indirect trauma. Where
And rea is und er regular care w ith her d entist and has had
there is no signi cant displacement o the coronal portion o
local anaesthetic w ithou t p roblem .
a tooth with a root racture, then an anterior occlusal
■ What specif c questions would you ask and why? radiograph will o ten detect root ractures that may not be so
Was there any loss o consciousness? I there was, then evident on periapical views. These radiographs will serve as
Andrea should be re erred to an accident and emergency baseline views be ore repositioning.
department. Vitality tests o all upper and lower incisors.
• 176
31 THEROOTFRACTUREDPERMANENTINCISOR
Cu rrent gu id ance ad vises a exible/ fu nctional splint for all
d isplacem ent inju ries includ ing root and alveolar fractu res.
The only d ifference betw een inju ries is the d u ration of
sp linting p rop osed .

Key point
Splinting in dentoalveolar trauma:
• Flexible 2 weeks or avulsions.
• Flexible 4 weeks or luxations, dentoalveolar ractures
and apical and middle third root ractures.
• Flexible 4 months or coronal third root ractures.

■ Describe step by step your procedure or reduction and


splinting Andrea’s 1 and 1.
1. Give topical and local anaesthesia labially and palatally.
Good and widespread anaesthesia is important to enable
pain- ree manual reposition.
2. While the local anaesthetic is taking e ect, bend your
splint so that it will sit passively on the upper labial
segment o all our incisor teeth. When splinting teeth, i
Fig. 31.1 Middle third root ractures. the splint is not passive, active orthodontic movement
will be seen in the splinted teeth.
■ Wha t is the diagnosis based on the clinical and 3. Gently reposition 1 and 1 and hold these in approximate
ra diogra phic f ndings? position. The repositioned tooth/teeth are unstable and
For both 1 and 1 a d iagnosis of a m id third root fractu re will need to be held in their new position. Although it is
w ith an extru sion of the coronal fragm ent. enticing to use ngers to hold the repositioned teeth, this
makes the rest o the splinting procedure more complex,
as both hands are needed. Some red wax palatally, which
Treatment extends over the incisal edges o 1 and 1, or cotton wool
■ Wha t design o splint would you use or 1 and 1? You have rolls between the upper and lower incisors are appropriate
conf rmed on ra diography tha t they have middle third root methods to hold the repositioned teeth in the correct
ractures (Fig. 31.1). position. Etch the labial sur aces o 2 and 2; wash, dry and
place bonding resin and a spot o composite in the centre
The sp lint shou ld be exible/ fu nctional and d esigned to o the labial sur ace. Place the splint into position on 2
have one sou nd (u ninju red ) abu tm ent tooth on either sid e and 2. Use a bonding brush to mould the composite
of the root-fractu red teeth. already on the teeth over the wire. Add extra composite i
■ How long should the splint be in place in root ra ctures? required. Cure the composite.

For 4 w eeks in apical and m id d le third fractures u ntil the 4. Remove the wax or cotton wool rolls, and with a
m ajority of p eriod ontal ligam ent bres have healed . This nonworking hand, bring 1 and 1 into an accurate
time can be extend ed to 4 m onths in coronal third fractu res. position against the wire splint. Hold them in this
The old regim en of rigid splinting for root fractu res has been position while etching, washing, bonding and adding
show n to have no bene t. Rigid splinting w as m eant to give composite with the working hand. Cure the composite.
the fractu re its best chance of a hard tissu e union. Research 5. Smooth any rough areas with sandpaper discs.
has show n that hard tissu e u nion is m ost likely to occu r if ■ What materials could be used or splinting?
there w as little d isplacem ent at the tim e of the original
injury rather than being a fu nction of the typ e of sp lint An ideal splint should be simple and quick to place and
emp loyed . In other w ord s, the larger the d isp lacem ent at remove, easy to bend to ensure it is passive, biologically com-
the fracture line at the tim e of the injury, the sm aller the patible and relatively cheap. The splint must allow for func-
chance of a hard tissu e u nion betw een the fractu re end s tional movement of the teeth while holding the teeth in their
after red u ction and splinting. new position and allow ing thorough cleaning around the gin-
gival margins. The most common materials used are pieces of
■ Do a ny orms o dentoa lveolar injury need to be
orthod ontic w ire, for example 0.014-in or 0.016-in stainless
rigidly splinted?
steel or twist ex (w hich is three pieces of 0.010-in wire twisted
N o. H istorically, rigid sp linting w as used for d entoalveolar around each other). Each of these wires requires moderate
fractures. A rigid sp lint involved tw o sou nd abu tm ent teeth skills in wire bending to achieve a passive splint. New materi-
on either sid e of the inju ry and a stiffer arch w ire. It w as als such as titanium are now available for splinting, and these
prop osed that this style of sp linting w ou ld ensu re no m ove- have the bene t of being much easier and therefore quicker
m ent betw een bony fragm ents, thereby allow ing healing. to passively ad apt to teeth (Chapter 32, Fig. 32.1B).
177 •
THEROOTFRACTUREDPERMANENTINCISOR 31
■ On removal o the splint how o ten would you Place non-setting calcium hydroxide 1 mm short o racture
review Andrea? line with aim o inducing barrier ormation.

After 1 m onth, 3 m onths, 6 m onths and then yearly. Change non-setting calcium hydroxide 3-monthly until
barrier orms, or alternatively use MTA to generate a 4 5-mm
■ What tests would you complete at each o these reviews?
barrier.
For all trau m atized teeth, a stand ard regimen shou ld be Obturate with gutta percha to barrier.
follow ed . If this is carried ou t in every case, then om issions
are less likely to be m ad e. Annual radiographic review.
• Clinical:
■ What ha ppens to the a pical ragment?
Colour (palatal sur ace best).
In nearly all cases this w ill u nd ergo pu lp canal obliteration
Buccal sulcus sensitivity to digital pressure.
and w ill not need treatm ent. If there is infection of the apical
Sinus or swelling presence.
fragm ent, it m ay requ ire su rgical removal. Root canal
Tenderness to percussion.
therapy across a root fracture is frau ght w ith d if cu lty
Percussion sound. becau se of p roblem s keeping the d istal canal d ry.
Mobility.
■ Is the prognosis good in coronal or gingival third
Sensibility testing.
root ractures?
• Radiography: long cone periapicals.
N o. Decisions need to be m ad e early regard ing options.
■ Is sensibility testing accurate?
Long-term stability and long-term retention of the w hole
N o form of sensibility testing on its ow n is accu rate. Electric tooth in these inju ries are uncom m on. Research show s a
pu lp testing (EPT) is p robably the m ost accu rate, bu t its real survival rate of one-third at 10 years post-traum a, w ith
valu e lies in successive num erical read ings w ith the sam e m any of these teeth being lost as a resu lt of fu rther trau m a
type of EPT. N u m erical valu es can then be comp ared . or excessive m obility. These consequ ences relate to the u nfa-
At each visit, resp onses (both positive and negative) vourable crow n root ratio.
shou ld be com p ared against u ninju red teeth and also if each
■ Wha t are the treatment options in coronal or gingival third
tooth is consistent in its response and the d u ration before
root ractures?
any p ositive sensation is acknow led ged . Cu rrent sensibility
testing relies on sensation of stim u li, w hereas true pu lp al Retain crown temporarily with an endodontic post across the
vitality is m ore appropriately tested by the presence of racture line.
blood ow. In the fu tu re, the w id espread u se of Dop pler in Remove coronal ragment. Root treat apical ragment and
d etecting blood ow in a trau m atized tooth w ill inform all orthodontically extrude prior to post, core and crown
d ecisions regard ing p u lp al necrosis and the necessity for placement.
extirp ation.
Remove coronal ragment. Cover, ‘bury’, the apical ragment
with a mucoperiosteal f ap. The width and height o the
alveolus are thus retained or uture implant placement.
Key point
Prosthetic replacement required.
What types o healing occur in root ractures?
Remove all portions o tooth. Prosthetic replacement
• Hard tissue. required. Future implant placement.
• Connective tissue (see Fig. 31.2). This type o healing is
o ten combined with osseous healing. As the two ■ Ca n root ra ctured teeth maintain vitality?
ragments grow apart over time, bone will f ll in the gap
Yes. The m ajority d o so (literatu re rep orts betw een 72% and
that was originally only f lled with connective tissue.
90% m aintain their vitality). Figs 31.2 and 31.3 show
• Granulation tissue. Technically this is not a type o And rea’s teeth 3 years after the original injury. The teeth
healing but is seen when the coronal ragment is m aintained vitality and u nd erw ent p rogressive p u lp canal
in ected. Once the in ection is treated, healing will take obliteration. In ad d ition, the d istal fragm ents are resorbing.
place by connective tissue. Im portantly, there has been no infection.
■ Ca n root ractured teeth be moved orthodontically?
Yes, bu t w ith cau tion. If there is not a hard tissu e u nion
■ Wha t is the likely radiographic appearance at the racture
betw een the fractu re end s, then the apical p ortion w ill
line i the coronal tooth portion becomes non-vital?
remain static and only the coronal portion w ill m ove.
There w ill be increasing w id ening and translu cency su r- Regu lar rad iograp hic review w ill be necessary d u ring any
round ing and betw een the fractu red end s of the root. orthod ontic treatm ent. Orthod ontic m ovem ent m ay cau se
■ I the coronal portion o an apical or middle third root fu rther root resorption, w hich w hen com bined w ith an
ra ctured tooth beca me non-vita l, how would you root treat alread y com prom ised crow n root ratio can lead to long-
the tooth? term m obility of the coronal fragm ent at the end of ortho-
d ontic treatment. Consequently, great care is need ed w ith
Extirpation to the racture line only. the orthod ontic treatm ent to m inimize the risk of further
Establish working length to racture line. root resorp tion.
• 178
31 THEROOTFRACTUREDPERMANENTINCISOR

Primary resources and


recommended reading
And reasen JO, Ahrensbu rg SS, Tsilingarid is G 2012 Root fractu res:
the in u ence of typ e of healing and location of fractu re on tooth
su rvival rates – an analysis of 492 cases. Dent Trau m atol 28
(5):404–409.
And reasen JO, And reasen FM, Mejare I et al 2004 H ealing of 400
intra-alveolar root fractu res. 1. Effect of pre-inju ry and inju ry
factors su ch as sex, age, stage of root d evelopm ent, fractu re
typ e, location of fractu re and severity of d islocation. Dent
Trau m atol 20 (4):192–202.
And reasen JO, Lau rid sen E, Bakland L et al 2014 Dental Trau m a
Gu id e. Available at: http :/ / w w w.d entaltrau m agu id e.org.
Diangelis AJ, And reasen JO, Ebelesed er KA et al 2012
International Association of Dental Trau m atology gu id elines for
the m anagem ent of trau m atic d ental inju ries: 1. Fractu res and
lu xations of perm anent teeth. Dent Trau m atol 28 (1):2–12.

For revision, see Mind Map 31,


page 252.

Fig. 31.2 Radiographic appearance a ter 3 years.

Fig. 31.3 Clinical appearance a ter 3 years.


32
laceration to the upper lip as well as an avulsion o 1
and subluxation o 1 (Fig. 32.1A)
4. Give labial and palatal local anaesthesia.
5. Irrigate socket with normal saline to remove the
blood clot.
6. Recontour labial plate with f at plastic instrument, i
required.
7. Gently reposition the avulsed tooth.

The avulsed incisor 8. Bend a titanium trauma splint (or alternative splinting
material as discussed in Chapter 31) to include one
uninjured tooth on either side o the avulsed tooth
CC
( ) and ollow guidance described in this chapter or

splint placement. Fig. 32.1B shows the splint in situ.


9. Prescribe chlorhexidine mouthwash. Antibiotics (such as
Amoxicillin or Penicillin V or 5 7 days) can be
prescribed at the clinician’s discretion. Chlorhexidine
0.2% should be used twice daily to enhance oral

CASE1 10.
hygiene.
Arrange a review appointment or 14 days.
SUMMARY ■ What a ctors a re important when deciding whether root
ca nal treatment is necessary in Ka thryn’s case?
Kathryn is 9 years old. She was playing with her
riends at a brownie camp with a skipping rope, Root development.
when the rope caught behind one o her upper How long the tooth was out o the mouth
central incisors and avulsed it. Her teacher has got (extra-alveolar time).
the tooth. How would you manage this problem? A tooth that has com pleted root d evelopm ent w ill not
■ Kathryn’s teacher phones your surgery or advice. She ha s u nd ergo revascu larization. The only tooth that has a chance
the tooth in a handkerchie . The accident occurred 10
minutes ago. Wha t is your a dvice?
Check that Kathryn has no other injuries, i.e. head injuries that
require re erral to an accident and emergency department.
Check or any known medical history.
Hold tooth by crown and wash gently under cold water to
remove any debris or 10 seconds.
Replant in socket ideally.
Hold tooth in socket by biting on a handkerchie . Come to
surgery.
I replantation is not possible, place the tooth in either milk or
normal saline ( rst aid box) and bring it to surgery with
Kathryn as quickly as possible. A

■ The tooth is brought to the surgery in milk. How would


you proceed?
1. Check medical history or any reason not to replant. For
children who are immunosuppressed (e.g. mid-
chemotherapy, organ transplant or primary immune
de ciency) or at risk o in ective endocarditis, a
discussion with their medical consultant would be
prudent be ore replantation.
2. Gently shake the pot containing the tooth and milk to
remove any oreign bodies. Check state o root
development.
3. Undertake a brie history and examination to ensure B
there are no other injuries (see structure history orm
shown in Appendix 6). Examine the patient and check Fig. 32.1 (A) 1 avulsed and 1 subluxed. (B) 1 replanted and
or other injuries. Examination shows a small intraoral splinted with a titanium trauma splint.
• 180
32 THEAVULSEDINCISOR
of revascu larization is the im m atu re tooth w ith an open radiographs as saucer shaped cavities along the mesial and
apex that is replanted w ithin 30–45 m inu tes. All other teeth distal sur aces o the root. Following the avulsion injury and
should be extirpated p rior to sp lint rem oval at 7–10 d ays. extraoral storage there is damage to the periodontal ligament
Different gu id elines argu e for d ifferent lengths of total and and cementum. Following replantation these areas
d ry tim e w here revascu larisation is u nlikely (for exam p le, o damage are removed including areas o adjacent root
British Society of Paed iatric Dentistry (BSPD) – less than 30 sur ace giving these saucer shaped cavities. Adjacent
m inu tes d ry tim e and / or 90 m inu tes total tim e; Interna- cementum and periodontal healing then grows over these
tional Association of Dental Trau m atology (IADT) – less cavities, and the periodontal and cemental architecture is
than 60 m inu tes d ry tim e). The reason for giving the tooth restored.
tim e to revascularize is only reserved for teeth w here if the In ection-related resorption (previously called external
pu lp is extirpated it w ou ld leave a non-vital im m atu re tooth in ammatory resorption): this type o resorption is only seen
w ith increased risk of late stage crow n root fractu re. where there is damage to the periodontal and cemental
H ow ever, if revascu larization d oes not occu r, there is a architecture ( or example ollowing trauma) combined with a
strong chance of infection-related resorp tion, w hich can non-vital and in ected root canal. The immunological
lead to rap id d estru ction of the tooth. Therefore in situ ations response (destructive phase) removes the necrotic and
w here an im m atu re tooth is given a chance to revascu larize, in ected material on the root sur ace, thus opening up the
very carefu l follow -u p is required to ensure any com plica- dentinal tubules. This opens up a pathway through which the
tions are id enti ed and treatm ent instigated as soon as bacteria toxins are transmitted, thus exacerbating the
possible. immunological response and stimulating urther the root
Kathryn w as seen for sp lint rem oval at 14 d ays after the resorption. Radiographically, the sur aces o the roots have
injury. The rad iograph show ed an im m atu re root form . The punched-out craters with radiolucency seen in the adjacent
tooth w as rep lanted w ithin 30 m inu tes, w ith 10 m inu tes bone. Only when the necrotic pulp is extirpated and the root
stored d ry and the rem aind er of the tim e kep t in m ilk. The canal disin ected will this resorption stop. The root sur ace is
d ecision w as therefore taken to see if the tooth w ou ld then able to repair (healing phase) with the type o healing
revascu larize. dictated by the amount o damage. Either the cementum and
■ Wha t actors are important in predicting root resorption? periodontal ligament will re-establish themselves (repair-
related resorption) or the adjacent bone will use to the root
Extra-alveolar dry time (EADT).
(ankylosis-related resorption).
Extra-alveolar time (EAT).
Ankylosis-related resorption (previously called replacement
Storage medium prior to replantation. resorption): this is characterized by the absence o the
Contamination. periodontal ligament with bone used to the cementum and
Root maturity. dentine. The tooth becomes part o the bone and is
constantly remodelled, resulting in progressive resorption o
Research has show n that teeth w ith a d ry tim e of greater the root until the entire root is resorbed and the crown
than 5 m inu tes have a signi cantly increased risk of resorp- ractures o . In growing children, ankylosis-related resorption
tion. The tw o p referred storage m ed ia, m ilk and norm al causes the local cessation o growth with the appearance o
saline, are iso-osmolar, and that is w hy they are the recom - the a ected tooth or teeth staying still (in raoccluding) while
m end ed storage m ed ia. Even in these ap prop riate m ed ia, the adjacent teeth continue to erupt.
period ontal ligam ent vitality is not m aintained for long.

Key point Key point


Critical in ormation to ask about in avulsion cases: Types o root resorption ollowing trauma:
• EADT. • Repair-related.
• EAT. • In ection-related.
• Storage medium and duration in it. • Ankylosis-related.
• Contamination o the root sur ace.

■ What is the treatment or in ection-related resorption


■ What types o resorption are there? ollowing trauma?
Repair-related resorption (previously called sur ace resorption): The only m ed icam ent that is proven to arrest infection-
most commonly seen as blunting o tooth apices a ter related resorp tion is non-setting calcium hyd roxid e. Its anti-
application o excessive orthodontic orces or on the roots o bacterial prop erties relate to its alkalinity (p H 11–12). It
upper lateral incisors as a result o canine tooth impaction. should be placed w ithin the root canal at least tw ice w ith a
The vitality o the tooth is not related to this type o healing, m onth gap betw een ap p lications. Applications continu e
e.g. the tooth may be vital or non-vital. u ntil rad iographs show the lack of rad iolucency in the bone
Following trauma this can be present on any part o ad jacent to the affected tooth. Once this is d etected , the root
the root sur ace but is only visible on conventional 2D can be obtu rated w ith gu tta p ercha.
181 •
THEAVULSEDINCISOR 32

Fig. 32.3 (A) Clinical appearance o 1 12 months ollowing


avulsion injury. (B) Radiographic appearance o 1 12 months
ollowing avulsion injury, showing urther root development.

B CASE2
Fig. 32.2 (A) A post-replantation radiograph taken at 14-day SUMMARY
review, 1 showing an immature root orm. (B) 1 radiograph
taken at 6-month review showing continued root development. Justin is 11 years old. He ell o his scooter, avulsing
his 1 . His riends at the park ran home to get his
parents, who arrived within 15 minutes. They took
Justin and his tooth to an accident and emergency
Follow ing splint rem oval, Kathryn’s 1 w as carefully department. At hospital his tooth was placed in
review ed at 6 w eeks, 3 m onths, 6 m onths and 12 m onths. milk. Three hours later his tooth was replanted and
At each visit, special tests (d escribed in Chap ter 31) and splinted. The EAT was 4 hours, and the EADT 1 hour.
rad iographs w ere taken. This w as to ascertain the type of ■ Justin’s parents book a n emergency a ppointment so see
period ontal healing and to look for signs of p u lp al necrosis you the ollowing day, what will you do?
and or infection-related resorp tion. As can be seen in Fig.
32.2, continu ed root form ation w as seen d em onstrating Fig. 32.4 show s a su tu re splint. This typ e of splint is inap-
pu lp revascu larization. At 12 m onths (Figs 32.3A and B) prop riate, as the patient w ill nd it very d if cu lt to m aintain
there w as clear clinical and rad iographic signs of period on- good oral hygiene. Moreover, this typ e of sp lint is unable to
tal healing (re-establishm ent of a norm al period ontal liga- hold the tooth in the rep lanted p osition w ith the 1 slightly
m ent w ith a few sm all areas of rep air-related resorp tion) extru d ed on presentation. The sp lint w as rep laced for
and continu ed root grow th and ap ical closu re (signs of a more conventional sp lint (d iscu ssed in Chapter 31);
revascu larization). Kathryn w as therefore d ischarged back how ever, Ju stin w ou ld not tolerate fu rther rep ositioning of
to her general d ental practitioner w ith the prognosis that the tooth to im p rove its p osition. Ad vice d escribed earlier
the 1 w ill be retained as a fu nctional tooth for the rest of in this chap ter is reiterated , and Ju stin w as booked for a
her life. review appointm ent 8 d ays after his accid ent.
• 182
32 THEAVULSEDINCISOR
resorp tion. Therefore early referral to an interd iscip linary
team is essential. At this interd iscip linary clinic they w ill
d iscu ss the d ifferent treatm ent op tions w ith Ju stin and his
parents.
BSPD and IADT gu id elines d iffer slightly w ith their cu t
off p oints of w hen ankylosis-related resorp tion is highly
likely. Beyond EADT of 30 m inu tes and EAT of 90 m inutes,
BSPD gu id elines rep ort that there is less than a 10% chance
of p eriod ontal healing. For IADT, beyond EADT of 60
m inutes there is less than a 1% chance of p eriod ontal
healing. Justin is beyond both of these cu t off p oints, and
therefore ankylosis-related resorption is almost guaranteed .
H ow ever, m aintaining a tooth w hich w ill u ltim ately fail is
very im p ortant over the short to m ed iu m term . The tooth
Fig. 32.4 Showing 1 replanted with an inappropriate suture w ill help m aintain good aesthetics for the p atient as w ell as
splint. Reproducedwith permission romthe RCSEDen project, Royal College o Surgeons o England. the height and w id th of the alveolus. Careful follow -u p is
essential to d iagnose infraocclu sion as soon as it becom es
■ Wha t is the cha nce o pulp survival? ap parent.
Rad iograp hs show 1 has com p lete root d evelop m ent. There- ■ What long-term treatment options are available?
fore there is no chance of p u lpal revascu larization. In ord er
There are a nu m ber of d ifferent op tions w hich w ere assessed
to prevent infection-related resorp tion, root canal treatm ent
for Ju stin. These inclu d ed d ecorontation, orthod ontic space
is com m enced at this visit. It is good practice to extirp ate
closure and cam ou age of the lateral incisor, d entu re, resin
the necrotic pu lp and d isinfect the root canal before rem ov-
retained brid ge and p rem olar transplant. In a grow ing
ing the splint. Justin w as booked for a fu rther review and
patient (e.g. a child or ad olescent) an osseointregrated
d isinfection of the root canal 1 m onth later.
im plant is contraind icated as they have not com p leted their
■ Wha t intracana l medicament should be pla ced in the facial grow th.
extirpated tooth? The d ifferent op tions w ere d iscu ssed w ith Ju stin and his
There has been som e anim al-based research that has show n parents at the interd iscip linary clinic. Justin had crow d ing
that w hen non-setting calciu m hyd roxid e is placed too in his u pper and low er arches and w as keen on und ergo-
early after rep lantation (w ithin the rst 7 d ays) in a tooth ing orthod ontics to im prove his app earance. As p art of the
w ith a d am aged p eriod ontal ligam ent, this can aggravate treatm ent p lan a p rem olar w as to be extracted in each
favou rable p eriod ontal healing and lead to ankylosis-related qu ad rant. The best prognosis for prem olar transp lant is
resorp tion. With this in m ind som e au thorities ad vocate that w hen the root form ation is betw een three-qu arter and
a steriod / polyantibiotic p aste shou ld be the rst d ressing comp lete root length d evelopm ent bu t w ith an open ap ex.
placed in root canals of avu lsed teeth as soon after the Ju stin tted the criteria for a p rem olar transp lant and both
inju ry as p ossible. They argued that the steroid / antibiotic he and his parents w ere keen to pu rsu e this option (see
paste w ill d am pen the im m u ne resp onse p rom oting favou r- Figs 32.5 A-D).
able healing. Cu rrently there is no d e nitive research su p- Ju stin had a tooth extracted in each qu ad rant includ ing
porting one app roach over another. Therefore gu id elines the 5 w hich w as u sed for the p remolar transp lants. This w as
ad vocate either ap proach. For Ju stin’s tooth, a non-setting bu ilt up initially d u ring the healing p hase of the transplant
calcium hyd roxid e d ressing w as p laced at his d ay-8 review before the orthod ontic treatm ent w as u nd ertaken. On the
appointm ent. comp letion of his orthod ontic treatm ent, a nal com posite
bu ild u p w as p rovid ed . The transplant d em onstrated both
■ Wha t are the cha nces o periodontal healing?
pu lp al and period ontal healing. The prognosis for the trans-
There is very little chance of period ontal healing w ith the plant is that it w ill be retained as a fu nctional u nit for the
tooth almost gu aranteed to heal by ankylosis-related rest of Ju stin’s life.
183 •
THEAVULSEDINCISOR 32

Fig. 32.5 (A) 1 week a ter the premolar transplant procedure with initial composite build up. (B) Radiograph o the premolar transplant
taken 1 week a ter the premolar transplant procedure. Reproduced with permission romthe RCSEDen project, Royal College o Surgeons o England. (C) Radiograph o
premolar transplant taken 6 months a ter the premolar transplant, showing periodontal and pulpal healing (continue root growth).
Reproduced with permission romthe RCSEDen project, Royal College o Surgeons o England. (D) Premolar transplant (1 ) a ter orthodontic treatment and nal composite
build up. Reproducedwith permission romthe RCSEDen project, Royal College o Surgeons o England.

Primary resources and Day PF, Kind elan SA, Sp encer J et al 2008 Dental trau m a: p art 2.
Managing poor prognosis anterior teeth - treatm ent options for
recommended reading the su bsequ ent sp ace in a grow ing p atient. J of Orthod
And ersson L, And reasen JO Day P et al 2012 International 35:143–155.
Association of Dental Trau m atology gu id elines for the Trope M 2011 Avu lsion of p erm anent teeth: theory to p ractice.
m anagem ent of traum atic d ental inju ries: 2. Avu lsion of Dent Traum atol 27 (4):281–294.
p erm anent teeth. Dent Trau m atol 28 (2):88–96.
And reasen JO, Laurid sen E, Bakland L et al 2014 Dental trau m a For revision, see Mind Map 32,
gu id e. Available at: http :/ / w w w.d entaltraum agu id e.org/
Exam ination.aspx. page 253.
Day P, Gregg T 2012 Treatm ent of avu lsed p erm anent teeth in
child ren. British Society of Paed iatric Dentistry. Available at:
http :/ / bspd .co.u k/ Resou rces/ BSPD-Gu id elines.
33
■ What a re the main problems associated with natal and
neona tal teeth?
Mobility.
Ulceration o ventral sur ace o tongue.
Nipple soreness (breast eeding mothers).
The teeth are m obile becau se the d evelop m ent of the
tooth is consistent w ith age. Only abou t ve-sixths of

Disorders o eruption the crow n, and u sually no root, is form ed . Ad d itionally, the
crow n is occasionally d ilacerated and the enam el hypoplas-
tic or hypom ineralized .

and exoliation
Excessive mobility is a d anger to the airw ay from aspira-
tion, and the tooth should be rem oved . Care shou ld be taken
to ensure that the entire tooth inclu d ing the p ulpal tissue is
removed , otherw ise d entine and a root w ill form , w hich w ill
requ ire eventual rem oval. If teeth can be left, then continued
root d evelopm ent w ill occu r. N ipp le soreness m ay occasion-
ally necessitate tooth rem oval.

CASE1 Ulceration on the ventral su rface of the tongu e m ay


respond to carm ellose sod iu m oral p aste. Sm oothing of the
incisal ed ges w ith sand p ap er d iscs m ay also help .
SUMMARY If the d ecision is m ad e to rem ove the tooth, d o rem em ber
Beth was only 20 days old when it was noticed she the local anaesthetic d oses, as these you ng babies are likely
had two teeth at the ront o her lower jaw (Fig. 33.1). to w eigh betw een 2–4 kg. N ew borns are given vitam in K in
the rst w eek of life. Check that they have received this as
■ Wha t is the correct terminology or these early
either an injection or an oral d ose in hosp ital or from their
erupting teeth?
m id w ife.
If the teeth are present at birth, ‘natal’ is the correct term . If
they are not present at birth but eru pt w ithin the rst month
of life, ‘neonatal’ is correct. Key point
The prevalence rates for both natal and neonatal teeth are Natal and neonatal teeth may need to be removed i :
reported as 1 in 2000–3000 live births. The m ost com m only
• Mobility causes concern about inhalation.
presenting tooth is the low er central incisor. More rarely,
• Ulceration under the ventral sur ace o the tongue
m axillary incisors or rst m olars have been rep orted . The
persists.
early eru ption is thou ght to be caused by the ectopic posi-
tion of the tooth germ d u ring fetal life. N atal and neonatal • Nipple soreness is signif cant.
teeth m ay follow a sp orad ic p attern, or they m ay be fam ilial.
H ow ever, they can be associated w ith speci c synd rom es:
■ What actors ca n cause generalized premature eruption but
Pachyonychia congenita.
still be considered as ‘normal’?
Ellis van Creveld.
Familial a amily history is a common nding.
Hallermann Strei .
Children with high birth weights.
Maternal smoking during pregnancy.
Lower socioeconomic status.
Reduced maternal physical activity.
Race generally Negroids tend to erupt their permanent
teeth earlier than Mongoloids, who are in turn in advance o
Caucasians and nally Asian children. Racial group can a ect
eruption times and eruption patterns o the primary
dentition.
Sex emales tend to erupt permanent teeth several months
ahead o males.
The opposite of prem atu re erup tion is d elayed eru ption.
■ When is generalized delay in eruption o primary
teeth expected?
Preterm in ants.
Fig. 33.1 Natal teeth. Very low birth weight in ants.
185 •
DISORDERSOFERUPTIONANDEXFOLIATION 33
■ What conditions may lead to a generalized delayed
eruption o teeth in both primary a nd permanent
dentitions?
Chromosomal abnormalities Down syndrome and Turner
syndrome.
Gross nutritional de ciency.
Hypothyroidism/hypopituitarism. A

Hereditary gingival bromatosis (HGF).


Acquired gingival overgrowth (drug-induced phenytonin,
cyclosporin, sodium channel blockers).
Acquired reduction in bone turnover (drug-induced
bisphosphonates).

■ What specif c condition is a ssociated with grossly delayed


or ailed eruption o teeth in the permanent dentition?
Cleid ocranial d ysp lasia – this is an au tosom al d om inantly
B
inherited cond ition w here, in ad d ition to m u ltip le su pernu -
m erary teeth cau sing d elayed exfoliation of p rim ary teeth Fig. 33.2 (A) Cyclic neutropenia. (B) Cyclic neutropenia.
and d elayed eru p tion of permanent teeth, there is ap lasia of
the d istal end or total absence of the clavicles.
■ What local a ctors ca n account or delayed eruption o Box 33.1 Di erential diagnosis o causes o premature
permanent teeth? ex oliation o primary and permanent teeth
Supernumerary teeth or odontomes. • Neutropenias and qualitative neutrophil de ects:
Ectopic crypt positions o permanent teeth. • Cyclic neutropenia.
• Congenital neutropenia (Kostmann disease).
Cystic change in the ollicle o permanent teeth.
• Prepubertal periodontitis.
Crowding.
• Juvenile periodontitis.
Thickened mucosa due to early primary tooth removal.
• Leucocyte adhesion de ect.
Exfoliation of teeth (like eru ption) can be either prema- • Papillon–Le èvre syndrome.
ture or d elayed .
• Chediak–Higashi disease.
• Langerhans cell histiocytosis – leading to bony destruction.
• Hypophosphatasia with aplasia or hypoplasia o cementum.
CASE2 • Sel -injuryin either a psychotic disorder or the congenital insensitivity to pain
syndrome.
SUMMARY • Ehlers–Danlos syndrome (type VIII) – disorder o collagen ormation causing
progressive periodontal destruction.
George was 3 years o age when his mother
• Scurvy – loss o tooth due to ailure o proline hydroxylation and collagen
f rst noticed that his lower primary incisors were synthesis.
loose.

Prem ature loss of prim ary teeth is an im portant d iagnos-


History tic event, as m ost cond itions cau sing it are potentially
George w as born after a norm al p regnancy and d elivery bu t serious and w arrant im m ed iate investigation (Box 33.1).
had p roblem s after birth w ith recu rrent cou ghs and cold s, Generally teeth m ay be lost early becau se of:
u p per and low er resp iratory tract infections, and oral u lcer-
Metabolic disturbances.
ation. H e w as extensively investigated and w as con rm ed
as having a cyclic neu trop enia. Severe periodontal disease.
Loss o alveolar bone support.
Dental history Sel -injury or non-accidental injury.
George and his m other had regu lar toothbru sh instruction, George w ill continu e to have regu lar d ental care and
and his oral hygiene w as excellent. H e also u sed 0.2% chlo- supervision of bru shing. Even in the p resence of im m acu -
rhexid ine gel at night instead of u orid ated toothp aste. late p laqu e control w e can exp ect that his neu trop hil d efect
Desp ite these efforts his low er p rim ary incisors exfoliated w ill p red isp ose him to p eriod ontal d isease and p rem atu re
betw een age 4 and 5 years, and by his sixth birthd ay he had loss of som e if not all his p erm anent teeth.
erup ted his low er perm anent central incisors and rst p er- The opposite of prem ature exfoliation is d elayed
m anent m olars (Figs 33.2A and B). exfoliation.
• 186
33 DISORDERSOFERUPTIONANDEXFOLIATION

■ Wha t ca uses delayed ex oliation o prima ry teeth? ■ How is in raocclusion graded?


Double primary teeth. • Grade I occlusal level above contact point o
Hypodontia a ecting permanent successors. adjacent tooth.
• Grade II occlusal level at contact point o adjacent
Ectopically placed permanent successors.
tooth.
Trauma or periradicular in ection o primary teeth causing • Grade III occlusal level below contact point o
interruption o physiological resorption. adjacent tooth.
In raocclusion or ankylosis. Grad e III infraocclu sions, if progressive, m ay be com -
In 40% of cases, d ou ble p rim ary teeth are associated w ith p letely subm erged by the surround ing hard and soft
an abnormality in the p erm anent d entition nu m ber. Parents tissu es. Rad iographs of infraocclu d ed teeth show blu rring
shou ld be ad vised of this and a d ental p anoram ic tom ogram or absence of the period ontal sp ace.
shou ld be taken arou nd the age of 6. Fu rther information
on d ou ble teeth is p rovid ed in Chap ter 37. Primary resources and
Infraocclu sion is the p referred term for either ‘su b- recommended reading
m erged teeth’ or ‘ankylosis’ w hen d escribing teeth that Ald red M, Cam eron A, Georgiou , A 2013 Paed iatric oral
have failed to achieve or m aintain their occlu sal relationship m ed icine, oral p athology and rad iology. In: Cam eron A, Wid m er
to ad jacent or op p osing teeth. Most com m only, p rimary R (ed s), H and book of Ped iatric Dentistry, 4th ed . Mosby Wolfe,
teeth have reached a norm al occlu sal level before becom ing St Lou is.
infraocclud ed . Craw ford PJM, Ald red MJ 2012 Anom alies of tooth form ation and
The tooth m ost com m only affected is the mand ibu lar eru p tion. In: Welbu ry RR, Duggal MS, H osey MT (ed s),
rst p rim ary m olar. Males and females are affected equ ally. Paed iatric Dentistry, 4th ed . Oxford : Oxford University Press.
Infraocclu d ed prim ary teeth are associated w ith a higher Managem ent of u nerup ted m axillary incisors 2010. Lond on:
incid ence of absent p erm anent su ccessors. Fu rther d etails of Royal College of Su rgeons of England , Facu lty of Dental
Su rgery. Available at: http s:/ / w w w.rcseng.ac.u k/ fd s/
their m anagem ent is d iscu ssed in Chapter 8.
p u blications-clinical-gu id elines/ clinical_gu id elines/ d ocu m ents/
ManMaxIncisors2010.p d f.
N tani G, Day PF, Baird J et al 2015 Maternal and early life factors
of tooth em ergence patterns and nu m ber of teeth at 1 and 2
years of age. J Dev Orig H ealth Dis 6:299–307.
Key point
In raocclusion: For revision, see Mind Map 33,
• Mandibular f rst primary molar is most commonly page 254.
a ected, with mandibular primary molars more
commonly a ected than maxillary primary molars.
• More common in primary teeth than in permanent
teeth.
• Equal sex ratio.
• Higher incidence o absent permanent successors.
• Prevalence o 2 8%.
34
the labial su rfaces of the new ly eru pted u pper and low er
p erm anent central incisors (Figs 34.1 and 34.2).
Up p er and low er arches w ere u ncrow d ed . Space assessed
from the d istal of 2’s to m esial of 6’s in each qu ad rant w as
21.5 m m in the low er arch and 22 m m in each u pp er arch
qu ad rant. On average, 21 m m and 22 m m are requ ired for
these d istances in the low er and u pper arches, to p rovid e
suf cient space for the p rem olars and canines to align
w ithin the arch. The incisor relationship w as Class I on

Poor qualityfrst 1 1; the m olar relationship w as Class I bilaterally.

permanent molars Key point


• Hypoplasia is the name given when there is a reduced
thickness o enamel but the enamel is well mineralized
(e.g. there is reduced quantity but normal quality o
enamel).
• Hypomineralization is the name given when there is
reduced mineral content o enamel but a normal
SUMMARY amount o enamel has been laid down (e.g. there is
Lisa is 9 years old. Her mother has brought her to reduced quality o enamel but normal quantity).
your surgery because her recently erupted • Post-eruptive breakdown is where the tooth erupts
permanent teeth at the back o her mouth are with hypomineralized enamel. Shortly a ter eruption
this area o enamel ractures o exposing underlying
brown and there are creamy patches on her adult
dentine.
incisors. What has caused these discolorations? • Atypical restorations is where the classical appearance
How may they be treated? o the restoration or dental caries ( ollowing Black’s
cavity design) is not seen.
History
Lisa has com p lained that for the p ast few m onths these teeth
have been p ainfu l on eating hot and cold food s. The pain is
of 1–2 m inu tes’ d u ration. Toothbru shing also has cau sed
sensitivity at the back of the m ou th. There has been no pain
on biting and eating food s. Very recently Lisa felt that one
of the back teeth has started to cru m ble. H er m other has also
noticed this w hen she has help ed w ith bru shing. Lisa has
not requ ired any analgesics for the d iscom fort cau sed by
her teeth.

Medical history
Lisa is a healthy child w ho has never been in hosp ital.

Dental history
Lisa and her fam ily are regu lar d ental attend ers. The fam ily
m em bers all have a low caries risk. Lisa has had no restora- Fig. 34.1 Hypomineralized and hypoplastic upper rst permanent
tions in her p rim ary d entition. She u sed a child ren’s tooth- molars.
p aste (1050 p p m) u ntil recently and is now u sing the sam e
ad u lt toothp aste as her p arents (1450 p p m ).

Examination
Intraoral examination revealed that all fou r rst perm anent
m olars (FPMs) w ere hyp om ineralized w ith areas of brow n
enam el. In ad d ition, there w ere areas of post-eru ptive
breakd ow n (hypoplasia) w here the enam el had chipped
aw ay, exp osing the u nd erlying d entine. The m axillary
m olars w ere the w orst affected by p ost-eru p tive break-
d ow n. 6 had a large atyp ical am algam restoration. In ad d i-
tion, some cream y hyp om ineralized areas w ere visible on Fig. 34.2 Hypomineralized upper and lower permanent incisors.
• 188
34 POORQUALITYFIRSTPERMANENTMOLARS

■ Do you think that the enamel hypomineralization and ■ What other di erential diagnoses might you consider?
hypopla sia noted on the f rst perma nent molars and the
• Caries. Newly erupted teeth are particularly prone to
permanent incisors ollows a chronological pattern? I so, a t
dental caries until their enamel maturation is complete.
what time wa s the a ected enamel ormed?
However, it seems very unlikely, even in the presence o
Yes, it is p ossible that there is a chronological pattern. The particularly deep ssures, that Lisa, who has no caries in
cu sp tip s of the rst p erm anent molars begin mineralizing her primary teeth, should develop such caries in her
from the eighth m onth of p regnancy, and the cu sp tips of permanent teeth. In addition, the overall colour o the
the incisors and cu spid s (canines) from abou t 3 m onths of permanent molars is not consistent with caries in a tooth
age (the u pp er lateral incisor slightly later at 10–12 m onths). o normal morphology. The exposed dentine is slightly
Mineralization d ates for the p erm anent d entition are given so tened and does have caries, but the overall pattern
in Table 34.1. o destruction o the tooth suggests that caries is
■ Wha t specif c questions would you like to ask Lisa’s mother? secondary to some other predisposing actor, such as
hypomineralization/hypoplasia. Another actor to
Pre-natal. Mother’s health during pregnancy. Were there any consider that is against a diagnosis o caries is the creamy
concerns such as high blood pressure, proteinuria or hypomineralized areas on the incisors. They are neither
preeclampsia? the shape nor the distribution o white spot or precarious
Peri-natal. Di cult birth. Was the delivery prolonged? Was lesions that one would expect with poor oral hygiene.
there a need or assisted delivery by orceps, ventouse or • Amelogenesis imper ecta. Although the presenting
caesarean? All o these can be associated with etal distress. eatures could be a hypomature orm o amelogenesis
Post-natal. Did Lisa spend any time in the special care baby imper ecta, there are several actors that suggest this is
unit (SCBU)? not the case: there is no amily history; the primary
dentition is not a ected; the pattern o the de ects is
Illnesses in the f rst 2 years o li e, e.g. meningitis, measles,
chronological. All o these combined make an inherited
respiratory in ections, chickenpox. These disturbances may
abnormality unlikely.
mani est as enamel de ects distributed in the enamel that
• Fluorosis. The severe enamel de ects on the molar teeth
was orming around birth and in the rst 2 years o li e.
could only occur i there was a history o very high
Fu rther qu estioning revealed that Lisa w as born after endemic f uoride levels, probably in excess o 6 ppm.
a norm al p regnancy and d elivery bu t had a signi cant Such levels do not occur in the UK. Mild f uorosis is seen
nu mber of respiratory infections d u ring the rst year of life. where children have swallowed toothpaste or have been
This con rm s you r d iagnosis of chronological hyp op lasia. given supplementation in addition to swallowing
The correct nam e for this cond ition is m olar incisor hypom - toothpaste. However, in such cases there are usually ne,
ineralization (MIH ). Post-eru ptive breakd ow n and atyp ical opaque, white lines ollowing the perikymata and small
restorations are consistent w ith this d iagnosis. irregular enamel opacities or f ecks that merge into the
background enamel colour. Fluorosis does not produce
well-demarcated opacities like those seen on the incisors.

Table 34.1 Mineralization times or the permanent dentition Key point


Tooth Mineralization begins (months)
In patients with MIH, your questioning should include:
Upper • Pre-natal period.
Central incisor 3–4 • Natal period.
Lateral incisor 10–12 • Post-natal period.
Canine 4–5 • Systemic illnesses in f rst 2 years o li e.
First premolar 18–21
Second premolar 24–27 ■ Is pain rom such molar teeth common?
First molar At birth
Yes. There is evid ence that these teeth have 5–10 tim es
Second molar 30–36
greater treatm ent need than norm al teeth and are m ore d if-
Third molar 84–108 cult to anaesthetize. A p alatal as w ell as a bu ccal in ltra-
Lower tion is essential. There is also evid ence that child ren w ith
Central incisor 3–4 hypom ineralized rst permanent m olars (H FPMs) have
m ore behaviou r management p roblem s than other child ren,
Lateral incisor 3–4
necessitating ad juncts to treatm ent su ch as sed ation. In
Canine 4–5
recent years, the prevalence of H FPMs varies across d iffer-
First premolar 21–24 ent pop u lations in the w orld . The average prevalence is 16%
Second premolar 27–30 w ith a range betw een 3% and 44%.
First molar At birth The histology of extracted H FPMs show s that the yellow /
brow n areas are m ore p orou s and occu p y the entire enam el
Second molar 30–36
layer. The w hite/ cream y areas occu py the inner parts of
Third molar 96–120
enam el. The affected areas have a higher carbon and low er
189 •
POORQUALITYFIRSTPERMANENTMOLARS 34
calcium and p hosp hate content (e.g. less m ineral and m ore Composite These can be d e nitive restorations w hen there
protein). are sm all areas of hypom ineralization. It is im p ortant w hen
restoring these teeth that all the hypomineralized area is
removed . This is achieved w ith gently ru nning a slow hand -
Investigations p iece w ith a large rose bur over the d iscolou red enam el. The
■ What investigations are indicated and why? soft enam el w ill give w ay, leaving hard , less affected enam el
behind .
Intraoral radiographs are ind icated in ord er to assess the p rox-
imity of the coronal d efects to the d ental p u lp . These can be Stainless steel crowns These are the m ost d u rable restora-
d if cu lt to interp ret, as MIH -affected teeth have a d ifferent tion and can m aintain a tooth u ntil a perm anent crow n
rad iographic app earance to d ental caries. A panoramic tomo- can be placed in the teenage years, or until a planned
gram is necessary to ascertain the p resence and stage of extraction.
d evelopm ent of the rem aining p erm anent d entition in view Adhesively retained copings These m ay be su itable for
of the p oor long-term p rognosis of the rst p erm anent teeth that are not signi cantly affected by hypomineraliza-
m olars. tion. When the d efect has been removed and replaced by
A panoram ic tom ogram revealed all p erm anent teeth, GIC, a cop ing can be placed on top of the restoration and
includ ing third m olars, to be present. The fu rcation of 7’s cover the rem aind er of the occlu sal and cu sp al su rface.
w as calcifying. Second ary caries w as evid ent in all rst
6 6 Extraction For m od erate to severely affected H FPM this is
perm anent m olars but w as m ost p ronou nced in . the preferred treatm ent option because:
6
• HFPMs are o poor long-term prognosis. Although ull
coverage coronal restorations could be undertaken to
Treatment retain them, this is an ambitious plan in a 9-year-old
child. The restorations undoubtedly would require
■ What are the main clinical problems in this case?
replacement at some uture date due to possible
Loss o tooth substance: breakdown o enamel tooth wear microleakage at the margins and caries. This would incur
secondary caries. additional inconvenience, expense and li e-long
Sensitivity. treatment or the patient.
• It is the optimal developmental stage to remove 6’s
Appearance.
as the urcation o 7’s is calci ying and predictable bodily
movement o 7 orward is most likely. Timing o 6
Key point removal is more critical than that o 6 as the mesial dri t
Children with MIH: tendency is greater in the upper arch.
• Have a higher treatment need. • Third molars are also developing and should erupt
• Have signif cantly higher behaviour management eventually i the 7’s migrate orward to occupy the
problems. position o the 6’s (Fig. 34.3).
Achieving good local anaesthetic o the entire tooth is
essential, as requently these teeth can be very sensitive
(histology o a ected MIH teeth shows signif cant pulpal
in ammation).
Treatment under rubber dam is requently needed to
prevent sensitivity rom a ected adjacent teeth.

■ What are the treatment options or the HFPMs in this case?


Sensitivity.
A
Composite.
Stainless steel crowns.
Adhesively retained copings.
Extraction.
Sensitivity This can be m anaged in a nu m ber of d ifferent
w ays. Ad vising p atients to u se antisensitivity toothpaste
and / or toothmousse (see Chap ter 23) can be affective. For
child ren over the age of 10, u se of a high- u orid e toothp aste
(e.g. 2800 pp m ) can also w ork. For localized areas that are B
sensitive to toothbrushing or cold d rinks and food , tem po-
rary glass ionomer cem ents (GICs) are often u sed to red u ce Fig. 34.3 (A) Dental panoramic tomogram be ore removal o all
the sensitivity sym p tom s. This is often u sed to maintain rst permanent molars. (B) Dental panoramic tomogram
sym p tom atic m olars u ntil the id eal age for their extraction. ollowing removal o all rst permanent molars.
• 190
34 POORQUALITYFIRSTPERMANENTMOLARS
• As the molar relationship is Class I bilaterally, removal o betw een the tw o. Im portantly this is the least invasive treat-
6’s necessitates the removal o 6’s to encourage m ent w ith no rem oval of enam el.
maintenance o the buccal segment relationship. This is Controlled enam el m icroabrasion m ay p rod u ce a m ore
known as compensating (removal o the equivalent acceptable result w ithou t rem oving the w hite areas. This is
opposing tooth) extractions and preventing over- becau se the su rface enamel layer after m icroabrasion is rela-
eruption o the 6’s. tively p rism less and w ell com pacted . The optical properties
are changed and a w hite area m ay becom e less p erceptible.
The techniqu e shou ld not be u sed if there is a red u ced thick-
Key point ness of enam el.
• Ensure all permanent teeth, especially 5’s and 8’s, are Localized com p osites can give very accep table resu lts
present radiographically be ore considering FPM bu t are d estru ctive of enam el and m ay w eaken the tooth
extraction. In some situations the FPM may be extracted stru ctu re if large areas are rem oved .
even i the 8’s are absent. Fu ll com posite veneers w ith a thin layer of a relatively
• An orthodontic assessment o the patient is essential to op aqu e com p osite m ay p rod u ce an accep table resu lt w ithou t
establish their malocclusion. A consultation with an any, or very little, enam el red u ction. This w ill, how ever,
orthodontist is prudent in Class II and III cases. m ake the tooth bu lkier.
• Timing o the extraction o 6 is more critical than that o
6. Ideal timing or a 6 is between the ages o 8.5 9.5 Primary resources and
(when there is calcif cation o the bi urcation o the 7). recommended reading
The timing or the extraction o a 6 is be ore the 7 D3 Grou p 2016 Develop m ent Dental Defects w ebsite. Available
erupts. at: http :/ / w w w.thed 3grou p .org.
• Consider compensating extractions o upper FPM where Jalevik B 2010 Prevalence and d iagnosis of m olar-incisor-
there is moderate to severe MIH in lower FPM. hyp om ineralisation (MIH ): a system atic review. Eu r Arch
Paed iatr Dent 11:59–64.
Jalevik B, Klingberg GA 2002 Dental treatm ent, d ental fear and
behaviou r m anagem ent p roblem s in child ren w ith severe
■ What are the treatment options or the incisors in this ca se? enam el hyp om ineralisation of their p erm anent rst m olars.
Bleaching. Int J Paed iatr Dent 12:24–32.
Lygid akis N A, Wong F, Jalevik B et al 2010 Best clinical p ractice
Microabrasion.
gu id ance for clinicians d ealing w ith child ren p resenting w ith
Localized composite restoration. m olar-incisor-hyp om ineralisation (MIH ): an EAPD p olicy
d ocu m ent. Eur Arch Paed iatr Dent 11:75–81.
Full composite veneer.
Royal College of Su rgeons A Gu id eline for the Extraction of First
Bleaching is controversial in these cases. Firstly, there Perm anent Molars in Child ren 2009. Available at: http :/ /
are legal issu es arou nd bleaching child ren’s teeth (see w w w.rcseng.ac.u k/ fd s/ p u blications-clinical-gu id elines/
Chap ter 36). Second ly, the bleaching can cau se sensitivity, clinical_gu id elines/ d ocu m ents/ A%20Gu id eline%20for%
and therefore p atients m ay stru ggle to com p ly w ith instru c- 20the%20Enforced %20Extraction%20of%20First%20Perm anent
tions for u se w hen their teeth are alread y sensitive. Finally, %20Molars%20in%20Child ren%20rev%20March%202009.pd f.
the im p act of the bleach is m ore likely to lighten the ad ja-
cent tooth arou nd the d efect, thereby red u cing the contrast For revision, see Mind Map 34, page 255.
35
Medical history
■ What specif c questions do you need to a sk his mother with
rega rd to potential causes o discoloration?
The pregnancy. The health of Sim on’s m other d u ring her
pregnancy and Sim on’s health d u ring the birth and d elivery
are im portant w hen consid ering the cond ition of the rst
perm anent m olars (FPMs). The FPMs w ere the only perm a-

Tooth discoloration,
nent teeth that had started to m ineralize before birth (arou nd
the eighth m onth of pregnancy). Cond itions that m ay
suggest som e fetal d istress and d ysm ineralization m ay be

hypomineralization
raised m aternal blood p ressu re; early ad m ission to hosp ital;
prematu re d elivery; p rolonged d elivery; assisted d elivery,
e.g. forcep s or ventou se, em ergency caesarean section; or

and hypoplasia
ad mission to the sp ecial care baby u nit (SCBU).
Childhood illnesses. These m ay resu lt in a ‘chronologi-
cal hyp op lasia’ affecting those p arts of the teeth that w ere
m ineralizing at the time of the illness. Although ‘chrono-
logical hyp op lasia’ u su ally involves som e failu re of d evel-
opm ent of enam el m atrix giving obvious lines or rid ges on
the teeth, there m ay be m ild er form s that can only be felt

CASE1 w ith a p robe and that p resent for care becau se they attract
extrinsic stain.
Tablets or medications taken during childhood. Tetra-
SUMMARY cycline staining shou ld not occu r now in child ren w ho have
Simon is 8 years old. He has been brought to your been brought u p in d evelop ed cou ntries. It is still com m on
in child ren from d evelop ing cou ntries w here tetracycline
surgery by his mother because his teeth are very
is still u sed becau se it is a very effective, cheap , broad -
dark and he is being teased at school. How would spectru m antibiotic. The only child ren w ho m ay still be
you determine the origin o the discoloration? affected in d eveloped countries are those w ith cystic brosis
w ho have d evelop ed mu ltip le d ru g resistances as a resu lt
History of recu rrent resp iratory infection.
Sim on w as born w ith prim ary biliary atresia. This
Sim on says that the colou r of his perm anent teeth has resulted in progressive liver failure, increasing levels of cir-
rem ained the sam e since they erupted (Fig. 35.1). cu lating biliru bin and eventu ally a liver transp lant at the
■ What other questions do you need to ask about the teeth? age of 2.5 years. All the perm anent teeth d eveloping prior
to the transp lantation w ill have intrinsic d iscoloration as a
Do they chip or wear?
result of the high circu lating biliru bin. The p rim ary d enti-
Are all the teeth a ected? tion w ill be affected to a lesser extent as a resu lt of staining
Was the primary dentition a ected? in second ary d entine. Althou gh not seen w ith Sim on, som e
Has anyone else in the amily got, or had, similar teeth? child ren can have gingival overgrow th as a resu lt of im m u -
Positive answ ers to the last three qu estions m ay su ggest nosu p p ressive treatm ent w ith cyclosp orine.
an inherited abnormality of the teeth su ch as am elogenesis The colour of the second perm anent m olars is likely to
or d entinogenesis im p erfecta. be entirely norm al as these teeth started m ineralizing
arou nd the age of 3 w hen there w as a fu nctioning new liver
and norm al levels of biliru bin.

Dental history
■ Wha t other lines o questioning do we need to explore i we
are considering all the possible causes o intrinsic
discoloration?
Was there a history o in ection and/or extraction or decay o
any o the primary teeth?
Localized infection on p rim ary teeth can cau se local-
ized abnorm alities of enam el form ation and m ineraliza-
tion of p erm anent teeth (Tu rner ’s tooth).

Fig. 35.1 Intrinsic discoloration.


• 192
35 TOOTHDISCOLORATION, HYPOMINERALIZATIONANDHYPOPLASIA
Was there ever any trauma to the primary teeth? ■ What is the only method o treatment that will help Simon’s
There is a 50% chance of enam el anom alies of perm a- appea ra nce?
nent su ccessor teeth after p rim ary trau m a (see Chap ter Veneering. Com p osite veneers should be provid ed u ntil the
29). These w ill be localized anom alies. age of 16 years w hen they can be replaced w ith porcelain
Fluoride history. veneers. Com posite veneers m ay not m ask the severe green
A fu ll history from birth, inclu d ing areas that the child stain u nless op aquing agents are u sed . Porcelain veneers
has lived in, u orid e su p p lem entation and bru shing m ay be the only realistic alternative; how ever, they are not
habits is requ ired . Flu orosis w ill p rod u ce a system ic or ap propriate until the gingival m argin has m atu red and sta-
chronological d istribu tion affecting the teeth that w ere bilized in late teens or early tw enties. The m ethod of place-
form ing w hen excess u orid e w as taken. m ent of comp osite veneers is covered in Chap ter 36.
The im p ortant categories and qu estions for a history An alternative, less d estru ctive ap p roach is the u se of
into intrinsic tooth d iscoloration and hypoplasia are vital nightgu ard bleaching (see Chap ter 36, together w ith
show n in Box 35.1. d iscu ssions abou t the cu rrent legality of bleaching for chil-
d ren und er 18 years old ). This requ ires no irreversible
d estru ction of enam el, and althou gh it m ay not fu lly correct
Examination the d iscolou ration, it m ay be su f cient for the patient. For
The im portant features to note about any intrinsic d iscolora- severe d iscolou ration several months of night-tim e bleach-
tion or hypoplasia are: ing m ay be requ ired . At 3 years there is rep orted colou r
relap se in a third of all cases. H ow ever, top up bleaching is
Is it generalized or localized? feasible and an accep table ou tcom e for som e p atients and
Does it a ect the primary and permanent dentitions? their parents.
■ In the major ca tegories or questioning shown in Box 35.1,
which a re likely to ca use genera lized discolora tion and
which are likely to ca use localized discoloration? Key point
Generalized: medical; amily; f uorosis. Lines o questioning in discoloration, hypomineralization
Localized: pregnancy; dental; trauma. and hypoplasia:
• Maternal.
Sim on has generalized intrinsic d iscoloration as a resu lt
of biliary atresia cau sing increased levels of biliru bin. • Trauma.
• Medical.
• Fluoride.
Box 35.1 Intrinsic tooth discoloration and hypoplasia • Dental.
Maternal and neonatal history • Family.
• History o pregnancy – maternal problems.
• History o birth – caesarean, orceps, etal distress.
Family history
■ I a patient ca me to you with a single discoloured root-f lled
incisor, what orm o treatment should you consider f rst?
• Is anyone in the amily similarly a ected.
Medical history N on-vital bleaching: this techniqu e has certain ad vantages,
especially in the you nger and ad olescent p atient:
• Dates o prolonged illnesses, e.g. childhood in ections, haematological disorders,
nutritional diseases etc. Non-destructive o tooth tissue (already root lled).
• Medications taken (such as topical or systematic tetracyclins). No irritation to gingival health that can occur with veneers.
Dental history No change in contour o tooth that may make oral hygiene
• Does the discoloration a ect all the teeth or just a ew. more di cult.
• Does the discoloration a ect both primary and permanent dentitions. Patient has some control over the amount o colour change
• Has the colour got worse or did the teeth erupt with this appearance. achieved, as they are responsible or the requency and
• History o abscesses o the primarydentition. duration o application, although as the dentist, you control
how many tubes o bleach (10% carbamide peroxide) are
• Anypain or sensitivity rom teeth.
used.
• Are the teeth chipping or wearing away.
The only contraind ications to non-vital bleaching w ou ld
Trauma history
be teeth that are p oorly obtu rated as they w ould need
• Has the child any historyo an accident to primaryor permanent teeth.
retreatm ent.
Fluoride history Patients w ith teeth that have com posites need to be
• Where has the child lived. w arned that these m ay need to be replaced once bleaching
• History o uoride supplements. has nished .
• Age at commencement o brushing. Althou gh there are other method s, su ch as sealing in
sod iu m perborate, these have very mu ch been su p ersed ed
• Amount and type o toothpaste.
by insid e outsid e technique and the d if culty in obtaining
• Toothpaste eating habits.
sod iu m perborate.
193 •
TOOTHDISCOLORATION, HYPOMINERALIZATIONANDHYPOPLASIA 35
Ideally the de nitive composite should be avoided
Technique or several days as the bleaching may inter ere with
composite bonding.
Visit 1
1. Make a diagnosis or the discolouration. Take
Visit 4
pre-operative periapical radiographs; these are essential
13. Remove the glass ionomer cement and cotton wool
to check i there is an adequate root lling and no
ball and provide a de nitive composite.
symptoms or signs o apical pathology.
Slight overbleaching is d esirable, as there is likely to be
2. Clean the teeth with pumice and make a note o the
some relap se. Although bleaching has been associated w ith
shade o the discoloured tooth. It is prudent to take
later occu rrence of external cervical resorp tion, resorption
clinical photographs o the anterior teeth with and
has never been reported w ith 10% carbam id e peroxid e.
without the shade guide visible.
3. Take an upper alginate impression o the entire arch.
Ask the technician to cast up the impression and to Key point
block out a well on both the buccal and palatal aspects
Restorative techniques in discoloration:
o the a ected tooth. Then to construct a so t vacuum
• Bleaching.
ormed tray made o thermoplastic material. The tray
• Microabrasion.
should be cut back and scalloped around the gingival
margin. • Localized composite.
• Composite veneers.

Visit 2
4. Check that the bleaching tray ts and trim back any

5.
rough edges that may irritate the gingival margin.
Place rubber dam, isolating the a ected tooth. Ensure
CASE2
adequate eye protection or the patient, operator and
SUMMARY
dental nurse.
6. Remove the palatal restoration to permit access to the Tony is 14 years old. He has come to your surgery
root canal. because he is concerned by the colour o his teeth
7. Remove root lling to 1 2 mm below the dentogingival and his bad breath (Fig. 35.2).
junction you may need to use adult burs in a
miniature contra-angle head.
History
8. Place 1 mm o glass ionomer cement over the gutta
percha. H e has noticed his teeth changing colou r over the p ast year.
9. Use an ultrasonic to remove debris blocking the H is friend s have also com mented on his bad breath over
dentinal tubules on the labial sur ace. Alternatively, very this period of tim e.
lightly use a slow hand piece with a round bur. Do not The colour of the teeth in Fig. 35.2 is a resu lt of extrinsic
remove excessive dentine. staining from chrom ogenic bacteria d u e to inad equate oral
hygiene. The staining is classically in the gingival and cervi-
10. Show the patient (or parent) how to ll up the access
cal areas of the teeth.
with 10% carbamide peroxide and then seat the
bleaching tray. A small amount can also be placed in ■ Are there any other causes o extrinsic staining?
the well o the bleaching tray around the discoloured Food and drink. Tea, co ee and dishes such as curry can cause
tooth. staining o the teeth. Most commonly this occurs in the
11. Provide written and verbal instructions. In summary gingival or cervical areas initially, but i oral hygiene is poor,
advise the patient to remove the bleaching tray and then it can a ect a signi cant part o the whole tooth sur ace
replace the bleach 3 5 times a day. This should (Fig. 35.3).
continue or several days up to 2 weeks until the
Arrested caries. This produces a brown stain as a result o
patient is happy with the colour. It is sensible to provide
chromogenic bacteria.
the patient with a maximum o two tubes o bleach so
that they are orced to return or a review appointment
a ter a week. The bleaching tray should be worn at all
times except or tooth brushing. Phoning the patient a
couple o days a ter their visit is sensible to check how
they are getting on. O ten at this point they will not
have seen any change in colour.

Visit 3
12. Check that the patient is happy with the colour. Wash
out the access cavity and place a cotton wool ball
and a temporary restoration (glass ionomer cement). Fig. 35.2 Extrinsic chromogenic staining.
• 194
35 TOOTHDISCOLORATION, HYPOMINERALIZATIONANDHYPOPLASIA
be ore school, and then last thing at night with a 2800 ppm
f uoride toothpaste based on a high caries risk.
Show him the ‘white spot’lesions. Explain that these will
progress to decay i the brushing is not corrected.
Daily f uoride mouthwash (0.05% sodium f uoride) used at a
separate time to brushing to encourage enamel
remineralization.

■ What actors in children a nd adolescents are important in


halitosis (bad brea th)?
Plaque index.

Fig. 35.3 Extrinsic ood staining. Bleeding sites in gingiva.


Food impaction.
Medical condition. In biliary atresia and jaundice it is possible Nasal in ection.
or bile pigments in the gingival crevicular f uid to cause Tonsillar and adenoidal in ection.
extrinsic staining. This is yellow/green in colour.
Furred tongue, especially posterior tongue.
Drugs. Ferrous sulphate in liquid iron preparations can result
in black staining. Ri abutin, an antituberculous drug, is ■ As well as improving his gingiva l health with improved
excreted in the crevicular f uid. This results in an orange-red toothbrushing, what else could be done with the
staining. toothbrush?
Chlorhexidine mouthwash, when used requently, can cause a Bru shing the d orsu m of his tongu e! Alternative m ethod s
brown-black staining. can be used to clean the su rface of the tongu e, and variou s
All these extrinsic stains originate initially arou nd the types of ‘tongu e scrap ers’ are available com m ercially. Very
gingival or cervical area of the tooth but can progress to occasionally in a child or ad olescent the origin of halitosis
involve a signi cant am ou nt of the tooth su rface. can be ulceration of oesophageal or gastric origin. If
there are sym ptom s of this, then a m ed ical referral should
■ How ca n you conf rm your diagnosis o extrinsic
be m ad e.
discolora tion?
Extrinsic stains can be p olished off. Carry ou t a Primary resources and
prop hylaxis.
recommended reading
■ What additiona l clinical signs are there in Fig. 35.2 to back Bharath KP, Su bba Red d y VV, Poornim a P et al 2014 Com p arison
up your diagnosis o chromogenic sta ining secondary to of relative ef cacy of tw o techniques of enam el stain rem oval on
poor ora l hygiene? u orosed teeth. An in vivo stu d y. J Clin Ped iatr Dent
38:207–213.
Marginal gingivitis.
Bu rrow s S 2009 A review of the ef cacy of tooth bleaching. Dent
‘White spot’demineralization lesions in the gingival third o Up d ate 36:537–538, 541–44, 547–548.
the labial enamel are visible on the anterior teeth. Bu rrow s S 2009 A review of the safety of tooth bleaching. Dent
Up d ate 36:604–606, 608–610, 612–614.
Kilp atrick N M, Bu rbrid ge L 2012 Anom alies of tooth form ation
Treatment and eru p tion. In: Welbu ry RR, Duggal MS, H osey MT (ed s),
Paed iatric Dentistry, 4th ed n. Oxford : Oxford University Press.
■ How would you treat Tony’s bad brea th?
Poyser N J, Kelleher MG, Briggs PF 2004 Managing d iscoloured
Encourage him. Remember teenagers do not take criticism non-vital teeth: the insid e/ ou tsid e bleaching techniqu e. Dent
well! Tell him that he is not alone and lots o ‘busy’young Up d ate 31:204–210, 213–214.
people orget twice daily brushing.
Toothbrushing instruction utilizing disclosing tablets or For revision, see Mind Map 35,
disclosing solution. Brushing twice daily, a ter break ast and page 256.
36
■ What childhood illnesses and in ections did she ha ve, and
when?
A chronological hyp om ineralization or hyp op lasia w ou ld
suggest a system ic origin. There w ere no signi cant
illnesses.
■ Wha t is Sophie’s uoride history?
This m u st inclu d e w here she has lived , any history of su p-

Mottled teeth
p lem ents, type and am ount of toothp aste used and any
history of eating toothp aste.
It transp ired that Sop hie had never received any su p p le-
m ents or u sed toothpaste excessively. H ow ever, she lived
on a farm w ith its ow n ‘w ell’ w ater sup p ly. This w as su b-
sequ ently analysed and w as fou nd to be over 1 ppm uo-
rid e. The d iagnosis w as one of u orosis.

Examination
■ Wha t is the distribution o the mottling that you can see in
SUMMARY Fig. 36.1?
Sophie is 8 years old. Her main concern is that White and brow n m ottling of the incisal half of the labial
some o her permanent teeth have white patches, surface of 1 1 and w hite m ottling of the incisal third of the
especially the upper central incisors. She is getting labial su rface of 2 2.
teased at school because the upper centrals also ■ Do you know why the labia l sur aces o the upper
have brown patches. What are the causes o the permanent central incisors are o ten more a ected
by mottling?
white patches? How may they be treated?
Research has show n that these teeth are p articu larly su scep -
tible to an excess ingestion of uorid e betw een 24 and 30
History m onths of age.
Sophie noticed that the p erm anent teeth cam e through w ith Mild uorosis gives a d iffu se m ottling that m ay m anifest
the w hite and brow n patches (Fig. 36.1). They have not as d iffu se lines or patches that m erge into the backgrou nd
changed in ap p earance since eru p tion. enam el. When the uorosis becom es m ore severe the lines
■ What important questions would you now a sk her mother? and patches coalesce to p rod u ce a con u ent w hite su rface.
In very severe cases there is also p itting of the enam el. Well-
Were the p rim ary teeth norm al? d e ned or w ell-d em arcated p atches (su ch as m olar incisor
If the answ er to this is yes, then it is u nlikely to be an hypom ineralization; see Chapter 34) that d o not follow a
inherited d efect and m ore likely to have a system ic origin. system ic or chronological d istribu tion, or are localized , are
■ Is anyone else in the a mily a ected? u nlikely to be d ue to u orosis.
Unless siblings are subjected to exactly the sam e systemic ■ Which part o the enamel does mild uorosis a ect?
d iseases and cond itions it is likely that an affected sibling The ou ter 200–300 µm .
w ill ind icate an inherited d efect.
■ How ca n you use this knowledge to your adva ntage during
Sop hie’s p rimary teeth w ere norm al, and she has no sib-
your clinica l examination?
lings. N either her m other ’s nor her father ’s fam ily has
anyone w ith sim ilar d ental ap pearance. Look for areas of the d entition that are su bject to erosion or
attrition, e.g. the occlusal su rfaces of the rst perm anent
m olars. If the m ottled enam el has been rem oved on these
surfaces then that con rm s that the m ottling is in the outer
aspect of the enam el and the d iagnosis is likely to be one of
u orosis.

Key point
History taking in suspected uorosis:
• Fluoridated water in places o residence.
• Amount o paste and age brushing started.
• Type o toothpaste used and uoride concentration.
• Flouride supplements.
Fig. 36.1 Dis guring f uorotic mottling a ecting upper • Toothpaste eating habits.
permanent central incisors.
• 196
36 MOTTLEDTEETH

■ In some cases o uorosis there is, in addition to white


mottling, some brown stain. What is the cause o the
brown sta ining?
This is u su ally d u e to extrinsic agents becom ing incorp o-
rated into the m ore p orou s w hite m ottled areas of enam el.
The brow n m ottling tend s to get w orse w ith tim e.

Treatment
■ What treatment options or Sophie would you consider or
uorotic mottling?
Fig. 36.2 A ter acid pumice microabrasion.
Microabrasion.
Composite veneers. 5. Mix 18% hydrochloric acid with pumice into a slurry and
apply a small amount to the labial sur ace on either a
Vital Bleaching.
rubber prophylaxis cup rotating slowly or a wooden
Microabrasion can be carried ou t in a variety of w ays and stick rubbed over the sur ace or 5 seconds, be ore
is a controlled rem oval of the su rface layer of enam el in washing or 5 seconds directly into an aspirator tip.
ord er to im p rove d iscolorations that are limited to the ou ter Repeat until the stain is reduced, up to a maximum o
enam el layer. It is not su itable for d eep enam el or d entine 10 × 5 second applications per tooth. Any improvement
d iscoloration. One of the m ost reliable m ethod s that has that is going to occur will have done so by this time.
been u sed extensively since 1986 is the hyd rochloric acid 6. Apply the f uoride drops to the teeth or 3 minutes.
(H Cl)-pu m ice m icroabrasion techniqu e. It is achieved by a
7. Remove the rubber dam.
combination of abrasion and erosion – the term ‘abrosion’
8. Polish the teeth with the nest sandpaper discs.
is som etim es u sed . In the clinical technique that w ill be
d escribed , no m ore than 100 µm of enam el is rem oved . Once 9. Polish the teeth with f uoridated toothpaste or 1
comp leted the p roced u re shou ld not be rep eated again in minute.
the fu ture. Too m u ch enamel rem oval is p otentially d am ag- 10. Review in 1 month or vitality tests and clinical
ing to the p ulp , and cosm etically the u nd erlying d entine photographs (Fig. 36.2).
colou r w ill becom e m ore evid ent. 11. Review biannually, checking pulpal status.
Ind ications: Critical analysis of the effectiveness of the techniqu e
• Fluorosis. shou ld not be m ad e im m ed iately but d elayed for at least 1
• Idiopathic speckling. m onth, as the appearance of the teeth w ill continu e to
• Post-orthodontic treatment demineralization. im prove over this tim e. Exp erience has show n that brow n
m ottling is rem oved m ore easily than w hite, bu t even w here
• Prior to veneer placement or well-demarcated stains.
w hite m ottling is incom p letely rem oved it nevertheless
• White/brown sur ace staining, e.g. secondary to primary
becomes less percep tible. This phenom enon has been attrib-
predecessor in ection or trauma (Turner’s teeth).
uted to the relatively prismless layer of com p acted su rface
Materials: enamel p rod u ced by the ‘abrosion’ techniqu e, w hich alters
• Bicarbonate o soda/water. the op tical p rop erties of the tooth su rface.
• Copalite varnish/Vaseline. Long-term stu d ies of the techniqu e have found no asso-
• Fluoridated toothpaste. ciation w ith p u lpal d am age, increased caries su sceptibility
• Non-acidulated f uoride (f uoride drops). or signi cant p rolonged therm al sensitivity. Patient com pli-
• Pumice. ance and satisfaction is good , and any d issatisfaction is
• Rubber dam. usu ally d ue to inad equ ate pre-op erative explanation. The
techniqu e is easy to p erform for the op erator and p atient
• Rubber prophylaxis cup.
and is not tim e consum ing. Removal of any m ottled area is
• Sandpaper polishing discs.
perm anent and achieved w ith an insigni cant loss of surface
• 18% hydrochloric acid. enamel. Failu re to im p rove app earance by the H Cl-p u m ice
Technique: m icroabrasion techniqu e has lim ited harm fu l effects bu t
1. Per orm pre-operative vitality tests; take radiographs w ill be less d estru ctive than localized or fu ll-face com posite
and photographs. veneers.
2. Clean the teeth with pumice and water, wash and dry.
3. Isolate the teeth to be treated with rubber dam, and Key point
paint Copalite varnish around the necks o the dam
(alternatively place Vaseline around the necks o the Microabrasion:
teeth under the rubber dam). • Will improve sur ace de ects, e.g. uorosis.
4. Place a mixture o sodium bicarbonate and water on • Will not improve deeper de ects ( ull thickness o
the dam behind the teeth, as protection in case o enamel lesions), e.g. amelogenesis imper ecta.
spillage.
197 •
MOTTLEDTEETH 36
Fig. 36.2 show s the appearance of Sophie’s 1 1 1 m onth allow them to d iffu se rap id ly throu gh enam el and d entine
after H Cl-p u m ice m icroabrasion. and thus explains the transient pu lp al sensitivity occasion-
ally exp erienced w ith hom e bleaching system s.
■ Is vital bleaching lega l or children?
Pulpal histology w ith regard to these m aterials has not
Changes in EU legislation in 2012 perm it d ental bleaching been assessed , bu t no clinical signi cance has been attrib-
to be u nd ertaken for ad u lts. This gu id ance sp eci ed that the u ted to the changes seen w ith 35% hyd rogen peroxid e over
m axim u m concentration of 6% hyd rogen p eroxid e w as p er- 75 years of usage, except w here teeth have been overheated
m itted for d ental bleaching; 10% carbam id e p eroxid e breaks or trau m atized . By extrapolation, 3.4% hyd rogen peroxid e
d ow n to 3.4% hyd rogen p eroxid e. For child ren u p to the age in the hom e system s is therefore safe.
of 18 it is illegal. It w as only in 2014, that the General Dental Althou gh m ost carbamid e peroxid e m aterials contain
Council changed its gu id ance for d entists. Their statem ent trace am ounts of p hosphoric and citric acid s as stabilizers
id enti ed that d entists cou ld bleach child ren’s d iseased and p reservatives, no ind ication of etching or a signi cant
teeth w ithout fear of d iscip linary action. It is alw ays sensi- change in the su rface m orp hology of enam el has been
ble to check w ith you r d ental d efence organization on the d em onstrated by scanning electron m icroscopy analysis.
latest p osition. N o matter w hich treatment op tion you There w as early concern that bleaching solutions w ith a low
choose, you r d iscu ssions w ith the p arents need to be p H w ou ld cau se d em ineralization of enam el w hen the pH
record ed in the patient’s clinical record s and w ritten consent fell below the ‘critical’ p H of 5.2–5.8. H ow ever, no evid ence
taken. of this process has been noted to d ate in any clinical trials
■ Has blea ching o teeth any part to play in the treatment o or laboratory tests, and this m ay be d ue to the u rea (and
sur ace enamel discoloration? subsequ ently the am m onia) and carbon d ioxid e released on
d egrad ation of the carbam id e p eroxid e elevating the p H .
Yes, external vital bleaching has been u sed as an initial treat- There is an initial d ecrease in bond strength of enam el to
m ent for u orotic m ottling and as a second ary treatm ent for com p osite resin im m ed iately after hom e bleaching, bu t this
resid ual brow n staining after m icroabrasion. It is, how ever, retu rns to norm al w ithin 7 d ays. This effect has been attrib-
usually associated w ith the bleaching of the yellow d iscol- u ted to the resid ual oxygen in the bleached tooth surface,
oration of ageing – so-called ‘nightgu ard vital bleaching’. w hich inhibits p olym erization of the com posite resin. The
This technique involves the d aily p lacem ent of 10% car- hom e bleaching system s d o not affect the colou r of restora-
bam id e p eroxid e gel into a cu stom - tted tray of either the tive m aterials. Any perceived effect is p robably d ue to
up per or the low er arch. As the nam e su ggests it is carried super cial cleansing.
ou t by the p atient at hom e and is initially d one on a d aily Minor u lceration or irritation m ay occu r d u ring the
basis. initial treatm ent. It is im p ortant to check that the m ou th-
Materials: gu ard d oes not extend on to the gingivae and that the ed ges
• Upper impression and working model. of the gu ard are sm ooth. If u lceration persists, a d ecreased
• So t mouthguard avoiding the gingivae. exposu re time m ay be necessary. If there is still a problem
• 10% carbamide peroxide gel. then allergy is a possibility.
Techniqu e: The exact mechanism of bleaching is u nknow n. Theories
of oxid ation, photo-oxid ation and ion exchange have been
1. Take an alginate impression o the arch to be treated and
su ggested . Conversely, the cau se of red iscoloration is also
cast a working model in stone.
u nknow n. This m ay be a com bination of chem ical red u ction
2. Relieve the labial sur aces o the teeth by about 0.5 mm
of the oxid ation p rod ucts previou sly formed , m arginal
and make a so t, pull-down, vacuum- ormed splint as a
leakage of restorations allow ing ingress of bacterial and
mouthguard. The splint should be no more than 2 mm
chem ical byp rod ucts, and salivary or tissu e uid contam i-
in thickness and should not cover the gingivae. It is only
nation via p erm eable tooth stru ctu re.
a vehicle or the bleaching gel and not intended to
protect the gingivae. ■ Wha t are the indica tions or composite veneers?
3. Instruct the patient on how to f oss the teeth thoroughly. Com p osite veneers m ay be requ ired to m ask d ense w hite
Per orm a ull mouth prophylaxis and instruct them how u orotic p laqu es that are refractory to m icroabrasion and
to apply the gel into the mouthguard. bleaching. Most com p osite veneers p laced in child ren and
4. Note that the length o time the guard should be worn ad olescents are the d irect type fashioned at the chairsid e
depends on the product used. rather than the ind irect or laboratory-m ad e typ e.
5. Review about 2 weeks later to check that the patient is Before p roceed ing w ith any veneering techniqu e, the
not experiencing any sensitivity, and then at 6 weeks, by d ecision m u st be m ad e w hether to red u ce the thickness of
which time 80% o any colour change should have labial enam el before placing the veneer. Certain factors
occurred. Sensitivity occurs in almost all patients, and should be consid ered :
reducing the duration o mouthguard wear is e ective • Increased labiopalatal bulk makes it harder to
at minimizing symptoms. However, around 14% maintain good oral hygiene. This may be courting
may stop this technique owing to the sensitivity it disaster in the adolescent with a dubious oral hygiene
causes. technique.
Carbam id e p eroxid e gel (10%) breaks d ow n in the m ou th • Composite resin has a better bond strength to
into 3.4% hyd rogen p eroxid e and 7% urea. Both u rea and enamel when the sur ace layer o 200 300 µm is
hyd rogen p eroxid e have low m olecu lar w eights, w hich removed.
• 198
36 MOTTLEDTEETH
• I a tooth is very discoloured, some sort o reduction will instrument, then use a brush lubricated with un lled
be desirable, as a thicker layer o composite will be resin to ‘paddle’and smooth it into the desired shape.
required to mask the intense stain. Cure or 60 seconds gingivally, 60 seconds mesioincisally,
• I a tooth is already instanding or rotated, its appearance 60 seconds distoincisally, and 60 seconds rom the
can be enhanced by a thicker labial veneer. palatal aspect i incisal coverage has been used. Di erent
N ew generation, highly p olishable, hybrid com posite shades o composite can be combined to achieve good
resins can replace relatively large am ou nts of m issing tooth matches with adjacent teeth and a transition rom a
tissu e and be u sed in thin sections as a veneer. Com bina- relatively dark gingival area to a lighter more translucent
tions of shad es can be u sed to sim ulate natu ral colour gra- incisal region. For some darkly stained dentine, a thin
d ations and hu es. Within these system s, op aqu ers can be layer o composite opaquer is an alternative to removing
very u sefu l at blocking ou t the transm ission of the d iscol- urther enamel and dentine. This is placed as the rst
ou red enam el d efect throu gh the com p osite. layer beneath the incremental composite build-up o
Ind ications for com posite veneers: di erent composite shades.
• Discoloration. 7. Flick away the un lled resin holding the contour strip
• Enamel de ects. and remove the strip.
• Diastemata. 8. Finish the margins with diamond nishing burs and
interproximal strips and the labial sur ace with graded
• Malpositioned teeth.
sandpaper discs. Characterization should be added to
• Large restorations.
improve light ref ection properties.
Contraind ications:
The exact d esign of the com posite veneer w ill d ep end on
• Insu cient available enamel or bonding. each clinical case, bu t w ill u su ally be one of fou r types:
• Oral habits, e.g. woodwind musicians. intra-enam el or w ind ow preparation; incisal bevel; over-
■ How do you undertake a composite veneer? lapped incisal ed ge; or feathered incisal ed ge.
Tooth preparation w ill not u su ally expose d entine,
The m aterials required are: bu t this w ill be u navoid able in som e cases of localized hypo-
• Rubber dam/contoured matrix strips. plasia or caries. Sou nd d entine m ay need to be covered by
• Preparation and nishing burs. glass ionom er cem ent p rior to placem ent of the com posite
• New generation, highly polishable, hybrid composite veneer.
resin.
• So -Lex polishing discs and interproximal polishing strips. Primary resources and
Technique: recommended reading
1. Use a tapered diamond bur to reduce labial enamel by Bu rrow s S 2009 A review of the ef cacy of tooth bleaching. Dent
0.3 0.5 mm. Identi y the nish line at the gingival margin Up d ate 36:537–538, 541–544, 547–548.
and also mesially and distally just labial to the contact Bu rrow s S 2009 A review of the safety of tooth bleaching.
points. Dent Up d ate 36:604–606, 608–610, 612–614.
2. Clean the tooth with a slurry o pumice in water. Wash Kilp atrick N M, Bu rbrid ge LA 2012 Anom alies of tooth form ation
and dry and select the shade. and eru p tion. In: Welbu ry RR, Duggal MS, H osey MT (ed s),
3. Isolate the tooth either with rubber dam or a contoured Paed iatric Dentistry, 4th ed n. Oxford University Press, Oxford ,
p p . 174–198.
matrix strip. Hold the latter in place by applying un lled
resin to its gingival side against the gingiva and curing Poyser N J, Kelleher MG, Briggs PF 2004 Managing d iscoloured
non-vital teeth: the insid e/ ou tsid e bleaching techniqu e. Dent
or 10 seconds.
Up d ate 31:204–210, 213–214.
4. Etch the enamel or 60 seconds, wash and dry.
5. Where dentine is exposed, apply dentine primer.
6. Apply a thin layer o bonding resin to the labial sur ace For revision, see Mind Map 36,
and roughly shape it into all areas with a plastic page 257.
37
■ There are a signif cant number o syndromes o the head
and neck tha t mani est with missing teeth. Can you
name some?
Ectodermal dysplasia.
Cle t lip and/or palate.
Down syndrome.
Chondro-ectodermal dysplasia (Ellis van Creveld syndrome).

Multiplemissingand Reiger syndrome.


Incontinentia pigmenti.

abnormallyshapedteeth
Oro- acial-digital syndrome (types I and II).
Ectodermal dysplasia d escribes a group of inherited d isor-
d ers involving ectod erm ally d erived stru ctures, i.e. hair,
teeth, nails, skin and sw eat gland s. The m ost com m on form
is the hyp ohid rotic X-linked form . The usu al p resentation is
a male child w ith:
Multiple congenital absence o teeth.
CASE1 Fine, sparse hair with sha t abnormalities.
Dry skin.
SUMMARY Frontal bossing.
Ellen is almost 11 years old. She is very concerned Maxillary hypoplasia.
by the gaps between her upper and lower ront
Thin lips showing little vermilion margin.
teeth. She is hoping to become an actress. She is a
Heterozygous emales can be identi ed dentally with
new patient to your surgery and has been brought
microdontia and hypodontia.
by her mother. What are the causes o the problem,
Down syndrome affects 1 in 700 births, is com m oner in
and how may it be treated?
offsp ring of old er m others and is a resu lt of chrom osom al
translocation. Ap art from the morphological featu res char-
History acteristic of the synd rome, there is com m only d elayed eru p-
tion and generalized m icrod ontia, as w ell as hypod ontia of
Ellen is a regu lar attend er w ith no caries. H er oral hygiene
some teeth.
is excellent. There is no history of tooth extraction in either
The d iagnosis and m anagem ent of all d ental anom alies
the prim ary or perm anent d entitions.
is extrem ely im p ortant and shou ld not be d elayed . Genetic
consu ltation is often d esirable to not only con rm the d iag-
Medical history nosis bu t also to help p arents u nd erstand the risk of fu tu re
Ellen has no m ed ical p roblem s. She is an active girl w ho is offsp ring and generations being affected . Conversely, genet-
involved in orienteering w ith her fam ily as w ell as partici- icists m ay requ ire help from p aed iatric d entists to clarify a
p ating in the local am ateu r d ram atic society. d iagnosis.
■ What question do you need to a sk Ellen’s mother?
Is there a fam ily history of m issing teeth or gaps?
Key point
As you are sp eaking to Ellen’s m other you notice that she
has a m issing u p p er right lateral incisor and the canine Hypodontia:
tooth is p rom inent. She con rm s your qu estion that not only • Prevalence is 3.5 6.5% in permanent dentition
has she got a missing tooth bu t her brother and her nep hew (excepting third molar).
(son of her brother) have either m issing or very sm all u p p er • May be associated with a number o syndromes.
lateral incisors. • Requires interdisciplinary care.
■ How prevalent are missing teeth in the population?
0.1 0.9% in the primary dentition.
3.5 6.5% in the permanent dentition (discounting third
■ What a ctors would you consider important in the
management o dental anomalies?
molars).
More common in emales than males (1.4 4×). The most Reassurance o child and parent.
common teeth to be absent are the last teeth in each series Elimination o pain.
(i.e. lateral incisor, second premolar, third molar). The presence
Prevention o caries and periodontal disease.
o a conical (peg) tooth is requently associated with a
missing tooth on the opposite side o the arch. There is Genetic counselling.
requently a amily history. Restoration o aesthetics.
• 200
37 MULTIPLEMISSINGANDABNORMALLYSHAPEDTEETH
Provision o adequate unction. • Fixed appliance to close the upper median diastema;
Maintenance o vertical dimension o occlusion. retain with a palatal bonded retainer.
• Poor prognosis B B. As the spaces are o concern to Ellen,
Interdisciplinary ormulation o de nitive treatment plan.
consider extraction o B B and replacement with an
upper partial denture with prosthetic 2 2. Alternative
Examination approaches include the extraction o the B B and the
provision o resin retained cantilever bridges. Another
Extraoral short- to medium-term option is to build up the primary
Ellen had a norm al facial ap p earance w ith a Class 1 skeletal lateral incisors with composite.
pattern. • In adulthood there will be the option o implants to
replace the missing maxillary and mandibular teeth.
Intraoral Hence the importance o interdisciplinary planning to
ensure short-term solutions do not compromise
The follow ing teeth w ere present:
long-term plans.
6E D C B1 1 BC D E 6
D D w ere m obile. The other
6E 321 234E6
prim ary teeth w ere rm . The p rim ary m olars w ere
infraocclu d ed . CASE2
■ Wha t special investigations are required?
SUMMARY
A d ental p anoram ic tom ogram (Fig. 37.1) to check on the
Cameron was 3 years old when his mother brought
presence/ absence of other p erm anent teeth.
him to the surgery because she was concerned that
■ Wha t is visible rom the radiograph?
two o his teeth were joined together (Fig. 37.2).
87532 234578 What is the cause, and how may it be treated?
Absence of: .
8754 1 57

■ Wha t is the condition known as? Medical and dental history


It is term ed severe hyp od ontia. This is d e ned as having six Cam eron is an active child w ith no m ed ical p roblem s. H e
or more p erm anent teeth m issing, not inclu d ing the m issing has no caries.
w isd om teeth.
■ What can you see in Fig. 37.2?
■ Wha t would you do?
The crow ns of the u p p er central and lateral p rim ary incisors
Take im p ressions and a w ax registration for stu d y m od els. are joined together.
Arrange a joint consultation w ith paed iatric d ental/ A nu m ber of d ifferent term s have been u sed to d escribe
orthod ontic/ restorative colleagu es. Frequ ently these inter- the p rocess of the form ation of d ou ble teeth either on
d iscip linary team s d iscu ss the cases before they see the the p rim ary or perm anent d entitions: fu sion, gem ination,
patient. Taking facial and intraoral view s as w ell as relevant d ichotom y, synod ontia, schizod ontia and connation. The
and justi able rad iograp hs w ill all help at this p lanning m od e of d evelop m ent given in old er textbooks for the d if-
m eeting. ferent nam es are unclear and unproven. The neu tral term
■ What treatment is likely to be required? ‘d ou ble teeth’ is not contentious and d escribes accu rately
w hat the tooth looks like clinically.
• Attempt to retain the primary molars or as long as
possible. ■ How prevalent do you think double teeth are?
• Management o in raoccluded primary molars (see 0.5 1.6% in primary dentition.
Chapter 8).
0.1 0.2% in permanent teeth.
• Assess mesiodistal width 11. Probable addition o
No clear mendelian trait established.
composite to distal aspect o these teeth to achieve
mesiodistal width o 8.5 9 mm.

Fig. 37.1 Severe hypodontia. ( romMillett andWelbury(2000) with permission). Fig. 37.2 Double incisor teeth.
201 •
MULTIPLEMISSINGANDABNORMALLYSHAPEDTEETH 37
The clinical m anifestation of the anom aly m ay vary con-
sid erably from a m inor notch in the incisal ed ge of an abnor-
m ally w id e incisor crow n to the ap p earance of tw o sep arate
crow ns. There m ay be hard tissu e continu ity betw een either
the crow ns or the roots of the tw o elements or betw een both.
Sim ilarly there m ay be one u ni ed p u lp cham ber and rad ic-
ular p u lp or sep arate ones.
In the prim ary d entition, d ou ble teeth m ore com m only
occu r in the labial segm ents of the arches and m ost fre-
qu ently in the m and ible. Double permanent teeth can occu r
anyw here in the arch and frequ ently involve the incisor
teeth.
■ What are the most important clinical aspects o a double
tooth in the prima ry dentition?
Fig. 37.3 Talon cusp.
Caries m ay occu r in the ‘join’ betw een the tw o coronal ele-
m ents if it is not easily accessible to a toothbru sh. This risk
can be red u ced by ssu re sealing the groove if there is food ■ What other types o crown abnormalities do you know?
stagnation, staining or enam el d ecalci cation.
The p resence of a nu m erical abnorm ality of the perm a- Accessory cusps.
nent d entition. Cou nting a d ou ble tooth as one u nit in the Invaginations.
primary d entition, w ith relation to the total nu mber of teeth
Evaginations.
present, m ay be help fu l in p red icting the typ e of nu m erical
abnorm ality of the p erm anent d entition. H ypod ontia is Ad d itional p rim ary cu sp s are seen on the m esiobu ccal
usually follow ed by m issing p ermanent teeth. A norm al aspect of maxillary rst m olars and the m esiop alatal asp ect
number of teeth in the p rim ary d entition is often associated of maxillary second m olars.
w ith p erm anent su p ernu m eraries. The overall frequency of The com m onest ad d itional cusp in perm anent teeth is
perm anent nu m erical abnorm alities follow ing prim ary often on the lingual cingulu m , m ore com m only in the
d ou ble teeth is betw een 30% and 50% in Caucasians and is m axilla than the m and ible. The nam e ‘talon cu sp’ is often
75% in the Jap anese p op u lation. given to the ad d itional cu sp (Fig. 37.3). The cu sp is com -
Rad iograp hy is therefore im p ortant at an ap p rop riate posed of enam el, d entine and a horn of p ulp tissu e. Talons
time, so the p arent can be ad vised of fu tu re treatm ent and in maxillary anterior teeth su ch as that show n in Fig. 37.3
prognosis and a treatm ent p lan formu lated . Extraction of can cau se a nu m ber of p roblem s:
d ou ble p rim ary teeth m ay be necessary if p hysiological Appearance.
root resorp tion is signi cantly retard ed . Su rgical rem oval of Occlusal inter erence.
su pernu m erary teeth m ay be requ ired to facilitate eru p tion Caries in the deep grooves between the cusp and the tooth.
of norm al u nits. Often treatm ent p lanning is m ost ap prop ri-
ately u nd ertaken w ithin an interd iscip linary team . The com m onest accessory cu sp in the perm anent d enti-
tion is the m esiopalatal sited tu bercle of Carabelli on the
m axillary rst m olar (10–60%).
Key point ■ Wha t are the treatment options or a talon cusp on a
Double primary teeth may: maxillary tooth?
• Be associated with numerical abnormalities o the Selective grinding o cusp to encourage obliteration o the
permanent dentition. pulp horn by secondary dentine.
• Be a caries risk. Aseptic removal o the cusp under rubber dam ollowed by a
• Not undergo normal physiological resorption. limited pulpotomy procedure (immature or mature root).
Aseptic removal o cusp and one-stage endodontic treatment
(mature root).
■ What are the important actors tha t will dictate whether Failure of a pu lp otomy techniqu e in an im m atu re tooth
you retain or extract double permanent teeth? w ill resu lt in the need for ind u ced ap ical closu re w ith
Space in the arch. Mineral Trioxid e Aggregate. Many clinicians favou r w aiting
u ntil fu ll root form ation has occu rred p rior to rem oval of
Aesthetics.
the cu sp. Invaginations or invagination of the enam el ep i-
Morphology o pulp chambers and roots. thelium into the d ental papilla of the u nd erlying tooth germ
If the coronal parts of the d ou ble teeth are joined bu t the has been d escribed as ‘d ens in d ente’, ‘gestant com posite
roots are sep arate, then it m ay be feasible to d ivid e the od ontome’ and ‘d ilated com posite od ontom e’. The correct
crow n and extract one p ortion of the crow n and its root, d escriptive term is ‘d ens invaginatus’ or ‘invaginated tooth’.
thereby retaining the other portion of the crow n w ith its The anom aly m ay vary clinically from a d eep cingulum pit
root. The retained p ortion w ill requ ire root canal treatm ent in a tooth of norm al form (Fig. 37.4) to a tooth w ith grossly
and su bsequ ent coronal restoration. d istorted crow n and root. Invaginated teeth are relatively
• 202
37 MULTIPLEMISSINGANDABNORMALLYSHAPEDTEETH
Evaginations consist of enam el, d entine and a pu lp al
extension. Evagination shou ld be su sp ected if a prem olar
w ithou t caries d evelop s a p eriap ical lesion shortly after
eruption.
Treatm ent is the same as ‘talon cu sp’.
■ What abnormalities o root orm do you know?
Taurodontism.
Accessory roots.
Pyramidal roots.
Tau rod ontism (bull-like teeth) ap plies to m u ltirooted
teeth in w hich the bod y of the tooth is enlarged coronoapi-
cally at the exp ense of the roots. Rad iographs show appar-
ent enlargem ent of the coronal p u lp cham ber, u su ally
extend ing below the level of the alveolar crest before root
d ivision. The norm al constriction at the level of the am el-
ocem ental ju nction is frequ ently absent in affected teeth.
The cond ition is uncom m on in p rim ary teeth but m ay occu r
in perm anent m olars in 6% of the p op u lation.
Fig. 37.4 Dens in dente o lateral incisor. ■ What conditions ma y taurodontism be associated with?
Amelogenesis imper ecta.

com m on, occurring in 1–5% of p erm anent teeth. Prim ary Trichodento-osseous syndrome.
invaginations are rare. Ectodermal dysplasia with hypodontia.
The m ain p roblem w ith invaginations is infection. The
Ellis van Creveld syndrome.
enam el lining of the invagination is either incom p lete or
very thin and easily breached , resu lting in d entinal caries Achondroplasia.
that qu ickly progresses to involve the pu lp cau sing a rapid ly Kline elter syndrome.
spread ing infection presenting w ith acu te facial cellu litis or Accessory roots can occu r in alm ost any tooth. Rarely
acu te d entoalveolar abscess. Rarely a large invaginated they m ay be d ue to early traum a to a form ing root. The
tooth m ay cause im paction and non-eru ption of an ad jacent m ajority are likely to be d u e to genetic factors that rem ain
tooth. to be speci ed .
There is an association betw een invaginated teeth and Pyram id al roots d escribe the red uction in root nu m ber in
supernu m erary teeth. A fu ll rad iograp hic exam ination is m ultirooted teeth. Any molar tooth m ay be affected .
ju sti ed if an invaginated tooth is id enti ed .
Invaginations in crow ns of norm al m orphology should Primary resources and
be sealed prop hylactically as soon as p ossible after eru ption.
If these ‘high’ invaginations d o become pu lpally involved recommended reading
then root treatm ent is p ossible, becau se the invaginated H olid ay R, Lu sh N , Chap ple J et al 2014 Aesthetics and fu nction
portion is accessible and can be rem oved w ith a crow n bu r p art 2: m anagem ent. Dent Upd ate 41:891–898.
allow ing access to the norm al root canal. All other invagina- Lu sh N , H ollid ay R, Chap ple J et al 2014 H ypod ontia: aesthetics
tions u su ally requ ire extraction ow ing to their com p lex root and fu nctions p art 1: aetiology and the p roblem s. Dent Up d ate
41:811–815.
m orp hology.
Millett D, Welbu ry R 2000 Orthod ontics and Paed iatric Dentistry:
‘Evaginated teeth’ or ‘d ens evaginatus’ or ‘tu bercu lated
Colou r Gu id e. Chu rchill Livingstone, Ed inbu rgh.
teeth’ occu rs com m only as a conical tu bercu lar p rojection
Tahm assebi JF, Day PF, Tou m ba KJ et al 2003 Paed iatric d entistry
arising from the occlu sal su rface of the central ssu re or the
in the new m illenniu m : 6. Dental anom alies in child ren. Dent
lingual p lane of the bu ccal cu sp . Prem olar teeth are the com -
Up d ate 30:534–540.
m onest teeth affected , w ith p erm anent m olars and canines
less com m only affected . The cond ition is m ainly seen in
people of Mongolian race (1–4%), althou gh rarely it is seen For revision, see Mind Map 37,
in Cau casians. page 258.
38
either hypop lasia, d ue to d e cient m atrix prod u ction, or
hypom ineralization, from im perfect m ineralization of the
m atrix proteins. In hyp op lasia the enam el m ay be thin,
grooved or p itted , w hereas in hyp om ineralization it m ay
ap pear m ottled bu t of norm al thickness. The com p lete range
of cau ses of d evelopm ental abnorm alities of enam el are
show n in Box 38.1.
Enam el d efects of genetic origin m ay occu r either as a
p henom enon p rim arily involving the enam el, w ith p ossible

Amelogenesis second ary effects in other d ental tissues (su ch as d elayed


eru p tion, anterior op en bite, id op athetic resorption) and
craniofacial stru ctures (eye or hearing anom alies), or as

imper ecta a com p onent of a m ore com p lex synd rom e in w hich d efec-
tive enam el is only one of a num ber of m ore generalized
abnorm alities.

Medical history
Mark has no m ed ical p roblem s. H e is d oing very w ell at
school and is a keen basketball player.
SUMMARY
Mark is 10 years old. He and his parents are Examination
concerned because his teeth seem rough. They also
Intraoral exam ination revealed that all the su rfaces of all the
stain easily. What is the cause o these problems?
eru p ted p erm anent teeth are affected by a rou ghness or
What treatment is possible? ‘p itting’ (Fig. 38.1). The second prim ary m olar teeth that are
■ Wha t ca n you see in Fig. 38.1? still p resent have a sim ilar roughness that, although not as
evid ent on visu al exam ination, w as obviou s on tactile exam -
All erup ted teeth have pitted hypoplastic enam el.
ination w ith a probe. There w as no tooth w ear.
■ Why is this pattern o enamel hypopla sia unlikely to be
History caused by systemic (chronological) in uences?
Mark’s m other says that she noticed w hen the teeth rst Primary and permanent teeth all a ected.
erupted that they w ere rough. They picked up stain and w ere
d if cult to clean. What key questions do you need to ask? All the enamel sur ace o the teeth is a ected.

■ Was there any systemic illness rom birth to early Positive amily history.
childhood? Am elogenesis im p erfecta (AI) occu rs as a resu lt of gene
m u tations that follow au tosom al-d om inant, au tosom al-
N o.
recessive or X-linked patterns of inheritance. The prevalence
■ Were the primary teeth similarly a ected? varies arou nd the w orld w ith rates of 1 in 718 in northern
There w as a slight rou ghening of the prim ary teeth, althou gh Sw ed en and 1 in 14 000 in Michigan, USA. A global qu oted
not as bad ly affected as the p erm anent teeth. gu re is often 1 in 10 000.
■ Is anyone else in the amily similarly a ected?
Mark’s father and his cou sin (father ’s brother ’s son) have Box 38.1 Di erential diagnosis o enamel de ects
similar rou ghness of their teeth. After obtaining this history General actors
it is m ost likely that Mark has an inherited enam el d efect • Genetic:
involving enam el – am elogenesis im p erfecta. H ow ever, • Primarily involving enamel – amelogenesis imper ecta.
d efective enam el form ation m ay be cau sed by genetic or • Associated with generalized de ects.
environm ental factors. The d efective enam el w ill exhibit
• Systemic (chronological):
• Nutritional defciencies.
• Metabolic or biochemical disorder.
• Toxic substances.
• In ectious illnesses: pre-natal; peri-natal; neonatal; in ancy; earlychildhood
(see Chapter 34).
• Excess Fluoride ingestion (see Chapter 36).
• Idiopathic.
Local actors
• Trauma.
• In ection.
Fig. 38.1 Hypoplastic amelogenesis imper ecta.
• 204
38 AMELOGENESISIMPERFECTA
Table 38.1 Treatment modalities or amelogenesis imper ecta
Restoration Aesthetics
Primarydentition: Adhesive restorations Minimal intervention
0–5 years Stainless steel crowns (SSCs) Composite veneers
especiallyon E’s
Mixed dentition: Adhesive restorations/SSCs on Composite veneers
6–16 years primarymolars
SSCs/adhesive castings on
permanent molars
Permanent dentition: Adhesive castings on premolars Porcelain veneers
Fig. 38.2 Hypomineralized amelogenesis imper ecta. 16+ years Full mouth rehabilitation ± Full crowns
crown lengthening

■ Wha t are the main types o AI?


• Hypoplastic.
Re erral or genetic testing In the UK testing for d ifferent
• Hypomineralized: hypocalci ed or hypomature. genetic types of AI is now available for N H S patients.
• Mixed pattern o both types.
Althou gh historically, p hysiological categories of d iffer-
ent typ es of AI w ere d escribed , there is grow ing realization
Treatment
the classi cations related to the genetic m utation are m ore The treatm ent of both am elogenesis and d entinogenesis
approp riate. Consequ ently, rad iograp hic and clinical phe- im perfecta requires early d iagnosis in ord er to im prove the
notyp es shou ld sim p ly d escribe their ap p earance (for long-term p rognosis of teeth. Parents need to be ed u cated
exam p le, hypop lastic, hyp om ineralized ). A good d iagnostic as to the im p lications of the cond ition, and prevention (d iet
gu id e is to exam ine rad iographs for u nerupted teeth to cou nselling, uorid e su pp lementation, oral hygiene instru c-
id entify the qu antity of enam el visible and the d ifferentia- tion (OH I)) is a cru cial elem ent in the su ccess of any restora-
tion betw een the enam el and d entine (e.g. the qu ality of tive treatm ent.
m ineralization). Affected teeth, once eru pted into the m ou th, There are fou r m ain clinical problem s associated w ith
can qu ickly p ick u p p igmentation (for exam p le, change inherited enam el and d entine d efects:
colou r from cream y to brow n), and the enam el su rface can • Poor aesthetics.
break d ow n (giving the im p ression of being hypoplastic • Chipping and attrition o the enamel.
w hen it w as originally hyp om ineralized ). With a better
• Exposure and attrition o the dentine causing sensitivity.
u nd erstand ing of the genetic m u tations, the p athw ays
• Poor oral hygiene, gingivitis, calculus and caries.
through w hich these genes in u ence enam el form ation is
slow ly being u nd erstood . Long-term treatm ent need s – this m eans that treatm ent
Genetic enam el d efects can be associated w ith general- shou ld aim to m inim ize treatm ent in child hood to red u ce
ized d isord ers in a nu m ber of u ncom mon or rare genetically bu rn out and p otential d ental anxiety.
d eterm ined d iseases and clinical synd rom es. These d iseases Althou gh it is im p ossible to d raw up a d e nitive treat-
and com plex synd rom es inclu d e ep id erm olysis bu llosa, m ent p lan for all cases, it is p ossible to d e ne the p rinciples
tuberous sclerosis, pseudohypoparathyroidism, trichodento- of treatm ent p lanning for this group of patients. It is im por-
osseous synd rom e, ocu lod ento-osseou s d ysp lasia, vitam in tant to realize that not all child ren w ith am elogenesis im p er-
D-d ep end ent rickets, amelo-cerebrohyp ohid rotic synd rom e, fecta or d entinogenesis im perfecta are affected equally.
amelo-onychohypohid rotic synd rom e and som e types of Many w ill not have m arked tooth w ear or sym p tom s and
m u cop olysaccharid osis. w ill not requ ire ad vanced intervention. Table 38.1 d escribes
Mark has a rou gh, hypop lastic typ e of AI. An exam ple of the p rincip les of treatm ent in term s of the age of the child /
a hyp om inerliazed form of AI is show n in Fig. 38.2. ad olescent and w ith regard to the three aspects of care:
prevention, restoration and aesthetics.
Clinicians m u st be very aw are of the im p act that AI
Investigations can have on the affected child and the w id er fam ily. The
poor app earance of som e typ es of AI, especially those w ith
■ Wha t investigations are necessa ry? brow n staining, can have a signi cant im pact on child ren,
especially w hen starting at a new school and m aking new
Dental panoramic tomogram This w ill con rm the p res-
friend s. The im pact is not lim ited to child ren bu t also
ence of all the perm anent d entition. In ad d ition, it w ill d iag-
affects their fam ilies w ho need to take tim e off w ork to
nose any tau rod ontism w hich is associated w ith som e typ es
bring them to appointm ents and m ay experience possible
of AI.
guilt associated w ith genetic cond itions. For a few ad u lts
Family examination If it is possible to examine Mark’s w ith AI, the negative im pact of their teeth has led them to
father ’s and his cou sin’s teeth, this w ill help to con rm you r conclu d e that they d o not w ant child ren in case they are also
d iagnosis. affected .
205 •
AMELOGENESISIMPERFECTA 38
be rep laced in ad olescence/ early ad ulthood by a porcelain-
Key point bond ed full crow n (Table 38.1). A rand om ized controlled
Main treatment aims or dental anomalies: trial rep orting on the bene ts and longevity of tw o d ifferent
• To alleviate symptoms. types of conventional crow ns has recently been pu blished
• To maintain/restore occlusal height. show ing good ef cacy for tw o types of crow n placed in
ad olescence and early ad u lthood .
• To improve aesthetics.
• Try to maintain a positive dental attitude, prevent and
minimize the likelihood o dental anxiety.
Primary resources and
recommended reading
Am erican Acad em y of Ped iatric Dentistry 2013 Gu id eline on
d ental m anagem ent of heritable d ental d evelopm ental
anom alies. Reference Manu al 37 (6):266–271.
Mark’s m ajor concern w as the staining and rou ghness McDonald S, Arku tu N , Malik K et al 2012 Managing the
of his front teeth. Fortu nately there w as no w ear of his p aed iatric p atient w ith am elogenesis im p erfecta. Br Dent J 212
posterior teeth and his p roblem, therefore, w as solely a (9):425–428.
cosmetic one. The pitting or hypop lasia on the upp er and Parekh S, Alm ehateb M, Cu nningham SJ 2014 H ow d o child ren
low er incisors can be m asked by a thin, d irectly applied w ith am elogenesis im p erfecta feel abou t their teeth? Int J
comp osite veneer. This shou ld be extend ed to inclu d e the Paed iatr Dent 24 (5):326–335.
canines and rst p rem olars w hen the arches are com p lete. Pou sette Lu nd gren G, Morling Vestlu nd G, Tru lsson M et al 2015
On occasions, if the sm ile is a very ‘w id e’ one, it m ay be A rand om ized controlled trial of crow n therap y in young
ind ivid u als w ith am elogenesis im p erfecta. J Dent Res 94
necessary to inclu d e second p rem olars. Com posite veneers
(8):1041–1047.
can be rep laced by p orcelain veneers w hen the gingival
Seow WK 2014 Develop m ental d efects of enam el and d entine:
contou r has m atu red in late ad olescence or early tw enties.
challenges for basic science research and clinical m anagem ent.
If the am elogenesis had been of the hyp om ineralized
Au st Dent J 59 (Su p p l. 1):143–154.
variety w ith m ore d estru ction of enam el, it m ay have been
necessary to consid er fu ll com p osite crow ns for aesthetics
in the anterior teeth and either stainless steel crow ns or For revision, see Mind Map 38,
ad hesive castings on the p osterior teeth. These w ill need to page 259.
39
■ What investiga tions do you need to do to conf rm your
suspicions?
Dental panoramic tomogram If this is DI, the d ental pano-
ramic rad iograph w ill p robably show the follow ing:
• Bulbous crowns with pronounced cervical constriction.
• Shortened roots.
• Progressive pulp chamber and canal obliteration
(Fig. 39.2 (di erent case)).

Dentinogenesis • Spontaneous periapical abscess ormation.

imper ecta

SUMMARY
Siobhan is 9 years old. She and her parents are
concerned because her permanent teeth are darker
than normal and she is getting teased at school.
What is the cause o the discoloration? How would A
you treat it?

History
Siobhan’s m other says she noticed that w hen the perm anent
teeth eru pted they looked d arker. Siobhan is very u nhapp y
at school and refu ses to sm ile for any photograp hs. When
she talks she has the habit of covering her mouth w ith a
hand so it is imp ossible to see her teeth. What can you see
in Fig. 39.1A? What key qu estions d o you need to ask?
■ Was there any systemic illness rom birth until early
childhood?
N o. Siobhan had no illness.
■ Were the primary teeth similarly a ected? B
The p rim ary teeth eru pted normally but very qu ickly
Fig. 39.1 (A) Dentinogenesis imper ecta. (B) Dentinogenesis
chip ped aw ay, becom ing w orn to gu m level (see Fig. 39.1B).
imper ecta.
■ Is anyone else in the a mily similarly a ected?
Siobhan’s brother and her father have a problem w ith their
teeth. Siobhan’s brother is 14, and he has need ed crow ns on
his back teeth and veneers on his front teeth. Siobhan’s
father need ed a lot of treatm ent w hen he w as you nger and
has had som e teeth crow ned . Many of his p osterior teeth,
how ever, w ere extracted .
Even before you have exam ined the m ou th, the history
suggests that Siobhan has an inherited d efect. Fig. 39.1A
con rm s you r su sp icion that this is d entinogenesis imp er-
fecta (DI).
■ Why is this DI and not amelogenesis imper ecta (AI)?
The teeth are translucent.
The enam el is poorly ad herent to the u nd erlying d entine
and easily chip s and w ears. The rem aining p rim ary canines
and m olars in Figs 39.1A and B are w orn to gingival level Fig. 39.2 Obliteration o root canals and pulp chambers in
and have a translu cent op alescent ap p earance. dentinogenesis imper ecta.
207 •
DENTINOGENESISIMPERFECTA 39
• Shell teeth. This is rare and seen in the primary dentition.
Box 39.1 Hereditary dentine de ects
The pulp remains large and the thin enamel and dentine
Limited to the dentine
rapidly ragments to cause pulpal in ection.
• Dentinogenesis imper ecta type II (hereditaryopalescent dentine). • DI type III (brandy wine type). This was rst described in
• Dentine dysplasia type I (radicular dentine dysplasia). Maryland, USA, and has been traced back to East Anglia
• Dentine dysplasia type II (coronal dentine dysplasia). in England. It was apparently taken to the USA by one o
• Fibrous dysplasia o dentine. the sailors who accompanied the Pilgrim Fathers to
Associated with generalized disorder Maryland. Type III de ect has been linked to the same
locus on chromosome 4q21 as DI type II.
• Osteogenesis imper ecta (dentinogenesis imper ecta type I).
• Ehlers-Danlos syndrome. DI type I associated with osteogenesis imper ecta Osteo-
• Brachioskeletogenital syndrome. genesis im perfecta is a grou p of connective tissu e d isord ers
• Vitamin Dresistant rickets. involving inherited abnorm alities of typ e I collagen.
Increased bone fragility is only one aspect of the cond ition
• Vitamin Ddependent rickets.
w hich m ay includ e lax joints, blue sclerae, opalescent teeth,
• Hypophosphatasia.
hearing loss and a variable d egree of bone d eform ity. The
inheritance p attern is either au tosom al recessive or d om i-
nant. The recessive form is often lethal arou nd birth.
Opalescent teeth are only rarely seen in surviving reces-
sive types. They are com m only a featu re of the d om inant
Family examination variety w ith accom p anying bone fragility, bone d eform ity
• Family examination o a ected members. and blu e sclerae. Ow ing to the bone fragility, frequently
• Dentine de ects like those o enamel may be subdivided child ren w ith osteogenesis im p erfecta are treated w ith
by cause into two main groups based on whether they bisphosp honates. In ad u lts bisp hosphonates, esp ecially
are o genetic or environmental origin. those given intravenou sly, are associated w ith osteonecro-
• Dentine anomalies that are genetically determined may sis. Although no cases of osteonecrosis have been reported
appear to be limited to the dentition or orm part o a in child ren, great care, thought and interd isciplinary
more complex generalized disorder (Box 39.1). w orking w ith m ed ical colleagu es is requ ired w hen m anag-
ing these com p lex child ren (see Chapter 28).
The m ost w ell d ocu m ented hered itary d entine d efects
The prim ary teeth in DI type I resem ble exactly those in
are DI type II (hered itary op alescent d entine), w hich only
DI typ e II. H ow ever, in the perm anent d entition the d efect
affects teeth, and DI typ e I, in w hich abnorm alities of teeth
is extrem ely variable. In m any cases the u pp er anterior teeth
are associated w ith osteogenesis im p erfecta.
m ay have a norm al colour and app earance, w hereas the
DI type II Both d entitions are u su ally affected . The severity low er incisors and canines are opalescent, d iscoloured
of the d efect varies consid erably betw een fam ilies and bluish-brow n and w ear at the incisal ed ges. In m ost cases
w ithin fam ilies. Prim ary teeth tend to be m ore severely the enam el d oes not chip aw ay from the und erlying d entine
affected than perm anent teeth, and the later form ing p er- as read ily as in typ e II.
m anent teeth m ay be the least affected . Enam el tend s to chip Rad iographic appearances are as alread y d escribed w ith
aw ay from the u nd erlying am elod entinal ju nction (ADJ), the excep tion that u pper teeth m ay retain their pu lp spaces
exposing the abnorm ally soft d entine that u nd ergoes rap id long after those in the low er jaw. H istological app earances
w ear. This is m ost m arked in the p rim ary d entition w here, are ind istingu ishable from type II.
w ithin 2 years, the crow ns m ay be w orn to the gingival Environm entally d eterm ined d entine d efects d o exist bu t
m argin and ap p ear as am ber-colou red rem nants (Fig. are less w ell d ocu m ented than correspond ing anom alies of
39.1B), w hich m ay be frequ ently infected and abscessed . In enam el: trau m a, nu tritional d e ciencies (m inerals, proteins
the p erm anent d entition, follow ing eru p tion, the enam el and vitam ins) and d ru gs (tetracycline, chem otherapeu tic
m ay look reasonably norm al, bu t histological stu d ies have agents – cyclophospham id e) w ill likely prod u ce increased
show n hyp om ineralized areas in ap p roxim ately one-third interglobular d entine, p red entine and osteoid .
of cases. Rad iograp hic signs are d escribed previou sly. H is-
tologically, the ADJ m ay ap p ear attened , and w hile the
su bad jacent p erip heral d entine m ay ap p roach normality, Key point
the rem aind er is grossly d isord ered w ith an am orp hous
Dentinogenesis imper ecta:
m atrix containing areas of interglobular calci cation, abnor-
• Occurs in 1 in 8000 o the population.
m ally shap ed and sized tu bu les and cellu lar inclu sions.
• May be associated with osteogenesis imper ecta.
■ Is DI more preva lent than AI?
Possibly, gu res of 1 in 8000 have been estim ated (AI
1:10 000).
■ Has DI got as many inheritance patterns as AI? Treatment
N o. Invariably it is au tosomal d om inant w ith m arked The m ain clinical problems associated w ith AI and DI
expressivity and good p enetrance. Tw o clinical variants of and the key points of treatm ent objectives are covered in
the cond ition have been d escribed : Chapter 38. The princip les of treatm ent for DI are the sam e
• 208
39 DENTINOGENESISIMPERFECTA
as those for AI w ith the excep tion that in the p ermanent ap propriate, and orthod ontic band s rather than brackets
d entition from age 16, crow n-lengthening p roced u res are w ill m inimize the risk of d am age to the abnorm al enam el.
m ore com m on in DI and the p rovision of overd entu res and The p roblem is tw ofold : there m ay be frequ ent bond failu re
fu ll d entu res is not u ncom m on. The role of imp lants in these d u ring active treatm ent or the enam el m ay be fu rther
patients has yet to be d e ned . d am aged d uring d ebond ing. Som e orthod ontists prefer to
Siobhan’s m ajor concern w as of the colour of her perm a- use band s even for anterior teeth, w hile others w ill use glass
nent incisors. There w as som e w ear of rst p erm anent ionom er cem ent as the bond ing agent in p reference to m ore
m olars. The rst p erm anent m olars w ere treated w ith conventional resin-based agents. In other instances, cos-
ad hesive castings w ith m icrom echanical retention to lu ting m etic restorative techniqu es (veneers and crow ns) m ay be
cem ent (Fig. 39.1B). The u p per and low er incisors w ere m ore ap p rop riate than orthod ontic treatm ent.
veneered w ith com p osite resin. This can be extend ed to
inclu d e the canines and p rem olars w hen the arches are com -
plete. The com p osite veneers can be rep laced by p orcelain
Primary resources and
veneers around the age of 18 years. recommended reading
You ng child ren w ith DI often pose the greatest p roblem s. Am erican Acad em y of Ped iatric Dentistry 2014/ 2015 Gu id eline
The p rim ary teeth und ergo su ch excessive w ear that they on d ental m anagem ent of heritable d ental d evelopm ental
becom e w orn d ow n to the gingival level and are unrestor- anom alies. Reference Manu al 36 (6):264–269.
able. Teeth affected by DI are also prone to sp ontaneous Barron MJ, McDonnell ST, Mackie I et al 2008 H ered itary d entine
d isord ers: d entinogenesis im p erfecta and d entine d ysplasia.
abscesses d u e to the p rogressive obliteration of the pu lp
Orp hanet J Rare Dis 20 (3):31.
cham bers. In these cases p u lp therapy is often u nsu ccessful
Dhaliw al H , McKaig S 2010 Dentinogenesis im p erfecta—clinical
and extraction of the affected teeth is necessary.
p resentation and m anagem ent. Dent Up d ate 37 (6):364–366,
Early consu ltation w ith an orthod ontist is ad visable in
369–371.
inherited abnorm alities of enam el and d entine in ord er to
keep the orthod ontic requ irem ents sim ple. Treatm ent for
these p atients is possible and in m any cases proceed s For revision, see Mind Map 39,
w ithou t p roblem s. The u se of rem ovable ap p liances, w here page 260.
40
su ch as titratable acid ity, the in uence of p laqu e pH and the
buffering capacity of saliva w ill all in u ence the erosive
p otential of a su bstrate. Fou r things, how ever, are clear w ith
erosive loss:
• It is worse i consumption is high.
• It is worse i consumption occurs at bedtime.
• It is worse i brushing occurs directly a ter consumption.
• It is worse i children undertake swishing or holding

Dental erosion History


habits when drinking.

Tom ’s only com p laint w as occasional sensitivity on his back


teeth as a result of the visible d entine.
■ Wha t is the best way to f nd out about Tom’s diet?
A 4-d ay w ritten d ietary history is the only w ay to accurately
elu cid ate constitu ents of the d iet that m ay be erosive.
■ Ca n the pattern o erosion caused by dietary constituents
SUMMARY be rela ted to the manner in which the substrate is
Tom is 9 years old. He is a new patient to your consumed?
practice. On examination you are concerned by the This is ind eed the case. ‘Frothing’ of a d rink betw een the
appearance o the occlusal sur aces o his lower u pper anterior teeth w ith its retention labially can lead to
primary molars. What has caused this, and how palatal, interproximal and labial erosion. Retention of a
may it be managed? d rink sp eci cally on one sid e of the m ou th can lead to
■ What do you see in Fig. 40.1? erosion on that sid e only.
■ You have covered Tom’s dietary history. Is your history now
There is erosion of the cu sp s of Tom ’s p rim ary m olars giving
complete, or a re there other questions you need to ask with
cu pp ing or p erim olysis of the cu sp s w ith loss of enam el and
relation to erosion?
visible u nd erlying d entine.
■ How would you def ne erosion? It is very im portant to consid er gastric acid as a cau se of
erosion, even in a you nger patient. The cond itions in chil-
An irreversible loss of tooth su bstance brou ght about by a d ren that are associated w ith chronic regu rgitation are
chem ical p rocess that d oes not involve bacterial action. show n in Box 40.2. The acid ity of the stom ach contents is
■ What oods and drinks have erosive potential? below pH 1.0 and therefore any regu rgitation or vom iting
is d am aging to the teeth.
See Box 40.1. While a w id e range of food and d rinks is
implicated in the p roblem , the bu lk of the d am age is d one ■ Wha t question would you ask to give you an indication that
by soft d rinks, especially carbonated d rinks, w hich are regurgitation was occurring?
increasingly available from vend ing m achines in schools ‘Do you ever have a bitter taste in your m ou th?’ There is a
and recreational facilities. All carbonated d rinks and fru it- group of patients w ho have gastro-oesophageal re ux
based d rinks have low ered pH valu es, bu t the d irect rela- d isease (GORD). This m ay be either sym p tomatic, in w hich
tionship betw een p H and erosion is u nclear. Other factors the ind ivid u al know s w hat provokes the re u x, or m ore
insid iously, asym ptom atic GORD, w here the p atient is
u naw are of the p roblem . The latter case is m ost likely
to occur at night w hen the horizontal sleeping p osition
m akes it m ore likely that acid w ill re u x throu gh the low er

Box 40.1 Foods and drinks with erosive potential


• Citrus ruits, e.g. lemons, oranges, grape ruits.
• Tart apples.
• Vinegar, pickles, ketchup and brown sauce.
• Yoghurt.
• All ruit juices, including resh juice and ruit-based squashes.
• Carbonated drinks, including low-calorie varieties, ‘sports drinks’and sparkling
mineral water.
• Vitamin Ctablets and iron preparations (some medication will cause a drymouth
therebyexacerbating other causes).
Fig. 40.1 Erosion o cusp tips o primary molars.
• 210
40 DENTALEROSION

Box 40.2 Conditions associated with chronic regurgitation ■ What advice would you give to Tom regarding his high
in children intake o f zzy drinks?
• Gastrooesophageal re ux. It is critically imp ortant w hen d ealing w ith child ren and
• Oesophageal stricture. ad olescents not to be too d ogm atic in your ad vice, and it is
• Chronic respiratorydisease, e.g. asthma. u nrealistic to expect youngsters w ho have been brought
• Disease o the liver/pancreas/biliary tree.
u p w ith a high intake of carbonated beverages to stop
altogether.
• Over eeding.
They shou ld be ad vised to erad icate betw een-m eal zzy
• Feeding problems/ ailure to thrive conditions. d rink consu m p tion bu t to have the zzy d rink w ith m eals
• Learning delay. and preferably to d rink it w ith a straw. The presence of food ,
• Anorexia. and the extra saliva that is generated at m ealtim es, w ill help
• Bulimia nervosa. neu tralize the acid ity. In ad d ition, a straw w ill d ep osit the
• Cyclic vomiting syndrome. m ajority of the carbonated beverage beyond the teeth.
Make su re that the betw een-m eal carbonated d rink is not
• Cerebral palsy.
su bstitu ted by som ething w ith a sim ilar erosive p otential,
• Rumination.
e.g. fresh fru it ju ice or a ju ice-based squash.
Milk and w ater are the m ost ap p rop riate betw een-m eal
d rinks. If either of these proves im possible, then an
oesop hageal sp hincter. In this case the qu estion about a extrem ely w ell-d ilu ted ‘no ad d ed su gar ’ squ ash can be
bitter taste in the m ou th shou ld have the suf x ‘w hen you accepted .
w ake u p ’. N o carbonated d rinks or fru it d rinks should be given last
■ What is the common pa ttern o erosive loss when there is thing at night.
chronic gastric regurgitation? Ad vocate the consu m p tion of a neu tral food im m ed i-
ately after a m eal, e.g. cheese.
Initially there is erosion of the palatal su rfaces of the up per
incisors, canines and prem olars. With time this extend s to
the occlu sal and buccal surfaces of the low er m olars and Management
prem olars.
The m ost im p ortant asp ect of the m anagem ent of Tom ’s
Whenever there is u nexp lained erosive loss, an eating
erosion w as early d iagnosis before there had been d am age
d isord er should be su sp ected . There are three su ch d isor-
to the p erm anent teeth, and su bsequently to establish the
d ers: anorexia nervosa; bulim ia nervosa; and ru m ination.
aetiology and elim inate the cau se.
The latter is a cond ition in w hich food is volu ntarily regu r-
gitated into the oral cavity and either expelled or sw allow ed
again.
Key point
■ Is there a specif c pattern o erosive loss in recurrent
vomiting? Management o erosion:
• Early diagnosis.
All tooth surfaces can be affected w ith the relative exception
of the lingu al su rfaces of the low er teeth, w hich are p ro- • Establish aetiology.
tected by the tongu e and the saliva from the sublingu al • Eliminate cause.
papillae. • Monitor or urther tooth sur ace loss.
■ Wha t would you do i you suspect, a ter questioning Tom
and his parents, that there may be asymptoma tic GORD?
Referral to a p aed iatrician w ith an interest in gastrointesti- ■ Tom only has occasional sensitivity. Wha t treatment, i a ny,
nal d isease w ou ld be ap p rop riate. The p aed iatrician w ill does he need?
seek to elim inate organic d isease and then attem p t to qu an-
Probably none. The follow ing w ou ld be realistic initially:
tify the p roblem . The latter m ay involve 24-hou r p H m oni-
Daily neutral sodium f uoride mouthwash (0.05%) to maximize
toring of the oesop hagu s, w ith p robes in the low er and
the resistance to remaining enamel and desensitize the
u pper oesop hagu s. An ad d itional p robe cou ld be ad d ed to
dentine.
an intraoral ap pliance to measu re m ou th p H . Med ical and /
or su rgical treatment m ay be requ ired to control GORD. High concentration sodium f uoride varnish (Duraphat) to be
Chronic regu rgitation can lead to scarring of the oesop ha- applied three to our times a year.
gu s and d ysp lastic change, and this is therefore an im p or- High f uoride toothpaste (2800 ppm or children aged
tant cond ition to d iagnose and treat. In su ch cases m ed ication 10 years and older, 5000 pmm or children aged 16 years
or su rgery are ind icated to p revent regu rgitation. and older).
Summary o Tom’s history There w as no evid ence of any If there is p rogressive sensitivity then the areas of enam el
gastrointestinal illness, bu t Tom d id consu m e a nu m ber of loss and d entine exp osu re cou ld be p rotected by an ad he-
zzy d rinks, especially betw een m eals and w hen he w as at sive restoration. In m any cases, if erosion is d iagnosed early
the local sp orts centre. In ad d ition to these, he rarely d rank then p reventive cou nselling and the above ad vice m ay be
w ater and m ilk. su f cient. It is a good id ea to take photographs and m ake
211 •
DENTALEROSION 40
stu d y casts of all p atients w ith signs of erosion or attrition
or abrasion to m onitor the rate of p rogression. In m ore
Key point
ad vanced cases than Tom ’s, as in Figs 40.2A and B, w here Treatment objectives or erosion:
there are signi cant sensitivity or cosm etic p roblem s, m ore • Resolve sensitivity.
active intervention is requ ired . Table 40.1 show s the m erits • Restore missing tooth sur ace.
of the d ifferent options available.
• Prevent urther tooth tissue loss.
• Maintain a balanced occlusion.

■ Erosion is only one element o tooth sur ace loss or wear.


What are the other elements?
Attrition: the wear o the tooth as a result o tooth-to-tooth
A contact.
Abrasion: physical wear o tooth substance produced by
something other than tooth-to-tooth contact.
In child ren, abrasion is usu ally d ue to overzealou s tooth-
brushing, w hich tend s to d evelop w ith increasing age.
The abnorm al bru shing techniqu e m u st be corrected before
signi cant tooth tissu e is rem oved and pu lpal exp osu re
occu rs. Attrition cau sed by norm al m astication is com m on,
B especially w ith the ageing p rim ary d entition. Alm ost all
prim ary teeth show signs of attrition by the tim e they
Fig. 40.2 (A) Signi cant erosive tooth sur ace loss o labial sur aces exfoliate.
o upper permanent incisors. (B) Signi cant erosive tooth sur ace
■ Wha t categories o patient exhibit more attrition
loss o palatal sur aces o upper permanent incisors.
than norma l?

Table 40.1 Treatment techniques or tooth sur ace loss Those w ith signi cant p arafu nctional activity, e.g. cerebral
palsy and other p hysical and d evelop m ental d isord ers w ith
Technique Advantages Disadvantages
intracranial abnorm alities. Controlling attritional w ear in
Cast metal Fabricated in thin section – Maybe cosmeticallyunacceptable due these p atients can be very d if cu lt. Som e d rugs act to try
(nickel/chrome requires only0.5 mmspace to the‘shine through’o metallicgrey
to red u ce su ch p arafu nctional activity, bu t even if this is
or gold) Veryaccurate ft possible Cannot be simplyrepaired or added to successful in the lim bs, there is often still resid u al oral
intraorally parafu nction. This is p robably d u e to the neu ronal sensitiv-
Verydurable ity of the m outh and the structu res w ithin it. For child ren
Suitable or posterior w ith p rofou nd neu rological cond itions w ho stru ggle to
restorations in para unction comm u nicate, grind ing is very com m on. If p arents p resent
Does not abrade opposing w ith an increase in frequ ency or d urations of grind ing, a
dentition thorou gh oral exam ination is essential to elim inate any
Composite: Least expensive Technicallydi cult or palatal veneers
intraoral p athology and / or p ain w hich m ay have led to this
direct activity.
Can be added to and repaired Limited control over occlusal and
intraorally interproximal contour ■ Wha t restorative materials are the most durable or
Aestheticallysuperior to cast Inadequate as a posterior restoration attritional wear a s a result o para unction?
metal Amalgam and stainless steel crow ns. Carefu l consid eration
Composite: Can be added to and repaired Requires more space – minimumo w hen using these m aterials is necessary, as occlu sal d erange-
indirect intraorally 1.0 mm m ent m ay exacerbate parafu ctional habits. One research
Aestheticallysuperior to cast Unproven durability pap er id enti ed higher levels of stainless steel crow n failu re
metal in child ren w ith d evelopm ental d isabilities, w hich they su g-
Control over occlusal contour gested m ay related to bru xist habits.
and vertical dimension
Porcelain Best aesthetics Potentiallyabrasive to opposing
dentition Primary resources and
Good abrasion resistance In erior marginal ft recommended reading
Well tolerated bygingival Verybrittle – has to be used in bulk Kilp atrick N M, Bu rbrid ge LA 2012 Anom alies of tooth form ation
tissues section and eru ption. In: Welbu ry RR, Du ggal MS, H osey MT (ed s),
Paed iatric Dentistry, 4th ed . Oxford University Press,
Hard to repair
Oxford .
• 212
40 DENTALEROSION
N g MW, Tate AR, N eed lem an H L et al 2001 The in u ence of Su rgeons. Available at: <http :/ / w w w.rcseng.ac.u k/ fd s/
m ed ical history on restorative p roced u re failu re rates follow ing p u blications-clinical-gu id elines/ clinical_gu id elines/ d ocu m ents/
d ental rehabilitation. Ped iatr Dent 23:487–490. d iagnosis-prevention-and -m anagem ent-of-d ental-erosion.
O’Su llivan E, Barry S, Milosevic A, 2013 Diagnosis, Prevention
and Managem ent of Dental Erosion. Lond on: Royal College of For revision, see Mind Map 40, page 261.
41
been p oor, but generally her control is now good w ith a
stable regim en. She is seen every 2 m onths by her d octor,
and she monitors her blood glu cose and u rinary glu cose at
hom e herself.

Dental history
Kayleigh and her family are regu lar d ental attend ers and
have ju st m oved to the area w ith her father ’s job. This is the

Gingival bleeding
rst tim e you have seen her.

Examination
and enlargement Extraoral exam ination is norm al, w ith no signs of infection.
Intraorally there is w id espread m arginal gingivitis, w hich
is p articu larly bad in the u p p er right qu ad rant anteriorly
(Fig. 41.1). Clinical and rad iographic exam inations of the
teeth reveal a low caries rate, w ith only the need to replace
a cracked and d e cient restoration in a low er rst perm a-
nent m olar that has recu rrent caries.
SUMMARY ■ What periodontal screening should be underta ken
Kayleigh is 15 years old. She is concerned that her or Ka yleigh?
upper gums look abnormal, and they bleed Period ontal screening shou ld be und ertaken for all child ren
whenever she brushes them (Fig. 41.1). over the age of 7 years. For child ren in the m ixed d entition
(e.g. 7–12 years old ) a sim p li ed basic p eriod ontal exam ina-
History tion (BPE) is recom m end ed . BPE is m easu red at six points
(m esiobuccal, buccal, d istobu ccal, d istolingu al, lingual and
Kayleigh has noticed bleed ing w hen she has been bru shing 61 6
for the last year. She is frightened of bru shing becau se of m esiolingu al) on six ind ex teeth ( ), and only cod es 0,
6 16
the bleed ing and feels that it is getting w orse. She is also 1 and 2 are u sed . For old er child ren (13–17 years old ) the
very socially conscious of her gu m s because they look very same ind ex teeth are m easured in the sam e w ay, bu t a full
red and are ‘bigger ’ than norm al. range of cod es (0, 1, 2, 3 and 4) are used . Any scores over
2 require further investigation, inclu d ing a fu ll p ocket
Medical history charting for all teeth and , w here ind icated , referral to sp e-
Kayleigh has insu lin-d ep end ent d iabetes. She takes her cialist care.
insu lin by subcu taneou s injection at 07.30 and 17.30 hours. ■ Wha t actors are contributing to the chronic
She has a regu lated gram intake of carbohyd rate at 07.30, marginal gingivitis?
11.00, 13.00, 15.00, 17.30 and 21.00 hou rs. Ap art from this,
she is an active girl w ho p lays basketball and hockey at Poor oral hygiene.
school and has learnt to increase her carbohyd rate intake Hormonal changes o puberty.
ap prop riately to cover her sp orting activities. H er m u m Poorly controlled diabetes mellitus.
reports that Kayleigh has had the occasional ‘rebellion’
against her cond ition and at these tim es d iabetic control has
Key point
Gingival bleeding can be as a result o :
• Local causes.
• Systemic causes.

The com m onest local and system ic causes of gingival


bleed ing in child hood / ad olescence are show n in Box 41.1.
■ Wha t do you think may ha ve precipitated the
initia l gingivitis?
This is likely to have coincid ed w ith one of the period s
w here d iabetic control w as p oor. Fu rther questioning
revealed that abou t a year ago Kayleigh w as stru ggling to
come to term s w ith her insu lin-d epend ent d iabetes. She
refused to take her insulin regu larly and end ed u p being
Fig. 41.1 Chronic gingivitis. ad mitted to hosp ital in com a w ith ketoacid osis. H er blood
• 214
41 GINGIVALBLEEDINGANDENLARGEMENT
intravenou s access is d if cu lt, give 1 m g intram uscu larly of
Box 41.1 Commonest causes o gingival bleeding in childhood
and adolescence glu cagon. In p ractical term s, 10 g glu cose app roxim ates to:
Local causes 2 tsp sugar.
• Eruption gingivitis. 3 lumps sugar.
• Acute/chronic gingivitis. 3 Dextrosol tablets.
• Chronic periodontitis. 60 ml Lucozade.
• Foreign body entrapment.
15 ml Ribena ( ull sugar type).
• Acute necrotizing ulcerative gingivitis.
90 ml cola (not diet variety).
• Haemangioma.
1
3 pint o milk.
• Reactive hyperplasias, such as pyogenic granuloma.
• Factitial injury.
Systemic causes Treatment
• Hormonal changes such as pregnancy or puberty.
Kayleigh’s gingivitis p robably started as a resu lt of p oor
• Diabetes mellitus – poor control. d iabetic control and u nfortu nately has been com p ou nd ed
• Anaemia. by poor oral hygiene and the horm onal changes of puberty.
• Leukaemia. ■ Why is the gingivitis worst in the anterior part o the upper
• Anyplatelet disorder. right quadrant?
• Clotting de ects.
She is right-hand ed , and this is often the case w hen a right-
• Drugs (e.g. anticoagulants). hand ed p erson changes their bru shing action from the left
• Scurvy. hand sid e of the m ou th to the right hand sid e. The op posite
• Human immunodefciency virus-associated periodontal disease. w ould be true for the left-hand er.
■ What other generalized ca uses o gingiva l enlargement do
you know?
There are a nu m ber of cau ses, w hich can be classi ed into
sugar at that tim e w as very high and her breath sm elt of congenital and acqu ired (Box 41.2).
‘p ear d rop s’ d ue to ketone bod ies. This w as a hyp erglycae- ■ Why is it important to eradicate Ka yleigh’s gingivitis?
m ic com a. She w as resu scitated w ith intravenou s u id s, as
she w as severely d ehyd rated , p rior to restabilization on an Poor oral hygiene in com bination w ith d iabetes can resu lt
insu lin regim en. in rapid period ontal d estru ction and attachm ent loss. An
exam ple of this in another su bject w ith d iabetes is show n in
■ Wha t is the other cause o diabetic coma and what are
Fig. 41.2. There is som e evid ence that signi cant period on-
its signs?
titis can u p set glycaem ic control.
H yp oglycaem ic com a occu rs d u e to inad equ ate carbohy- Kayleigh need s reassu rance that the bleed ing w ill red u ce
d rate intake (m issed m eal), exercise or excess insu lin. The and stop w hen her oral hygiene im p roves. She need s to
onset is qu icker than the hyperglycaem ic com a. The signs appreciate the im portance of good oral hygiene and the
of hyp oglycaem ic com a are very sim ilar to having ‘a d rink problem s that w ill occu r if oral hygiene is p oor.
too m any’, and can be su m m arized into those cau sed by ■ Why is it important not to lea ve ca ries in a diabetic?
ad renaline release and cerebral hyp oglycaem ia:
Ad renaline release: Infection of any origin can resu lt in an increased need for
insulin. Withou t an insu lin increase there w ill be a rise in
• Sweaty warm skin.
blood su gar resu lting in ketosis. Therefore all infections in
• Rapid bounding pulse.
a d iabetic, includ ing those in the orofacial region, should be
• Dilated (reacting pupils). treated vigorou sly w ith antibiotics. Caries should be treated
• Anxiety, tremor. early to p revent pu lpal necrosis and subsequ ent infection.
• Tingling around mouth.
Cerebral hypoglycaem ia:
• Con usion, disorientation. Key point
• Headache. In diabetics:
• Dysarthria. • Poor oral hygiene will accelerate attachment loss.
• Unconsciousness. • In ection can inter ere with diabetic control.
• Focal neurological signs, e.g. ts.
If an ind ivid u al having a hyp oglycaem ic ep isod e is con-
sciou s, they shou ld be given su gar orally – 25 g glu cose. In
■ Why is the timing o the appointment to restore Kayleigh’s
m any em ergency d ru g boxes are sachets of thick glu cose
f rst perma nent molar important?
syru p , w hich can be slow ly squ irted u nd er the tongue.
If com atosed , they requ ire 20 m g of 20% d extrose intra- To not interfere w ith Kayleigh’s carbohyd rate intake and
venou sly, follow ed by 25 g orally on arousal. Alternatively, if p recip itate hypoglycaem ia, it is probably best to give her an
215 •
GINGIVALBLEEDINGANDENLARGEMENT 41
Box 41.2 Systemic causes o gingival enlargement ■ What dieta ry a dvice should you give to diabetic pa tients?
Congenital Do not change your requ ired carbohyd rate intakes, as these
• Hereditary gingival fbromatosis. are critical to d iabetic control.
• Mucopolysaccharidoses. Tailor the d ental ad vice to the speci c need s of the
p atient, i.e. take you r toothbru sh to school if p ossible. Try
• In antile systemic hyalinosis.
to clean you r teeth after snacks and at lu nchtim e. Try to take
Acquired snacks that provid e the necessary su gar but d o not rem ain
• Puberty/pregnancy gingivitis. stuck to the teeth. Use su gar-free gu m if it is not possible to
• Plasma cell gingivitis. brush the teeth d u ring the d ay. In sum m ary, changing
• In ections: herpes simplexvirus. d ietary su gar intake or frequ ency is d if cu lt and therefore
• Haematological: acute myeloid leukaemia, preleukaemic leukaemia, aplastic attention shou ld be concentrated on increasing the fre-
anaemia, vitamin Cdefciency (scurvy). qu ency of toothbru shing, strength of u orid e toothp aste
• Drugs: phenytoin, cyclosporin, calcium-channel blockers, vigabatrin. (e.g. 2800 p pm for Kayleigh, increasing to 5000 ppm w hen
she is 16 years old ) and the effectiveness of bru shing.
• Deposits: mucocutaneous amyloidosis.
• Chronic granulomatous disorders: sarcoidosis, Crohn disease, oro acial
■ What other oral mani estations can occur in diabetes?
granulomatosis. Dry mouth.
Swelling o salivary glands (sialosis).
Glossitis.
Burning o tongue.
Oral candidosis i control is poor.
These mani estations are more commonly seen in adults.

Primary resources and


recommended reading
Chap p le ILC, Wilson N H F 2014 Manifesto for a p arad igm shift:
p eriod ontal health for a better life. Br Dent J 216:159–162.
Clerehu gh V 2008 Period ontal d iseases in child ren and
ad olescents. Br Dent J 204:469–471.
Clerehu gh V, Kind elan S 2011 Gu id elines for period ontal
screening and m anagem ent of child ren and ad olescents und er
18 years of Age. Lond on. Available at: http :/ / w w w.bsp erio
.org.uk/ p u blications/ d ow nload s/ 54_090016_bsp_bsp d -perio
-gu id elines-for-the-und er-18s-2012.pd f.
Fig. 41.2 Gingival and periodontal disease.
Firatli E, Yilm az O, Onan U 1996 The relationship betw een
clinical attachm ent loss and the d u ration of insu lin d epend ent
d iabetes m ellitu s (IDDM) in child ren and ad olescents. J Clin
Period ontol 23:362–366.

ap pointment either rst thing in the m orning or d irectly Karjalainen KM, Knu u ttila MLE, Kaar M-L 1997 Relationship
betw een caries and level of m etabolic balance in child ren and
after lunch. For any p rolonged su rgical p roced ure in a d ia-
ad olescents w ith insulin-d ep end ent d iabetes m ellitu s. Caries
betic p erson, or any treatm ent that requ ires general anaes-
Res 31:13–18.
thesia (GA), a referral to hospital is requ ired . GA w ill requ ire
ad m ission p re-op eratively to stabilize insu lin and glu cose
requirem ents via a d rip so that hypoglycaem ic com a d oes For revision, see Mind Map 41,
not occu r w ith p re-op erative GA starvation. page 262.
42
■ What is the diagnosis?
Prim ary herpetic gingivostom atitis.
There are tw o typ es of herp es sim p lex viru s: herp es
simplex type 1 (H SV-1) and herp es sim plex typ e 2 (H SV-2).
Classically H SV-1 cau ses oral d isease and H SV-2 cau ses
genital d isease. The viru ses, how ever, are very sim ilar, and
both can cau se both oral and genital d isease, althou gh there
are d ifferences in recu rrence rates. Prim ary exposure to H SV

Oral ulceration in the m outh cau ses acute prim ary herpetic gingivostom a-
titis (Fig. 42.1). The viru s cau ses a viraem ia, fever, m alaise
and lym phad enopathy. All the surfaces of the m ou th,
inclu d ing the hard p alate and attached gingiva, can be
involved initially w ith a vesicular rash that u lcerates and
can become su perinfected . The illness lasts for 10–14 d ays
before resolving spontaneou sly. Diagnosis is u sually m ad e
on clinical grou nd s but can be con rm ed by a threefold rise
in the convalescent antibod y titre over that seen in the acute
phase or by d irect im m u no uorescence of vesicular uid
using speci c antisera.
SUMMARY
Alan is 8 years old. He has been brought to the
surgery by his mother because his mouth is so Key point
pain ul he ca nnot eat (Fig. 42.1). What could cause Systemic signs in HSV:
this problem? How would you treat it? • Fever.
• Malaise.
History • Lymphadenopathy.
• Di culty eating and drinking.
Alan has not been w ell for a cou p le of w eeks. H e had a
‘viru s’ that resu lted in his being p ut to bed and m issing
school. Just as he w as im proving, his m ou th becam e very
sore. H e has been unable to eat solid food for 3 d ays and The primary infection m ay be m ild and subclinical in the
has been on liqu id s only. H e feels hot and lethargic. H is majority of you ng child ren w ho are exp osed to it, though
gu m s bleed w hen he tries to bru sh them . the cond ition m ay be severe and d ebilitating. In an im m u -
nocom prom ised ind ivid u al, it m ay lead to severe illness and
Medical history som etim es herp etic hepatitis or encep halitis, w hich m ay be
Alan is generally a healthy boy. H e has had a cou p le of fatal in the absence of treatm ent.
cou rses of antibiotics for ear infections bu t has had no real Oral infection arises from d irect contact w ith secretions
illnesses. H e has never been in hosp ital and is not on any from an ind ivid u al w ho has either p rim ary or recu rrent
tablets or m ed icines from his d octor. H SV infection. Direct inocu lation of the ngers or skin w ith
virally contam inated secretions or u id can lead to local
■ Describe the appea rance o the upper and lower gingiva e
infection, e.g. herp etic w hitlow of the nger.
in Fig. 42.1.
H SV is a neu rogenic virus, and on recovery from the
There is erythem atou s gingival enlargem ent w ith sm all prim ary infection the viru s m ay becom e latent w ithin the
u lcerations of the gingival m argin. trigem inal ganglion or basal ganglia of the brain and m ay
subsequ ently be reactivated to cau se a second ary infection.
The second ary infection m ay cau se a ‘cold sore’ on the lip
or Bell’s palsy. Best p ractice for p atients presenting w ith a
cold sore for regu lar d ental treatm ent (e.g. not in acu te pain)
is to rebook them once the lesion has d isapp eared . When a
cold sore is present, there is increased risk to the p atient of
exacerbating the infection and infecting one or m ore
mem bers of the d ental team (eye infection).

Treatment
Alan resp ond ed w ell to rehyd ration and analgesia su ch as
paracetam ol, w hich is also antip yretic, and an antiseptic
m ou thw ash su ch as chlorhexid ine (Corsod yl) or benzyd am -
ine hyd rochlorid e (Dif am ). H ad Alan not been able to
Fig. 42.1 Gingival enlargement and ulceration. m aintain hyd ration, he w ou ld have had to be ad m itted for
217 •
ORALULCERATION 42
intravenous uid therapy. There is no evid ence that sys- Coxsackie viruses. These are RN A viru ses and resp onsi-
tem ic aciclovir is of any bene t at the relatively late stage of ble for herpangina and hand , foot and m ou th d isease.
the cond ition that he p resented . If he had been seen w ithin H erp angina is a Coxsackie A viru s that cau ses a herp es-
72 hou rs of the onset of the infection and the clinical severity like orop haryngitis w here the u lceration is p red om inantly
w arranted it, aciclovir cou ld have been p rescribed . The d ose on the tonsil, soft palate and u vula. The m ild illness is asso-
is 200 m g ve tim es d aily for 5 d ays in p atients over 2 years ciated w ith fever and m alaise bu t only p ersists for a few
of age and 100 m g ve tim es d aily for 2 d ays in patients d ays.
u nd er 2 years of age. Althou gh aciclovir can shorten the H and , foot and m ou th d isease is also Coxsackie A virus
cou rse of the prim ary infection, there is no evid ence that it and occurs pred om inantly in child ren and their fam ilies.
red u ces the incid ence of recurrent herpetic lesions. The cond ition is characterized by u lceration affecting the
■ What are the reasons given or the reactivation o HSV to gingiva, tongu e, cheeks and p alate. It is associated w ith
produce a cold sore (herpes labialis) (Fig. 42.2)? vesicles and u lcers on the p alm s and soles. It m ay persist
for 2 w eeks.
See Box 42.1. Human papilloma virus (HPV). An increasing num ber
■ How should herpes labialis be treated? of H PV typ es have been id enti ed . Typ es 2 and 4, the
common w art, occu r in child ren usually d u e to au toinocula-
App ly aciclovir 5% cream to the lip lesion as soon as tingling
tion by biting ngers or hand w arts. Cond ylom a acu m inata
or prickling of the prod rom al p hase is felt. This is u sually
or venereal w arts can occu r on the oral m u cosa and are
24 hou rs prior to vesicu lation and p ain.
associated w ith H PV types 6, 11 and 60.
■ What other viral in ection can occur in the mouths o There are a nu mber of other cau ses of oral u lceration in
paediatric patients? child ren and ad olescents that are not associated w ith infec-
Varicella zoster virus (VZV). VZV is a neu rogenic DN A tions. These are show n in Box 42.2.
virus that causes a prim ary infection in the form of chick- ■ What types o a phthae are there?
enpox. Thereafter it rem ains d orm ant u ntil reactivation as
Minor.
shingles.
Epstein–Barr virus (EBV). EBV is a herp es viru s w ith a Major.
pred ilection for infecting B lym p hocytes. This infection Herpeti orm.
causes the B lym p hocytes to becom e activated and p rod u ce
Behçet syndrome.
their ow n antibod ies. A p rim ary infection w ith EBV causes
gland u lar fever or infectiou s m ononu cleosis. Recurrent aphthae a ect up to 20% o the population.
Cytomegalovirus. This is associated w ith a gland ular M inor aphthae are com m onest and account for 85% of
fever-like illness in child hood . On occasion it is found in recurrent aphthae. Ulcers are less than 1 cm in d iam eter,
severe atyp ical oral u lceration in hu m an im mu nod e ciency u sually only 2–3 m m . They occu r singly or in crop s of u p to
virus (H IV). 10, last 3 d ays to 3 w eeks and heal w ithou t scarring. Ulcers
Herpes virus type 8. This has been d em onstrated in are round or oval w ith a yellow ish-grey base and su r-
Kaposi sarcom a in H IV infection. round ed by an erythem atous halo. They affect only non-
keratinized m u cosa.
M ajor aphthae are m ore severe and u su ally m ore than
1 cm in d iam eter w ith an irregular ou tline. They are u su ally
single and often affect the fau ces. They are often d eep er
and bleed , and m ay last for several w eeks or m onths
before healing w ith scarring. Again, they tend to affect non-
keratinized m u cosa.
Herpetiform aphthae look sim ilar to p rim ary herpes. The
u lcers are 1–2 m m in d iam eter and large nu m bers m ay
occu r sim u ltaneou sly. They last from 2 d ays to 2 w eeks
before healing w ithout scar form ation. Again they affect
Fig. 42.2 Recurrent herpes ‘cold sore’. only non-keratinized m u cosa.

Box 42.1 Reactivation o HSV Box 42.2 Other causes o oral ulceration
• Trauma. • Aphthae.
• Chemicals. • Gastrointestinal disease.
• Heat. • Haematological disease.
• Hormones. • In ections.
• Sunlight. • Mucocutaneous disorders.
• Emotion. • Radiotherapy.
• Immunosuppression. • Trauma.
• Concurrent in ection. • Carcinoma.
• 218
42 ORALULCERATION

Box 42.3 Common aetiological actors in recurrent aphthae Box 42.5 Therapy or recurrent aphthae
in children Topical
Host actors • Coating agents (Gelclair, Orabase).
• Genetics. • Antimicrobials (chlorhexidene mouthwash, tetracycline mouthwash).
• Nutrition. • Corticosteroids (beclomethasone inhaler spray, betamethasone mouthwash,
• Systemic disease. triamcinolone dental paste).
• Immunity. • Local analgesics (lidocaine spray, lidocaine ice lolly, topical benzocaine (20%).
Environmental actors Systemic
• Trauma. • Systemic corticosteroids.
• Allergy. • Colchicine.
• In ection. • Thalidomide.
• Stress.

Box 42.4 Dietary allergens in recurrent aphthae Patients u su ally d evelop fresh u lceration w ithin 12–24
• Cheese. hours of ingesting the su sp ect food . Food s m ay be id enti ed
• Chocolate.
more objectively by patch testing. Dietary avoid ance is often
attend ed by clinical im p rovem ent.
• Nuts.
• Tomatoes.
• Citrus ruits.
• Benzoates.
Key point
• Cinnamon aldehyde.
Aetiological actors in recurrent aphthae:
• Host related.
Behçet syndrome d escribes the triad of oral and genital • Environment related.
u lceration and anterior u veitis. It is rare in the UK and USA.
■ Wha t aetiological a ctors are importa nt in recurrent
aphthae?
See Box 42.3. Althou gh only 20% of the p op u lation experi- Treatm ent of recurrent aphthae can be grou ped into
ence recu rrent ap hthae, there is a p ositive fam ily history in topical and system ic m ed ication d ep end ing on the severity
50% of fathers and 60% of m others. There is, in ad d ition, a and frequ ency of the u lceration (Box 42.5).
w eak hu m an leu cocyte antigen (H LA) association w ith
H LA typ es A2 and B12. Primary resources and
N u tritional d e ciencies of iron, folic acid and B12 can
occu r singly or in com bination. Many of these are latent and recommended reading
have a normal p eripheral blood p ictu re. It is necessary to Al Johani KA, Moles DR, H od gson TA et al 2010 Orofacial
assay ind ivid u al levels of ferritin, folic acid and vitam in B12, granu lom atosis: clinical featu res and long-term ou tcom e of
therap y. J Am Acad Derm atol 62:611–620.
as w ell as d o a fu ll blood cou nt (FBC).
Montgom ery-Cranny JA, Wallace A, Rogers H J et al 2015
■ What systemic diseases in children are commonly Managem ent of recu rrent aphthou s stom atitis in child ren.
associated with aphthae? Dent Up d ate 42:564–566, 69–72.
Coeliac disease. Scu lly C, Welbu ry R, Flaitz C, et al 2002 A Color Atlas of
Orofacial H ealth and Disease in Child ren and Ad olescents, 2nd
Crohn disease. ed . Martin Du nitz, Lond on.
Ulcerative colitis. Scottish Dental Clinical Effectiveness Program (SDCEP) 2011
Dru g Prescribing for Dentistry, 2nd ed . SDCEP, Du nd ee.
In old er p atients and less com m only in child ren, aphthae
Available at: http:/ / w w w.sd cep .org.u k/ p u blished -gu id ance/
m ay also be associated w ith p erniciou s anaem ia, H IV infec-
d rug-p rescribing/ .
tion and m alabsorption.
There is an increasing aw areness of the role of d ietary
allergens in recurrent ap hthae in child ren. Dietary allergens For revision, see Mind Map 42,
imp licated in recu rrent ap hthae are show n in Box 42.4. page 263.
43
Mind maps
43
• 220

c
h e c
i
n
k c
c
s
f
h i
t
o
s
r e
r
e
i
v c
t
n
e
c k
o
b
h p
f
t
c
e
i
o
c n
a
h
t
h
r
w g
p
e
b
i e
c
i
l
o
n
t l
l
k
e
o
a
a f
i
n
o n
n
f
o
u
o
t c
t
1
h
p
n
e
r h
|
p
r s
1
/
i
u
n
d
e u
p
p
g
e
r
p a
w
n
l
e
o t
i
a
t
p
h r
f
a
r
n
o l
d
l
u b
o
m
o
g n
y
e
p
e r
f
a
l
a
r a
l
e
r
a b
n
y
t
d
i a
a
a
e l
l
l n
a
f
t a
t
a
t p
?
m
a
l a
c
p
c
a
i
m
i l
l
h
h
e
n
p
y
b m
o
m
a
r
c
a
o
s
h
i
c r
i
a
a
r
t
e
d
e
c i
t
i
e
s
p
y
o
i n
e
l
n
n
u
t
i n
c
t
n
c
t
o
b
g
s s
o
2
a
c
r
e
a u
y
m
e

l
s
l
p d
t
v

o
o
u
m 2
c
p
u
i
e
d
r
b
l %
l
e
h
i
e
o n
w
o
e
y
c
D
n
m
t
i n
r
a
c
p
o
o
c P
t
i
e
a
b
o
n
n
o
u d
T
n
l
s
d
i
m
l
c i
a
f
t
e
g
o e
a
a
t
e
n
i
n
e m
l
l
t
e
j
t t
u
r
s
i a
x
2
a
a
v
u
a l

d
t
e
e
s
c d
y
m
i
n
s
k
o
p e
i
i
i
g
h
d
n
l
e
v n
e
r
i
e
e
g
s
a s
a
a
c
p
l
p
n t
t
o
p
b
y
i
r
o e
t
o
o
p
s
h
o
s r
r
n
m
m
y
t e
i
a
i
c
o s
/
n
l
p
i
i
e
v
d n
n
u
t
t
r
e
n
u
o c
e
p
o
o
o
t
I
m
n
i n
e
c
a
n
s
r
l
t
a
t l
r
o
l
o
v
i
p
i t
n
n
r
l
(
r
i
u
e a
e
s
u

s
a
d
u
s
r g
b
m
t
i
g
t
i
-
i
s o
a
i
s
l
e
ga
p y
n
l
h
r
p
r f
a
d
a
o
r
a
o t
r
e
u
p
p
i t
f
y
o
h
n c
e
o
u
n
o
k
u
t d
r
e
p
t
l l
m
s
i
o
i
e 2
p
n
o
g
t

e
g n
s
h
y

r ) A
e t
i o l
o g y M
a n
a g e m
e n
t r
m
e
d
e n
i
a
m
a
b c
s
y
e
o
o t
f
e
u
v
c o
c
s
e
l
m
r y
p
r
o
a
e s
a
s
s
a
g t
c
e
u
a e
/
>
e
n
p s
t
d
y
e
u
4 f
p
o
i
r m
g
o
o
n
m
r
i
n r
t
u
o
2
t m
t
h
s
m
u
a
’ s
u
o
r
n :
e
b
c
d b
e
c
r
e
k
o
a
e o
o
f
i
n
n
r
o
f
u n
o
y
t
g
r
s
s
i r
e
c
l
t
e
i y
h
o
d
3
t a
o
e
c
r
a
|
e b
l
t
r
3
s
o
h a
i
i
t
s
n 3
e
t
/
m
u t
t

r
o s
t
e
r
u
e e
e
r
p
p
s
e
p
e
d
a v
n
c
t
e
t
r
o
t
e
u
p
o
t o
p
h
u
r
a
s
n
r
p
n
p a
m
p
i
n
s
o
t
n
l
s l
i
t
i
i
a
d
a
o
a r
g
i
b
d t
t
n
n
n
l
l
a
o e
s
e
a
c
e
l p
r
c
e
2
n
r r
i
a
f
k
e
n
e
’ t
r
s
c s
e
t
t
b
o
e a
e
t
(
n
o
f
o r
?
f
i
r
e
n
i
c
c n
e
r
x
a
c
a
e
v
l d
m
e
o
e
t
t
n
r
e d
o
s
i
o
v
n
i d
u
m
n
e
v
e r
e
a
y
e
e t
b
r r
w
e
s l
e
i a
)
d t
t
m h e n
p
u
m
m
t r
s
i
i
i u
n
m
x
o
a
e a
r
l
d
l r
y
i
y n
d
c
c
e d
o
r
n
e e
m
t
n a
i
t
p
t i
s
i
o
t l
e
e
i n
o
t
:
t
n e
o
~
: ~
~
~
3 9
1
1
m 3

y
m
e
2 y
a
e
m i
r
n
a s
m
c
r s
r
e i
n a
c s
r e
e a s e
MINDMAP1A

M e d
i a n D
i a s t
e m
a
221 •
MINDMAP1B 43

significantly larger 6
local more mesial angle of eruption of 6
Indicative of crowding
small maxilla
Aetiology

hereditary familial tendency

non-cleft 2–6%
Prevalence
cleft lip and palate 20–25% 6 disimpacts
reversible 6 erupts spontaneously
rare after 8 years
Classification
until treated (see below)
6 remains impacted
irrreversible E lost spontaneously

Impacted Upper First


Permanent Molar ? E mobile
clinical
? caries 6
Investigations
? DPT
radiographs extent of E resorption
? bitewing
? caries 6

brass wire separator


disc distal E
without extraction E
separating spring
move 6 distally
Management irreversible impaction marked resorption
when 6 cannot be disimpacted
with extraction E access required to restore 6
6 more mesial eruption path
consider move 6 distally
6 eruption blocked by distal E Clinical features leads to space loss
accept and treat later
distal E resorbs
• 222
43 MINDMAP2

absence – very rare


genetic
gender male : female (2:1)
hypodontia tuberculate (~ 12%) most common with unerupted 1
supernumerary teeth family history conical (~ 75–78%)
cleft lip and palate types
history supplemental
trauma supernumerary teeth odontome
previous surgery to premaxilla incidence 1.5–3.5% permanent dentition (exct 8’s)
swelling inverted
dental charting Investigations Aetiology everted
labial position
palpation clinical examination midline (mesiodens)
palatal maxilla / mandible (9:1)
space analysis cleidocranial dysplasia
adjacent incisors possible sensibility test multiple Gardner syndrome
DPT cleft lip and palate
upper anterior occlusal or periapical view radiographic ectopic tooth germ
? CBCT dilaceration
trauma to primary dentition
arrested development
previous surgery
scar tissue
cleft lip / palate repair
tumour
pathology
dentigerous cyst
Unerupted Upper Central Incisor

joint orthodontic / oral surgical planning


assess patient wishes / likely compliance with future treatment
remove A – and any pathology
prosthesis; bridge; implant later
ectopic tooth germ probable surgical removal 1– replace 1– 2 drift mesially; ? orthodontic alignment; crown 2
– –
prosthesis; bridge; implant later
arrested development probable surgical removal 1– replace 1– 2 drift mesially; crown 2
– –

Management
surgically expose and align orthodontically if 1 apex not destined to perforate cortical plate
mild
dilaceration prosthesis; bridge; implant later
severe surgical removal 1 replace 1
– 2 drift mesially; crown 2
– – –
tumour biopsy for further diagnosis
dentigerous cyst marsupialize
surgically expose 1–, closed technique: bond attachment activation
retention
design using ARAB
? start with URA anchorage
baseplate
surgical / orthodontic orthodontic alignment fixed appliance
bonded retainer open space for 1 always balance for unilateral loss/extraction of C
r
e p
l a n
r
e p
l a r
e p
o
l a
n n
r
e e
r o
o l
t
o o
n
n y
e
r r
n
r
e o
t
a
o
h
t a
r
m
m p
o
l
h r
r
n
a
e o
e
e
n
m t
t
h m
p
r
a
o
p a
l
a
n
a
e r
n
o
t e
r
r
a
o
e e
m
a
n
m r
o
t
a n
a
e
o
r
t r
o
t
e
l a
t
o
e
r e
r
n
r
l
a
m
e
o e
n
n
a
e l
t
p
a
a
o
t e
l
h
a
r e
r
l
r
l
e
r
o
m
a
o
n p
y
o
r
e
t
r
l
n
t
r
t h
e
l
a
o
o
r
p
a
e
m o
o
a
e
r
p
n
t
e
r a
e
n
a
e
t
r a
r
n
o
m a
o
p
t
a
e
n
a
h p
n
e
t
a
r
o
o
l
e
p e
l
r
t
t
o
m
o n
t
m
a
e
n e
l
l
t
t
o
l
a
m o
r
o
a
a
h
T
a
n
p
e r
r
n
r
a
o
e n
t
a
y
r
e
e l
m
n
p
m
a n
n
e
a
a
t
n
t r
n
e
p
m
e
n
p
l
m
e p
r
e
n
p
r
l
o n
e
a
a
a
n
e
e
y n
n
l
t
r
m
n
t y
t
t
o
a
a y
t
n
a
e
r
r
h e
e
n
t
i
y
p
o e
m
o
m
r
p
l
o
n n
r
t
l
a
h
s
a a
m
r
l
o
m l
p
o
e
e e
m
n
l
y
m
p
o
a p
n
r
p
n
m
t y
l
t
e
h e
n
a
a
t n
m
r
e
o p
t
a m
a
a
r
r o
M
y
m a
e
p
a
e
e n
l a
e
g o
n
p
n
e
r
e h
a
e
m
t
e a
r
y
n e
n
o n
t
t y
p e I
n H
v
y
e p
s o
t
i d
g
o a
t
n i
t
on i
a s r
a e
a
l e
n r
o a
m
s
m l
p
e
t y
a
i
o
r
l
v d
a
t
d
r
e
a o
p
e
(
r
l y

e
h
r
t
n
e
a
e
e
a l
(
t
n
n
l
2
e >
e
t
t
)
t a
a
l
6
(
o l
3
t
l
a
)
a l
r

o t
e
l
r
t
n 5
n
o
o
l
a
a
) o
t
t
t t
h
p e
e
a
o o
r
r
n n
n
n
a
l
l
r
r
s
i
a
d n
e
e
o
m
n
e
e
c e
d
t
h
l
a r
m
a
p
a
t
h
u
e
i
o i
y
d
l
a
n
c l
e
a
e
r
a e
e
o
t
r
e
s
o
e
d l
a
d
d
r
p
e
n
t t
e
r
i
o
h d
d
t
c
p
l
y
h
t
o
h n
o
h
e
t
r
o
e
e
w
a e
i
n
n
m
y v
c
e
p
a
e
t
e
t a
t
e
a
t
r
o
o r
l
r
d
f
b
n y
t
d
a
e i
t
e
t
r
c t
e
r
e
e v
i
m
a e
e
p
l
a t
l
a
o
h
h l
t p
e
d
h
t m
i
y
r
o
g e
s
l
h
e
o a
p
t
n t
l
m
a t s
e
y i
a
n t t
a
h n
n e n n e o
o p
h
a a
l
m a
r r e
n o r
m a l p h e n
o t
y p e A e t
i ol og y
e n t p
p e r a t
e r
a l n o
r
223 •
MINDMAP3
43
• 224
43 MINDMAP4

refer to dermatologist
females > males nickel allergy medical history
if confirmed use nickel-free
appliance components arc usuall crowded
crowding spacing Buccal 3–'s las per anen oo o erup an erior o
angula ion o lower incisors re ainedC
canine inclina ion lower arc irs
o er i e dep
idline s i
ild
odera e degree o crowding
assess space
se ere increased o er i re e uire en s
cen reline s i

assess oo o e en appliance pe re uired o align lower arc


Treatment planning
i agine correc ed posi ion os
en all reposi ion
s o Class rela ions ip wi correc ed
posi ion o s a e allowance or lower la ial seg en spacing
and or oo si e discrepancies e g pegs s aped
decide w e er space as o e crea ed a oun and pe o
o e en re uired
plan upper la ial seg en and rela ed ec anics

decide inal olar rela ions Crowding and Buccal Upper Canines
ip and rela ed ec anics
anc orage
re en ion

prognosis
si e o crowding
e racs ion
e en o crowding
indi idual oo posi ions
surgical e posure i re uired
crea e spac
e arc e pansion er rarel undern a e
in lower arc
dis al o e en
ena el s ripping
co ina ion
re o a le
Treatment id e
appliances
eadgear pala al arc
id e
re ainer
s
re o a le
incidenc
e
s and s
decalci ica ion
care ul pa ien selec ion
ain ris s
pre en iondie ar ade ic
applica ion o luo
wee l around rac e
roo resorp ion sodiu luoride e
relapse
225 •
MINDMAP5 43

pulpal pathology Turner’s hypoplasia


caries
causes severe crowding
trauma

inherent dentoalveolar
disproportion
age at removal
arch from which removed
Causes early loss of primary teeth space loss affected by: degree of crowding
tooth lost: greater if posteriorly in arch 6 rotates mesiopalatally following loss of E
occlusion
supernumerary teeth
megadont
space lost by mesial drift
premolar / molar rotation
possible effects premolar / canine impaction
crossbite of premolar with / without mandibular displacement
centreline shift

Severe Crowding

DPT
radiographic
Investigations
average space for 3,4,5 = 22 mm
space analyses
average space for 3,4,5 = 21 mm

made more complex by loss of 6 if severe labial segment crowding


URA with headgear support
upper arch headgear support to upper molar bands
Nance button palatal arch with/without transpalatal arch
TAD
Management where anchorage demands are maximal
lingual inclination of molars
lower removal appliance not acceptable
encroachment on tongue space
lingual arch
lower arch
bond/band 7's and ligate to 6's
TAD
• 226
43 MINDMAP6

o ilit
olour
lini al wear
palpate palatall
in lination
tooth with longest path of eruption
r pt ispla e ent he wi th of an for
in isor pre olar h po ontia erti al or hori ontal tu t e shif
two fil s
ost o on ou les paralla
feaf ts s in i en peg e or a sent
infrao lu e pri ar olar Aetiology
ost
usuall un row e ar h resorption of a a ent teeth
pol geni Investigations
i pa te geneti un ertaint a out position on stan sar fil
asso iate
other e topi teeth root length
ano alies
transposition ra iographi erti al position ofto roots of in isor
s
lass i ision assess esio istal position of to roots of ins isor
uropeans a ial in lination
fe ales ales ape lo ation
ilateral with greater fre uen than e pe te in isor root resorption esti ate higher wit
anterior a illar trau a than with on entio
patholog ra iographs
italit test
au asians
a sent
au asians of whi eh topi
Retained C
ilateral – alatal anines ean anterior ratio
olton anal s si
ean o erall ratio

re o e s ut no fir resear h e as
s ear
inter eption
o erlaps istal half root
a ti e treat ent if no h
onitor onths within onths
patient not een for treat ent
no patholog resorption
retain an o ser e prognosis goo
an in onta t goo s aestheti
se erel ispla e no patholog
highl oti ate patient
Management e pose an align e ellent ental healt
h
spa e for or possi le to reate spa e
position fa oura e l
no signifi ant ifferen e l enta
open lose te hni health
ue ental aestheti
s
patient not een for align ent
st for ation
poor prognosis to align
re o e
an goo t onta
goo aestheti s prognosis
earl resorption of a a ent
h teet
a e uate spa e
transplant inta t re o al possi le
a e uate u al palatal one
rarel signifi ant if
a ress
lini all signifi ant if s
e on olton ean alues
227 •
MINDMAP7 43

maxilla: a a m a a
ma i l :a a alm x l i l
la i i a i : a ial m l
al :
a i l : i a i m al

l i a l i a
i i i
i x a i ima
Transposition
a
i i
a mai ai
a i i a ma i m l
a
a m a
a a i
x a i i la
ix a lia i
i la i i i a i
i ali m

a i l m

i i
m mm i mal
i i a i a la i mi li
xi a i
m ili
li i al l
i i a i i ili
Associated incisor root resorption
i ll x l
i i l i al a i a
a i a i

i i i i
x a i a
a m i
x a i a ai i al x
a a m i

m i i a i a i all i a

ma al ll i ma a m a ia
l a i ma i i a i a
i i a m
• 228
43 MINDMAP8

ti li
o i tio it l t l to i
Aetiology i il t m o
ylo i
o ti l ol o t it

m t titio
i by o ti t Prevalence
y o o ti

f m l m l

f fo ll yt ti
Latex allergy?
if o fi m l t f

f o l im y ol

mobility of E
if
ob tio o
t t of i f o l io of E
li i l o o i oo o
ti o to io
Investigations
o tio of o o i t t t i o b t
Management l o l lo l y
b t l
io i o o i oo o t
bm b lol i i l
mo
oot fo m tio o o o
open or close space
229 •
MINDMAP9 43

e tea
a c ea e eate May predispose to a a
c pete t p ppe c t a a a a e c e c
e eta
ta t e ce ta a a
c e a a a a
e p e
Aetiology t t pe eact e e p
e e t e t t
t c a t
a a p ace e t
c a p ace e t
c at
t t e e
t pe
t t e
e et e e a a
t p e p e ta
accept c pete t p
ea a e a c
a e a c e
p t e a e
ct a a a a
c ea e e et app ea c app e a
te a a t act
e e e
e ac cea
e e ate t t e pt
e t ac a
e eta
pat e t t cat
e ect e t a e a
ate a cep a a ea t
e t te t Diagnosis e ce c e ce
t a a
pe ap ca a ap a e e e t act
a e app a ce
a a p t
ea ea
Management e
e eta
ct a ea ea
eta t e t c p
eate e eta
e t ca ac a p p
ca a e
a
c e a e
a e a t act e app a
pat e t a e e eta
e e ce c ea
t at c ac a pe p t
e

e et c p ete e ce
e p c e t t a
a a ace
ta e c ect
t e t t
te c a a e a
a e ate ete t
• 230
43 MINDMAP10

erio ont st t s o o er inciso


c n c ie e e e to e e in
n ib r tis ce en
ret ine or c inic
ecto ic osition o toot sin
b o nt o o erbite
e incisor Investigations
Aetiology incisor inc in ntio
s ern er r
o nt o cro in
see re erse o er et or ore incisors
root st t s
r io r ic
s ern er r
Prevalence

May predispose to
erio ont tr

Incisor Crossbite

o nt o oe erbit
Stability of correction
no n ib r is ce ent s bse ent n ib
no erio ont tr cce t n re ie
ini o erbite
n ib r is ce ent
Management
erio ont tr
e tr scttoCenco r e o erbite incre se o r b e o erbiteroc i ine
incisor e
s rin
consi er
osterior c in
o re o er
i e e i nc
cost ess t n
231 •
MINDMAP11 43

? familial tendency
profile
A-P
skeletal vertical
growth direction / extent extent greater in males
clinical
? mandibular displacement
periodontal trauma
Investigations gingival recession
amount of overbite
site and extent of crowding
ANB < 1º
long mandible lateral cephalogram growth pattern
anterior position of glenoid fossa radiographic ? dentoalveolar compensation
skeletal III
short and / or retrognathic maxilla tooth number
DPT
short anterior cranial base tooth position
mandibular displacement
Aetiology
retained BA | AB
excess
mandibular growth
forward pattern
restraint of maxillary growth

Reverse Overjet

see ‘incisor crossbite’


positive family history
accept and monitor
? direction / extent of future mandibular growth
Frankel III / chincup
mild/moderate Class III (maxillary retrusion)
growth modification facemask (protraction headgear) year
s
su ess ul at year ollo
no ad erse e e ts on
one lates ase o y omati ar and anterior mandi
BAMP
s eletal mo ement t an a emas
retrocline 21|12
Management ? procline 21|12

camouflage usuallly extract 4’s only if prognosis favourable
Class III intermaxillary traction avoid –6 extrusion
fixed appliances
severe skeletal III
surgery fixed appliances
align and decompensate arches pre-surgery
patient not keen for surgery
accept and align arches
severe skeletal III
• 232
43 MINDMAP12

red ced s e eta


anteri r andib ar r t r tati n
Aetiology
a s eractive i er i er i ine
absence cin red ced and
er i eve
c inica in iva tra a
incisor mobility
cr din
Assessment red ced
red ced
radi ra icatera ce a ra
assess incis r
inc inati ns

be are er arc e tracti ns n


Treatment planning
s e r c inati n a be stab e

i ed a iances it it t s inc de s
Increased Overbite
ti it arc incisor
es intrusion
di ic t
n molar eruption
cervica ead ear Methods of overbite reduction
ec anics t intr de incis rs molar
isted ab extrusion
ve
ass II III inter a i s ar e astic
lower incisor proclination

at anteri r bite ane s n e eta I i d II


ncti na a iance r in atient
i d derate s e eta II Management
i ed a iances a ter (non-extraction approach
e tracti ns favoured for Class II
i ed a iances
n e tracti ns division 2 malocclusion)
i d derate s e eta II
t r e er incis rs
c rrect interincisa an e n n r in
atient
derate severe s e eta II
i ed a iances
verbite dee tra a
s r er at anteri r bite ane n
Stability
b nded retainer a ata t in ass II div
233 •
MINDMAP13 43

in re e A n e et
o n r n i r ro t p ttern
en o eno ton t e t r
i it in er p i ier
o on in e r i oo Aetiology
re e it non ne triti e in it ( ) in re e A n
oe not pre i t e n to o io e et
ini
etri AOB e i tit i
Assessment
e t ip n op ton et r t
ite e i re o eo r e e op ent
in re e A
r io r p i ter ep o r
in re e

Anterior Open Bite (AOB)

AOB i
t i it n i eept
re ie e ro in i n r e on
i e r it re e
po iti e or ne ti e rein or e ent
ot
po terior o er e i r p int
e et i p ero t e or i i tion
n tion pp i n e it ppin
r AOB
e Treatment
r n e it ip e tr inin
i p in p
A it ppin
AOB i
re ie e ro in
o e
i e pp i n e it it o t A
oi e tr ion
AOB e ere
ro t o p eter er
t i it
• 234
43 MINDMAP14

lass
s eletal
emimandibular pertr
c ec centrelines mandibular displacement di it suc in
inclinati n p steri r teetclinical Aetiology cr din in sciss rs bite
cr din Assessment
earl l ss in
c ec c nd lar anat m radi rap ic cr ded arc buccal cr ssbite

unilateral
bilateral Classification
it it ut mandibular displacement

mandibular displacement
Treatment needed
ma predisp se t steri r r ssbite

sprin r scre c nsider relie


secti n n cr din e tract se erel it ut mandibular
e displaced t
i ed applianc t in cr ssbite
teetdisplacement
it cr ss elastics accept teet unilateral buccal cr ssbite d buccal interdi nitati
rinds Stability n displacin c ntact s
it midline scre it mandibular displacement
a urable r t
uad eli ma be m re success ul
bilateral buccal cr ssbite Treatment n abits
t an in mi ed dentiti n usuall accept
see ind ap
c nsider rapid ma illar e pansi n
i ed appliances
it mandibular displacement
e pand l er c ntract upper arc c mplete unilateral
sur ern mandibular displacement lin ual cr ssbite
c mplete bilateral
usuall accept
see ind ap
c nsider sur er
235 •
MINDMAP15 43

eletal o o it la alo l io
o i le ole o a e oi to il
aetiology alte e ea o t e e to o t eat i g
o t ti e
lo to g e o itio
a ti e e to le t e ai
ll le gt o e i i o
a o e a
elate i te o i al gi gi ae
li i al eat e
att a ti e ile e i i o e ge o to at e lo e li
oo ile ae t eti
late al e te io at lea t to
a o al o i o
a o iate a i la i la e e t
i e tigatio i li atio o al eg e t teet
t i e o gi gi al ti e a al o e

Buccal
atie t ot ee o t eat e t
a e t a o eito lyi g eletal li ely to o e
it a i la g o t
i a y ea ly i e e titio
lo e a io
it i li e e o o ea lia
y ie li eit eai g a a ti atio ay e o le ati

ot o et tie e o e ti e e i e to o e e i e e
e e e ea ly i e e titio
a ti ate al ola toot i t
a age e t e a io e i e
to a e o e o e
a eli
a t to gi e o e e a io o te io ly o a te io ly
o ie ie et lo e a io
e otate ola g o
o e o a e a lia e o e e t e ay
ilate al o ite e ia ia te a al i te

i o e o e t
eletal o ite it e alatala io t e et o ei ei g al o
g a a tee e o e age etai o o
i e eaa lia
t o g ela ye e te
it alatal a
o e age o i
a ea y e

i e o t o teoto y ig t o o i e loo
eletally at e it e a io e i e
a e ate a o
e a io ally ta t y it i ay o ge
t a o e o t o teoto y t it o t
a illa y o a t e
i e i g o a illa agai t o ly o t ti e e i ta
o e ta le t a gi al i e i g o a illa

eletal o o it la alo l io
o e i le ig to g e o itio
o t ti
aetiology ea ly lo o
lo al

e ie e o i a o aly
a
Lingual o i e e t a tio o i la e toot
i gle toot e te
a o ea i e
o i e o i atio o ealatal
te o e e t teet
o a
al o e ee tet o lo ae teet
t eat e t
a el e a lia
g o t o i i atio
el ligati g i e a lia
ae ot a lage
tio o e t a tio
e e al teete te a o ea o it eo o ti
allo o teoge e i a i toge e i
ay late y e io
i lie et i e y e a
i t a tio o teoge
toot o e i t a to o e e
o li atio ai ly il a t a ie

o t og at i a i
ge y la at a e e
• 236
43 MINDMAP16

ti actoria
orward andi ar growt w ere a i ar
growt as sto ed aided e i ress r c inica se erit o crowding
transse ta i res s
Investigations radiogra ic
anterior co onent st d ode s
o occ sa gingi
orces a and or occ saAetiology
orces
ead to esia igration
o osterior teet
red ction in intercanine widt
e ert esia ress re d ring n er tio
re ent osterior teet ot ird
ing os ar
dista in res onse to ot er orces

Late Lower Incisor Crowding

nti ates teen


crowding a increas e
acce t and onito
r
deter ine ate o d t ir
o ars i resent
Management
i d crowding
c ear a rigne
inter ro i a strig a in iance t era
i ed a iance a
onded retainer
i ed a iance a ia ing a
e tract a ower incisor
onded retainer
237 •
MINDMAP17 43

c olog cal
auma c
ae olog mul fac o c alo e
occlu al la
a e o e o
lac g co ac
e
TMJDS a afu c o
e ma ageme
ma ula g o l e a
ae olog Prominent chin
a e o g o o a o occlu al e u l a o
ma ageme
o o o o c gua a ee

o o o c
o og a o c gua a ee

om e a

eam a oac
camouflage
full e e of ma ula g o
u ge
la e ee
fac al occlu al a l
ela e afegua e
fac al
o og a
e al
u l ca e fo
mo el u ge
f ma lla u ge u mo
la ele
a cula o mou e e ga o

com a e ual a a o mal Treatment


a a ma c e fo age ecge al e e ace
ce alome c a al e
la e al
com u e og am ce alog am a og a c
l g al e of cle omage
ce alome c ac g
mo mage o ual l el oo
of a ou u g cal la
fac al mage
a e a
m l g
a e e l el of le eou come
c o
c olog cal a e me
ma a fe la e mo eme
o u l ca e e of u mo el o e fo ma o leafle
e ac o
ecom e a ee u g cal
al g coo o a eo ao c e c
ec e a e cal c o o o
e u g cal ec a gula e a all oo u le ac e eg a e oo
m l a e e c e
ou a le eel a c e
o u ge
occlu al e l g l g o ela c
follo u m mum ea
mo e u g cal mo e
a l
m mal of ue a
• 238
43 MINDMAP18

dia ete
periodontal disease edi al in l en e on periodonti
periapi al pat olog istor attendan e
dental
st treat ent
t o r s or e
r ing Aetiology aries
lini al dental stat s
nail iting a it sin s
Investigations gingi al s elling le
pen e ing
andi lar displa e ent e a ination periodontal stat s toot o ilit
o l sal periodontal po eting
la o posterior teet
andi lar displa e ent
o l n sio
e tent o posterior s tppor
or periapi als
radiograp i
pat olog al eolar one le els

ri ting n isors

assess dental and periodontal prognosis


assess patient s is es
s o ing essation ad i e
ontrol periodontal diseas
e
root anal t erap and periapi al s rger
re o e st t o r
ridge
e tra t teet o repla
opeless prognosi
s e it partial dent er
Management
i plants
dis o rage a its
o sal ad st ent re ao ilitationsal splint
ens re periodontal ondition sta le pre treat
a oid ands
se stainless steel ligat sre
in periodontall in ol ed dentition lig t or es
reg lar periodontal re all
ort odontis per anent retention
aest eti ra ets
onsider applian e
aest eti s in ads lt lear aligners

ling al applian es
239 •
MINDMAP19 43

Candida

Red palate

Bond failure

Overextended archwire T
T

Broken vacuum-formed retainer

Broken bonded retainer

T
• 240
43 MINDMAP20

too o
or e
operator e e e
a r ter ere e
app a e e
orre t re a eter o pr
ort a p ate o ta t
aet o o a ato a
reta e root
ter tte t re o e at ea
pat e app
t a e ear
orre t po t o o pr re tort o
o a ter ere e
a t
t pp
o
tr o
Slow / no tooth movement rotat o or e tr o
or e
pre re
p
te o
ta e
reorpt o
o e
epo t o
appropr ate or e e e
orre t app a e e
appropr ate app a e ear
a r
a a e e t re
re o e o tr o t o a
r a re o a or
reta e ea e
root a o tor
o ra e a t

e o e o
t a tor a

o pre e e ort o o t treat e t


t e e e
ea or e o t ar
aet o o r pre o tra a t o tor o
a tortt e e e e
tho ont ll a toot orp o o
n e oot
ra et pre r pt o e at o
e o t on o e e e
oot o e e t a ar re e e e

e ate ro e
pre treat e appropr
t ate ra o rap e p a r

p at o or
a o ea or e
a pra t e
o t pa e t
treat e t ear re orpt o o
o t pro re ra o rap
o tor o o r
e root
t a a t erap

e ere re orpt o o e
pport t e rea e treat e t ter
p a ate treat e t
e t e
oro a a o t t e a ra o rap
or e o tor
aet o o o o e ere re orpt o o ear
a a ro t po t treat e e t o treat e t ra root
o rapa a t erap
el e appropr ate rete t o re e o e p t
re ap e rete t o a part o or e o e t
pre
pre e t o treat e t
t re po t t ee or o ter rete t o
a ept a o tor
a or e po ret a e
ea
a a e e t re o a ea e t t o t a r ate a a o
t pe
rap aro e
p o pete t a ter o er et re t o
re treat o pro e pa e e t t o
ar e a terat o a tero po ter or po t o o o er
e e ere rotate teet
appropr ate rete t o re e pa te
per o o ta o e e t t o
e t p a pa ate
o ter
re o a e
t o
rat o
e o ter

appropr ate o er t ort o o t t


241 •
MINDMAP21 43

CLP fa il i t
eneti left palate (CP fa il i t )
Aetiology
eate i it CLP t an CP
en i n ental

p i a e palat
e nda e palat
nilate al
Classification ilate al
Ca a ian eplet
ale Prevalence
e n in eplet
left ide t lef

Cleft Lip and Palate (CLP)

et nat i a illa and andi le


inte di iplina
ed ed ppe fa ial ei t
eletal pa ental n ellin
in ea ed l e fa ial ei t
feedin ad e i
Cla ealt i it
ne natal p e enti e dental ad i e
ante i
ite lip nt
p te i l al plan e
palate nt
late al pen ite
Clinical features a e pee peeea in t e ap
a ent
all left ide p i a dentiti n e i e lip ea
i aped p e enti e dental ade dental
i t eat ent

tated tilted t a dental


d left
Management e t in i al ite
e all dela ed e pti n
e t ea inte it
e all all t t i e
al e la ne
ala a e t pp
pe n e a teet aft ea all e pti n epla e
i in teet i edpe anent n idee i e ape n
all pa e l
dentiti n
ind palatal p f f n
l e na al fi t la
e t ent eline
elie e din n n left ad ant and l e a
pe anent nded etenti n
late teena e ea t nat i e
• 242
43 MINDMAP22

sweetened or fruit-flavoured drinks


sugar added to drinks sugar
in some lactulose-free milk
frequency Aetiology
prolonged bottle or feeding
on-demand nocturnal breast feeding eruption sequence
submandibular ducts
Pattern of caries bow upper lip
gravity
tongue position

Managing cooperation (see Chapter26)


Nursing and
Early Childhood Caries

clinical evidence
dietary factors
social history
use of fluoride Risk assessment
plaque control
saliva
medical history

diet diary (4 day) positioning


toothbrushing instructions parent and child parental undertaking of brushing parenting skills
managing unwilling or uncooperative child
toothpaste – 1450 ppm
fluoride
topical-varnish
Treatment
prevention only
restoration seal +/– partial caries removal
(see Chapter 24)
complete caries removal
extraction
243 •
MINDMAP23 43

c ri s
c inic i r nc ious nt tr ction
r ious r stors tion
h o stic n
nt ctors og n sis i r ct
oor or h g in
ntinog n sis i r ct
ti ing
r u nc
sug r Aetiology High caries risk i t r ctors
in u t uori r gi ns oor nt tt n nc
soci histor
i it nt s s ir tion
in ro ri t tooth st str
uori us
in ro ri t ti ing o uors
u contro

s i rosto i

ic histor

igh ri s is o sc nts

onstrn tio
toothbrushing instruction
isc osing t b ts
tooth st str ngth ic
high r str ngth tooth st
su nts n ti uori ing
rnish ic tion
oo
rin i t ic
ch ing gu s nt r s stor tion
issur s Treatment
nts / preventive care Restorative care nt rior co osit s
sth tic or
sug r r icin ic bri g or
• 244
43 MINDMAP24

pain to hot and cold


reversible pain transient / short duration
pain ‘sharp’ in nature
pain on chewing
site Characteristics irreversible dull ache – continuous / long duration
onset spontaneous pain
character abscess
radiation sinus
History
association swelling
non-vital
time course tender to percussion
exacerbating perifurcation radiolucency
severity internal resorption

Pain Control and Treatment Planning


for Carious Primary Teeth

identify caries (clinically and radiographically)


problem list depth of caries
treatment options per tooth
prevention
visit-by-visit plan behaviour management
Treatment plan
temporary/definitive treatment
child’s cooperation
parent’s expectation
influence by parent’s engagement with prevention
further pain and symptoms
dental team’s expertise and experience
prevention only
indirect pulp therapy
seal in caries Hall crowns
fissure seal
Definitive treatment composites
stainless steel crowns
complete caries removal pulpotomy
pulpectomy
extraction

paracetamol
pain control
ibuprofen
introduction to dental procedures
Acute treatment reduces Streptococcus mutans count
why
temporization cement fluoride reservoir
increase comfort for eating, brushing

reversible pulpitis Glass Ionomer Cement


how poly-antibiotic dressing
irreversible pulpitis + Glass Ionomer Cement
245 •
MINDMAP25 43

reduced intake of food and drink


pain
sick child
acute
temperature raised
red, swollen face
lymphadenopathy
Type
sinus
mobile tooth
halitosis
chronic
discoloured tooth
pain
lymphadenopathy

dehydration
temp > 39ºC
culture and sensitivity
Hospital admission
trismus
complete closure of eye
breathing and swallowing difficulties

Facial Swelling and


Dental Abscess

extraction
incision and drainage
culture and sensitivity
IV antibiotics
Hospital treatment
fluid maintenance
saline mouthwash
pain control
chloramphenicol eye drops

extract
remove cause
RCT
incision
dentist Management
RCT local drainage
extract
oral antibiotics if systemic involvement
• 246
43 MINDMAP26A

a l an io t
tain ntall h i all i all
in i ation o o i ati nt
a l int ith ntal a
inhalation h o o n
lo al ana th ia annot
ation h oni ti o t l ona i a
otional i t an
lat n n i
ont ain i ation
i t t i t o nan
ntal i l ll i a
i al ati nt t at ith l o in l hat
a il a
History ntal la o holo i al o
na l to oo at
ntal h i al o
lo nt la o nition a t in tion
lo al ana th ia tiin anato i al a iation
all
t l n oo ati a
in i ation
an io o n o ni ati
Pharmacological i ni i ant i al o
n al ana th ia ot t lo in h
an o i al i
i iat o h n i o al
h alth oo ati ati nt
ith ini al ntal n

ont ain i ation i al on ition h


n al ana th ia
ina i a l
a l an io ati nt h n
a i ha io ana nt
holo i al otional i at it
in i ation an o ntal h i al i al
h n oo ati hil
o al ation a ot t lo in h
an o i al i
oo ati ith ini al ntal n
ont ain i ation
i al on ition h ation ina
Preventive plan (see Chapter 22)

in i ation an ati nt
non al
ont ain i ation non

in i ation an ati nt
t ll ho o
ont ain i ation non
mother
i iat l
father (if ...) who has parental
on i t nt
step-parents (if ...) responsibility Consent l a
relative, friends (if ...) Communicative
o iti in o ontin nt nt
social services (if ...)
in i ationan ati nt

ont ain i ationnon

in i ation an ati nt
i t a tion non
ont ain i ation
in i ation an ati nt
oi ont ol
non
ont ain i ation
in i ationan ati nt
a ntal n a non n
ont ain i ation
247 •
MINDMAP26B 43

lo el e tee
Peer pressure
tool it bullying ocial e ia
iet iary an tailore iet a vice
patient contract Preventing disease
cognitive behavioural therapy
behaviour change theorie recreational rug
otivational intervie ing
Habits o ing
alcohol

perio ontal i ea e
u ing concern to otivate Dental diseases carie carie ri a e ent
veneer aci ero ion
Concerns about aesthetics non carie tooth ur ace lo
bleaching attrition
previou entoalveolar trau a
ortho ontic treat ent
pre olar e traction Consent co petency

he n iou ole cent


intravenou e ation
po itive rein orce ent Conscious sedation inhalation e ation
tell ho o ng
gui eline
chil a ent ocu o control e g Behavioural
management
increa ing el e tee top han ignal General anaesthetic (see Mind Map 26A)
techniques
onitor having a ay in eveloping
blunter the treat ent plan
giving a han irror pa t trau atic ental even
t
care ul ue tioning
perceive re e bere trau atic even
cognitive behavioural therapy Assessing anxiety
pecial techni ue pa t trau atic e ical event e g
he an

ea uring an iety
ealing ith parental eparation
an co ple a ily relation Changing
hip relationship with parents
groo ing
neglect
Self-guarding
abu e phy ical e ual e otional
orce arriag
e
43
• 248

f
s
l
a i a
n
n s
s o
i o
n
s
s i
l
n
o i
c a l s
s i
s f
a
l i
c a
a o
i
i
l
l o
i
c i i
o
a
o
n i
n i
l o
a
o
a n
o a
l l
a a
i a
i
s s
i s
o i
n i o
c
s s
s
s i
a
s s n s
n a s s s i
i c
o
i a l s c
l
l o
o
i s
s
n
o o l f
f
l
a i
n s
o
o s c o a
o o l s c o
o s
i i
c o
n a
l l
a s
c s a
f i
c
l l
i s n
i
a c
a
MINDMAP27

a
s s i
n a
l n
a
f
o
i s
n
n s a
l
a
o s o
a
n n
s
o
o a i
s
n a l
a n
a
a a
n f
i n
a
i n c
o
i l
a
n l i
o
l o
n s
o s s
o l
c s a a i
c a
a c
c c
c s
s s
s o c
a I n e q
u al i t
y S
c h o ol a n d s o c i
a l s u p p o r t s e r v
i c e s i
c a l a s s s s n s a o l
o i
n s
o n i
n c o
o
s f
a a i
a
f i
s i
l
i i
l
f
a
i c a
a n i
o o
i
s s
n
o
c i
f a n
l
o
i
n a
n
i
o
f s
i
c
s o
s
o
l n a
a
i
f
o i
a
l
l
n
l l
l o
a
i
c
l o
s
i
i
l
s
o a
s
i a
n
n
o
n
s a
f
f
n
o
n
o a
R
o
s E
n
n
n
e
a
a
i
B
l
s
c l
a
i
l
s
l
a
r
i
o
i
i
e a
n
c
i
l
a
s
o
c n
h
t
e
u
a
e s
a
s
s
l
d
t
i o
o
of
v
i
n
a
nc i
d
a
ou o
i
l
e
a
s
a e
a
i
s
l
c n
d
n
r
c
c
o
a
t
e a
n
a
a
o
a
e
a
l
n a
f
l
a
n
p
c
m f
t
a
s
a t
f
s
a
a
n l n
n
o
c
i
c
a a
o
m
a
e
g
r
o n
s
e
a
e of
o
l
o
m
i f
e
li
o e
c
sn i
n
o
c t
i
s
i c
i s
n a
i s s
i s
i
f a
f ic i
l i
l i s
a n
i
n i
l a n i n n
i
l a l i a c o o s o i
n o n s n o
C H
on i gh
s c i
c
ou a r
s i
e s s
e r
d i
s a k
t i
o n c
i
c
n a
l
o
c a
s i
s
l a
a
l i
i
a
a
a i
a
n
c
s
i s
s o
s n
n
a
n
o
i i
a
n l
c
o o
o o
c
f
i
i o
s
a
n i
n
i
n
c
c n
c
c i
a
s
i l
s i
a
o i
n
c
n c
l i
o
s i
l
o
s
f
a i
o l
i
n s
i c
c
o
s a
i
n f
o
l s
c
s
c i
i
o
a
n l l
l a n a
s
o
a i
o
n f s
a l
i
c
i
a
c a
a a l i
n l
i s s i
i
i
c
o s
n
n c a
i n a c a
n f
n
i
f
i
n
o s
a
f
n a
l
c
s n
a n
a
a l a n
s
a n s s
l s s
i i
a i
s
c i
s
c a s i
a
a i
i
n
c i
a n f
i n i s s f
o
M e d i
c a l c o m or b i
d i
t y
G e n
T
e
y r
p
a e
l s
a o
n f
a e
d s
i
s t
h a
e b
s
i l
i i
a
t y
249 •
MINDMAP28 43

histor
infecti e endocarditis clinical exa
leedin Identify extent of disease radio ra s h
i aired i nitBroad implications dia nosis
increase likelihood of extraction a oid infection hich a ro le list see ha ter
rather than co lex restorati e careco ro ise eneral health

direct conse ence of treat ent edication


Impact of medical treatment on oral health
direct conse ence of disease

ana e ent of ental aries


in hildren ith o on Liaise with medical colleagues
edical ro le s

i act of edical treat ent on coo eration


Cooperation of child
decrease ca acit to ndertake lon and co licated treat ent lans

toolkit
Comprehensive and rigorous prevention
a ha e to odif as
of edical treat ent
• 250
43 MINDMAP29

? accident, if so how
does injury fit the age and clinical findings
is explanation consistent with injury infection
? delay in seeking advice pain
required if
? story of accident mobility
relationship between parent and child ? Child physical abuse occlusal interference
to other people concussion and subluxation observation
reactions
to medical or dental examinations spontaneous reposition
demeanour of child lateral luxation
extract
comments by child / parent that give concern spontaneous reposition
about upbringing / lifestyle of child intrusion
extract

extrusion extract

avulsion replantation not indicated


Treatment
1 week
1 month
loss of consciousness then 3 months
when? symptoms
if retain luxated tooth review
where? History colour
what surface? sinus
how? tenderness
lip/tongue radiograph 6 monthly
swallow no periapical pathosis review
fragments
inhale discoloration extract
pathosis
spat out RCT with zinc oxide paste

general demeanour
tetanus status to other people
reactions
to medical / dental treatment
congenital heart disease / rheumatic fever Displaced Primary Incisor
immunosuppression Medical history bruising
bleeding disorders extraoral abrasions
allergies lacerations
Examination ? child physical abuse
burns
bites
fractures
tooth
mobility
intraoral dentoalveolar segment
occlusion
periapical concussion and subluxation
periapical lateral luxation
periapical intrusion radiographs may occur in 50% of cases
Investigations
periapical extrusion intrusive trauma most damaging
periapical avulsion enamel
hypomineralization
not indicated vitality testing
enamel hypoplasia
Effect on permanent successor crown dilaceration
crown / root dilaceration
root dilaceration
odontome formation
disturbance in eruption
sequestration of permanent tooth germ
251 •
MINDMAP30 43

swelling
previous treatment extraoral asymmetry
previous local anaesthetic occlusion
Dental history
attitude to lip laceration – check
treatment soft tissue for teeth fragments
Examination colour
mobility
intraoral teeth
percussion
congenital heart sensibility
disease / rheumatic fever occlusion
immunosuppression bone
bleeding disorders Medical history
allergies
tetanus status

Fractured Immature Permanent Incisor Crown

how
when
where
check for root and alveolar fractures
fragments History Radiographs periapical
check periapical status
previous injury

loss of consciousness and


chest x-ray missing fragment

smooth/restore clinically
enamel fracture monitor vitality
radiographically
reattach fragment Review monitor root
temporary adhesive bandage enamel / dentine fracture maturation
Treatment
acid etch tip monitor adjacent teeth
partial pulpotomy enamel / dentine /
complete coronal pulpotomy pulp fracture
calcium hydroxide pulpectomy
• 252
43 MINDMAP31

a ri i ec i e e car i i
pre i rea e
i ppre i
Medical history Dental history pre i l cal c a ae e i
lee i i r er
a i e rea e
aller ie

e
ere
History
pre i i r elli
l c ci e e ra ral
a e r
ccl i
e i
c l r
Examination
ili
ee
i ra ral perc
e r c re Structured
i r and examination
history r
rac re er a e ci r e i
ccl i
e

pli re al
i r e i ili Review rac re ee
periapical
i r e i ili a ace ee Radiographs
a eri r

re c i
i ili a i lile i le irepli e i
Treatment
c l r e i i e a
ie
253 •
MINDMAP32 43

previous treatment
at risk of infective endocarditis Dental history previous local anaesthetic
immunosuppression attitude to treatment
Medical history
bleeding disorders
allergies

how
swelling
when
extraoral asymmetry
where
occlusion
extra-alveolar dry time
History soft tissue
storage medium
colour
total extra-alveolar time Examination
intraoral mobility
previous injury adjacent teeth
percussion
contamination
sensibility
place in normal saline/milk
tooth in medium
establish apical status

decoronation
denture
The Avulsed Incisor 30 min DT
Early referral to
BSPD
resin retained bridge interdisciplinary team open apex replanted – may revascularize 90 min EAT
orthodontic space closure IADT 60 min DT
transplant
extirpate prior to splint removal
Endodontics
all others initial dressing antibiotic / steroid paste (days 0–10)
change non-setting calcium hydroxide 3 monthly
or initial dressing with non-setting
repair calcium hydroxide (days 7–10)
infection monitor resorption
ankylosis
Review
obturate if no progressive resorption
monitor sensibility of adjacent teeth

flexible splint 7–14 days periapical


Radiographs
may need antibiotics already replanted anterior occlusal
chlorhexidine 0.2%
irrigate socket
Treatment
replant
flexible splint 7–14 days milk or normal saline
may need antibiotics
chlorhexidine 0.2%
replant
flexible splint 4 weeks
dry > 60 min
may need antibiotics
chlorhexidine 0.2%
• 254
43 MINDMAP33

spontaneous
aetiology familial
syndromic
mobility
double primary teeth
problems tongue ulceration
hypodontia affecting
permanent successor Delayed exfoliation Natal / neonatal teeth nipple soreness
of primary teeth
ectopic successor mobility extract
abnormal physiological resorption nipple soreness extract
infraocclusion / ankylosis treatment carmellose paste
tongue ulceration smooth tooth
extract

Disorders of Eruption and


Exfoliation

familial
Generalized high birth weight
premature eruption race
of permanent teeth
sex
neutropenia maternal smoking in pregnancy
neutrophil defect reduced maternal physical exercise
histiocytosis Metabolic lower socioeconomic status
hypophosphatasia Premature loss
scurvy of primary teeth
Ehlers–Danlos syndrome periodontal disease
psychosis pre-term
self injury primary teeth
congenital insensitivity to pain very low birth weight
non-accidental injury
chromosomal
nutritional
hypothyroid
primary and permanent teeth hypopituitarism
hereditary gingival
Generalized delay in eruption fibromatosis (HGF)
acquired gingival overgrowth
generalized cleidocranial dysplasia
supernumerary / odontome
ectopic
permanent teeth
localized cystic follicular change
crowding
thickened mucosa
255 •
MINDMAP34 43

first permanent molars mineralize around birth


maternal illness
pre-natal
early admission to hospital
prolonged delivery family history
natal primary teeth affected
assisted delivery Amelogenesis imperfecta
special care baby unit other permanent teeth affected
Hypomineralized / hypoplastic Dentinogenesis imperfecta
meningitis generalized
measles type
respiratory illness post-natal
childhood illness
chronological duration and time
drugs

endemic F levels
Fluorosis
toothpaste ingestion

occlusal
interproximal
Caries recurrent caries
oral hygiene Poor Quality First Permanent Molars Restored
periapical pathology
diet
antisensitivity toothpaste
sensitivity toothmouse
discrete creamy or brown hypomineralized GIC temporary
areas affecting permanent molars and incisors preventive oral hygiene
Molar incisor hypomineralization
post-eruptive breakdown measures diet advice
atypical restorations ? fluoride supplements
composite
restoration stainless steel crown
adhesively retained copings
Treatment
assess all 6’s
buccal 6’s ensure all permanent teeth
check cervically
lingual 6’s caries dental status especially 5’s and 8’s present
diet diary clinical extraction
timing more critical for lower
first permanent molars
occlusion
Investigations compensate
6’s caries / hypoplasia
number, position, status of unerupted teeth DPT
radiographic
? furcation calcifying developmental state 7’s colour brown worse than creamy
periapical / bitewing What is the long-term prognosis? size of area
cusps affected
• 256
43 MINDMAP35

50% chance of damage to permanent successor hypomineralized


Infection of primary teeth localized defect
hypomineralized Trauma to primary teeth hypoplastic
localized defect
hypoplastic

special care baby unit


Post-natal
can affect 6’s

age started brushing


type of paste
amount of paste
Fluorosis
swallowing of paste
supplements
generalized
prolonged delivery
Natal assisted delivery
can affect 6’s
Tooth Discoloration,
family history Hypomineralization and Hypoplasia
primary teeth affected
Amelogenesis imperfecta
permanent teeth affected
generalized

family history
primary teeth affected
Dentinogenesis imperfecta
permanent teeth affected maternal illness
generalized Pre-natal early admission to hospital
can affect primary molars and 6’s

type
duration and time (chronological)
drugs Childhood illness
can affect 6’s and other permanent teeth
generalized

amelogenesis
requires a primary predecessor inherited
dentinogenesis
trauma to primary teeth Localized Generalized
systemic illness
infection to primary teeth fluorosis
257 •
MINDMAP36 43

microabrasion
composite
Diffuse defect veneer
porcelain
external bleach

how does it work?


what do you do?
indications?
Techniques
legality?
advantages and disadvantages
long-term outcomes

Treatment of Mottled Teeth

composite veneer
external bleach
leave defect
microabrasion (how superficial within the enamel is the defect?)
porcelain veneer in late adolescence Demarcated defect
composite veneer remove defect
• 258
43 MINDMAP37

taurodontism
accessory roots Root abnormalities
pyramidal roots

0.1–0.9% primary
prevalence
3.5–6.5% permanent (excluding 8’s)
1.4–4 times more common in females than males
hair, teeth, nails, skin, sweat glands
most common hypohidrotic X-linked
usually male
multiple congenital absence of teeth
fine, spare hair with shaft abnormalities ectodermal dysplasia
dry skin
frontal bossing accessory cusps
maxillary hypoplasia Other crown abnormalities invagination
syndromes
thin lips, little vermillion border evagination
cleft lip and / or palate
Multiple missing teeth
1:700
delayed eruption Down syndrome
microdontia / hypodontia
Ellis–van Creveld syndrome
Multiple Missing and
Reiger syndrome Abnormally Shaped Teeth
Orofacial digital syndrome (types I and II)
reassure child and parent
eliminate pain
genetic counselling
restoration of aesthetics management
provide adequate function
maintain vertical dimension
formulate interdisciplinary treatment plan

high chance of abnormal number


0.5–1.6% primary dentition in permanent dentition
prevalence
0.1–0.2% permanent teeth
primary check normal resorption
space in arch
Double teeth aesthetics
treatment
morphology of pulp
permanent chambers and roots ? surgical division
mask with restorations
extraction
259 •
MINDMAP38 43

autosomal dominant
Inheritance autosomal recessive
X-linked

1 in 10000 Incidence

epidermolysis bullosa
tuberous sclerosis
pseudohypoparathyroidism
trichodento-osseous syndrome
occulodento-osseous dysplasia Generalized disorder association
vitamin D dependent rickets
Amelo-cerebrohypohidrotic syndrome
Amelo-onychohypohidrotic syndrome hypoplastic
mucopolysaccharidosis Main clinical types hypomineralized
mixed

Amelogenesis Imperfecta

adhesive restorations
alleviate symptoms SSCs
maintain / restore occlusal height cast onlays
improve aesthetics Treatment modalities
full crowns
maintain positive dental attitude Treatment principles composite veneers
prevention porcelain veneers
minimize likelihood of dental anxiety
interdisciplinary care
• 260
43 MINDMAP39

1 in 8000 Incidence

Inheritance autosomal dominant

osteogenesis imperfecta
Ehlers–Danlos syndrome
brachio-skeletogenital syndrome
Generalized disorder association
vitamin D resistant rickets
vitamin D dependent rickets
hypophosphatasia

type I (associated with


osteogenesis imperfecta)
Main clinical types type II (not associated with
osteogenesis imperfecta)

Dentinogenesis Imperfecta

shortened roots
progressive pulp chamber and canal obliteration
Complicating factors wear
spontaneous periapical abscesses
bulbous crowns

adhesive restorations
alleviate symptoms SSCs
maintain / restore occlusal height Treatment principles cast onlays
Treatment modalities
improve aesthetics full crowns
maintain positive dental attitude composite veneers
prevention porcelain veneers
minimize likelihood of dental anxiety
interdisciplinary care
261 •
MINDMAP40 43

diet diary
study models
Dental investigation antisensitivity toothpaste
photographs
high fluoride paste
diagnostic index
low abrasive paste
Desensitization fluoride varnish
fluoride supplements
sugar-free gum
bonding agents

bulimia / anorexia
drugs for reflux
Medical treatment and investigation
surgery
oesophageal monitoring
limit acids to meals
reduce frequency
finish meal with alkaline foods
Dietary advice
avoid acid foods at night
avoid toothbrushing after acid foods
check pH of medications / mouthwashes

citrus fruits Dental Erosion


tart apples
vinegar and pickles
yoghurt foods and drinks
fruit juices extrinsic
carbonated drinks
vitamin C tablets
swimming pools leisure
medications and oral hygiene products
gastro-oesophageal reflux Aetiology
oesophageal stricture
chronic respiratory disease
liver / pancreas / biliary disease
overfeeding
failure to thrive gastric acid intrinsic
learning delay
cerebral palsy
rumination cast metal onlays
anorexia / bulimia direct
Restorative treatment composite
drug induced indirect
porcelain
• 262
43 MINDMAP41

eruption gingivitis
acute / chronic gingivitis
chronic periodontitis
foreign body entrapment
local
ANUG
haemangioma
reactive hyperplasia
factitial
Bleeding hormonal
diabetes mellitus
anaemia
leukaemia
systemic platelet disorder
clotting defects
anticoagulants
scurvy
HIV

7–12 years old


s o assess e
Assessment
Gingival Bleeding and Enlargement de ee
1 –17 years ol
d
s ores 2 1
s o
s l ed as er odo al e a
de ee
s ores 1 2

hereditary gingival fibromatosis


congenital mucopolysaccharidosis
infantile systemic hyalinosis
puberty / pregnancy gingivitis
plasma cell gingivitis
Enlargement infections – HSV
acute myeloid
leukaemia
preleukaemic
haematological aplastic anaemia
vitamin C deficiency (scurvy)
acquired
phenytoin, ciclosporin, calcium channel
drugs blockers, vigabatrim
deposits mucocutaneous amyloidosis
sarcoidosis, Crohn disease, orofacial
chronic granulomatous disorders granulomatosis
263 •
MINDMAP42 43

coeliac disease
genetic Gastrointestinal disease Crohn disease
nutritional ulcerative collitis
host factors
systemic disease
immunity
trauma
cheese
chocolate Aetiology
nuts
tomatoes allergy
environmental factors
citrus fruits
benzoates
iron deficiency
cinnamon aldehyde
Haematological disease B12
infection
folic acid
stress
haematinic replacement
treat gastrointestinal disease
exclusion diet
minor treat infections
major topical anaesthesia
Aphthae Treatment
herpetiform Oral Ulceration topical antimicrobial oral rinse
Behçet topical anti-inflammatory paste
symptomatic
topical coricosteroid
medications/mouthwashes
systemic corticosteroids

breakdown of surface epithelium Carcinoma herpes labialis


VZV
EBV
Infections cytomegalovirus
herpes virus
Coxsackie virus
human papilloma virus

direct damage Trauma


lichen planus
pemphigus
Mucocutaneous disorders
pemphigoid
lupus erythematosus

reduced saliva
Radiotherapy
reduced life span of oral epithelium

FromHeasman P2008 Master DentistryVol. 2, Restorative Dentistry, PaediatricDentistryand Orthodontics, 2nd Edn. Edinburgh, Churchill Livingstone. Fig. 110, with permission.
A1
The indexo orthodontic
treatment need: dental
health component

Grade Characteristics
1. None Extremelyminor malocclusions including displacements <1 mm
2. Little a. Increased overjet >3.5 mmbut ≤6 mmwith competent lips
b. Reverse overjet >0 mmbut ≤1 mm
c. Anterior or posterior crossbite with ≤1 mmdiscrepancybetween retruded contact position and intercuspal position
d. Displacement o teeth >1 mmbut ≤2 mm
e. Anterior or posterior open bite >1 mmbut ≤2 mm
. Increased overbite ≥3.5 mmwithout gingival contact
g. Prenormal or postnormal occlusions with no other anomalies; includes up to hal a unit discrepancy
3. Moderate a. Increased overjet >3.5 mmbut ≤6 mmwith incompetent lips
b. Reverse overjet >1 mmbut ≤3.5 mm
c. Anterior or posterior crossbites with >1 mmbut ≤2 mmdiscrepancybetween retruded contact position and intercuspal position
d. Displacement o teeth >2 mmbut ≤4 mm
e. Lateral or anterior open bite >2 mmbut ≤4 mm
. Increased and complete overbite without gingival or palatal trauma
4. Great a. Increased overjet >6 mmbut ≤9 mm
b. Reverse overjet >3.5 mmwith no masticatoryor speech di culties
c. Anterior or posterior crossbite with >2 mmdiscrepancybetween retruded contact position and intercuspal position
d. Severe displacements o teeth >4 mm
e. Extreme lateral or anterior open bites >4 mm
. Increased and complete overbite with gingival or palatal trauma
h. Less extensive hypodontia, requiring prerestorative orthodontics or orthodonticspace closure to obviate the need or a prosthesis
l. Posterior lingual crossbite with no unctional occlusal contact in one or both buccal segments
m. Reverse overjet >1 mmbut <3.5 mm, with recorded masticatoryand speech di culties
t. Partiallyerupted teeth, tipped and impacted against adjacent teeth
x. Supplemental teeth
5. Verygreat a. Increased overjet >9 mm
h. Extensive hypodontia with restorative implications (more than one tooth missing in anyquadrant) requiring prerestorative orthodontics
i. Impeded eruption o teeth (with the exception o third molars) owing to crowding, displacement, the presence o supernumeraryteeth, retained deciduous
teeth and anypathological cause
m. Reverse overjet >3.5 mmwith reported masticatoryand speech di culties
p. De ects o cle t lip and palate
s. Submerged deciduous teeth

FromHeasman P2013 Master DentistryVol. 2, Restorative Dentistry, Paediatric Dentistryand Orthodontics, 3rd edn. Churchill Livingstone, Edinburgh, with permission.
A2
Classifcation and
defnitions

Extraoral Skeletal pattern Class I Mandible 2–4 mmbehind maxilla


(Anteroposterior) Class II Mandible >4 mmbehind maxilla
Class III Mandible <2 mmbehind maxilla
Skeletal pattern Average Frankort plane (FP; superior aspect o external auditorymeatus to lower border o orbit) and mandibular
(Vertical: FMPA) plane (MP; lower border o mandible) intersect at occiput (backo head)
Increased FPand MPintersect in ront o occiput
Reduced FPand MPintersect behind occiput
Skeletal pattern Symmetrical Midpoint o eyebrows, nasal tip, middle o upper lip at vermillion border and chinpoint coincident with
(Transverse) acial midline
Asymmetrical Midpoint o eyebrows, nasal tip, middle o upper lip and chinpoint non coincident with acial midline
Skeletal pattern Retrognathia Retrusion o maxilla and/or mandible relative to cranial base
Prognathism Protrusion o maxilla and/or mandible relative to cranial base
Nasolabial/lips/smile Nasolabial angle Angle between line drawn through midpoint o nostril aperture and line perpendicular to FPwhile
intersecting subnasale
Competent Upper and lower lips contact without muscular activityat rest
Incompetent Some muscular activityrequired or lips to meet together
Posed smile Voluntarysmile, with no emotional linkand airlyreproducible
Spontaneous smile Involuntarysmile, linked with emotion, maximal elevation o upper lip
Smile arc Relationship between curvature o maxillaryincisal edges and canine tips to curvature o lower lip during
posed smile.
TMJ/mandibular position Centricrelation Condyle in most superior anterior position in glenoid ossa
Rest position (RP) o the Position in which muscles acting on mandible showminimal activity; determined byresting lengths o
mandible muscles o mastication
Postured position Position in which mandible habituallymaintained to acilitate anterior oral seal or aesthetics
Freewayspace Space between occlusal sur aces o the teeth when the mandible is in RPor a position o habitual posture
Premature contact Occlusal contact during centricpath o closure o mandible be ore maximumintercuspation; mayproduce
mandibular displacement, tooth movement or both
Mandibular displacement When closing romRP,mandible displaces (either laterallyor anteriorly) to avoid premature contact
Mandibular deviation Closure path o mandible starts roma postured position
Incisor relationship Class I Lower incisor edges occlude with or lie immediatelybelowcingulumplateau o upper central incisors
(British Standards Class II Lower incisor edges lie posterior to cingulumplateau o upper incisors
Institute
classifcation) Division 1 Upper central incisors are proclined or o average inclination; overjet increased
Division 2 Upper central incisors are retroclined; overjet usuallyminimal or maybe increased
Class III Lower incisor edges lie anterior to cingulumplateau o upper incisors; overjet reduced or reversed
Continued
• 266
A2 CLASSIFICATIONANDDEFINITIONS

Molar relationship Class I Mesiobuccal cusp o 6 occludes with buccal groove o 6


(Angle’s
Class II Mesiobuccal cusp o 6 occludes anterior to buccal groove o 6
classifcation)
Class III Mesiobuccal cusp o 6 occludes posterior to buccal groove o 6
Dental Malocclusion Variation romideal occlusion with dental health and/or psychosocial implications; division between normal occlusion and malocclusion
debatable
Cingulumplateau Convexityo cervical third o lingual/palatal aspect o incisors and canines
Angulation Degree o tip o a tooth in mesiodistal plane
Inclination Degree o tip o a tooth in labio palatal plane
Proclined Upper or lower incisors incline labiallyto a greater degree than normal
Retroclined Upper or lower incisors incline palatally/linguallyto a greater extent than normal
Overjet Distance between upper and lower incisors in horizontal plane; normal = 2–4 mm
Reverse Lower incisors lie anterior to upper incisors; i one or two incisors are involved, termanterior crossbite used
Overbite Vertical overlap o upper and lower incisors when viewed anteriorly: normal = 1
3 – 12 coverage o lower incisors; > 12 = increased;
< 13 = reduced
Curve o Spee Curvature o occlusal plane in sagittal plane
Complete Lower incisors occlude with upper incisors or palatal mucosa
Traumatic Occlusion o lower incisors with palatal mucosa with ulceration
lncomplete Lower incisors do not occlude with opposing upper incisors or palatal mucosa when buccal segment teeth are in occlusion
Open bite Anterior No vertical overlap o incisors when buccal segment teeth are in occlusion
Posterior When teeth are in occlusion, there is space between posterior teeth
Bimaxillary Proclination Upper and lower incisors are proclined relative to skeletal base
Retroclination Upper and lower incisors are retroclined relative to skeletal base
Dentoalveolar compensation Inclination o teeth compensates or underlying skeletal pattern, so occlusal relationship less marked
Bolton (tooth size) discrepancy Mismatch between sumo m d widths o maxillaryand mandibular dentition
Dilaceration Abnormal bend or curve in root or crown o ten ollowing trauma
Supernumeraryteeth Teeth in excess o normal series
Occlusion Centric Position o maximuminterdigitation
Ideal Anatomicallyper ect arrangement o the teeth; rare
Normal Acceptable variation romideal occlusion
Crossbite Buccal Buccal cusps o lower premolars and/or molars occlude buccallyto buccal cusps o upper premolars and/or molars
Lingual (scissors bite) Buccal cusps o lower premolars and/or molars occlude linguallyto palatal cusps o upper premolars or molars
Crowding/spacing Crowding Insu cient space to accommodate teeth in per ect alignment in arch, or segment o arch
Rotation Tooth twisted around long axis
Impaction lmpeded tooth eruption, maybe as a result o displacement o tooth, crowding or supernumerary
Leewayspace Di erence in diameter between C, D,Eand 3, 4, 5
Midline diastema Space between 1–1; more common in upper arch
Spacing Teeth do not touch interproximally; localized or generalized
Hypodontia One or more permanent teeth (excluding third molars) congenitallyabsent
Anchorage/tooth Anchorage Source o resistance to orces generated in reaction to active components o an appliance
movement lntermaxillary Between arches
lntramaxillary Within same arch
Tilting Movement o root apexand crown o tooth in opposite directions around a ulcrum
Bodily Equal movement o root apexand crown o tooth in same direction
Uprighting Mesial or distal movement o root apexso root and crown o tooth are at ideal angulation
Torque Movement o root apexbuccolingually, with no or minimal movement o crown in same direction
Centroid Imaginarypoint in root, ~ 13 romapex, about which a tooth will tip when orce applied to crown
Moment (o a orce) Tendencyo a orce to cause rotation
Migration Physiological (minor) movement o tooth
Relapse Return, ollowing correction, o eatures o original malocclusion
Hyalinization Loss o cells roman area as per light microscopy
Transseptal fbres Periodontal fbres interconnecting adjacent teeth
Camou age Occlusal compensation byorthodontictooth movement or skeletal discrepancies
267 •
CLASSIFICATIONANDDEFINITIONS A2
Extractions Balancing Extraction o same (or adjacent) tooth on opposite side o arch to maintain symmetry
Compensating Extraction o same tooth in opposing arch
Serial Extract C’s at 8.5–9.5 years, D’s ~1 year later, 4’s as 3’s erupting
Functional Working side Side to which mandible shi ts during normal masticatory unction
occlusion Non working side Side away romwhich mandible moves during normal masticatory unction
Non working side inter erences Occlusal contacts present on non working side during lateral excursion o mandible
Disclusion Dynamic separation o opposing teeth during mandibular movements
Canine guided Contact maintained on working side canine teeth during lateral excursion o mandible
Group unction Contacts maintained between several teeth on working side during lateral excursion o mandible
Appliances Removable Appliance removable rommouth consisting primarilyo wire and acryliccomponents; maybe active or passive; used almost exclusively
in upper arch; most unctional appliances are removable
Fixed Appliance fxed to teeth byattachments through which orce application is byarchwires or auxiliaries
Archwire Wire engaged into orthodontic brackets to provide active orces or tooth movement or to stabilize teeth
Anchorage Lingual arch Mandibular fxed anchorage rein orcing appliance, wire soldered onto 6 6 bands extends orward to
contact lingual sur aces o incisors, to maintain arch length
Nance palatal arch Maxillaryfxed anchorage rein orcing appliance, wire soldered onto the 6 6 bands connected to acrylic
button contacting anterior vault o palatal.
Quadhelix Maxillaryexpansion appliance, stainless steel wire, our helices, attached to 6 6 bands
Transpalatal arch Maxillaryfxed anchorage appliance, wire connecting 6 6 bands, to maintain intermolar width
Two by our Fixed appliance to 6 6 and 21 12
Couple Pair o equal and opposite parallel orces applied to a body
Functional Appliance using, removing or modi ying orces generated byoro acial musculature, tooth eruption and dento acial growth
Headgear Extraoral appliance using cervical or cranial anchorage (or both) to apply orces to teeth or jaws or tooth movement or growth
modifcation
Facemask Extraoral appliance using anchorage romchin and orehead to applyanterior orces to maxillarydentition and/or maxilla; requently
used in class IIImalocclusion
Cephalometric Cephalometricanalysis Evaluation and interpretation o both lateral and p a radiographs o head (usuallyconfned to ormer)
analysis Ricketts’ELine Tangent to chin and nose used to assess lip ullness; see Appendix5, Fig. A5.3
Ortho gnathic Pre surgical orthodontics Orthodontictreatment in preparation or OGS
surgery(OGS) Decompensation Removal o dentoalveolar compensation prior to OGS
OGS Surgical repositioning o mandible and/or maxilla or correction o dento acial de ormity
Osteotomy Bilateral sagittal split Surgical mandibular procedure, ramus split parallel to sagittal plane, used to advance, setbackor rotate
(BSSO) mandible
Le Fort 1 Surgical maxillaryprocedure, maxilla osteotomised above tooth apices, used to advance or vertically
reposition maxilla
Autorotation Rotation o mandible around condylar axis a ter vertical maxillaryrepositioning
Distraction osteogenesis Surgical technique or lengthening bones and their associated so t tissue envelope, involving corticotomy, ollowed bygradual
separation (distraction) o bone segments (1 mm/day) and osseous infll
Genioplasty Surgical chin procedure to reposition bonychin point a p, verticallyand/or transversely
Tooth sur ace loss Abrasion Loss o tooth substance as a result o wear caused bydissimilar materials
Attrition Loss o tooth substance as a result o tooth wear
Erosion Loss o tooth substance as a result o chemical dissolution
Growth Growth rotation Rotation o core o mandible and maxilla in relation to cranial base; occurs with normal growth; maybe clockwise or anticlockwise
Functional matrixtheory Theorysuggesting skeletal growth determined by unctional spaces and so t tissues associated with skeletal unit
A3
Primary resources and
recommended reading
British Orthod ontic Society 2010 Managing the Develop ing
Occlu sion – A Gu id e for Dental Practitioners, Lond on.
Cobou rne M 2014 N ational clinical gu id elines for the extraction
of rst p erm anent m olars in child ren. Br Dent J 217:643–648.
Scott JE, Atack N E 2015 The d evelop ing occlu sion of child ren and
you ng p eop le in general p ractice: w hen to w atch and w hen to

Orthodontic problems: refer. Br Dent J 218:151–156.

re erral guide

When to re er What to re er
Primarydentition • Gross skeletal discrepancy
• Markedlyslowdental development (see Chapters 1 and 33)
• Cranio acial anomalies, especiallycle t lip and/or palate (unless
care provided alreadybyspecialist team) (see Chapter 21)
Mixed dentition • Increased overjet with associated severe teasing or marked lip
incompetence (greater trauma risk1) (see Chapter 9)
• Severe reverse overjet2 (see Chapter 11)
• Anterior crossbite with marked mandibular displacement, with
or without associated periodontal trauma (see Chapter 10)
• Posterior crossbite with marked mandibular displacement
(see Chapter 14)
• Unerupted upper permanent incisor(s) (see Chapter 2)
• Ectopic eruption o 6 (see Chapter 1)
• Poor qualityfrst permanent molars (see Chapter 34)
• Developmentallyabsent permanent teeth (see Chapter 8)
• Palatal/ectopic canine (see Chapter 6)
• Pathology, e.g. root resorption o 2’s by3’s (see Chapter 6)
• Medicallycompromised (see Appendix4)
Permanent dentition • Mani est malocclusion with or without skeletal problem
(see Chapter 5)
• Dri ting incisors (see Chapter 18)
• Temporomandibular joint problem(see Chapter 17)
• Complexcombined orthodontic restorative or orthodontic
orthognathicproblems (see Chapter 17)
1
Earlygrowth modifcation indicated
2
Likelyto beneft romearlyorthopaedic intervention
Adaptedbypermission romScott JK&AtackNE, 2015The developingocclusiono childrenandyoung
people in general practice: when to watch and when to re er. Br Dent J. Macmillan
Publishers Ltd.
Adapted with permission o BOS.
A4
Implications o some
medical problems
or orthodontics

Problem Implications
Asthma • Risko intraoral candidiasis with steroid based inhalers
• Maintain excellent oral and appliance hygiene, especiallywith URA
Allergy
Latex • Re er or testing to confrm
• Avoid latexgloves and latex containing orthodontic products
• Treat earlymorning to avoid exposure to airborne latexproducts
Nickel • Uncommon or intraoral allergy
• Place plastic sleeving on headgear to avoid extraoral dermatitis; i intraoral atopyconfrmed, use nickel ree appliance components
Bleeding diathesis • Liaise with medical specialist regarding extractions/surgery
• Mayusuallyproceed with orthodontictreatment
• Avoid mucosal/gingival irritation romappliances
Biphosphonates • Discuss potential treatment with physician, especiallyi extraction is likely
• Assess risko osteonecrosis and advise on slowtooth movement/reduced bone healing
Cardiac de ect with IErisk • Current guidelines indicate ABCis not recommended routinely or procedures likelyto produce bacteraemia
• Consult patient’s cardiologist i anyconcern
Diabetes • Onlyconsider treatment i diabetes is well controlled
• Greater risko periodontal disease
• Time appointment to minimize inter erence with control regime and possible hypoglycaemic attack
Drugs • Sloworthodontic tooth movement
Steroids
NSAIDs
TSADs
Epilepsy • Treat onlyi epilepsyis well controlled
• Avoid headgear and removable appliances due to risks associated with seizure
• Phenytoin induced gingival hyperplasia
• Ensure patient has rested, eaten and taken antiepileptic medication be ore appointment to reduce risko seizure
Juvenile rheumatoid arthritis • Steroid injections intoTMJ mayreduce jawgrowth
• Prone to periodontal breakdown due to long termsteroids
• Controversywith regard to role o unctional appliances
• Avoid prolonged orthodontictreatment
Leukaemia • Delaytreatment to >2 years post BMT
• Mayhave short blunt roots and increased risko root resorption
• Reduced resistance to oral in ections
• Growth modifcation guarded prognosis, as growth maybe suppressed

URA, upper removable appliance; IE, in ective endocarditits; ABC,antibiotic cover; NSAIDs, non steroid anti in ammatorydrugs; TSADs, tricyclic antidepressants; BMT,bone marrowtransplant.
• 270
A4 IMPLICATIONSOFSOMEMEDICALPROBLEMSFORORTHODONTICS

Primary sources and


recommended reading
Patel A, Burd en DJ, Sand ler J 2009 Med ical d isord ers and
orthod ontics. J Orthod 36:1–21.
A5
Cephalometric interpretation
For Cau casians, com p are ind ivid u al valu es w ith Eastman
norm s (Table A5.2).

Skeletal relationships
• A-P. I SNA is < or >81° and S N/maxillary plane is within

Lateral cephalometric
8° ± 3°, correct ANB as ollows: or every degree SNA is
>81°, subtract 0.5° rom the ANB value and vice versa.
• Vertical. MMPA and acial proportion should lend support

analysis
to each other usually.

Tooth position
• To assess i overjet reduction is possible by tipping
movement, do a prognosis tracing (Fig. A5.2), or or
every 1 mm o overjet reduction, subtract 2.5° rom 1
angulation. I the nal angulation is not <95° to maxillary
Aim and objective o plane, tipping is acceptable.
cephalometric analysis
Aim : To assess the anteroposterior and vertical relationships
of the up p er and low er teeth w ith su p p orting alveolar bone
to their respective m axillary and mand ibu lar bases and to
Table A5.1 De nition o commonly used cephalometric points
the cranial base.
and planes
Objective: To com p are the p atient to norm al p op u lation
stand ard s ap p rop riate to his or her racial grou p , id entifying Points and planes De nition
any d ifferences betw een the tw o. S Sella: midpoint o sella turcica
N Nasion: most anterior point o the rontonasal suture (may
use the deepest point at the junction o the rontal and nasal
Practice o cephalometric analysis bones instead)
Ensu re that teeth are in occlu sion and that the p atient is not P Porion: uppermost, outermost point on the bonyexternal
p ostured forw ard . auditorymeatus (upper border o the condylar head is at the
In a d arkened room , by tracing or d igitizing, id entify the same level, which helps location)
p oints and p lanes listed in Table A5.1 (Fig. A5.1); alw ays O Orbitale: most in erior anterior point on the margin o the
trace the m ost prom inent im age. For bilateral land m arks, orbit (use average o the le t and right orbital shadows)
u nless d irectly su p erim p osed , take the average. Calcu late ANS Tip o the anterior nasal spine
angu lar and linear measu rem ents.
PNS Tip o the posterior nasal spine (pterygomaxillaryfssure is
directlyabove, which helps location)
A Apoint: most posterior point o the concavityon the anterior
sur ace o the premaxilla in the midline belowANS
B Bpoint: most posterior point o the concavityon the anterior
sur ace o the mandible in the midline above pogonion
N Pog Pogonion: most anterior point on the bonychin
Me Menton: lowermost point on the mandibular symphysis in
S the midline
P FP O Go Gonion: most posterior in erior point at the angle o the
mandible (bisect the angle between tangent to the posterior
PNS ANS ramus and in erior bodyo the mandible to locate)
A
Planes S–Nline Line drawn through Sand N
Frankort plane Line connecting porion and orbitale
Go B Maxillaryplane Line joining PNSand ANS
Pog Mandibular plane Line joining Go and Me
G
M Functional occlusal plane Line drawn between the cusp tips o the frst permanent
molars and premolars/primarymolars
Fig. A5.1 Standard cephalometric points. FromHeasman P2013 Master DentistryVol. 2, Restorative Dentistry, PaediatricDentistryand
(FromHeasman P2013 Master DentistryVol. 2, Restorative Dentistry, PaediatricDentistryand Orthodontics, 3rd edn. Churchill Livingstone, Edinburgh, with permission.
Orthodontics, 3rd edn. Churchill Livingstone, Edinburgh, with permission).
• 272
A5 LATERALCEPHALOMETRICANALYSIS
Table A5.2 Normal Eastman cephalometric values or Caucasians
Holdaway line
Parameter Value (± SD)
Ricketts’ E-line
SNA 81 ± 3°
SNB 78 ± 3°
ANB 3 ± 2°
S–N/Max 8 ± 3°
1 to maxillaryplane 109 ± 6°
1 to mandibular plane 93 ± 6°
Interincisal angle 135 ± 10°
MMPA 27 ± 4°
Facial proportion 55 ± 2%

FromHeasman P2013 Master DentistryVol. 2, Restorative Dentistry, PaediatricDentistryand


Orthodontics, 3rd edn. Churchill Livingstone, Edinburgh, with permission.

Fig. A5.3 So t tissue lines.

• Check 1 angle to mandibular plane in conjunction with


ANB and MMPA. There is an inverse relationship between
1 angle and MMPA.
• Interincisal angle: as this increases, overbite deepens.
• 1 to APo: this is an aesthetic re erence line, but it is
unwise to use or treatment planning purposes.

So t tissue analysis
• Holdaway line. Lower lip should be ±1 mm to this line.
Predicted
reduction • Ricketts’ E-line. Lower lip should be 2 mm (±2 mm)
in ront o this with the upper lip slightly behind
(Fig. A5.3).
Overjet reduction by tipping movement unacceptable
(note upper incisor root through labial plate)

Fig. A5.2 Prognosis training.


A6
Astructured dental
trauma history orm

THE Leeds Dental Institute


LEEDS
TEACHING
Department of Paediatric Dentistry
HOSPITALS Trauma History &Diagnosis Form WUN178

Name ............................................................................................... Hospital No. ......../..................................


Date ........../.........../.......... Referral Source .....................................................................................................
History of Injury
Date of injury ........../........../.......... Time .........................................................................................................
Location ..........................................................................................................................................................
Cause ..............................................................................................................................................................
Nature of Dental Injury
Other injuries? ....................................................................................................... KO’d? Yes No
Previous dental opinion/treatment?
Radiographs? ..................................................................................................................................................
Symptoms now?
Nausea/dizziness/diplopia? .............................................................................................................................
PDH Relevant PMH
EXAMINATION
Extraoral ..........................................................................................................................................................
Soft tissues ............................................................................................................................................
Facial skeleton .......................................................................................................................................
Mandibular movement/occlusion ............................................................................................................
Intraoral ...........................................................................................................................................................
Soft tissues ............................................................................................................................................
Oral hygiene/perio status .......................................................................................................................

Teeth Present

8 7 6 5 4 3 2 1 1 2 3 4 5 6 7 8

8 7 6 5 4 3 2 1 1 2 3 4 5 6 7 8
• 274
A6 ASTRUCTUREDDENTALTRAUMAHISTORYFORM

Teeth Injured
Vitality Tests Transillu- Root dev
Tooth Clinical Injury Mobility TTP
Eth. Cl ETP mination stage

Vitality Mobility TTP Transillumination Root Dev Stage


- = no resp Grade 0–3 0 = nil N = normal 1 = < 2/3
N = normal + = yes 2 = > 2/3
+ = hypersens ++ = severe 3 = complete
ANK = ankylosis (apex open)
4 = complete
(apex closed)
For Avulsion Only

1. Extra Alv Period .................................................... 2. Method of Storage .................................................

Radiographs Views

Report .............................................................................................................................................................
........................................................................................................................................................................
........................................................................................................................................................................
........................................................................................................................................................................

Diagnosis
Tooth Diagnosis

Treatment Plan
.........................................................................................................................................................................
.........................................................................................................................................................................
.........................................................................................................................................................................
.........................................................................................................................................................................
.........................................................................................................................................................................
.........................................................................................................................................................................

Signed Student ............................................................ Staff ....................................................................

Formreproduced courtesyo MontyDuggal, Leeds Dental Institute.


Index
Page nu m bers follow ed by “f” ind icate gu res, “t” ind icate tables, and “b” ind icate boxes.

A w ith incisor d rifting, 112 expansion, in bu ccal u p p er canines, 23


in increased overbite, 75 length d iscrep ancy, 37
Abnorm ally shaped teeth, m ultip le, w ith p alatal canines, 41 u pp er, severe crow d ing and , 26
199–202 w ith u p p er and low er arches, 34, Archw ire
Abscess, 146–148 34f–35f ad justm ent of, 115f
Absent u p p er lateral incisors, 11–15, Angu lation, canines/ incisors, 13 p rojection of, 116
11f–12f, 223f Ankylosis-related resorp tion, 180 Arrested caries, 193
treatm ent op tions for, 13, 13b Anterior occlu sion, 25f, 36f Asp erger ’s synd rom e, 158b
Accep tance Anterior op en bite (AOB), 81–86, 81f–82f, Asthm a, im p lications of, 269
of infraocclu d ed prim ary m olar, 53 233f Attention d e cit hyp eractivity d isord er
of root resorption, 49–50 cau ses of, 82, 82b, 82t, 83f (ADH D), 158b
of transposition of canines, 47 d iagnosis of, 84 Attrition
Accessory cu sp, 201 exam ination of, 81–83, 81f–82f in late low er incisor crow d ing,
Accessory roots, 202 treatm ent of, 84–86, 84b, 86b ap p roxim al, lack in m od ern d iet, 100
Aciclovir, for prim ary herpetic Antibiotics w ear d u e to, 211
gingivostom atitis, 216–217 intravenous, 147, 147t Au tism , 158b
Acrylated labial bow, low er incisor oral, 146–147, 147t Avu lsed incisor, 169, 179–183, 179f, 253f
crow d ing, 101 p rop hylaxis, 166 long-term treatm ent op tions for, 182
Acrylic allergy, 114 Antifu ngals, p alatal stom atitis, 114
Activator-type fu nctional ap pliance, for Anxiety B
increased overbite, 75 d ental, 152
Active ligatu res, 54 m anagem ent, 152–153 Bacteria, chrom ogenic, 193
Acu te lym phoblastic leu kaem ia, d ental care Ap hthae Bad breath, 194
and , 163 recu rrent. see Recu rrent ap hthae BAMP. see Bone anchored m axillary
ADH D. see Attention d e cit hyp eractivity typ es of, 217–218 p rotraction (BAMP)
d isord er (ADH D) Ap ical p eriod ontitis, acute, 140 Band s
Ad hesively retained cop ings, 189 Ap pliance-related p roblem s, 239f avoid ance w ith p eriod ontal d isease,
Ad olescent Ap pliances 112
anxiou s, 247f in anterior open bite, 84 for d entinogenesis im p erfecta, 208
consent for, 155 ARAB (activation, retention, anchorage, Basep late
gingival bleed ing in, 214b basep late), 9, 9b in bilateral crossbite, 93
high caries risk, 135–139, 135f–136f, in bilateral crossbite, 93–94, 93t, 94f in increased overbite, 75
136t w ith canine transp osition, 47–48 w ith palatal canines, 41
preventive care for, 137–138, 138b, w ith crow d ing, 28–29, 28f Begg retainer, 122, 123f
138f d em ineralization in, 19–20, 20b Behaviou r m anagem ent, 149–150
uncoop erative child and , 149–156, 152b xed nonp harm acological, 152
Aesthetics for absent u p p er lateral incisors, 13 Behçet synd rom e, 218
for am elogenesis im p erfecta, 204t w ith crow d ing and buccal upp er Bilateral crossbite, 91–98, 91f, 92t, 235f
app liances, d rifting incisors, 111 canines, 17–18 Bip hosp honates, im p lications of, 269
AI. see Am elogenesis im p erfecta (AI) hygiene, 114 Bitew ing rad iograp hs
Alignm ent for incisor crossbite, 64 of caries, in ad olescents, 136f
m inor shifts in, 117, 117f w ith increased overbite, 73, 75 for cariou s p rim ary teeth, 141, 141f
in transp osition, 47 in lingu al crossbite, 97–98, 97f Bleaching, for enam el d iscoloration, 197
of u pper canine, 118 low er incisor crow d ing, 101 Bleed ing d iathesis, im p lications of, 269
Allergens, in recurrent ap hthae, 218b orthod ontic, 112t Bolton d iscrep ancy, 37, 38b
Allergy for p osterior crossbite, 89–90 Bond ed retention, for cleft lip and palate,
acrylic, 114 qu ad helix, for buccal up per canines, 23, 128
im p lications of, 269 24f Bone anchored m axillary p rotraction
Alveolar bone grafting, for cleft lip and related problem s, 113–117 (BAMP), 72
palate, 128, 129b rem ovable, w ith absent u p p er lateral ad vantages and d isad vantages of, 72
Alveolar d evelop m ent, localized failu re of, incisors, 14, 14b, 14f Brackets
in anterior op en bite, 82t rem ovable p roblem s, 113f bond failu re, 115f, 115t
Am elogenesis im p erfecta (AI), 188, upp er, 113 d rifting incisors, 112t
203–205, 259f rem oval of, in u neru p ted u pp er Brand y w ine typ e (DI typ e III), 207
com p ared w ith d entinogenesis perm anent central incisor, 9, 9f Breastfeed ing, and caries, 130
im perfecta (DI), 206, 206f repair of, 115 Breath, bad , 194
Analgesics, for cariou s prim ary teeth, 141t risk of, 19 Brow n staining, u orosis, 196
Anchorage u p p er xed , 111f Bu ccal crossbite, bilateral, 91, 91f, 92t
in bilateral crossbite, 93 Arch Bu ccal eru p tion, of u p p er p erm anent
w ith crow d ing, 27, 28b affected by transp osition of canines, 47 canines, 16–20, 16f–17f, 17b, 19f
tem p orary, 28–29 crow d ing, treatm ent options for, 23 Bu ccal occlusion, right, 46, 46f
• 276
INDEX
Bu ccal segm ent Cervical pu lp otom y, for fractu red Com p osite veneers, u orotic m ottling,
of absent upper lateral incisors, 13–14 p erm anent incisor crow n, 172 197–198
crossbite, 87, 87f, 90, 90b Cervical vertebral m atu ration (CVM) Com p u tational m ethod , of tooth-size
Bu llying ind ex, 57 d iscrep ancy, 37–38
com p ared to teasing, 55 Cheek m u cosa, 115 Com p u ted tom ograp hy, of palatal canines,
consequ ences of, 55 Chem otherap y 38, 38b, 39f
p rom inent teeth and , 55 d ental care and , 163, 163f Cone beam com p u ted tom ograp hic (CBCT)
sid e effects of, 164b view, for u neru p ted u pp er perm anent
C Chew ing gu m , su gar-free, for ad olescents, central incisor, 8
w ith caries, 138 Consent, inform ed , 155, 155b
Candida, and p alatal stom atitis, 114, 114b Child p hysical abu se, 169 Cop ings, ad hesively retained , 189
Canine fossa infection, m axillary, 146, 146f Child ren Coronal p ortion, of non-vital, root
Canines w ith d isabilities and learning fractu red tooth, 177
eru ption d ates, 2t d if cu lties, 157–162, 161t Coxsackie viru s, 217
p alatal, 36–45 com m unication w ith, 160–161, 161b Crossbite, 32
p rim ary, u nilateral loss of, 6, 6b d ental m anagem ent for, 159 bilateral, 91–98, 91f, 92t–93t
p roblem s, 227f general anaesthetic for, 160 buccal segm ent, 87, 87f, 90, 90b
rem oval of, 27–28 m ed ical com orbid ities in, 160, 160b in cleft lip and p alate, 126b, 127
retraction and alignm ent of, 35, 35b, 35f nitrou s oxid e inhalation sed ation for, w ith p alatal canines, correction of, 41
transposition in, 46 159–160 p osterior, 87–90
u pp er, crow d ing and bu ccal, 16–20, nonp harm acological m anagem ent of, Crow d ing
16f–17f, 17b, 19f 159, 159b buccal u p p er canines and , 224f
Carbam id e p eroxid e gel, 197 rad iograp hic investigation for, in increased overjet aetiology, 57t
Carbonated d rinks, cau sing erosion, 209, 158–160 lack of, 53
209b treatm ent p lan for, 160 late low er incisor, 99–101
Card iac d efect w ith IE risk, im p lications of, m ed ical p roblem s in, 163–167, 167b, causes of, 100
269 167t d iagnosis of, 100–101
Caries, 188 acu te lym phoblastic leu kaem ia as, m anagem ent of, 100–101
in ad olescents, 135–139, 135f–136f, 136t 163, 163f–164f low er and u p p er arch, 42–44, 44f–45f
risk assessm ent of, 136b congenital heart d efect as, 166, 166f severe, 25–35, 26f
arrested , 193 haem op hilia A as, 165, 165f Crow n, shape abnorm alities of, 201
in d iabetic p atient, 214 vital bleaching for, 197 Crow n fragm ents, p artial p u lp otom y for,
early child hood , 130–134, 131f Chin, p rom inent, and TMJ p ain, 102–108, 173, 173f
d iet ad vice for, 132 102f Cryp t d isp lacem ent, 37
high-risk factors for, 132t Chronic m od erate p eriod ontitis, 111, 111b Cu sp, accessory, 201
hom e based ad vice for, 131 Chronic p eriod ontal d isease, 109 Cyclic neu trop enia, 185f
m ed ication for, 133 Chronological hyp op lasia, in child hood Cytom egaloviru s, 217
p rofessional interventions for, 133 illnesses, 191
rem oval of, 133 Class II d ivision 2 m alocclu sion, 42 D
m anagem ent of, for incisor crossbite, 64 Class III m alocclu sion
Cariou s prim ary teeth, p ain control and d iagnosis of, 105 Deep overbite, 77–78
treatm ent p lanning for, 140–145, 140f TMJ p ain and p rom inent chin and , Delaire-typ e facem ask, 70, 70f
Carisolv, for early child hood caries, 133 103–104 Delayed d evelop m ent, 149
Cast m etal, w ith tooth surface loss, 211t Classi cation and d e nitions, 265–267 Dens evaginatu s, 202
Cavernou s sinu s throm bosis, 146 Cleft lip and p alate (CLP), 125–129, 125f, Dens invaginatu s, 201–202, 202f
Cavitation, in early child hood caries, 130f, 241f Dental anom alies, treatm ent for, 205b
133 d iagnosis of, 128 Dental anxiety, 152, 152b
Cellulitis, facial, 146 exam ination of, 126–127 Dental caries, in child ren w ith com m on
Centerline, low er, m onitor of, for incisor extraoral, 126, 127f m ed ical p roblem s, m anagem ent of, 249f
crossbite, 64 intraoral, 125f, 126–127, 127f Dental erosion, 209–212, 261f
Central incisors, trau m a to, 171f genetic risk of, 125, 125b Dental health com p onent, 264
Centreline shift investigations of, 127 Dental m anagem ent, for child ren w ith
w ith bu ccal u p p er canines, 21, 22f p revalence of, 125, 125b d isabilities, 159
in p osterior crossbite, 88, 88b sex and sid e variation and , 125 Dental p anoram ic tom ogram (DPT), 31, 31f,
w ith u neru p ted u p p er central incisors, treatm ent of, 128–129 100f
6 Cleid ocranial d ysp lasia, 185 in am elogenesis im p erfecta, 204
correction of, 10, 10b CLP. see Cleft lip and p alate (CLP) of d entinogenesis im p erfecta, 206, 206f
p revention of, 6 Cold sore, 216, 217f Dentine d efects, environm entally
Cephalom etric analysis, lateral, 271–272 Colou r, canines/ incisors, 13 d eterm ined , 207
aim and objective of, 271 Com a, d iabetic, 214 Dentinogenesis im p erfecta (DI), 206–208,
interpretation of, 271, 272t Com bined orthod ontic-su rgical ap p roach, 207b, 260f
p ractice of, 271, 271f, 271t TMJ p ain and p rom inent chin and , treatm ent of, 204
Cep halom etric nd ings, 34 104–105, 105b Dentoalveolar inju ry, sp linting of, 176,
Cep halom etric rad iograp h Com bined orthod ontic-su rgical p lanning, 176b
in bilateral crossbite, 92 104t ’Dentu re’ stom atitis, w ith xed app liances,
for increased overjet, 57 Com m unication, w ith child ren w ith 114
for reverse overjet, 67, 70 d isabilities, 160–161, 161b cau ses of, 114t
Cephalom etric values, TMJ p ain and Com m u nication p assp ort, 157, 157f p rognosis of, 114
p rom inent chin and , 104, 104t Com m u nicative m anagem ent, 149–150 treatm ent of, 114
Cep halom etry Com posite, p oor qu ality rst perm anent Develop m ental abnorm alities
in anterior op en bite, 83 m olars and , 189 of enam el, 203b
of bu ccal u pper canines, 22 Com posite d irect/ ind irect, w ith su rface in u pper prem olar rotations, 27
in increased overbite, 74, 77 loss, 211t DI. see Dentinogenesis im p erfecta (DI)
277 •
INDEX
Diabetes Em ergency care, ap p liance related Facial sw elling, d ental abscess and ,
gingival bleed ing and enlargem ent in, p roblem s, 116, 116b 146–148, 245f
213 Enam el, u orosis-related , 195 Fissu re sealants, for ad olescents, w ith
im p lications of, 269 Enam el d efects, d evelop m ental cau ses of, caries, 137–138, 137f, 137t
period ontal p athology and d rifting 203, 203b Fixation, w ith TMJ p ain and p rom inent
incisor, 111 Enam el m icroabrasion, 190 chin, 107
Diastem a, m ed ian Enam el-d entine fractu re, 172 Fixed ap p liances
causes of, 2t End ocard itis, infective, risk of, 25 for absent u p p er lateral incisors, 13
ectopic eru ption and , 1–4, 1f Environm ental cau ses, d entine d efects, 207 in anterior open bite, 85
labial segm ent p roblem s in, 3, 3f Ep ilep sy, im plications of, 269 in bilateral crossbite, 93
up p er rst p erm anent m olar of, 1–4, 1f Ep stein-Barr viru s (EBV), 217 for cleft lip and p alate, 128
Dietary ad vice, for d iabetic p atients, 215 EPT. see Electric p u lp testing (EPT) w ith crow d ing and buccal upp er
Dietary allergens, in recu rrent aphthae, 218b Erosion, d ental, 209–212 canines, 17–18
Dietary causes (food and d rinks) m anagem ent of, 210–211, 210b w ith d eep overbite, 80f
of erosion, 209, 209b treatm ent for, 211b, 211t d rifting incisors, 109
staining d ue to, 193–194, 194f Eru p tion for incisor crossbite, 64–65
Dietary constituents, cau sing erosion, 209 d ates of, 2b, 2t w ith increased overbite, 73, 75–76
Dietary history, of ad olescents, w ith caries, exfoliation and , d isord ers of, 254f for infraocclu d ed p rim ary m olar, 53–54,
136 p rem atu re, 184 53f–54f
Digit-su cking habit, 121 Essix retainers, 28–30 in lingu al crossbite, 97–98, 97f
anterior op en bite d u e to, 83, 83f, 85b low er vacu u m -form ed , 116, 116f p roblem s, 114
in bu ccal segm ent, 91 Evaginated teeth, 202 in tooth m ovem ent, 121, 121f–122f
in increased overjet aetiology, 57t Exfoliation, 185, 185b for transp osition of canines, 47–48, 48f
w ith p osterior crossbite, 88b, 88t, 89 eru ption and , 184–186 Fixed bond ed retainer, 123, 123b–124b
Direct p ulp capping, 172 External in am m atory resorp tion, 180 Fixed retainers, 122
Direction of d isplacem ent, in d isplaced Extirpated tooth, intracanal m ed icam ent in, Fizzy (carbonated d rinks), cau sing erosion,
prim ary incisor, 169 182 210
Disability, in child ren, 248f Extraalveolar d ry tim e (EADT), 180 Flu orid e
Discoloration/ staining, 187 Extraction for ad olescents, 151–152
extrinsic, 193–194 in bu ccal u p p er canines, 23 history, 192b
restorative techniqu es in, 193b w ith canine transp osition (of m ost of ad olescents, w ith caries, 136
Disp laced prim ary incisor, 168–170, 250f d isp laced tooth), 47 m ottling, 195
Disp lacem ent, in p osterior crossbite, 88 of carious prim ary teeth, 141–142 toothp aste, for child ren, 152
Distraction, 150 w ith crow d ing, 27 Flu orid e m ou thw ash
Distraction osteogenesis, in lingual of rst p rem olars, 27 of ad olescents, w ith caries, 136
crossbite, 97t, 98 of u p p er canines, 27–28 for early child hood caries, 133
Dou ble perm anent teeth, 201 w ith crow d ing and bu ccal u p p er for erod ed teeth, 210
Dou ble prim ary teeth, 201, 201b canines, 17–18 Flu orid e su p p lem ents
Dou ble teeth, 200–201 of rst p rem olars, 18–19, 19b for ad olescents, w ith caries, 137, 137t
Dow n synd rom e, 199 of infraocclud ed prim ary m olar, 53 for early child hood caries, 132–133
DPT. see Dental p anoram ic tom ogram (DPT) in lingu al crossbite, 97t Flu orid e toothp aste
Drainage of pu s, 147 w ith p oor qu ality rst p erm anent for ad olescents, w ith caries, 137–138,
Dressing, for open cavities, 140, 140f, 141b m olars, 189–190, 190b 137t
Drifting w ith root resorp tion, 50 for early child hood caries, 131–132
incisors, 109–112, 238f of u pper canines, 119 Flu orid e varnish
sp eci cally relation to history, 110 Extraction-only p lan, 32, 32f ap p lication, for ad olescents, w ith
of low er second p rem olars into contact Extraoral exam ination caries, 137, 137t
w ith second m olars, 103 in bilateral crossbite, 91 for erod ed teeth, 210
Dru gs in increased overbite, 73, 73f, 76–77, Flu orosis, 188, 192, 195, 195b
d entinogenesis im perfecta d ue to, 207 77f FMPA. see Frankfort-m and ibular p lanes
im p lications of, 269 in late low er incisor crow d ing, 99 angle (FMPA)
staining d ue to, 194 in lingu al crossbite, 97–98 Forces, gingival/ occlu sal, 100
Dry tim e, teeth w ith, 180 in p osterior crossbite, 87, 87b Form ocresol, for p u lp otom y, of carious
Du m m y su cking, anterior op en bite d u e to, TMJ p ain and p rom inent chin and , p rim ary teeth, 144
83f 102–103 Fractu re
Extrinsic staining, 193–194, 193f clean-cut, 116
E Eye sw elling, root fractu red p erm anent incisor root, 109
incisors and , 175 root, d rifting incisors and , 109
EADT. see Extraalveolar d ry tim e (EADT) Fractu red incisor crow n, 171–174
Eating d isord er, 210 F im m atu re perm anent, 251f
EBV. see Epstein-Barr viru s (EBV) Fractu red p erm anent incisor, root,
Ectod erm al d ysp lasia, 199 Facem ask, 70, 72b 175–178
Ectop ic eru p tion d esign of, 70, 70f–71f Frankel II ap p liance, for lingu al crossbite,
clinical featu res of, 3–4 effects of, 71, 71f–72f 97, 97f
treatm ent of, 4, 4b, 4f p sychological bene ts of, 71 Frankel III ap p liance for reversed overjet,
up per canines, bu ccal, 16–20, 16f–17f, su ccess of, 72 67–68, 68f
17b, 19f long-term , 72 Frankfort-m and ibu lar p lanes angle
up per rst perm anent m olar of, 1–4, 1f. Facial asym m etry, m ild , in buccal u p p er (FMPA), w ith increased overjet, 56
see also Im p acted u pp er rst m olars canines, 16 Fu nctional ap p liances
Elastic traction, 41, 42f Facial convexity, 56 in anterior op en bite, 84–85
Electric p u lp testing (EPT), 177 Facial grow th, p ost-treatm ent, 122 w ith increased overbite, 75, 75b
for fractu red incisor crow n, 172 Facial height, low er, in increased overjet, 56 for increased overjet, 58, 58b–60b, 60f
for root resorp tion, 49 Facial p ro le, in anterior op en bite, 81, 81f typ es for, 58
• 278
INDEX
Fu nctional app liances (Continued) H erp angina, 217 lateral, u p p er, absent, 11–15, 11f–12f
for reverse overjet, 67–68, 68f H erp es labialis, 217 m axillary, severely reabsorbed , 50b
effects of, 68 H erp es sim p lex viru s (H SV), 216, 216b p erm anent
w ear of, 68 reactivation of, 217b central, affected by m ottling, 195
H erp es viru s typ e 8, 217 eru p tion d ates, 2t
G H erp etic gingivostom atitis, p rim ary, 216, root fractured , 175–178
216f rotations, m ed ian d iastem a and , 2
Gastric acid , causing erosion, 209 H erp etiform ap hthae, 217 up per, space creation in d evelop m ent,
Gastro-oesop hageal re u x d isease (GORD), H igh caries risk ad olescents, 243f 2, 2b
gastric regu rgitation and , 209–210 H osp ital ad m ission, for orofacial infection, p rim ary, erup tion d ates, 2t
GDP. see General d ental p ractitioner (GDP) 147, 147b resorp tion
Gend er, affected by transp osition of H PV. see H u m an p apillom a viru s (H PV) d etection of, 49b
canines, 47 H SV. see H erp es sim plex virus (H SV) by ectop ic m axillary canine, 49b
General anaesthesia H u m an p ap illom a viru s (H PV), 217 u p p er
for child ren w ith d isabilities, 160 H yd rochloric acid (H Cl)-p u m ice w ith increased overbite, 75
contraind ications for, 154 m icroabrasion technique, 196, 196f trau m a of, 58
ind ications for, 154 H yp erglycaem ic com a, 213–214 Increased overbite, 232f
General d ental p ractitioner (GDP), 116–117 H yp od ontia, 199b, 201 Increased overjet, 229f
Generalised slight gingival erythem a, 42, 43f facial/ d ental/ occlu sal associated w ith, Ind ex of orthod ontic treatm ent need , 264
Generalized m arginal gingival erythem a, 13 Infection-related resorp tion, 180
30, 31f genes associated w ith, 13 Infective end ocard itis
Genetic enam el d efects, 203–204 p revalence of, 52–53, 53b d ental care and , 166, 166f
Genetic factors, in palatal canine ectop ia, u p p er incisors, 11–15, 11f–12f risk of, 25
36–37 H yp oglycaem ic com a, 214 Inferior alveolar block injection, 166
GICs. see Glass ionom er cem ent (GICs) H yp om ineralization In ltration injection, 166
Gingiva early child hood caries and , 131f Inform ed consent, 155, 155b
bleed ing of, 213b enam el, hyp op lasia and , 187b, 187f, 188 Infraocclu d ed p rim ary m olar, 51–54, 228f
enlargem ent, oral u lceration and , 216, treatm ent for, 190 aetiology of, 51
216f H yp om ineralized am elogenesis im p erfecta, cause of, 52, 52b
p ressure from , cau sing late low er 204f d ental history, 51
incisor crow d ing, 100 H yp op lasia d iagnosis of, 53
Gingival bleed ing and enlargem ent, early child hood caries and , 131f exam ination of, 51–52
213–215, 262f enam el extraoral, 51
Gingival enlargem ent, system ic cau ses of, cause of, 26 intraoral, 51–52, 51f–52f
214, 215b hyp om ineralization and , 187b, 187f, fam ily history, 51
Gingival health, w ith toothbru shing, 194 188 history of, 51
Gingival recession, 62–63 system ic (chronological) in uences in, com plaint, 51
Gingival third root fractu res, 177 203 investigations for, 52–53
Gingivitis, chronic, 213, 213f H yp op lastic am elogenesis im p erfecta, 203, clinical, 52, 52b
Gingivop lasty, labial, 10 203f rad iographic, 52–53, 53f
Gingivostom atitis, p rim ary herpetic, 216, IOTN DH C grad e, 53
216f I m ed ical history, 51
Glass ionom er cem ent (GICs), 189 p revalence of, 51
GORD. see Gastro-oesop hageal re u x Ibu p rofen, for cariou s p rim ary teeth, 141t treatm ent of, 53–54, 53f–54f, 54b
d isease (GORD) Im p acted u p p er rst m olars Infraocclu sion, 186, 186b
Grafting, alveolar bone, for cleft lip and cau ses, 3, 4t Infraorbital oor, fracture of, 175
p alate, 128, 129b frequ ency/ p revalence, 3 Infraorbital m argin, fractu re of, 175
Greater m axillary protraction, for reverse m anagem ent of, 4 Inhalational sed ation, 150, 151b
overjet, 72 p erm anent, 221f Insulin-d epend ent d iabetes, gingival
Grow th m od i cation, in lingu al crossbite, treatm ent of, 4, 4b, 4f bleed ing and enlargem ent in, 213–214
97t Im p ressions Intercanine w id th
of d ental arches, in p osterior crossbite, in d evelop ing d entition, 2b
H 88 red u ced , cau sing late low er incisor
u p p er and low er, 100 crow d ing, 100
H aem op hilia A, 165, 165f Incisor crossbite, 62–65, 62f, 230f Intercep tive m easu re, for absent u p p er
H alitosis, 194 d iagnosis of, 64 lateral incisors, 13–14, 14b
H all crow n technique, for child ren w ith extraoral exam ination of, 62 Intercep tive treatm ent, for transp osition of
m ed ical p roblem s, 166–167 featu res of, 63, 63b canines, 47
H all crow ns intraoral exam ination of, 62, 62f Interm axillary xation, w ith TMJ p ain and
for cariou s p rim ary teeth, 143–144 IOTN DH C grad e of, 64 p rom inent chin, 106
for child ren w ith d isabilities, 159 labial recession in, p rognosis for, 64 Interp roxim al strip p ing, late low er incisor
H and , foot and m outh d isease, 217 p revalence of, 63 crow d ing, 101
H aw ley retainer special investigations for, 63 Intracanal m ed icam ent, in extirp ated tooth,
for severe crow d ing, 29–30 treatm ent for, 64–65, 64b 182
in tooth m ovem ent, 122, 123f Incisors Intraoral exam ination
H ead and neck synd rom es, associated w ith angu lations of, 21–22 in bilateral crossbite, 91–92, 92f
m issing teeth, 199 central, u neru p ted u pp er, 5–10, 5f in increased overbite, 73–74, 73f–74f, 74b
H ead gear crow n, p erm anent, fractu red , 171–174 in late low er incisor crow d ing, 99–100
w ith severe crow d ing, 28 d ou ble (crow ns are joined ), p rim ary in p osterior crossbite, 87–88, 87f–88f
w ear, for absent u pper lateral incisors, teeth, 200, 200f TMJ p ain and p rom inent chin and,
14 d rifting, 109–112 103–104, 103f
H erbst ap pliance, increased overbite, 75 intrusion, in overbite red u ction, 78, 78b, Intraoral p eriap ical rad iographs, root
H ered itary d entine d efects, 207b 78f fractu res in, 175, 176f
279 •
INDEX
Intravenou s antibiotics, 147, 147t Mand ibu lar grow th, 24, 24f third
Intrinsic d iscoloration, 191f in late, low er incisor crow d ing in late low er incisor crow d ing, 100,
Invaginations, 201–202 cau sation, 100 100b
Invisalign, d rifting incisors and , 111 in posterior crossbite, 88t rem oval of, 101, 101b
IOTN DH C grad e, 100, 105 Mand ibu lar infections, 146 Mottled teeth, 195–198
Irreversible p u lp itis, 140, 140t Mand ibu lar m id line d istraction, in lingu al treatm ent of, 257f
crossbite, 98 Mottling, 195, 195f
J Mand ibu lar p ath of closu re, p rom inent Mouth
chin and TMJ p ain, 103 breathing, in bilateral crossbite, 91, 91b
Joint period ontal, referral for, 111 Mand ibu lar setback osteotom y, TMJ p ain in increased overbite, ap p earance of,
Ju venile rheum atoid arthritis, im p lications and prom inent chin, 107 73–74
of, 269 Maxillary canine fossa infections, 146, Mu ltip le m issing and abnorm ally shap ed
146f teeth, 258f
K Maxillary exp ansion, rap id , in bilateral Mu ltistrand w ire retainer, 122, 123f
crossbite, 93–94, 93b–94b, 93t, 94f
Kim m echanics, in anterior open bite, 85 Maxillary tooth, talon cu sp in, 201–202 N
MCDAS. see Mod i ed Child Dental
L Anxiety Scale (MCDAS) N atal teeth, 184, 184b, 184f
Med ian d iastem a, 220f N eonates
Labial bow causes of, 2t cleft lip and palate in, 126
w ith absent u p p er p erm anent central labial segm ent problem s in, 3, 3f teeth, 184, 184b
incisor, 9 u p p er rst p erm anent m olar of, 1–4, 1f N eu trop enia, cyclic, 185f
w ith increased overbite, 75 Med ical com orbid ities, in child ren w ith N ickel, im p lications of, 269
w ith low er incisor crow d ing (late d isabilities, 160, 160b N ickel allergy, 20–21, 20b, 21f
presentation), 101 Med ical cond ition, staining d u e to, 194 N ickel titaniu m archw ires, ad vantages of,
Labial gingivop lasty, 10 Med ical p roblem s, in child ren, 163–167, 45
Labial segm ent crow d ing, 26, 29, 29b, 29f 167b, 167t N ickel-titaniu m (N iTi) sp rings, 54, 54f
Labial segm ent problem s, in m ed ian acu te lym phoblastic leu kaem ia as, 163, N ightgu ard vital bleaching, 197
d iastem a, 3, 3f 163f–164f N ight-tim e feed ing, early child hood caries
Labial segm ent sp acing congenital heart d efect as, 166, 166f and , 131
d rifting incisors and , 111 haem op hilia A as, 165, 165f N itrou s oxid e, 151, 151b
up p er, causes of, 12, 12b, 12t Microabrasion, u orotic m ottling for, 196b, inhalation sed ation, 153
Labial su rface, of up per p erm anent 197 for child ren w ith d isabilities, 159–160
incisors, affected by m ottling, 195 Mid azolam N N SH s. see N onnu tritive su cking habits
Lain’s d entition, p rognosis of, 111 intravenou s sed ation, 154 (N N SH s)
Lateral cep halom etric analysis, 271–272 for oral sed ation, 154 N onextraction ap proach, in d eep overbite,
aim and objective of, 271 Mid d le third root fractures, 176f 78–79, 79f–80f, 80b
interp retation of, 271, 272t Mid line d istraction osteogenesis, in lingu al N onnu tritive su cking habits (N N SH s), 83,
practice of, 271, 271f, 271t crossbite, 97t 83b
Latex Mid -p alatal sutu re, in bilateral crossbite, N onp harm acological behaviou r
allergy, 51 93 m anagem ent, 152
im p lications of, 269 MIH . see Molar incisor hyp om ineralization for child ren w ith d isabilities, 159, 159b
Le Fort I ad vancem ent, TMJ pain and (MIH ) N on-verbal com m u nication, 149
prom inent chin, 107 Mild m arginal gingival erythem a, 32, 33f N on-vital bleaching, 192
Learning d if culties, in child ren, 248f Millard rep air, 126 N SAIDs, im plications of, 269
Left bu ccal occlu sion, 99f Mind m ap s, 219 N u rsing caries, 130–134, 130f, 131b
Lesions, in p alate, 113 Mineralization tim es, for p erm anent early child hood and , 242f
Leu kaem ia, im p lications of, 269 d entition, 188t
Lingu al appliances, of late low er incisor Minor ap hthae, 217 O
crow d ing, 101 Missing shap ed teeth, m u ltip le, 199–202
Lingu al crossbite, bilateral, 97b, 97t Missing teeth, 199 Occlu sal rad iograp h
Lip sw elling, fractu red p erm anent incisor Mixed d entition, referral gu id e for, 268 of cleft lip and p alate, 127, 128f
crow n and , 171 Mobility, of natal teeth, 184 of p alatal canines, 38
Low er arch Mod i ed Child Dental Anxiety Scale u neru p ted u p p er p erm anent central
crow d ing, 32, 42–44, 44f–45f (MCDAS), 152, 153f incisor, 7, 8f
eru p tion pattern in, 30 Mod i ed palatal arch, in anterior op en bite, Occlu sion
Low er bond ed retainer, 123f 84–85, 84f anterior, 94, 95f–96f
Low er centreline shift, 30, 31b Molar eru p tion, in overbite red u ction, 78 bu ccal, 96, 96f
Low er incisor Molar extru sion, in overbite red u ction, 78 nal, w ith d rifting incisors, 112f
crow d ing, late presentation, 99f, 236f Molar incisor hyp om ineralization (MIH ), forces, cau sing late low er incisor
extraction of, 101 188, 188b–189b crow d ing, 100
in overbite red u ction, 78 Molar intru sion, in anterior op en bite, 85f p osttreatm ent
Lu xated p rim ary tooth, 170 Molars in bilateral crossbite, 95, 96f
erosion in, 209, 209f in increased overbite, 76, 76f
M p erm anent in lingu al crossbite, 98, 98f
eru p tion d ates, 2t in p osterior crossbite, 89, 90f
Major ap hthae, 217 p oor quality rst, 187–190 at p resentation, w ith d rifting incisors,
MAKATON , for child ren w ith d isabilities, u p p er rst, ectop ic eru ption of, 1–4, 109f–110f
160–161 1f relap sed in, in increased overbite, 76,
Mand ibu lar d isplacem ent on closure, 63 p rim ary 76b
in p osterior crossbite, 88, 88b early loss, sp ace loss follow ing, 26, TMJ p ain and p rom inent chin, 107, 107f
prom inent chin and TMJ p ain, 103 26t, 27b Op alescent d entine, hered itary (DI type II),
reverse overjet and , 67 rst, infraocclusion of, 186, 186b 207
• 280
INDEX
Open bite, anterior, 81–86, 81f Palatal lu xation, of p rim ary incisor, 168f Perm anent tooth p u lp otom y, 172–173
Oral hygiene Palatal m u cosa, as d iagnosis of ap p liance, Physical abu se, 169
in bilateral crossbite, 93 114 Plaqu e control
d iabetes d ue to, 214b Palatal stom atitis, m anagem ent of, 114b in ad olescents, 136
w ith d rifting incisors, 109 Palatally d isp laced canines, 39–40, 40b, 40t in caries, 132t
instru ction, for incisor crossbite, 64 Palate, app earance of, 113 Polyu rethane p ow erchain, 54
p oor, chrom ogenic staining d ue to, 194 Panoram ic tom ogram Poor qu ality rst p erm anent m olar, 255f
severe crow d ing and , 25 acu te lym p hoblastic leu kaem ia and , Porcelain, for tooth su rface loss, 211t
Oral m anifestations, d iabetes, 215 164, 164f Position, tooth, cep halom etric analysis for,
Oral sed ation, 154 in anterior open bite, 83 271–272, 272f
Oral u lceration, 216–218, 217b, 263f in bilateral crossbite, 92 Positive reinforcem ent, 149–150, 150b
Orbital ow (blow ou t) fractu res, 175 for child ren w ith d isabilities, 158 Posterior crossbite, 87–90, 234f
Orofacial infection, 147, 147b for cleft lip and p alate, 127, 128f Posterior op en bite, 59
Orofacial soft tissu es, forces from , 121–122 crow d ing and buccal u pper canines, 17, Post-su rgical orthod ontics, phase of, 107
Orthod ontic alignm ent, 42 17f Post-su rgical stability, in u ence of, 107, 107b
Orthod ontic cam ou age, 60 in increased overbite, 74 Pre-ad ju sted ed gew ise ap p liance, 53–54,
in lingu al crossbite, 97t of late low er incisor crow d ing, 100 53f–54f
Orthod ontic consu ltation, referral for, 111 m ed ian d iastem a, 3, 3f Pregnancy, rst perm anent m olars (FPMs)
Orthod ontic m ovem ent, of root fractu red nickel allergy and , 20, 20f in, 191
teeth, 177 of palatal canines, 38, 39f Prem atu re eru p tion, 184
Orthod ontic p roblem s, referral gu id e for, of perm anent teeth, 200, 200f Prem axilla, anterior, vertical p arallax of, 7
268 p oor qu ality rst p erm anent m olars Prem olars
Orthod ontic treatm ent and , 189, 189f eru p tion d ates, 2t
need , ind ex of, 264 in posterior crossbite, 88–89, 89f rst, extraction of, 18–19, 19b
for unerup ted teeth, risks of, 9 of severe crow d ing, 27 low er, extraction of, 101
Orthod ontics, im plications of som e m ed ical of tooth m ovem ent, 119, 120f rotations, cau ses of, 27
p roblem s for, 269–270 of unerupted up per perm anent central TMJ p ain and p rom inent chin, 105
Orthognathic su rgery incisor, 7, 8f transp lant, avu lsed incisor and , 183f
in lingu al crossbite, 97t Paracetam ol, for cariou s p rim ary teeth, 141t Pressu re zones, in tooth m ovem ents, 118,
TMJ p ain and p rom inent chin, 105 Parafu nctional activity, attritional w ear 118f
Osteogenesis im p erfecta, DI typ e I w ith, 211 Pre-surgical orthod ontics, p hase of, 106,
associated w ith, 207 Parental involvem ent, early child hood 107b
Overbite caries and , 132 Prim ary d entition
am ou nt of, 63 Parental presence/ absence, in behaviou ral in cleft lip and palate, 126, 126b
increased , 73–80 m anagem ent, 150 d elayed or failed eruption of teeth in,
red u ction, 78b Partial pu lp otom y, for fractu red p erm anent 185
trau m atic, 73, 73f, 74t incisor crow n, 172, 173b, 173f referral gu id e for, 268
Overjet Periapical p eriod ontitis, d rifting incisors, Prim ary incisors
increased , 55–61, 55f 109 d iscoloured , 170, 170f
aim s of treatm ent for, 58 Periap ical rad iograp h d isplaced , 168–170
cau ses of, 56, 57t for canine transp osition, 47 p alatal lu xation of, 168f
d iagnosis of, 57–58 of fractured incisor crow n, 172 Prim ary teeth
early, treatm ent for, 58 fu ll-m ou th, d rifting incisors and , am elogenesis im p erfecta affecting, 203
extraoral exam ination of, 55–56, 56f 110–111, 110f carious, 140–145, 140f, 143b
intraoral exam ination of, 56, 57f intraoral, root fractu res in, 175, 176f exam ination of, 141, 141f
options for treatm ent of, 60 of root fractu res, 172 restorative treatm ent of, 142t, 143b
rad iographs for, 57 of tooth m ovem ent, 119, 119f–120f treatm ent p lanning for, 142–143, 142t
risk of traum a w ith, 55 Period ontal assessm ent, for root resorp tion, d elayed eru ption of, 184
reverse, 66–72, 66f, 70f 49 d elayed exfoliation of, 186
cau ses of, 67t Period ontal d isease, gingival and , 215f d entinogenesis im perfecta affecting,
d iagnosis of, 67, 70 Period ontal healing, 181–182, 181f 206
extraoral exam ination of, 66, 66f Period ontal ligam ent, d am aged , 182 d ouble, 201, 201b
intraoral exam ination of, 66–67, 67f Period ontal p robing d ep ths, in increased luxated , 170
IOTN DH C grad e of, 67 overbite, 74 trau m a, 169, 170b
prognosis of, 69 Period ontal treatm ent, 111 treatm ent m od alities for, in
treatm ent for, 67–72, 67b–68b, 68f–69f, Period ontally com p rom ised d entition, 112, am elogenesis im perfecta, 204, 204t
68t, 71b 112b Prom inent chin and TMJDS, 237f
Period ontitis Protraction (reverse-p u ll) head gear, 70, 72b
P chronic m od erate, 111, 111b d esign of, 70, 70f–71f
p eriap ical, d rifting incisors, 109 effects of, 71, 71f–72f
Pain Perm anent d entition long-term , 72
control d elayed or failed eru ption of teeth in, p sychological bene ts of, 71
for cariou s p rim ary teeth, 140–145, 140f 185 su ccess of, 72
and treatm ent planning for cariou s m ineralization tim es for, 188t Pu lp capp ing, d irect, 172
prim ary teeth, 244f referral gu id e for, 268 Pu lp cham bers, obliteration of, in
p oor quality rst perm anent m olars Perm anent incisor d entinogenesis im p erfecta, 206f, 208
and , 188–189 crow n, fractu red im m atu re, 171–174 Pu lp su rvival, 182
relief, for nu rsing caries, 133 erosive tooth su rface loss of, 211f Pu lp tester, electric, for fractu red incisor
TMJ, and prom inent chin, 102–108, 102f Perm anent teeth crow n, 172
Palatal canine ectop ia, 36–37, 37b, 37f d ou ble, 201 Pu lp therap y, ind irect, for cariou s p rim ary
Palatal canines, 226f p rem ature exfoliation of, 185b teeth, 143–144
exposu re, surgery of, 40t, 41, 41b treatm ent m od alities for, in Pu lp al exp osu re, of fractured p erm anent
Palatal crib, in anterior open bite, 84–85, 84f am elogenesis im p erfecta, 204, 204t incisor crow n, 172
281 •
INDEX
Pulpectom y, for carious p rim ary teeth, 142 w ith p alatal canines, 41 Severe crow d ing, 225f
Pu lp itis, 140–141, 140t in posterior crossbite, 89 Severe hyp od ontia, 200, 200f
Pu lp otom ies w ith severe crow d ing, 29, 30f Shap e, canines/ incisors, 13
for cariou s p rim ary teeth, 144 therap y, tooth m ovem ents and , 118 Shap ed teeth, abnorm ally, 199–202
for fractu red p erm anent incisor crow n, Rem ovable retainers, 122–123, 123b Shell teeth, 207
173 Rep air-related resorp tion, 180 Size
Pyram id al roots, 202 Rep lacem ent resorp tion, 180 canines/ incisors, 13
Rep lantation, avulsed incisor and , 180, 181f d iscrepancy, 37, 38b
Q Resin-retained brid ge, for absent u p p er Skeletal p attern
lateral incisors, 14, 14b, 15f in anterior open bite, 82t, 84
Qu ad helix Resorp tion (root), 49f in bilateral crossbite, 91, 92t
app liance, for bu ccal u p p er canines, 23, avu lsed incisor and , 180, 180b of bu ccal u p p er canines, 22
24f cau ses of, 49 in increased overbite, 73, 74t, 76–77
in bilateral crossbite, 93t d rifting incisors, 109 in increased overjet, 55–56, 57t
for cleft lip and p alate, 128, 128f incid ence of, 49 in lingu al crossbite, 97
in p osterior crossbite, 89–90, 90f orthod ontically ind uced , 119, 120b in posterior crossbite, 87, 88t
Qu ick-check m ethod , of tooth-size p revention of, 120, 120b Skeletal relationship s, cep halom etric
d iscrep ancy, 37 rad iographs of, 49 analysis for, 271
short- to m ed iu m -term p rognosis w ith, Sm ile, attractive, 95, 95b
R 50, 50f Sm oking
treatm ent op tions for, 49–50, 50b cessation of, 111
Rad iographic investigations Restoration p eriod ontal d isease and , 111
for child ren w ith d isabilities, 158–160, for am elogenesis im p erfecta, 204t SOCRATES, 140
159b of rst p erm anent m olar, 214–215 Soft tissu es
in increased overbite, 74 su rface tooth loss and , 211 analysis, cep halom etric analysis for,
Rad iography Retainer, typ e of, 45, 45f 272, 272f
of absent u pper lateral incisors, 12 Retaining, 41 increased overbite in, 73–74, 74t
for ad olescents, w ith caries, 136–138, Retention in increased overjet aetiology, 57t
136f in bilateral crossbite, 94 Sp ace
in am elogenesis im p erfecta, 204 in crow d ing, 28 closu re (therapeu tic), absent u pper
in anterior op en bite, 83 in increased overbite, 75 lateral incisors, 13
in bilateral crossbite, 92 orthod ontic, role of general d ental creation
of buccal upp er canines, 22 p ractitioner (GDP) in, 123, 124t absent u p per lateral incisors, 13
for cleft lip and p alate, 127 in p alatal canines, 41 for crow d ing and bu ccal u p p er
in d entinogenesis im p erfecta, 206 p lan, in tooth m ovem ent, 122, 122b canines, 18, 18b–19b
for d isp laced p rim ary incisor, 169–170 Retraction, of u p per canines, 119 for u p p er p erm anent incisors, 2, 2b
in d ou ble teeth, 201 Reverse overjet, 231f Sp eech
for incisor crossbite, 63 Reversible pu lp itis, 140, 140t w ith anterior op en bite, 81–82
for increased overjet, 57 RME. see Rap id m axillary expansion (RME) w ith cleft lip and p alate, 126
for infraocclu d ed p rim ary m olar, 52–53 Root Sp linting
intraoral, p oor qu ality rst p erm anent lled incisor, non-vital bleaching for, in root fractu res, 176
m olars and , 189 192 w ith TMJ p ain and p rom inent chin, 107
of late low er incisor crow d ing, 100 lling m aterial, orthod ontic tooth Stability
for reverse overjet, 67, 69–70 m ovem ent, 9 of corrected overjet, 60
root fractu res and , 175 fractu red p erm anent incisor, 175–178, of crossbite correction, 64, 64f
root resorption, 48–49, 49f 252f Stainless steel crow ns, p oor qu ality rst
of severe crow d ing, 27 in incisor trau m a, fractu re, 109 p erm anent m olars, 189
transp osition of canines, 47 resorp tion, m onitoring of, 49–50 Steroid s, im p lications of, 269
unerupted teeth, 7, 7b shap e abnorm alities of, 202 Stom ach, acid ity of, and regurgitation, 209,
Rad iology, in p osterior crossbite, 89 Root canal 210b
Rad iotherapy, sid e effects of, 165b obliteration of, in d entinogenesis Stom atitis, p alatal (red p alate), w ith xed
Rapid m axillary exp ansion (RME), in im perfecta, 206f ap pliances, 114
bilateral crossbite, 93–95, 93b–94b, 93t, treatm ent, avu lsed incisor and , 179–180 Stress, TMJ p ain and , 103
94f, 96b, 96f Rotational correction, in increased overbite, Structured history form , 273, 273f–274f
not feasible, 95 76 Stu d y m od els, of late low er incisor
su rgically assisted , 93t Rotations, u p p er p rem olar, cau ses of, 27 crow d ing, 100
Recession, labial, 64 Su blu xed incisor, 179f
Recurrent aphthae S Su gar intake, red u ce, 114
aetiological factors in, 218, 218b Su gar-free chew ing gum , for ad olescents,
d ietary allergens in, 218b SARPE. see Su rgically assisted rap id w ith caries, 138
therapy for, 218b m axillary exp ansion (SARPE) Su pernu m erary teeth
Red u ction Screw tu rning, w ith palatal canines, 41 aetiology of, 7, 7b
of d eep overbite, 78, 78b Sed ation, oral, 154 treatm ent p lan for, 8
proced ure in, of root fractu red incisor, Sensibility testing, 177, 177b u neru p ted u p p er p erm anent central
176 of incisor, 7 incisor d u e to, 6, 6b, 7f
Referral letter, to orthod ontist, 18f for root resorp tion, 49 Su rface
Regu rgitation, gastric, 210 Sensitivity resorp tion, 180
Reinforcem ent, positive, 149–150 w ith erosive loss, treatm ent of, 210–211 tooth, loss or w ear
Rem ovable ap pliances w ith p oor qu ality rst p erm anent erosion in, 211
w ith absent u p p er lateral incisors, 14, m olars, 189 in increased overbite, 74
14b, 14f Severe class III skeletal p attern, as Su rgically assisted rap id m axillary
in bilateral crossbite, 93t interpretation of nd ings, in TMJ p ain expansion (SARPE), in bilateral
in increased overbite, 75 and prom inent chin, 104 crossbite, 93t, 95, 95f–96f, 96b
• 282
INDEX
Su tu re splint, avu lsed incisor and , 181, 182f Tongue com m unicative m anagem ent for,
Sw allow ing p attern, in anterior op en bite, scrap ers, 194 149–150
81 sw allow ing p attern in, w ith anterior d ental anxiety in, 152
Sw eetened d rink, early child hood caries open bite, 81 inform ed consent for, 155
and , 130 Tooth d iscoloration, hyp om ineralization, inhalation sed ation for, 150–151
Synd rom es, associated w ith m issing teeth, and hypop lasia, 191–194, 192b, 256f treatm ent p lan for, 153–154
199 Tooth fragm ents, in fractu red perm anent Uneru p ted teeth
System ic d iseases, associated w ith ap hthae, incisor crow n and , 171, 171b, 172f u p p er canines, 21–24, 22f
218 Tooth m ou sse, for ad olescents, w ith caries, u p p er central incisor, 5–10, 5f, 222f
138 cau ses of, 6, 6b
T Tooth m ovem ent, 118–124 m anagem ent of, 10b
m echanism for, 119 Up p er arch crow d ing, 32, 42–44, 44f–45f
TADs. see Tem porary anchorage d evices and related p roblem s, 240f Up p er perm anent central incisors, affected
(TADs) slow rate of, 118 by m ottling, 195
Talon cusp, 201, 201f Toothbrushing, 194 Up p er rem ovable ap p liance therap y, 118,
Tau rod ontism , 202 for ad olescents, 151–152 119b
Teasing w ith caries, 137, 137t to p rocline, for incisor crossbite, 64,
com p ared to bu llying, 55 early child hood caries and , 131–132 64f
history of, and incisor trau m a, 55 overzealous, 211
Teens Tooth-size d iscrepancy (TSD), 37, 38b V
late, low er incisor crow d ing in, Transpalatal arch, 28–29
99–100 Transp osition, canines, 46, 47b Vacu u m form ed retainers, 30, 117, 122,
p rom inent chin and TMJ p ain, aetiology of, 47 123f
reassessed in, 103–104, 104b arch and teeth affected by, 47 Varicella zoster viru s (VZV), 217
Teeth, affected by transposition of canines, classi cation of, 47 Veneers, com p osite, 192
47 incid ence of, 47 d entinogenesis im perfecta and , 208
Tell-show -d o techniqu e, 149 positions of, correction of, 48 uorotic m ottling, 197–198
Tem p orary anchorage d evices (TADs) treatm ent for Vertical parallax, of anterior prem axilla, 7
in anterior op en bite, 85–86, 85f factors to consid er in, 47, 47t Vital bleaching, for u orotic m ottling,
in overbite red u ction, 78 op tions in, 47, 47b 197
in severe crow d ing, 28–29 Trau m a Vital p u lp ectom y, for cariou s p rim ary
Tem p orom and ibu lar joint d ysfu nction infection-related resorp tion and , teeth, 144
synd rom e, 103 180–181 Vitality
Tem p orom and ibu lar joint p ain and risk of, w ith increased overjet, 55, 55b of root fractured teeth, m aintain, 177,
p rom inent chin, 102–108, 102f up p er incisor, 58 178f
aim s of treatm ent of, 105 Trau m atic overbite, 73, 73f, 74t tests, 175
case m anage and , 105 TSADs, im p lications of, 269 for fractu red incisor crow n, 172
surgical planning and , 105–106, 106b, TSD. see Tooth-size d iscrep ancy (TSD) Voice control, 150
106f Tuberculated teeth, 202 Vom iting, recu rrent, erosive loss, 210
Ten H ove app liance, in d eep overbite, 79, Turner ’s tooth/ hyp op lasia, 26 VZV. see Varicella zoster viru s (VZV)
79f Tw in-Block ap pliance, 59, 59f
Tension zones, in tooth m ovem ents, 118, in anterior op en bite, 84 W
118f, 119b effects of, and other fu nctional
Throm bosis, cavernous sinu s, 146 ap p liances, 59 Water, ad olescent consu m p tion, 136
Thum b sucking, anterior op en bite d u e to, fabrication of, 58 Wax registration
83f in increased overbite, 75 for fu nctional ap p liances, reverse
Tip p ing m ovem ent, 118, 118b, 118f instructions for, 58–59 overjet, 68
Titaniu m trau m a splint, avulsed incisor of late low er incisor crow d ing, 100
w ith, 179, 179f U in posterior crossbite, 88
Tom ogram White sp ot d em ineralization lesions,
of incisor crossbite, 63, 63f Ulceration, oral, 216–218, 217b 194
for increased overjet, 57 Uncooperative child , 246f Wire
of infraocclud ed prim ary m olar, 52, and ad olescents, 149–156, 152b com posite, 117
53f anxiety m anagem ent for, 152–154 sp u rs, w ith absent u p p er lateral
for reverse overjet, 67, 69 behavior m anagem ent for, 149 incisors, 14, 14b, 14f

You might also like